Você está na página 1de 321

Soluções de Exercı́cios do Livro

“Curso de Análise”, Volume I,


de Elon Lages Lima

Cleber Fernando Colle,


Edson José Teixeira,
Júlio C. C. da Silva (jcconegundes@gmail.com) e
Rodrigo Carlos Silva de Lima (rodrigo.uff.math@gmail.com)

24 de dezembro de 2013
Sumário

1 Conjuntos
Exerc´ıcio 1.1e Funções
.................................................... 7 8
Exerc´ıcio 1.2 . . . . . . . . . . . . . . . . . . . . . . . . . . . . . . . . . . . . . . . . . . . . . . . . . . . . 9
Exerc´ıcio 1.3 . . . . . . . . . . . . . . . . . . . . . . . . . . . . . . . . . . . . . . . . . . . . . . . . . . . . 10
Exerc´ıcio 1.4 . . . . . . . . . . . . . . . . . . . . . . . . . . . . . . . . . . . . . . . . . . . . . . . . . . . . 11
Exerc´ıcio 1.5 . . . . . . . . . . . . . . . . . . . . . . . . . . . . . . . . . . . . . . . . . . . . . . . . . . . . 12
Exerc´ıcio 1.6 . . . . . . . . . . . . . . . . . . . . . . . . . . . . . . . . . . . . . . . . . . . . . . . . . . . . 13
Exerc´ıcio 1.7 . . . . . . . . . . . . . . . . . . . . . . . . . . . . . . . . . . . . . . . . . . . . . . . . . . . . 14
Exerc´ıcio 1.8 . . . . . . . . . . . . . . . . . . . . . . . . . . . . . . . . . . . . . . . . . . . . . . . . . . . . 15
Exerc´ıcio 1.9 . . . . . . . . . . . . . . . . . . . . . . . . . . . . . . . . . . . . . . . . . . . . . . . . . . . . 16
Exerc´ıcio 1.10 . . . . . . . . . . . . . . . . . . . . . . . . . . . . . . . . . . . . . . . . . . . . . . . . . . . . 17
Exerc´ıcio 1.11 . . . . . . . . . . . . . . . . . . . . . . . . . . . . . . . . . . . . . . . . . . . . . . . . . . . . 18
Exerc´ıcio 1.12 . . . . . . . . . . . . . . . . . . . . . . . . . . . . . . . . . . . . . . . . . . . . . . . . . . . . 19
Exerc´ıcio 1.13 . . . . . . . . . . . . . . . . . . . . . . . . . . . . . . . . . . . . . . . . . . . . . . . . . . . . 20
Exerc´ıcio 1.14 . . . . . . . . . . . . . . . . . . . . . . . . . . . . . . . . . . . . . . . . . . . . . . . . . . . . 21
Exerc´ıcio 1.15 . . . . . . . . . . . . . . . . . . . . . . . . . . . . . . . . . . . . . . . . . . . . . . . . . . . . 22
Exerc´ıcio 1.16 . . . . . . . . . . . . . . . . . . . . . . . . . . . . . . . . . . . . . . . . . . . . . . . . . . . . 23
Exerc´ıcio 1.17 . . . . . . . . . . . . . . . . . . . . . . . . . . . . . . . . . . . . . . . . . . . . . . . . . . . . 24
Exerc´ıcio 1.18 . . . . . . . . . . . . . . . . . . . . . . . . . . . . . . . . . . . . . . . . . . . . . . . . . . . . 25
Exerc´ıcio 1.19 . . . . . . . . . . . . . . . . . . . . . . . . . . . . . . . . . . . . . . . . . . . . . . . . . . . . 26
Exerc´ıcio 1.20 . . . . . . . . . . . . . . . . . . . . . . . . . . . . . . . . . . . . . . . . . . . . . . . . . . . . 28
Exerc´ıcio 1.21 . . . . . . . . . . . . . . . . . . . . . . . . . . . . . . . . . . . . . . . . . . . . . . . . . . . . 29

2 Conjuntos Finitos, Enumeráveis e Não-Enumeráveis 30


Exerc´ıcio 2.1 . . . . . . . . . . . . . . . . . . . . . . . . . . . . . . . . . . . . . . . . . . . . . . . . . . . . 31
Exerc´ıcio 2.2 . . . . . . . . . . . . . . . . . . . . . . . . . . . . . . . . . . . . . . . . . . . . . . . . . . . . 32
Exerc´ıcio 2.3 . . . . . . . . . . . . . . . . . . . . . . . . . . . . . . . . . . . . . . . . . . . . . . . . . . . . 33
Exerc´ıcio 2.4 . . . . . . . . . . . . . . . . . . . . . . . . . . . . . . . . . . . . . . . . . . . . . . . . . . . . 34
Exerc´ıcio 2.5 . . . . . . . . . . . . . . . . . . . . . . . . . . . . . . . . . . . . . . . . . . . . . . . . . . . . 38
Exerc´ıcio 2.6 . . . . . . . . . . . . . . . . . . . . . . . . . . . . . . . . . . . . . . . . . . . . . . . . . . . . 39
Exerc´ıcio 2.7 . . . . . . . . . . . . . . . . . . . . . . . . . . . . . . . . . . . . . . . . . . . . . . . . . . . . 40
Exerc´ıcio 2.8 . . . . . . . . . . . . . . . . . . . . . . . . . . . . . . . . . . . . . . . . . . . . . . . . . . . . 41
Exerc´ıcio 2.9 . . . . . . . . . . . . . . . . . . . . . . . . . . . . . . . . . . . . . . . . . . . . . . . . . . . . 42
Exerc´ıcio 2.10 . . . . . . . . . . . . . . . . . . . . . . . . . . . . . . . . . . . . . . . . . . . . . . . . . . . . 43
Exerc´ıcio 2.11 . . . . . . . . . . . . . . . . . . . . . . . . . . . . . . . . . . . . . . . . . . . . . . . . . . . . 44
Exerc´ıcio 2.12 . . . . . . . . . . . . . . . . . . . . . . . . . . . . . . . . . . . . . . . . . . . . . . . . . . . . 45
Exerc´ıcio 2.13 . . . . . . . . . . . . . . . . . . . . . . . . . . . . . . . . . . . . . . . . . . . . . . . . . . . . 46
Exerc´ıcio 2.14 . . . . . . . . . . . . . . . . . . . . . . . . . . . . . . . . . . . . . . . . . . . . . . . . . . . . 47
Exerc´ıcio 2.15 . . . . . . . . . . . . . . . . . . . . . . . . . . . . . . . . . . . . . . . . . . . . . . . . . . . . 48
Exerc´ıcio 2.16 . . . . . . . . . . . . . . . . . . . . . . . . . . . . . . . . . . . . . . . . . . . . . . . . . . . . 49
Exerc´ıcio 2.17 . . . . . . . . . . . . . . . . . . . . . . . . . . . . . . . . . . . . . . . . . . . . . . . . . . . . 50
Exerc´ıcio 2.18 . . . . . . . . . . . . . . . . . . . . . . . . . . . . . . . . . . . . . . . . . . . . . . . . . . . . 51
Exerc´ıcio 2.10 . . . . . . . . . . . . . . . . . . . . . . . . . . . . . . . . . . . . . . . . . . . . . . . . . . . . 52
Exerc´ıcio 2.20 . . . . . . . . . . . . . . . . . . . . . . . . . . . . . . . . . . . . . . . . . . . . . . . . . . . . 53
Exerc´ıcio 2.21 . . . . . . . . . . . . . . . . . . . . . . . . . . . . . . . . . . . . . . . . . . . . . . . . . . . . 54

1
Exerc´ıcio 2.22 . . . . . . . . . . . . . . . . . . . . . . . . . . . . . . . . . . . . . . . . . . . . . . . . . . . . 55
Exerc´ıcio 2.23 . . . . . . . . . . . . . . . . . . . . . . . . . . . . . . . . . . . . . . . . . . . . . . . . . . . . 56
Exerc´ıcio 2.24 . . . . . . . . . . . . . . . . . . . . . . . . . . . . . . . . . . . . . . . . . . . . . . . . . . . . 58
Exerc´ıcio 2.25 . . . . . . . . . . . . . . . . . . . . . . . . . . . . . . . . . . . . . . . . . . . . . . . . . . . . 59
Exerc´ıcio 2.26 . . . . . . . . . . . . . . . . . . . . . . . . . . . . . . . . . . . . . . . . . . . . . . . . . . . . 60
Exerc´ıcio 2.27 . . . . . . . . . . . . . . . . . . . . . . . . . . . . . . . . . . . . . . . . . . . . . . . . . . . . 61
Exerc´ıcio 2.28 . . . . . . . . . . . . . . . . . . . . . . . . . . . . . . . . . . . . . . . . . . . . . . . . . . . . 64
Exerc´ıcio 2.29 . . . . . . . . . . . . . . . . . . . . . . . . . . . . . . . . . . . . . . . . . . . . . . . . . . . . 67

3 NúmerosReais 69
Exerc´ıcio 3.01 . . . . . . . . . . . . . . . . . . . . . . . . . . . . . . . . . . . . . . . . . . . . . . . . . . . . 70
Exerc´ıcio 3.02 . . . . . . . . . . . . . . . . . . . . . . . . . . . . . . . . . . . . . . . . . . . . . . . . . . . . 71

Exerc´
Exerc´ııcio
cio 3.03
3.04 .. .. .. .. .. .. .. .. .. .. .. .. .. .. .. .. .. .. .. .. .. .. .. .. .. .. .. .. .. .. .. .. .. .. .. .. .. .. .. .. .. .. .. .. .. .. .. .. .. .. .. .. 73
74
Exerc´ıcio 3.05 . . . . . . . . . . . . . . . . . . . . . . . . . . . . . . . . . . . . . . . . . . . . . . . . . . . . 75
Exerc´ıcio 3.08 . . . . . . . . . . . . . . . . . . . . . . . . . . . . . . . . . . . . . . . . . . . . . . . . . . . . 76
Exerc´ıcio 3.09 . . . . . . . . . . . . . . . . . . . . . . . . . . . . . . . . . . . . . . . . . . . . . . . . . . . . 77
Exerc´ıcio 3.10 . . . . . . . . . . . . . . . . . . . . . . . . . . . . . . . . . . . . . . . . . . . . . . . . . . . . 78
Exerc´ıcio 3.11 . . . . . . . . . . . . . . . . . . . . . . . . . . . . . . . . . . . . . . . . . . . . . . . . . . . . 79
Exerc´ıcio 3.12 . . . . . . . . . . . . . . . . . . . . . . . . . . . . . . . . . . . . . . . . . . . . . . . . . . . . 80
Exerc´ıcio 3.13 . . . . . . . . . . . . . . . . . . . . . . . . . . . . . . . . . . . . . . . . . . . . . . . . . . . . 81
Exerc´ıcio 3.14 . . . . . . . . . . . . . . . . . . . . . . . . . . . . . . . . . . . . . . . . . . . . . . . . . . . . 82
Exerc´ıcio 3.15 . . . . . . . . . . . . . . . . . . . . . . . . . . . . . . . . . . . . . . . . . . . . . . . . . . . . 83
Exerc´ıcio 3.16 . . . . . . . . . . . . . . . . . . . . . . . . . . . . . . . . . . . . . . . . . . . . . . . . . . . . 84
Exerc´ıcio 3.17 . . . . . . . . . . . . . . . . . . . . . . . . . . . . . . . . . . . . . . . . . . . . . . . . . . . . 85
Exerc´ıcio 3.10 . . . . . . . . . . . . . . . . . . . . . . . . . . . . . . . . . . . . . . . . . . . . . . . . . . . . 86
Exerc´ıcio 3.19 . . . . . . . . . . . . . . . . . . . . . . . . . . . . . . . . . . . . . . . . . . . . . . . . . . . . 87
Exerc´ıcio 3.20 . . . . . . . . . . . . . . . . . . . . . . . . . . . . . . . . . . . . . . . . . . . . . . . . . . . . 88
Exerc´ıcio 3.22 . . . . . . . . . . . . . . . . . . . . . . . . . . . . . . . . . . . . . . . . . . . . . . . . . . . . 90
Exerc´ıcio 3.23 . . . . . . . . . . . . . . . . . . . . . . . . . . . . . . . . . . . . . . . . . . . . . . . . . . . . 92
Exerc´ıcio 3.24 . . . . . . . . . . . . . . . . . . . . . . . . . . . . . . . . . . . . . . . . . . . . . . . . . . . . 93
Exerc´ıcio 3.25 . . . . . . . . . . . . . . . . . . . . . . . . . . . . . . . . . . . . . . . . . . . . . . . . . . . . 94
Exerc´ıcio 3.26 . . . . . . . . . . . . . . . . . . . . . . . . . . . . . . . . . . . . . . . . . . . . . . . . . . . . 95
Exerc´ıcio 3.27 . . . . . . . . . . . . . . . . . . . . . . . . . . . . . . . . . . . . . . . . . . . . . . . . . . . . 96
Exerc´ıcio 3.28 . . . . . . . . . . . . . . . . . . . . . . . . . . . . . . . . . . . . . . . . . . . . . . . . . . . . 97
Exerc´ıcio 3.29 . . . . . . . . . . . . . . . . . . . . . . . . . . . . . . . . . . . . . . . . . . . . . . . . . . . . 98
Exerc´ıcio 3.30 . . . . . . . . . . . . . . . . . . . . . . . . . . . . . . . . . . . . . . . . . . . . . . . . . . . . 99
Exerc´ıcio 3.31 . . . . . . . . . . . . . . . . . . . . . . . . . . . . . . . . . . . . . . . . . . . . . . . . . . . . 100
Exerc´ıcio 3.32 . . . . . . . . . . . . . . . . . . . . . . . . . . . . . . . . . . . . . . . . . . . . . . . . . . . . 101
Exerc´ıcio 3.33 . . . . . . . . . . . . . . . . . . . . . . . . . . . . . . . . . . . . . . . . . . . . . . . . . . . . 102
Exerc´ıcio 3.31 . . . . . . . . . . . . . . . . . . . . . . . . . . . . . . . . . . . . . . . . . . . . . . . . . . . . 103
Exerc´ıcio 3.32 . . . . . . . . . . . . . . . . . . . . . . . . . . . . . . . . . . . . . . . . . . . . . . . . . . . . 104
Exerc´ıcio 3.33 . . . . . . . . . . . . . . . . . . . . . . . . . . . . . . . . . . . . . . . . . . . . . . . . . . . . 105
Exerc´ıcio 3.34 . . . . . . . . . . . . . . . . . . . . . . . . . . . . . . . . . . . . . . . . . . . . . . . . . . . . 106
Exerc´ıcio 3.35 . . . . . . . . . . . . . . . . . . . . . . . . . . . . . . . . . . . . . . . . . . . . . . . . . . . . 107
Exerc´ıcio 3.37 . . . . . . . . . . . . . . . . . . . . . . . . . . . . . . . . . . . . . . . . . . . . . . . . . . . . 108
Exerc´ıcio 3.38 . . . . . . . . . . . . . . . . . . . . . . . . . . . . . . . . . . . . . . . . . . . . . . . . . . . . 109
Exerc´ıcio 3.39 . . . . . . . . . . . . . . . . . . . . . . . . . . . . . . . . . . . . . . . . . . . . . . . . . . . . 110
Exerc´ıcio 3.40 . . . . . . . . . . . . . . . . . . . . . . . . . . . . . . . . . . . . . . . . . . . . . . . . . . . . 111
Exerc´ıcio 3.42 . . . . . . . . . . . . . . . . . . . . . . . . . . . . . . . . . . . . . . . . . . . . . . . . . . . . 112
Exerc´ıcio 3.43 . . . . . . . . . . . . . . . . . . . . . . . . . . . . . . . . . . . . . . . . . . . . . . . . . . . . 113
Exerc´ıcio 3.44 . . . . . . . . . . . . . . . . . . . . . . . . . . . . . . . . . . . . . . . . . . . . . . . . . . . . 114
Exerc´ıcio 3.45 . . . . . . . . . . . . . . . . . . . . . . . . . . . . . . . . . . . . . . . . . . . . . . . . . . . . 115
Exerc´ıcio 3.46 . . . . . . . . . . . . . . . . . . . . . . . . . . . . . . . . . . . . . . . . . . . . . . . . . . . . 116
Exerc´ıcio 3.47 . . . . . . . . . . . . . . . . . . . . . . . . . . . . . . . . . . . . . . . . . . . . . . . . . . . . 117
Exerc´ıcio 3.48 . . . . . . . . . . . . . . . . . . . . . . . . . . . . . . . . . . . . . . . . . . . . . . . . . . . . 118
Exerc´ıcio 3.49 . . . . . . . . . . . . . . . . . . . . . . . . . . . . . . . . . . . . . . . . . . . . . . . . . . . . 119

2
Exerc´ıcio 3.50 . . . . . . . . . . . . . . . . . . . . . . . . . . . . . . . . . . . . . . . . . . . . . . . . . . . . 120
Exerc´ıcio 3.51 . . . . . . . . . . . . . . . . . . . . . . . . . . . . . . . . . . . . . . . . . . . . . . . . . . . . 121
Exerc´ıcio 3.52 . . . . . . . . . . . . . . . . . . . . . . . . . . . . . . . . . . . . . . . . . . . . . . . . . . . . 122
Exerc´ıcio 3.53 . . . . . . . . . . . . . . . . . . . . . . . . . . . . . . . . . . . . . . . . . . . . . . . . . . . . 124
Exerc´ıcio 3.54 . . . . . . . . . . . . . . . . . . . . . . . . . . . . . . . . . . . . . . . . . . . . . . . . . . . . 125
Exerc´ıcio 3.55 . . . . . . . . . . . . . . . . . . . . . . . . . . . . . . . . . . . . . . . . . . . . . . . . . . . . 126
Exerc´ıcio 3.56 . . . . . . . . . . . . . . . . . . . . . . . . . . . . . . . . . . . . . . . . . . . . . . . . . . . . 128
Exerc´ıcio 3.57 . . . . . . . . . . . . . . . . . . . . . . . . . . . . . . . . . . . . . . . . . . . . . . . . . . . . 129
Exerc´ıcio 3.58 . . . . . . . . . . . . . . . . . . . . . . . . . . . . . . . . . . . . . . . . . . . . . . . . . . . . 130
Exerc´ıcio 3.59 . . . . . . . . . . . . . . . . . . . . . . . . . . . . . . . . . . . . . . . . . . . . . . . . . . . . 131
Exerc´ıcio 3.60 . . . . . . . . . . . . . . . . . . . . . . . . . . . . . . . . . . . . . . . . . . . . . . . . . . . . 132

4 Exerc´
Sequências
ıcio 4.1 e. Séries
. . . . . . . de
. . . Números
. . . . . . . . .Reais
................................ 134 135
Exerc´ıcio 4.2 . . . . . . . . . . . . . . . . . . . . . . . . . . . . . . . . . . . . . . . . . . . . . . . . . . . . 136
Exerc´ıcio 4.3 . . . . . . . . . . . . . . . . . . . . . . . . . . . . . . . . . . . . . . . . . . . . . . . . . . . . 137
Exerc´ıcio 4.4 . . . . . . . . . . . . . . . . . . . . . . . . . . . . . . . . . . . . . . . . . . . . . . . . . . . . 138
Exerc´ıcio 4.5 . . . . . . . . . . . . . . . . . . . . . . . . . . . . . . . . . . . . . . . . . . . . . . . . . . . . 139
Exerc´ıcio 4.6 . . . . . . . . . . . . . . . . . . . . . . . . . . . . . . . . . . . . . . . . . . . . . . . . . . . . 140
Exerc´ıcio 4.7 . . . . . . . . . . . . . . . . . . . . . . . . . . . . . . . . . . . . . . . . . . . . . . . . . . . . 141
Exerc´ıcio 4.8 . . . . . . . . . . . . . . . . . . . . . . . . . . . . . . . . . . . . . . . . . . . . . . . . . . . . 142
Exerc´ıcio 4.9 . . . . . . . . . . . . . . . . . . . . . . . . . . . . . . . . . . . . . . . . . . . . . . . . . . . . 143
Exerc´ıcio 4.10 . . . . . . . . . . . . . . . . . . . . . . . . . . . . . . . . . . . . . . . . . . . . . . . . . . . . 144
Exerc´ıcio 4.10 . . . . . . . . . . . . . . . . . . . . . . . . . . . . . . . . . . . . . . . . . . . . . . . . . . . . 145
Exerc´ıcio 4.11 . . . . . . . . . . . . . . . . . . . . . . . . . . . . . . . . . . . . . . . . . . . . . . . . . . . . 146
Exercı́cio 4.11a . . . . . . . . . . . . . . . . . . . . . . . . . . . . . . . . . . . . . . . . . . . . . . . . . . . 147
Exerc´ıcio 4.12 . . . . . . . . . . . . . . . . . . . . . . . . . . . . . . . . . . . . . . . . . . . . . . . . . . . . 148
Exerc´ıcio 4.14 . . . . . . . . . . . . . . . . . . . . . . . . . . . . . . . . . . . . . . . . . . . . . . . . . . . . 149
Exerc´ıcio 4.15 . . . . . . . . . . . . . . . . . . . . . . . . . . . . . . . . . . . . . . . . . . . . . . . . . . . . 150
Exerc´ıcio 4.18 . . . . . . . . . . . . . . . . . . . . . . . . . . . . . . . . . . . . . . . . . . . . . . . . . . . . 151
Exerc´ıcio 4.19 . . . . . . . . . . . . . . . . . . . . . . . . . . . . . . . . . . . . . . . . . . . . . . . . . . . . 152
Exerc´ıcio 4.20 . . . . . . . . . . . . . . . . . . . . . . . . . . . . . . . . . . . . . . . . . . . . . . . . . . . . 154
Exerc´ıcio 4.21 . . . . . . . . . . . . . . . . . . . . . . . . . . . . . . . . . . . . . . . . . . . . . . . . . . . . 155
Exerc´ıcio 4.22 . . . . . . . . . . . . . . . . . . . . . . . . . . . . . . . . . . . . . . . . . . . . . . . . . . . . 156
Exerc´ıcio 4.25 . . . . . . . . . . . . . . . . . . . . . . . . . . . . . . . . . . . . . . . . . . . . . . . . . . . . 157
Exerc´ıcio 4.31 . . . . . . . . . . . . . . . . . . . . . . . . . . . . . . . . . . . . . . . . . . . . . . . . . . . . 159
Exerc´ıcio 4.33 . . . . . . . . . . . . . . . . . . . . . . . . . . . . . . . . . . . . . . . . . . . . . . . . . . . . 160
Exerc´ıcio 4.35 . . . . . . . . . . . . . . . . . . . . . . . . . . . . . . . . . . . . . . . . . . . . . . . . . . . . 161
Exerc´ıcio 4.36 . . . . . . . . . . . . . . . . . . . . . . . . . . . . . . . . . . . . . . . . . . . . . . . . . . . . 162
Exerc´ıcio 4.40 . . . . . . . . . . . . . . . . . . . . . . . . . . . . . . . . . . . . . . . . . . . . . . . . . . . . 163
Exerc´ıcio 4.41 . . . . . . . . . . . . . . . . . . . . . . . . . . . . . . . . . . . . . . . . . . . . . . . . . . . . 164
Exerc´ıcio 4.42 . . . . . . . . . . . . . . . . . . . . . . . . . . . . . . . . . . . . . . . . . . . . . . . . . . . . 165
Exerc´ıcio 4.43 . . . . . . . . . . . . . . . . . . . . . . . . . . . . . . . . . . . . . . . . . . . . . . . . . . . . 166
Exerc´ıcio 4.44 . . . . . . . . . . . . . . . . . . . . . . . . . . . . . . . . . . . . . . . . . . . . . . . . . . . . 167
Exerc´ıcio 4.45 . . . . . . . . . . . . . . . . . . . . . . . . . . . . . . . . . . . . . . . . . . . . . . . . . . . . 168
Exerc´ıcio 4.46 . . . . . . . . . . . . . . . . . . . . . . . . . . . . . . . . . . . . . . . . . . . . . . . . . . . . 169
Exerc´ıcio 4.47 . . . . . . . . . . . . . . . . . . . . . . . . . . . . . . . . . . . . . . . . . . . . . . . . . . . . 170
Exerc´ıcio 4.48 . . . . . . . . . . . . . . . . . . . . . . . . . . . . . . . . . . . . . . . . . . . . . . . . . . . . 171
Exerc´ıcio 4.49 . . . . . . . . . . . . . . . . . . . . . . . . . . . . . . . . . . . . . . . . . . . . . . . . . . . . 173

5Topologia da
Reta 175
Exerc´ıcio 5.01 . . . . . . . . . . . . . . . . . . . . . . . . . . . . . . . . . . . . . . . . . . . . . . . . . . . . 176
Exerc´ıcio 5.02 . . . . . . . . . . . . . . . . . . . . . . . . . . . . . . . . . . . . . . . . . . . . . . . . . . . . 177
Exerc´ıcio 5.03 . . . . . . . . . . . . . . . . . . . . . . . . . . . . . . . . . . . . . . . . . . . . . . . . . . . . 178
Exerc´ıcio 5.04 . . . . . . . . . . . . . . . . . . . . . . . . . . . . . . . . . . . . . . . . . . . . . . . . . . . . 179
Exerc´ıcio 5.05 . . . . . . . . . . . . . . . . . . . . . . . . . . . . . . . . . . . . . . . . . . . . . . . . . . . . 180
Exerc´ıcio 5.06 . . . . . . . . . . . . . . . . . . . . . . . . . . . . . . . . . . . . . . . . . . . . . . . . . . . . 181

3
Exerc´ıcio 5.07 . . . . . . . . . . . . . . . . . . . . . . . . . . . . . . . . . . . . . . . . . . . . . . . . . . . . 182
Exerc´ıcio 5.08 . . . . . . . . . . . . . . . . . . . . . . . . . . . . . . . . . . . . . . . . . . . . . . . . . . . . 183
Exerc´ıcio 5.09 . . . . . . . . . . . . . . . . . . . . . . . . . . . . . . . . . . . . . . . . . . . . . . . . . . . . 184
Exerc´ıcio 5.10 . . . . . . . . . . . . . . . . . . . . . . . . . . . . . . . . . . . . . . . . . . . . . . . . . . . . 185
Exerc´ıcio 5.11 . . . . . . . . . . . . . . . . . . . . . . . . . . . . . . . . . . . . . . . . . . . . . . . . . . . . 186
Exerc´ıcio 5.12 . . . . . . . . . . . . . . . . . . . . . . . . . . . . . . . . . . . . . . . . . . . . . . . . . . . . 187
Exerc´ıcio 5.13 . . . . . . . . . . . . . . . . . . . . . . . . . . . . . . . . . . . . . . . . . . . . . . . . . . . . 188
Exerc´ıcio 5.14 . . . . . . . . . . . . . . . . . . . . . . . . . . . . . . . . . . . . . . . . . . . . . . . . . . . . 189
Exerc´ıcio 5.15 . . . . . . . . . . . . . . . . . . . . . . . . . . . . . . . . . . . . . . . . . . . . . . . . . . . . 190
Exerc´ıcio 5.16 . . . . . . . . . . . . . . . . . . . . . . . . . . . . . . . . . . . . . . . . . . . . . . . . . . . . 191
Exerc´ıcio 5.17 . . . . . . . . . . . . . . . . . . . . . . . . . . . . . . . . . . . . . . . . . . . . . . . . . . . . 192
Exerc´ıcio 5.18 . . . . . . . . . . . . . . . . . . . . . . . . . . . . . . . . . . . . . . . . . . . . . . . . . . . . 194
Exerc´
Exerc´ııcio
cio 5.19
5.20 .. .. .. .. .. .. .. .. .. .. .. .. .. .. .. .. .. .. .. .. .. .. .. .. .. .. .. .. .. .. .. .. .. .. .. .. .. .. .. .. .. .. .. .. .. .. .. .. .. .. .. .. 195
196
Exerc´ıcio 5.21 . . . . . . . . . . . . . . . . . . . . . . . . . . . . . . . . . . . . . . . . . . . . . . . . . . . . 197
Exerc´ıcio 5.22 . . . . . . . . . . . . . . . . . . . . . . . . . . . . . . . . . . . . . . . . . . . . . . . . . . . . 198
Exerc´ıcio 5.23 . . . . . . . . . . . . . . . . . . . . . . . . . . . . . . . . . . . . . . . . . . . . . . . . . . . . 199
Exerc´ıcio 5.24 . . . . . . . . . . . . . . . . . . . . . . . . . . . . . . . . . . . . . . . . . . . . . . . . . . . . 200
Exerc´ıcio 5.25 . . . . . . . . . . . . . . . . . . . . . . . . . . . . . . . . . . . . . . . . . . . . . . . . . . . . 201
Exerc´ıcio 5.26 . . . . . . . . . . . . . . . . . . . . . . . . . . . . . . . . . . . . . . . . . . . . . . . . . . . . 202
Exerc´ıcio 5.27 . . . . . . . . . . . . . . . . . . . . . . . . . . . . . . . . . . . . . . . . . . . . . . . . . . . . 203
Exerc´ıcio 5.28 . . . . . . . . . . . . . . . . . . . . . . . . . . . . . . . . . . . . . . . . . . . . . . . . . . . . 204
Exerc´ıcio 5.29 . . . . . . . . . . . . . . . . . . . . . . . . . . . . . . . . . . . . . . . . . . . . . . . . . . . . 205
Exerc´ıcio 5.30 . . . . . . . . . . . . . . . . . . . . . . . . . . . . . . . . . . . . . . . . . . . . . . . . . . . . 206
Exerc´ıcio 5.31 . . . . . . . . . . . . . . . . . . . . . . . . . . . . . . . . . . . . . . . . . . . . . . . . . . . . 207
Exerc´ıcio 5.32 . . . . . . . . . . . . . . . . . . . . . . . . . . . . . . . . . . . . . . . . . . . . . . . . . . . . 208
Exerc´ıcio 5.33 . . . . . . . . . . . . . . . . . . . . . . . . . . . . . . . . . . . . . . . . . . . . . . . . . . . . 209
Exerc´ıcio 5.34 . . . . . . . . . . . . . . . . . . . . . . . . . . . . . . . . . . . . . . . . . . . . . . . . . . . . 210
Exerc´ıcio 5.35 . . . . . . . . . . . . . . . . . . . . . . . . . . . . . . . . . . . . . . . . . . . . . . . . . . . . 211
Exerc´ıcio 5.36 . . . . . . . . . . . . . . . . . . . . . . . . . . . . . . . . . . . . . . . . . . . . . . . . . . . . 212
Exerc´ıcio 5.37 . . . . . . . . . . . . . . . . . . . . . . . . . . . . . . . . . . . . . . . . . . . . . . . . . . . . 213
Exerc´ıcio 5.38 . . . . . . . . . . . . . . . . . . . . . . . . . . . . . . . . . . . . . . . . . . . . . . . . . . . . 214
Exerc´ıcio 5.39 . . . . . . . . . . . . . . . . . . . . . . . . . . . . . . . . . . . . . . . . . . . . . . . . . . . . 215
Exerc´ıcio 5.40 . . . . . . . . . . . . . . . . . . . . . . . . . . . . . . . . . . . . . . . . . . . . . . . . . . . . 216
Exerc´ıcio 5.41 . . . . . . . . . . . . . . . . . . . . . . . . . . . . . . . . . . . . . . . . . . . . . . . . . . . . 217
Exerc´ıcio 5.42 . . . . . . . . . . . . . . . . . . . . . . . . . . . . . . . . . . . . . . . . . . . . . . . . . . . . 218
Exerc´ıcio 5.43 . . . . . . . . . . . . . . . . . . . . . . . . . . . . . . . . . . . . . . . . . . . . . . . . . . . . 219
Exerc´ıcio 5.44 . . . . . . . . . . . . . . . . . . . . . . . . . . . . . . . . . . . . . . . . . . . . . . . . . . . . 220
Exerc´ıcio 5.45 . . . . . . . . . . . . . . . . . . . . . . . . . . . . . . . . . . . . . . . . . . . . . . . . . . . . 221
Exerc´ıcio 5.46 . . . . . . . . . . . . . . . . . . . . . . . . . . . . . . . . . . . . . . . . . . . . . . . . . . . . 222
Exerc´ıcio 5.47 . . . . . . . . . . . . . . . . . . . . . . . . . . . . . . . . . . . . . . . . . . . . . . . . . . . . 223
Exerc´ıcio 5.48 . . . . . . . . . . . . . . . . . . . . . . . . . . . . . . . . . . . . . . . . . . . . . . . . . . . . 224
Exerc´ıcio 5.49 . . . . . . . . . . . . . . . . . . . . . . . . . . . . . . . . . . . . . . . . . . . . . . . . . . . . 225
Exerc´ıcio 5.50 . . . . . . . . . . . . . . . . . . . . . . . . . . . . . . . . . . . . . . . . . . . . . . . . . . . . 226
Exerc´ıcio 5.51 . . . . . . . . . . . . . . . . . . . . . . . . . . . . . . . . . . . . . . . . . . . . . . . . . . . . 227
Exerc´ıcio 5.52 . . . . . . . . . . . . . . . . . . . . . . . . . . . . . . . . . . . . . . . . . . . . . . . . . . . . 228
Exerc´ıcio 5.53 . . . . . . . . . . . . . . . . . . . . . . . . . . . . . . . . . . . . . . . . . . . . . . . . . . . . 229
Exerc´ıcio 5.54 . . . . . . . . . . . . . . . . . . . . . . . . . . . . . . . . . . . . . . . . . . . . . . . . . . . . 230
Exerc´ıcio 5.55 . . . . . . . . . . . . . . . . . . . . . . . . . . . . . . . . . . . . . . . . . . . . . . . . . . . . 231

Exerc´ııcio
Exerc´ cio 5.56 .. .. .. .. .. .. .. .. .. .. .. .. .. .. .. .. .. .. .. .. .. .. .. .. .. .. .. .. .. .. .. .. .. .. .. .. .. .. .. .. .. .. .. .. .. .. .. .. .. .. .. ..
5.57 232
234
Exerc´ıcio 5.58 . . . . . . . . . . . . . . . . . . . . . . . . . . . . . . . . . . . . . . . . . . . . . . . . . . . . 235
Exerc´ıcio 5.59 . . . . . . . . . . . . . . . . . . . . . . . . . . . . . . . . . . . . . . . . . . . . . . . . . . . . 236
Exerc´ıcio 5.60 . . . . . . . . . . . . . . . . . . . . . . . . . . . . . . . . . . . . . . . . . . . . . . . . . . . . 237
Exerc´ıcio 5.61 . . . . . . . . . . . . . . . . . . . . . . . . . . . . . . . . . . . . . . . . . . . . . . . . . . . . 238
Exerc´ıcio 5.62 . . . . . . . . . . . . . . . . . . . . . . . . . . . . . . . . . . . . . . . . . . . . . . . . . . . . 239
Exerc´ıcio 5.63 . . . . . . . . . . . . . . . . . . . . . . . . . . . . . . . . . . . . . . . . . . . . . . . . . . . . 240

4
Exerc´ıcio 5.64 . . . . . . . . . . . . . . . . . . . . . . . . . . . . . . . . . . . . . . . . . . . . . . . . . . . . 241

6 Limites de Funções 244


Exerc´ıcio 6.01 . . . . . . . . . . . . . . . . . . . . . . . . . . . . . . . . . . . . . . . . . . . . . . . . . . . . 245
Exerc´ıcio 6.02 . . . . . . . . . . . . . . . . . . . . . . . . . . . . . . . . . . . . . . . . . . . . . . . . . . . . 246
Exerc´ıcio 6.03 . . . . . . . . . . . . . . . . . . . . . . . . . . . . . . . . . . . . . . . . . . . . . . . . . . . . 247
Exerc´ıcio 6.04 . . . . . . . . . . . . . . . . . . . . . . . . . . . . . . . . . . . . . . . . . . . . . . . . . . . . 248
Exerc´ıcio 6.05 . . . . . . . . . . . . . . . . . . . . . . . . . . . . . . . . . . . . . . . . . . . . . . . . . . . . 249
Exerc´ıcio 6.06 . . . . . . . . . . . . . . . . . . . . . . . . . . . . . . . . . . . . . . . . . . . . . . . . . . . . 250
Exerc´ıcio 6.07 . . . . . . . . . . . . . . . . . . . . . . . . . . . . . . . . . . . . . . . . . . . . . . . . . . . . 251
Exerc´ıcio 6.08 . . . . . . . . . . . . . . . . . . . . . . . . . . . . . . . . . . . . . . . . . . . . . . . . . . . . 252
Exerc´ıcio 6.09 . . . . . . . . . . . . . . . . . . . . . . . . . . . . . . . . . . . . . . . . . . . . . . . . . . . . 253

Exerc´
Exerc´ııcio
cio 6.10
6.11 .. .. .. .. .. .. .. .. .. .. .. .. .. .. .. .. .. .. .. .. .. .. .. .. .. .. .. .. .. .. .. .. .. .. .. .. .. .. .. .. .. .. .. .. .. .. .. .. .. .. .. .. 254
256
Exerc´ıcio 6.12 . . . . . . . . . . . . . . . . . . . . . . . . . . . . . . . . . . . . . . . . . . . . . . . . . . . . 258
Exerc´ıcio 6.13 . . . . . . . . . . . . . . . . . . . . . . . . . . . . . . . . . . . . . . . . . . . . . . . . . . . . 259
Exerc´ıcio 6.14 . . . . . . . . . . . . . . . . . . . . . . . . . . . . . . . . . . . . . . . . . . . . . . . . . . . . 260
Exerc´ıcio 6.15 . . . . . . . . . . . . . . . . . . . . . . . . . . . . . . . . . . . . . . . . . . . . . . . . . . . . 261
Exerc´ıcio 6.16 . . . . . . . . . . . . . . . . . . . . . . . . . . . . . . . . . . . . . . . . . . . . . . . . . . . . 262
Exerc´ıcio 6.17 . . . . . . . . . . . . . . . . . . . . . . . . . . . . . . . . . . . . . . . . . . . . . . . . . . . . 263
Exerc´ıcio 6.18 . . . . . . . . . . . . . . . . . . . . . . . . . . . . . . . . . . . . . . . . . . . . . . . . . . . . 264
Exerc´ıcio 6.19 . . . . . . . . . . . . . . . . . . . . . . . . . . . . . . . . . . . . . . . . . . . . . . . . . . . . 265
Exerc´ıcio 6.20 . . . . . . . . . . . . . . . . . . . . . . . . . . . . . . . . . . . . . . . . . . . . . . . . . . . . 267
Exerc´ıcio 6.21 . . . . . . . . . . . . . . . . . . . . . . . . . . . . . . . . . . . . . . . . . . . . . . . . . . . . 268
Exerc´ıcio 6.22 . . . . . . . . . . . . . . . . . . . . . . . . . . . . . . . . . . . . . . . . . . . . . . . . . . . . 271
Exerc´ıcio 6.23 . . . . . . . . . . . . . . . . . . . . . . . . . . . . . . . . . . . . . . . . . . . . . . . . . . . . 273
Exerc´ıcio 6.24 . . . . . . . . . . . . . . . . . . . . . . . . . . . . . . . . . . . . . . . . . . . . . . . . . . . . 274

7 Funções
Contı́nuas 275
Exerc´ıcio 7.38 . . . . . . . . . . . . . . . . . . . . . . . . . . . . . . . . . . . . . . . . . . . . . . . . . . . . 276
Exerc´ıcio 7.39 . . . . . . . . . . . . . . . . . . . . . . . . . . . . . . . . . . . . . . . . . . . . . . . . . . . . 278
Exerc´ıcio 7.40 . . . . . . . . . . . . . . . . . . . . . . . . . . . . . . . . . . . . . . . . . . . . . . . . . . . . 279
Exerc´ıcio 7.41 . . . . . . . . . . . . . . . . . . . . . . . . . . . . . . . . . . . . . . . . . . . . . . . . . . . . 280
Exerc´ıcio 7.42 . . . . . . . . . . . . . . . . . . . . . . . . . . . . . . . . . . . . . . . . . . . . . . . . . . . . 282
Exerc´ıcio 7.43 . . . . . . . . . . . . . . . . . . . . . . . . . . . . . . . . . . . . . . . . . . . . . . . . . . . . 283
Exerc´ıcio 7.44 . . . . . . . . . . . . . . . . . . . . . . . . . . . . . . . . . . . . . . . . . . . . . . . . . . . . 284
Exerc´ıcio 7.45 . . . . . . . . . . . . . . . . . . . . . . . . . . . . . . . . . . . . . . . . . . . . . . . . . . . . 285
Exerc´ıcio 7.46 . . . . . . . . . . . . . . . . . . . . . . . . . . . . . . . . . . . . . . . . . . . . . . . . . . . . 286
Exerc´ıcio 7.47 . . . . . . . . . . . . . . . . . . . . . . . . . . . . . . . . . . . . . . . . . . . . . . . . . . . . 287

Derivadas
8 295
Exerc´ıcio 8.46 . . . . . . . . . . . . . . . . . . . . . . . . . . . . . . . . . . . . . . . . . . . . . . . . . . . . 296
Exerc´ıcio 8.47 . . . . . . . . . . . . . . . . . . . . . . . . . . . . . . . . . . . . . . . . . . . . . . . . . . . . 297
Exerc´ıcio 8.48 . . . . . . . . . . . . . . . . . . . . . . . . . . . . . . . . . . . . . . . . . . . . . . . . . . . . 298
Exerc´ıcio 8.49 . . . . . . . . . . . . . . . . . . . . . . . . . . . . . . . . . . . . . . . . . . . . . . . . . . . . 299
Exerc´ıcio 8.50 . . . . . . . . . . . . . . . . . . . . . . . . . . . . . . . . . . . . . . . . . . . . . . . . . . . . 300
Exerc´ıcio 8.51 . . . . . . . . . . . . . . . . . . . . . . . . . . . . . . . . . . . . . . . . . . . . . . . . . . . . 301
Exerc´ıcio 8.52 . . . . . . . . . . . . . . . . . . . . . . . . . . . . . . . . . . . . . . . . . . . . . . . . . . . . 303
Exerc´ıcio 8.53 . . . . . . . . . . . . . . . . . . . . . . . . . . . . . . . . . . . . . . . . . . . . . . . . . . . . 304
Exerc´ıcio 8.54 . . . . . . . . . . . . . . . . . . . . . . . . . . . . . . . . . . . . . . . . . . . . . . . . . . . . 305
Exerc´ıcio 8.55 . . . . . . . . . . . . . . . . . . . . . . . . . . . . . . . . . . . . . . . . . . . . . . . . . . . . 306
9Integral
deRiemann 307

5
10 Sequências e Séries de Fun¸ cões 308
Exercı́cio 10.44 . . . . . . . . . . . . . . . . . . . . . . . . . . . . . . . . . . . . . . . . . . . . . . . . . . . 309
Exercı́cio 10.45 . . . . . . . . . . . . . . . . . . . . . . . . . . . . . . . . . . . . . . . . . . . . . . . . . . . 310
Exercı́cio 10.46 . . . . . . . . . . . . . . . . . . . . . . . . . . . . . . . . . . . . . . . . . . . . . . . . . . . 311
Exercı́cio 10.47 . . . . . . . . . . . . . . . . . . . . . . . . . . . . . . . . . . . . . . . . . . . . . . . . . . . 313
Exercı́cio 10.48 . . . . . . . . . . . . . . . . . . . . . . . . . . . . . . . . . . . . . . . . . . . . . . . . . . . 314
Exercı́cio 10.49 . . . . . . . . . . . . . . . . . . . . . . . . . . . . . . . . . . . . . . . . . . . . . . . . . . . 316
Exercı́cio 10.50 . . . . . . . . . . . . . . . . . . . . . . . . . . . . . . . . . . . . . . . . . . . . . . . . . . . 317
Exercı́cio 10.51 . . . . . . . . . . . . . . . . . . . . . . . . . . . . . . . . . . . . . . . . . . . . . . . . . . . 318
Exercı́cio 10.52 . . . . . . . . . . . . . . . . . . . . . . . . . . . . . . . . . . . . . . . . . . . . . . . . . . . 319
Exercı́cio 10.53 . . . . . . . . . . . . . . . . . . . . . . . . . . . . . . . . . . . . . . . . . . . . . . . . . . . 320

6
Capı́tulo 1

Conjuntos e Funções

7
Exercı́cio 1.1:
Dados os conjuntos A e B , seja X um conjunto com as seguintes propriedades:
(1a ) X ⊃ A e X ⊃ B,
(2a ) Se Y⊃ A e Y ⊃ B então Y ⊃ X.
Prove que X = A ∪ B.

A inclusão A ∪ B ⊂ X é fornecida pela primeira hipótese. De fato, se x ∈ A ⊂ X ou x ∈ B ⊂ X (isto é, se


x ∈ A ∪ B) então x ∈ X .
E a segunda hip´otese fornece a inclus˜ao A B X pois A B A e A B B.
Portanto, X = A ∪ B. ∪ ⊂ ∪ ⊃ ∪ ⊃

8
Exercı́cio 1.2:
Enuncie e prove um resultado, análogo ao anterior, caracterizando A ∩ B.
Enunciado:
Dados os conjuntos A e B , seja X um conjunto com as seguintes propriedades:
1a X ⊂ A e X ⊂ B,
2a Se Y ⊂ A e Y ⊂ B então Y ⊂ X .
Prove que X = A ∩ B.
Prova:
A inclusão A ∩ B ⊃ X é fornecida pela primeira hipótese. De fato, se x ∈ X temos que A ⊃ X ∋ x e B ⊃ X ∋ x.
Consequentemente, se x ∈ X então x ∈ A ∩ B.
E a segunda hip´otese fornece a inclus˜ao A ∩ B ⊂ X pois A ∩ B ⊂ A e A ∩ B ⊂ B.
Portanto, X = A ∩ B.

9
Exercı́cio 1.3:

Sejam A, B E . Prove que A ∩ B = ∅ se, e somente se, A ⊂ E \B. Prove também que A ∪ B = E se, e somente
\ ⊂
se, E A B.

• A ∩ B = ∅ se e somente se A ⊂ E \B:
Suponhamos que A ∩ B = ∅. Se x ∈ A devemos ter que x pertence a E \B. De fato, como x pertence a A e
A está contido em E , segue que x pertence a B ou E \B. Como A ∩ B = ∅, temos que x ∈
/ B . Logo, x ∈ E \B.
Assim, A ⊂ E \B.
Consideremos o caso em que A E B. Se existisse x A B ter´ıamos que x Aex B. Mas, como A e´ um
\∩
subconjunto de E B, ter´
concluimos que A B = .
⊂ \
ıamos também que x
∅ ∈ E \B. Um∈ absurdo,
∩ pois se x ∈ E∈ \B então
∈ x ∈/ B. Desta forma,
• A ∪ B = E se e somente se E \A ⊂ B:
Suponhamos que A ∪ B = E . Se x ∈ E \A devemos ter que x pertence a B. De fato, como x pertence a E e
E = A ∪ B, devemos ter que x ∈ A ou x ∈ B. Além disso, como x ∈ E \A, temos também que x ∈ / A. O que nos
garante que x ∈ B. Logo, E \A ⊂ B.
Consideremos o caso em que E \A ⊂ B. Seja x ∈ E . Segue que, x ∈ A ou x ∈ E \A. Se x ∈ E \A então x
pertence a B pois E \A está contido em B. Logo, x ∈ A ou x ∈ B. Ou seja, x ∈ A ∪ B. Assim, devemos ter que
E ⊂ A ∪ B. E, como A e B estão contidos em E , segue (veja o exercicio 1.1) que E = A ∪ B.

10
Exercı́cio 1.4:
Dados A, B ⊂ E, prove que A ⊂ B se, e somente se, A ∩ (E \B) = ∅.
⊂ ∈ ∩ \ ∈ ∈ \ ∈
Suponhamos que A B. Se existisse x A (E B) ter´ıamos que x A e x E B. Isto é, existiria x E tal
∈ ∈ ⊂ ∈ ∈ ∩ \
que x A e x / B. Mas, isto é um absurdo, pois, como A B, se x A então x B. Portanto, A (E B) = .∅
∩ \ ∅ ∈
Consideremos, agora, o caso em que A (E B) = . Seja x A. Como A ⊂ ∈
E , temos que x E . Assim,
∈ ∈ \ ∈ ∈ \ ∈ ∩ \ ∅
x B ou x E B. Logo, x B pois se x E B ter´ıamos que x A (E B) = .

11
Exercı́cio 1.5:
Dê exemplo de conjuntos A , B,C tais que ( A ∪ B) ∩ C̸= A ∪ (B ∩ C ).
{ } { } { }
Tome A = 1, 2, 3 , B = 1, 3 e C = 1, 2 . Desta forma, temos

(A ∪ B) ∩ C = {1, 2} ̸
= {1, 2, 3} = A ∪ (B ∩ C ).

12
Exercı́cio 1.6:
Se A, X ⊂ E são tais que A ∩ X = ∅ e A ∪ X = E , prove que X = E \A.
Seja x ∈ X . Uma vez que x ∈ / ∅ = A ∩ X , temos que x ∈ / A. E, como x ∈ X ⊂ E , devemos ter, também, que
x∈ E . Logo, x ∈ E \A. Portanto, como x ∈ X e´ arbitráro, devemos ter que X ⊂ E \A.
Considere, agora, x ∈ E \A. Segue que x ∈ E e x ∈ / A. Como x ∈ E = A ∪ X e x ∈ / A, temos que x ∈ X .
Portanto, como x ∈ E \A e´ arbitráro, devemos ter que X ⊂ E \A.

13
Exercı́cio 1.7:
Se A ⊂ B, então
B ∩ (A ∪ C ) = (B ∩ C ) ∪ A,
para todo conjunto C . Por outro lado, se existir C de modo que a igualdade acima seja satisfeita, ent˜ ao A ⊂ B.
Primeiramente, mostremos que se A ⊂ B então, para qualquer conjunto C , temos
B ∩ (A ∪ C ) = (B ∩ C ) ∪ A.

Seja x ∈ B ∩ (A ∪ C ). Assim, x ∈ B e (x ∈ C ou x ∈ A).

• Se x ∈ C temos que x ∈ B ∩ C. Logo, x ∈ (B ∩ C ) ∪ A.


• Se x ∈ A temos imediatamente que x ∈ (B ∩ C ) ∪ A.
Segue, em todo caso, que x ∈ (B ∩ C ) ∪ A. Logo, concluimos que B ∩ (A ∪ C ) ⊂ (B ∩ C ) ∪ A.
Considere, agora, que x ∈ (B ∩ C ) ∪ A. Assim, x ∈ B ∩ C ou x ∈ A.
• Se x ∈ B ∩ C então x ∈ B e x ∈ C . Logo, x ∈ B, x ∈ A ∪ C e, consequentemente, x ∈ B ∩ (A ∪ C ).
• Se x ∈ A temos que x ∈ B, já que A ⊂ B. Assim, x ∈ B e x ∈ A ⊂ A ∪ C. Logo, x ∈ B ∩ (A ∪ C ).
Em ambos os casos, x ∈ B ∩ (A ∪ B). Desta forma, tem-se que B ∩ (A ∪ C ) ⊃ (B ∩ C ) ∪ A.
Portanto, se A ⊂ B então B ∩ (A ∪ C ) = (B ∩ C ) ∪ A, para qualquer conjunto C .
Reciprocamente, suponhamos que exista um conjunto C tal que x ∈ (B ∩ C ) ∪ A = B ∩ (A ∪ C ).
Se x ∈ A temos que x ∈ (B ∩ C ) ∪ A. Mas, como ( B ∩ C ) ∪ A = B ∩ (A ∪ C ), devemos ter que x ∈ B. Logo,
conclui-se que A ⊂ B.

14
Exercı́cio 1.8:
Suponhamos que A e B sejam subconjuntos de E . Prove que A = B se, e somente se,
( ∩ \ ∪(
A (E B) (E A)\ ∩B  = . ∅

Suponhamos que A = B. Neste caso, temos que

\
E A = E B. \
Logo, A ∩ (E \B) = A ∩ (E \A) = ∅
e
B ∩ (E\A) = B ∩ (E \B) = ∅.
Portanto, ( ∩ \  ∪ ( ∩ \  ∅∪ ∅ ∅
A (E B) B (E A) = = .
Reciprocamente, consideremos o caso em que
( ∩ \ ∪( ∩ \  ∅
A (E B) B (E A) = .

∈ A. Se supusermos, por absurdo, que


Seja x
(∈ ∩ ∈ \  ∪ ( ∩ \ ∈  ∩ ∅ \
x A
x / B teremos que x

(E B) B
A

(E A) = .
(E B) e, consequentemente,

Uma contradição. De modo inteiramente an´alogo é imposs´ıvel que x ∈ B e x ∈/ A. Portanto, A = B.

15
Exercı́cio 1.9:
Prove que
\ ∪ (B\A) = (A ∪ B)\(A ∩ B).
(A B)

• (A\B) ∪ (B\A) ⊂ (A ∪ B)\(A ∩ B)


Seja x ∈ (A\B) ∪ (B \A). Neste caso, x ∈ A\B ou x ∈ B \A. Se x ∈ B \A então temos que x ∈ A e x ∈ / B.
Logo, x ∈ A ∪ B e x ∈ / A ∩ B, ou seja, x ∈ (A ∪ B)\(A ∩ B). Analogamente, x ∈ B \A implica x ∈ (A ∪ B)\(A ∩ B).

• (A\B) ∪ (B\A) ⊃ (A ∪ B)\(A ∩ B)


Seja x ∈ (A ∪ B)\(A ∩ B). Neste caso, x ∈ A ∪ B e x ∈ / A ∩ B. Se x ∈ A então x ∈
/ B, uma vez que x ∈
/ A ∩ B.
Isto é, se x ∈ A então x ∈ A\B. Analogamente, se x ∈ B, temos que x ∈ B \A. Portanto, x ∈ (A\B) ∪ (B \A).

16
Exercı́cio 1.10:
Para conjuntos A e B , definimos o conjunto

\ ∪ (B\A).
A∆B := (A B)

Prove que A∆B = A∆C implica que B = C . Examine a validade um resultado an´alogo com , ∩ ∪ ou × em vez de
∆.

Suponhamos que A∆B = A∆C .


Mostraremos que os conjuntos B ∩ A e B\A estão contidos em C . Desta forma, como B = (B ∩ A) ∪ (B \A),
concluiremos que B C .
∈ ∈∩
⊂ \ ∪ ∈\ \ ∪ \ ∈ ∈\ \ ∈ ∈\
Seja x B A. Temos que x / A∆B = (A B) (B A), pois x / A B e x / B A. Assim, como A∆B = A∆C ,

temos que x / A∆C = (A C ) (C A) e, consequentemente, x / A C . Logo, x C pois x A e x / A C . Como∈ \
∈ ∩ ∩ ⊂
x B A e´ arbitrário, concluimos que B A C .
∈ \
Seja x ∈ \ ∪ \
B A. Logo, x (A B) (B A) = A∆B. E, como A∆B = A∆C , temos que x ∈
A∆C . Sendo
∈ \ ∪ \ ∈ \ ∈ \ ∈
x A∆C = (A C ) (C A), segue que x A C ou x / C A. Assim, j´a que x / A, devemos ter que x C A e, ∈ \
∈ ∈ \ \ ⊂
consequentemente, x C . Como x B A e´ arbitrário, concluimos que B A C .
∩ \ ⊂
Por fim, como B A e B A estão contidos em C , devemos ter que B C . E, de forma an´aloga, prova-se que
∩ \ ⊂
C A e C A estão contidos em B . Logo, C B. Portanto, supondo que A∆B = A∆C , temos que B = C .
∩∪ ×
Consideremos agora a validade dos casos análogos para , e ao invés de ∆.
Existem A, B e C tais que
• A ∩ B = A ∩ C e B ̸= C . Por exemplo: A = {1}, B = {1, 2} e C = {1, 2, 3};
• A ∪ B = A ∪ C e B ̸= C . Por exemplo: A = {1}, B = {2} e C = {1, 2};
• A × B = A × C e B ̸= C . Por exemplo: A = ∅, B = {1} e C = {2}.

17
Exercı́cio 1.11:
Prove as seguintes afirmações:
(a) ( A ∪ B) × C = (A × C ) ∪ (B × C );
(b) ( A ∩ B) × C = (A × C ) ∩ (B × C );
(c) ( A − B) × C = (A × C ) − (B × C );
(d) A ⊂ A′ , B ⊂ B ′ =⇒ A × B ⊂ A ′ × B ′ .

(a) Temos que a igualdad e ( A ∪ B) × C = (A × C ) ∪ (B × C ) é v´alida pois


∈ (A ∪ B) × C ⇐⇒ x ∈ A ∪ B e c ∈ C
(x, c)
⇐⇒ (x ∈ A e c ∈ C ) ou ( x ∈ B e c ∈ C )
⇐⇒ (x, c) ∈ A × C ou (x, c) ∈ B × C
⇐⇒ (x, c) ∈ (A × C ) ∪ (B × C ).
(b) Temos que a igualdad e ( A ∩ B) × C = (A × C ) ∩ (B × C ) é v´
alida pois

(x, c) ∈ (A ∩ B) × C ⇐⇒ x ∈ (A ∩ B) e c ∈ C
⇐⇒ (x ∈ A e c ∈ C ) e (x ∈ B e c ∈ C )
⇐⇒ (x, c) ∈ A × C e (x, c) ∈ B × C
⇐⇒ (x, c) ∈ (A × C ) ∩ (B × C ).
(c) Temos que a igualdad e ( A − B) × C = (A × C ) − (B × C ) é v´alida pois

(x, c) ∈ (A − B) × C ⇐⇒ x ∈ A − B e c ∈ C
⇐⇒ (x ∈ A e c ∈ C ) e (x ∈/ B e c ∈ C )
⇐⇒
⇐ ⇒ (x, c) ∈ A × C e (x, c) ∈
(x, c) ∈ (A × C ) − (B × C ).×
/B C

(d) Seja ( a, b) ∈ A × B. Então, a ∈ A′ e b ∈ B ′ pois A ⊂ A′ e B ⊂ B ′ . Logo, ( a, b) ∈ A′ × B ′ . Portanto,


concluimos que A × B ⊂ A ′ × B ′ .

18
Exercı́cio 1.12:
Dada uma fun¸cão f : A→ B:
(a) Prove que se tem f (X \Y ) ⊃ f (X )\f (Y ), sejam quais forem os subconjuntos X e Y de A;
(b) Mostre que se f for injetora então f (X \Y ) = f (X )\f (Y ) para quaisquer X e Y contidos em A.

(a)

∈ \ ∈
Suponhamos que z f (X ) f (Y ). Desta forma, temos que z f (X ) e, consequentemente, existe x X tal ∈
que f (x) = z. Como z / f (Y ) e z = f (x), devemos ter que x / Y . Logo, x X Y . Assim, concluimos que
∈ \
z = f (x) f (X Y ). ∈ ∈ ∈ \
\ ⊂ \
Portanto, devemos ter que f (X Y ) f (X ) f (Y ).

(b)

\ ⊂ \ \ ⊃
Pelo item (a), temos que f (X Y ) f (X ) f (Y ). Logo, basta verificarmos que f (X Y ) f (X ) f (Y ).\
∈ \ ∈ \ ∈
Seja z f (X Y ). Então, podemos escolher x X Y tal que f (x) = z. Assim, z = f (x) f (X ) pois x X . ∈
∈ ∈ ∈
Por outro lado, como f é injetivo, f (x) = z e x / Y , nenhum y Y é tal que f (y) = z. Logo, z / f (Y ). Portanto,
∈ \
z f (X ) f (Y ).
\ \
Com isso, concluimos que f (X Y ) = f (X ) f (Y ).

19
Exercı́cio 1.13:
Mostre que a fun¸cão f : A → B é injetora se, e somente se, f (A\X ) = f (A)\f (X ) para todo X ⊂ A.
Se f : A→ B é injetiva, pelo item (b) do exerc´ıcio 1.12, a igualdade f (A\X ) = f (A)\f (X ) é v´alida para todo
X ⊂ A.
Suponhamos que a igualdade f (A\X ) = f (A)\f (X ) seja válida para todo X ⊂ A. Seja a ∈ A e denotemos por
b o elemento f (a) ∈ B. Assim,
b∈/ f (A\{a}) = f (A)\f ({a}).
Logo, não existe a ′ ∈ A \{a} tal que f (a′ ) = b = f (a). Desta forma, como a ∈ A e´ arbitrário, concluimos que f é
injetivo.

20
Exercı́cio 1.14:
Dada a fun¸cão f : A → B, prove que:
(a) f −1 (f (X )) ⊃ X para todo X ⊂ A;
(b) f é injetora se, e somente se, f −1 (f (X )) = X para todo X ⊂ A.

(a)

Se x ∈ X então x ∈ f − (f (X )) pois f (x) ∈ f (X ). Assim, devemos ter que


1
f −1 (f (X )) ⊃ X.
(b)

Suponhamos que f é injetora e fixemos X A. Provaremos que f −1 (f (X ))


⊂ ⊂
X e concluiremos, pelo item
(a), que f −1 (f (X )) = X . Desta forma, podemos concluir que se f é injetora então f −1 (f (X )) = X , para qualquer

X A.
Seja y f −1 (f (X )). Segue que f (y) f (X ). Assim, existe x X tal que f (x) = f (y). Sendo f injetiva,
∈ ∈ ∈
conclui-se que y = x X . Portanto, como y f −1 (f (X )) é arbitrário, temos que f −1 (f (X )) X .
∈ ∈ ⊂
Suponhamos, por outro lado, que f seja tal que f −1 (f (X )) = X , para qualquer X ⊂
A. Sejam x e y A ∈
tais que f (x) = f (y). Neste caso, temos que f ( x ) = f ( x, y ). Assim, f −1 (f ( x )) = f −1 (f ( x, y )) e, pela
{} { } {} { }
hipótese,
x = f −1 (f ( x )) = f −1 (f ( x, y )) = x, y .
{} {} { } { }
Desta forma, y ∈{ }
x e, consequentemente, x = y. Com isso, concluimos que se x e y ∈ A são tais que f (x) = f (y)
então x = y. Portanto, f e´ injetiva.

21
Exercı́cio 1.15:
Dada f : A → B, prove:
(a) Para todo Z ⊂ B, tem-se que f (f − (Z )) ⊂ Z ;
1

(b) f é sobrejetora se, e somente se, f (f −1 (Z )) = Z para todo Z ⊂ B.

(a)

Seja z f (f −1 (Z )). Existe x f −1 (Z ) tal que f (x) = z. Assim, como x


∈ ∈ ∈ f − (Z ), z = f (x) ∈ Z .
1

Portanto, podemos concluir que f (f −1 (Z )) Z .



(b)

Suponhamos que f seja sobrejetora. Provaremos, para um Z B arbitrário, que f (f −1 (Z )) = Z .



Pelo item (a), temos que f (f −1 (Z )) Z .

∈ ∈ ∈
Seja z Z . Como f e´ sobrejetiva, existe x A tal que z = f (x). Desta forma, como f (x) = z Z , segue que
x f −1 (Z ). Logo, z = f (x) f (f −1 (Z )).
∈ ∈
Desta forma, concluimos que f (f −1 (Z )) Z . ⊃
Portanto, devemos ter que f (f −1 (Z )) = Z .
Suponhamos, por outro lado, que f (f −1 (Z )) = Z , para todo Z B. ⊂
∈ {}
Seja z B. Definindo Z = z , temos que

f (f −1 (Z )) = Z = z .
{}
Desta forma, temos que z ∈ f (f − (Z )). Assim, existe x ∈ f − (Z ) ⊂ A tal que f (x) = z.
1 1

Portanto, neste caso, f e´ sobrejetiva.

22
Exercı́cio 1.16:
Dada uma famı́lia de conjuntos (Aλ )λ∈L , seja X um conjunto com as seguintes propriedades:
(1a ) Para todo λ ∈ L, tem-se X ⊃ A ;λ

(2a ) Se Y ⊃A λ , para todo λ ∈ L, então Y ⊃ X.


Prove que, nestas condições, tem-se X =


λ L
Aλ .

Pela primeira condição, temos que X ⊃A λ para cada λ ∈ L. Assim, Aλ ⊂ X pois cada x ∈ Aλ pertence
λ L
∈ λ L

a A λ X , para algum λ L.

O conjunto Aλ e´ tal que∈ Aλ ⊃A λ, para todo λ ∈ L. Logo, pela segunda condi¸cão, Aλ ⊃ X.
λ L
∈ λ L
∈ λ L

Portanto, X = Aλ .
λ L

23
Exercı́cio 1.17:
Enuncie e demonstre um resultado análogo ao anterior, caracterizando

λ L

Aλ .

Enunciado: Dada uma famı́lia de conjuntos (Aλ )λ∈L , seja X um conjunto com as seguintes propriedades:
a
(1 ) Para todo λ ∈ L, tem-se X ⊂ A ; λ

(2a ) Se Y ⊂A λ para todo λ ∈ L, então Y ⊂ X .


Nestas condições, tem-se X =
λ L


Aλ .

Demonstração:

Todo elemento x de X pertence a


λ L


∈ X ⊂ A , pela primeira hip´otese sobre X . Logo,
Aλ pois x λ

λ L

Aλ ⊃ X.
O conjunto

λ L

Aλ e´ tal que

λ L

Aλ ⊂ A , para todo λ ∈ L. Assim, pela segunda hip ótese sobre X ,
λ

λ L

Aλ ⊂ X.
Portanto, X =

λ L

Aλ .

24
Exercı́cio 1.18:
P −→ P
Seja f : (A) ⊂ ⇒ ⊂
(A) uma função tal que X Y = f (Y ) f (X ) e f (f (X )) = X. Prove que f ( Xλ ) = ∪ ∩f (X ) λ
∩ ∪
e f ( Xλ ) = f (Xλ ).[Aqui X, Y e cada X λ são subconjuntos de A].

Façamos cada inclusão separadamente.

(i) f (
∪ ⊂∩
Xλ ) f (Xλ )

Como Xλ ∪ ⊃X λ, para todo λ, temos por hip´otese que f ( Xλ )∪ ⊂ f (X ), para todo λ. Da´ı, f (∪X ) ⊂ ∩f (X ).
λ λ λ

(ii) f ( Xλ ) f (Xλ )
∪ ⊃∩ ∩
Por (ii), temos que f ( f (Xλ )) ⊃ ∪f (f (X )) = ∪X . Da´ı, f (f (∩f (X ))) ⊂ f (∪X ). Logo, ∩f (X ) ⊂ f (∪X ).
λ λ λ λ λ λ

(iii) f (
∩ ⊃∪
Xλ ) f (Xλ )

Como Xλ ∩ ⊂ X λ , para todo λ, temos por hip´otese que f ( Xλ ) ∩ ⊃ f (X ), para todo λ. Da´ı, f (∩X ) ⊃ ∪f (X ).
λ λ λ

(iv) f (
∩ ⊂∪
Xλ ) f (Xλ )


Por (i), temos que f ( f (Xλ ))⊂ ∩f (f (X )) = ∩X . Da´ı, f (f (∪f (X ))) ⊃ f (∩X ). Logo, ∪f (X ) ⊃ f (∩X ).
λ λ λ λ λ λ

De (i) e (ii), temos que f (∪X ) = ∩f (X ) e de ( iii) e (iv), temos f (∩X ) = ∪f (X ).


λ λ λ λ

25
Exercı́cio 1.19:
Dadas as famı́lias (Aλ )λ∈L e (Bµ )µ∈M , forme duas fam´
ılias com ı́ndices em L × M considerando os conjuntos
(Aλ ∪B ) µ (λ,µ) L M
∈ × e (Aλ ∩B m u)(λ,µ) L M .∈ ×
Prove que se tem  
Aλ ∩ Bµ = (Aλ ∩ B ),
µ
λ L ∈ µ M
∈ ∈ ×
(λ,µ) L M
 
Aλ ∪ Bµ = (Aλ ∪ B ).
µ
λ L µ M (λ,µ) L M
∈  ∈  ∈ ×

Primeiramente provemos que


 
Aλ ∩ Bµ = (Aλ ∩ B ).
µ
λ L ∈ ∈
µ M (λ,µ) L M
∈ ×
Como
Aλ ⊃A ⊃A ∩B ,
λ λ µ
λ L ∈
para todo ( λ, µ) ∈ L × M, temos que
Aλ ⊃ (Aλ ∩ B ). µ
λ L
∈ (λ,µ) L M
∈ ×
Analogamente, mostra-se que
Bµ ⊃ (Aλ ∩ B ). µ

µ M (λ,µ) L M
∈ ×
Assim, segue que  
Aλ ∩ ⊃ Bµ (Aλ ∩ B ).
µ
λ L ∈ µ M
∈ (λ,µ) L M
∈ ×
Seja x ∈ ( ∪ ∈ A ) ∩ (∪ ∈ B ).
λ L λ µ M µ Desta forma, x ∈∪∈ λ L Aλ ex ∈∪ ∈ µ M Bµ . Assim, existem λ ∈L e µ ∈M
tais que x ∈ A e x ∈ B . Logo,
λ µ
x ∈ A ∩B ⊂
λ µ (Aλ ∩ B ). µ
(λ,µ) L M
∈ ×
Com isso, podemos concluir que
 
Aλ ∩ Bµ ⊂ (Aλ ∩ B ).
µ
λ L ∈ µ M
∈ ∈ ×
(λ,µ) L M

Mostremos agora que  


Aλ ∪ Bµ = (Aλ ∪ B ).
µ
λ L µ M (λ,µ) L M

Como

(Aλ
∪ ∈
Bµ )

⊃ ⊃ Aλ
∈ ×
Aλ ,
λ L∈
para todo ( λ, µ) ∈ L × M , temos que
(Aλ ∪B ) ⊃ µ Aλ .
(λ,µ) L M∈ × λ L ∈

26
Analogamente, mostra-se que
(Aλ ∪B ) ⊃
µ Bµ .
(λ,µ) L M
∈ × ∈
µ M

Assim, segue que  


(Aλ ∪B )⊃
µ Aλ ∪ Bµ .
(λ,µ) L M
∈ × λ L ∈ ∈
µ M

∈∩
Seja x ∪ ∈∩ ∪∩ ∈∩
(λ,µ)∈L×M (Aλ Bµ ). Suponhamos, por absurdo, que x / ( λ∈L Aλ ) ( µ∈M Bµ ). Então, x / λ∈L Aλ
∈∩ ∈ ∈ ∈ ∈ ∈ ×
e x / µ∈M Bµ . Assim, existem λ L e µ M tais que x / A λ e x / B µ . Com igual raz˜ao, existe (λ, µ) L M tal
∈ ∪ ∪ ⊂∩
que x / A λ Bµ . Um absurdo, pois como Aλ Bµ (λ,µ)∈L×M (Aλ Bµ ), ter´ ∪ ∈∩ ∪
ıamos que x / (λ,µ)∈L×M (Aλ Bµ ).
∈∩ ∪∩
Logo, devemos ter que x ( λ∈L Aλ ) ( µ∈M Bµ ). Com isso, concluimos que

(Aλ ∪B )⊂
µ Aλ ∪
  Bµ .
(λ,µ) L M
∈ × λ L ∈ ∈
µ M

27
Exercı́cio 1.20:
Seja (Aij )(i,j)∈N×N uma famı́lia de subconjuntos com ı́ndices em N N. Prove, ou disprove por contra-exemplo, a
igualdade
∞ ∞ ∞
 × 

Aij = Aij .
j=1 i=1 i=1 j=1

A igualdade é falsa em geral. De fato, tomando-se

Aij :=
{ 1}, se i = j,
, se i = j,

temos que ∅ ̸
∞ ∞ ∞
Aij = (∅) = ∅
j=1 i=1 j=1
e  
∞ ∞ ∞
Aij = {} {}
( 1 )= 1 .
i=1 j=1 i=1

28
Exercı́cio 1.21:
Dados os conjuntos A , B,C, estabeleça uma bije¸cão entre F (A × B; C ) e F (A; F (B; C )).
Seja f : A B× → C. Podemos definir uma fun¸cão ϕf : A → F (B; C ) definindo ϕ (a) : B → C como sendo a
f
função dada por ( 
ϕf (a) (b) := f (a, b),
para todo b ∈ B. Verificaremos que a fun¸cão ϕ : F (A × B; C ) → F (A; F (B; C )), dada por
ϕ(f ) := ϕ f ,

paraSuponhamos
cada f ∈F ×
(A
que fB;
e gC ), é (A
uma B;
∈F × bijeção.
C ) sejam tais que ϕ(f ) = ϕ(g). Assim, ϕ f = ϕ g . Logo, dado ( a, b) ∈ A × B,
temos que
ϕf (a) = ϕg (a)
e, consequentemente, (  ( 
f (a, b) = ϕf (a) (b) = ϕg (b) (b) = g(a, b).
Portanto, f = g. Com isso, concluimos que ϕ e´ injetiva.
→F C por
Seja ψ : A
( × →
(B; C ). Podemos definir uma fun¸cão f : A

f (a, b) := ψ(a) (b),


B

para todo ( a, b) ∈ A × B. Seja a ∈ A. Temos que


(  ( 
ϕf (a) (b) = f (a, b) = ψ(a) (b),


para todo b B . Desta forma ϕf (a) = ψ (a). Portanto, como a e´ arbitrário, conclu´
ımos que ϕf = ψ. Com isso,
concluimos que ϕ e´ sobrejetiva.
F ×
Portanto, ϕ : (A B; C ) →F F
(A; (B; C )) é uma bijeção como quer´
ıamos demonstrar.

29
Capı́tulo 2

Conjuntos Finitos, Enumeráveis e


Não-Enumeráveis

30
Exercı́cio 2.1:
Prove que, na presença dos axiomas P1 e P2, o axioma A abaixo é equivalente a P3:
A : Para todo subconjunto n˜ao-vazio X ⊂ N, tem-se X \s(X )̸= ∅.

Relembremos as propriedades:
P1 : s : N → N é injetora;
P2 : N s(N) = 1 ;
\ {}
⊂ ∈ ∈ ∈
P3 : Se X N e´ tal que 1 X e, para todo n X , s(n) X, então X = N.
Suponhamos que as afirmações P1, P2 e P3 sejam válidos. Concluiremos que o axioma A é valido mostrando que
se X N e´ tal que X s(X ) = então X = . Equivalentemente, se X s(X ) então N X = N . Primeiramente,
⊂ \ ∅ ∅ ⊂ \
temos que 1 N X , pois, caso contr´ario, 1 s(N) já que
∈ \ ∈
X ⊂ s(X ) ⊂ s(N),
contradizendo P2. Por P1,
s(N X ) = s(N) s(X )
\ ⊃ s(N)\X.
\
Desta forma, se n ∈ N\X então s(n) ∈
/ X e, consequentemente, s(n) ∈ N\X . Assim, por P3, conc luimos que
N\X = N.
Reciprocamente, suponhamos que os axiomas P1, P2 e A sejam v´ alidos. Seja X N tal que 1 X e, para
⊂ ∈
todo n X , s(n) X . Provaremos que X = N e concluiremos da´ı que P3 é válido. Suponhamos por absurdo que
∈ ∈
N X = . Por A, segue que existe
\ ̸ ∅
n ( N X ) s(N X ).
∈ \ \ \
Como 1 / N X , devemos ter que n = 1 e, por P2, existe m
∈ \ ̸ ∈ N tal que
s(m) = n.

Por P1, m / N X já que s(m) = n / s(N X ). Assim, m


∈ \ ∈ \ ∈ X e s(m) = n ∈/ X , contradizendo a hipótese sobre X .

31
Exercı́cio 2.2:
Dados os números naturais a e b, prove que existe um n´umero natural m tal que m a > b. ·
Se a = 1, basta tomar m = b + 1, pois
1(b + 1) = b + 1 > b.
̸
Se a = 1 então a > 1 já que a ∈Z +. Assim, pela monoticidade da multiplica¸ cão em Z + ,

ba > b.

Logo, para m := b, temos que ma > b .

32
Exercı́cio 2.3:
Seja a um número natural. Se um conjunto X e´ tal que a ∈ X e, além disso, n ∈ X ⇒ n + 1 ∈ X , então X contém

todos os números naturais a.

Seja
{ ∈Z
A := k +: a +k ∈ X }.
Pela definição da rela¸cão  em Z+ , b  a se e somente se b = a + k para algum k Z0 . Desta forma, provando

que A = Z+ podemos concluir que X cont´ em todos os números naturais  a.
∈ ∈
Como a X , temos, pela propriedade de X , que a + 1 X . Logo, 1 A. ∈
∈ ∈
Suponhamos que k A. Pela defini¸cão de A, isto implica que a + k X . Assim pela propriedade de X , temos

que a + k + 1 X . Logo, k + 1 A.∈
Portanto, p elo PIF, segue que A = Z+ .

33
Exercı́cio 2.4:
Tente descobrir, independentemente, algumas das demonstrações omitidas no texto.

Associatividade: m + (n + p) = (m + n) + p.

Provada no livro.

Comutatividade: m + n = n + m.

Primeiramente mostraremos que


m + 1 = 1 + m,
para todo m ∈Z +. O caso em que m = 1 é tautológico. Supondo, como hipótese de indu¸cão, que
m+1=1+ m
para algum m ∈Z +, segue que
s(m) + 1 = s(s(m)) = s(m + 1) = s(1 + m) = 1 + s(m).
Assim, pelo PIF, temos que m + 1 = 1 + m, para todo m Z+ . ∈
Por fim, provaremos, para m Z+ arbitrário e por indu¸cão em n
∈ ∈Z +, que
m + n = n + m.
O caso n = 1 foi provado no par´agrafo anterior. Supondo, como hip´otese de indu¸cão, que
m+n= n+m
para algum n ∈Z +, segue que
m + s(n) = s(m + n) = s(n + m)
= n + s(m) = n + (m + 1)
= n + (1 + m) = (n + 1) + m
= s(n) + m.
E o resultado segue pelo PIF.

Lei do Corte : m + n = m + p ⇒ n = p.
Sejam n e p Z+ . Provaremos, por indu¸cão em m Z+ , que se m + n = m + p então n = p.
∈ ∈
O caso em que m = 1 resume-se `a injetividade da fun¸cão s : Z + Z+ . Isto é, como

s(n) = n + 1 = 1 + n = 1 + p = p + 1 = s(p),
temos que
n = p.
Suponhamos, como hipótese de indução, que m + n = m + p implique que n = p. Assim, se s(m) + n = s(m) + p
então
s(m + n) = s(n + m) = n + s(m)
= s(m) + n = s(m) + p
= p + s(m) = s(p + m)
= s(m + p).
Assim, se s(m) + n = s(m) + p temos, novamente pela injetividade de s : Z + →Z +, que m + n = m + p e, pela
hipótese de indu¸cão,
E o resultado seguen pelo
= p. PIF.

Tricotomia: Dados m e n Z + , exatamente uma das três alternativas seguintes podem ocorrer: ou m = n,

ou existe p Z+ tal que m = n + p, ou, então, existe q Z+ com n = m + q .
∈ ∈
Dizemos que ( m, n) ∈Z ×Z
+ + satisfaz a condição C se exatamente uma das exatamente uma das três alterna-
tivas ocorre:

34
• m = n;
• m = n + p, para algum p ∈ Z ; +

• n = m + q, para algum q ∈ Z . +

Seja X o subconjunto de Z × Z definido por


+ +

T := {(m, n) ∈ Z × Z + +: (m, n) satisfaz C . }


Observemos que, como
T = {m} × T m,
m Z+∈
onde
{ ∈Z
Tm := n +: (m, n) satisfaz C , }
mostrando que
T m = Z+ ,
para cada m ∈Z +, podemos concluir que

T = {m} × T m = {m} × Z + = Z+ ×Z +.
m Z+∈ m Z+

Portanto, concluimos a Lei da Tricotomia.
Procederemos com a demonstração de que T m = Z+ por indução em m Z+ . ∈
Consideremos o caso em que m = 1. Se n = 1 temos que n = m. Além disso, como 1 / s(Z), segue que ∈
p
̸
m = 1 = s (n) = n + p

e
n = 1 = s q (m) = m + q,
̸
para todos p e q Z+ . Logo, (1 , 1) satisfaz a condi¸cão C e, consequentemente, 1 T1 . Supondo que n T 1 , como
∈ ∈ ∈
não se pode ter que 1 = m + q = s q (m) já que 1 / s(Z+ ), temos que exatamente uma das duas alternativas ocorre: ∈
• n = 1 e, equivalentemente, s(n) = 1 + 1;
• n = 1 + q e, equivalentemente, s(n) = s(1 + q) = 1 + s(q).
Logo, se n ∈ T então s(n) ∈ T . Com isso, concluimos, pelo PIF, que
1 1 T 1 = Z+ .
Suponhamos, como hipótese de indução, que T m = Z+ . Provaremos que T s(m) = Z+ .
Como X1 = Z+ , temos imediatamente que (1 , s(m)) satisfaz a condi¸cão C e, consequentemente, ( s(m), 1)
satisfaz a condi¸cão C. Logo, 1 ∈ ∈
T s(m) . Supondo que n T s(m) , temos que exatamente uma das três alternativas
ocorrem:
• n = s(m): Neste caso, s(n) = s(s(m)) = s(m) + 1;
• n = s(m) + q, para algum q ∈ Z : Neste caso, s(n) = s(s(m) + q) = s(m) + s(q);
+

• s(m) = n + p, para algum p ∈ Z : Neste caso, se p = 1 então s(m) = s(n). E, se p ∈ Z \{1} = s(Z
+ + + ), existe
p˜Z+ tal que p = s(˜
p), e assim

s(m) = n + p = n + s(˜
p) = n + (1 + ˜p) = (n + 1) + ˜p = s(n) + p.
˜

Assim, se n ∈T s(m) , temos que exatamente uma das três alternativas ocorrem:
• s(n) = s(m) (no caso em que n = s(m));
• s(n) = s(m) + q̃ (no caso em que n = s(m) ou n = s(m) + q, onde q̃ = s(q));
• s(m) = s(n) + p˜ (no caso em que s(m) = n + p, onde p = s(˜p)).

35
Logo, se n Ts(m) então s(n) Ts(m) . Com isso, concluimos, pelo PIF, que T s(m) = Z+ .
∈ ∈
Portanto, X m = Z+ , para todo m Z+ . ∈
Transitividade: se m < n e n < p então m < p.

Se, para m, n e p ∈Z +, tivermos que m < n e n < p então existem r e s ∈Z + tais que

m+r= n

e
n + s = p.
Desta forma,
p = n + s = (m + r) + s = m + (r + s).
Logo,
m < p.

Tricotomia: dados m e n ∈Z + exatamente uma das alternativas seguintes pode ocorrer: ou m = n, ou m < n
ou n < m.

Sejam m e n Z+ . Segundo a tricotomia da adi¸cão (provada acima), exatamente uma das três condições é

válida: ou m = n; ou existe p Z + tal que m = n + p e, portanto, m > n; ou existe q Z + tal que n = m + q e,
∈ ∈
portanto, m < n.

Monoticidade da adi¸cão: se m < n então, para todo p ∈Z + , tem-se m + p < n + p.

Provada no livro.

· ·
Associatividade: m (n p) = (m n) p. · ·
Provada no livro.

Comutatividade: m n = n m. · ·
Primeiramente, provaremos que m 1 = 1 m, para todo m Z+ . Depois, supondo, como hip´otese de indu¸cão,
· · ∈
que n Z + e´ tal que m n = n m, para todo m Z + , provaremos que n + 1 é tal que m (n + 1) = ( n + 1) m.
∈ · · ∈ · ·
Como isso, o resultado segue pelo Princı́pio da Indução Finita.
Provaremos a igualdade m 1 = 1 m por indução em m Z+ . Para m = 1 a igualdade é trivial. Suponhamos,
· · ∈
como hipótese de indu¸cão, que m 1 = 1 m, para algum m Z+ . Desta forma, temos que
· · ∈
·
(m + 1) 1 = m + 1 = m 1 + 1 = 1 m + 1 = 1 (m + 1). · · ·
Logo, pelo PIF, a igualdade é válida.
Suponhamos que n Z + seja tal que m n = n m, para todo m Z + . Mostraremos, por indu¸cão em m, que
∈ · · ∈
m (n + 1) = ( n + 1) m, para todo m Z+ . Para m = 1, o resultado segue do par´agrafo anterior. E, supondo que
· · ∈
· ·
m (n + 1) = ( n + 1) m, temos que

·
(m + 1) (n +1) = ( m + 1) n + (m + 1) ·
= n (m + 1) + ( m + 1) ·
= n m+n+m+1
=
=
···
m n+m+n+1
m (n + 1) + ( n + 1)
= ·
(n + 1) m + (n + 1)
= ·
(n + 1) (m + 1).

E temos o resultado.

Distributividade: m(n + p) = m n + m p. · ·
36
Provada no livro.

·
· ⇒ m = n.
Lei do Corte : m p = n p

Suponhamos que m, n e p ∈ Z são tais que


+

·
m p = n p. ·
Pela tricotomia, exatamente uma das três condições é satisfeita: ou m = n + q , para algum q Z+ ; ou m = n + q , ∈
m = n + q , para algum q Z + ; ou m = n. Provaremos que as duas primeiras condi¸cões não são poss´ıveis e, com

isso, teremos o resultado.
Suponhamos que m = n + q , para algum q Z+ . Segue que

· · · · ·
n p = m p = (n + q ) p = p (n + q ) = p n + p q = n p + p q. · · ·
Contradizendo a tricotomia.
De forma an´aloga, não podemos ter n = m + q , para algum q ∈Z +.

Monoticidade: m < n ⇒ m · p < n · p.


Sejam n e m ∈Z + tais que
m < n.
Provaremos que
·
m p < n p, ·
para todo p Z+ , por indu¸cão em p.

Para p = 1, a desigualdade é imediata.
·
Suponhamos, como hipótese de indu¸cão, que m p < n p, para um certo p· ∈Z +. Como m < n, existe q ∈Z
+
tal que
n = m + q.

Assim,
· · · · ·
n (p + 1) = ( m + q ) (p + 1) = ( p + 1) (m + q ) = (p + 1) m + (p + 1) q = m (p + 1) + ( p + 1) q. · ·
e, consequentemente,
·
n (p + 1) < m (p + 1). ·
E o resultado segue, como querı́amos, pelo PIF.

37
Exercı́cio 2.5:
Um elemento a Z+ chama-se antecessor de b Z quando se tem a < b mas não existe c
∈ ∈ ∈Z + tal que a < c < b .
Prove que, exceto 1, todo n´umero natural possui um antecessor.

Seja x Z+ 1 . Mostraremos que x possui um antecesor.


∈ \{ }
Pelo axioma de Peano P2, x = s(y) = y + 1 para algum y Z+ . Logo, y < x.

Suponhamos que z Z+ e´ tal que z < x. Mostraremos que z  y. Temos que

x = z + n,

para algum n Z+ . Se n = 1 temos que


∈ y+1 =x = z +1
e, consequentemente, pela Lei do Corte, y = z. Se n Z+ 1 então, novamente pelo axioma de Peano P2, existe
∈ \{ }
m Z+ tal que n = s(m). Assim,

s(y) = x = z + n = z + s(m) = s(z + m)

e, pela injetividade da fun¸cão s (axioma de Peano P1),

y = z + m.

Logo, z < y.
Portanto, y é um antecessor de x.

38
Exercı́cio 2.6:
Use indução para demonstrar os seguintes fatos:
a) 2(1 + 2 + 3 + ... + n) = n(n + 1);
b) 1 + 3 + 5 + ... + (2n + 1) = ( n + 1)2 ;
c) (a− 1)(1 + a + a 2
+ ... + an ) = a n+1 − 1, seja quais forem a, n ∈ N;
n
d) n ≥ 4 ⇒ n! > 2 .

Façamos as demonstrações de maneira bem resumida.


a) Para n = 1, temos obviamente a igualdade uma vez que
2(1) = 1(1 + 1) .

Suponhamos que a igualdade seja verdadeira para n = k, ou seja,


2(1 + 2 + 3 + ... + k) = k(k + 1)
e provemos a sua validade para n = k + 1.
Temos pela hipótese de indu¸cão que
2(1 + 2 + 3 + ... + k + (k + 1)) = 2(1 + 2 + 3 + ... + k) + 2(k + 1)
= k(k + 1) + 2(k + 1)
= (k + 1)(k + 2) = ( k + 1)((k + 1) + 1).

b) Para n = 1, temos a igualdade verificada obviamente pois


1 + 3 = 4 = (1 + 1) 2 .

Suponhamos que a igualdade seja verificada para n = k, ou seja,


1 + 3 + 5 + ... + (2k + 1) = ( k + 1) 2 .

Assim, temos que


1 + 3 + 5 + ... + (2(k + 1) + 1) = 1 + 3 + 5 + ... + (2k + 1)(2k + 3)
= (k + 1) 2 + 2k + 3
= k2 + 2k + 1 + 2 k + 3
= k2 + 4k + 4
= (k + 2) 2 = ((k + 1) + 1)2
2
c) Para n = 1, temos a igualdade verificada obviamente, pois (a − 1)(1 + a) = a − 1.
Suponhamos que a igualdade seja verdadeira para n = k, ou seja, ( a − 1)(1 + a + a 2
+ ... + ak ) = a k+1 − 1.
Assim, temos que
(a − 1)(1 + a + a 2
+ ... + ak + ak+1 ) = (a 1)(1 + a + a2 + ... + ak ) + (a
− − 1)(a k+1
)
= ak+1 1 + ak+2 ak+1
= ak+2 1. − −
d) Para n = 4, temos a igualdade verificada. Suponhamos que a igualdade seja verificad a para n = k, ou seja,
k! > 2 k .
Assim, temos que
(k + 1)! > (k + 1)k! > (k + 1)2k > 2 k+1

39
Exercı́cio 2.7:
Use o Segundo Princı́pio de Indução para demonstrar a unicidade de decomposi¸ cão de um n´umero natural em
fatores primos.

Seja n N. Suponha que todos os n´ umeros naturais menores do que n sejam escritos de forma ´unica como

produto de fatores primos. Suponhamos que n tenha duas decomposições

n = α1 α2 ...αm

e
n = β 1 β2 ...βp ,
com α i e β j números primos. Se α i = β j para determinados i,j, (neste caso podemos supor sem perda de generali-
dade α 1 = β 1 ) ent ao temos que
n = αi r
e
n = β 1 s,
com r = α 2 ...αm , s = β 2 ...βp e r = s < n. Pelo segundo princ´ ıpio de indução, temos que m = p e αi = β i , para
i = 2, 3,...,m.
̸
Se α i = β j para qualquer i = 1, 2,...,m e j = 1, 2,...,p, então temos que

mdc(n, n) = mdc(α1 ...αm , β1 ...βp ) = 1,

o que é um absurdo. Logo ocorre o primeiro caso e segue o resultado.

40
Exercı́cio 2.8:
Seja X um conjunto com n elementos. Use indu ¸cão para provar que o conjunto das bije¸cões (ou permuta¸cões)
f :X → X tem n! elementos.

Provemos este exerc´ | |


ıcio usando indução sobre o n´umero de elementos de X. Para X = 1, temos obvia-
| | | | | |
mente F = 1. Suponhamos que se X = k, então F = k!. Suponhamos que X = k + 1, digamos X = | |
{ }
x1 , x2 ,...,x n , xn+1 . Para cada i = 1, 2,...,k + 1, seja f i : X
 → X tal que f i (xk+1 ) = x i . Temos então k + 1 f i′ s.

Agora, por indu¸cão existem k! restrições f i X \{xk+1 } , pois cada restri¸cão f i X \{xk+1 } : X\{xk+1 } → \{
X xk+1 é}
| |
uma bijeção. Portanto F = (k + 1)k! = (k + 1)!, o que conclui a demonstra¸cão.

41
Exercı́cio 2.9:
Sejam X e Y conjuntos finitos.
a) Prove que card(X ∪ Y ) + card(X ∩ Y ) = card(X ) + card(Y ).
b) Qual seria a f´ormula correspondente para três conjuntos?
c) Generalize.

{
a) Sejam A = (1, x); x ∈ X } ∪ {(2, y); y ∈ Y } e B = {(3, z); z ∈ X ∪ Y } ∪ {(4, w); w ∈ X ∩ Y }. Definamos
f :A→ B como sendo f (1, x) = (3 , x)
(3, y); se y ∈ Y \ X .
f (2, y) =
(4, y); se y ∈ X ∩ Y

Temos trivialmente que f e´ uma bijeção entre A e B . Além disso, card(A) = card(X ∪ Y ) + card(X ∩ Y ) e
card(B) = card(X ) + card(Y ). Da´
ı segue o resultado.
b) card(X ∪ Y ∪ Z ) + card((X ∩ Y ) ∪ (X ∩ Z ) ∪ (Y ∩ Z )) = card(X ) + card(Y ) + card(Z ).
c) card
∪ n

i=1
Xi + card

̸
i =j
(Xi ∩X )
j = card(X1 ) + card(X2 ) + ... + card(Xn ).

42
Exercı́cio 2.10:
Dado um conjunto finito X, prove que uma função f : X → X é injetora se, e somente se, é sobrejetora.

( ) Temos que g : X → ̸
f (X ) dada por g (x) = f (x) é uma bijeção. Se f (X ) = X ter´
ıamos um absurdo pois
não pode haver bije¸cão entre um conjunto finito e um subconjunto pr´ oprio deste conjunto.
⇐ { } ̸
( ) Seja X = x1 , x2 ,...,x n . Suponha que f não seja injetora, ou seja, existem xi = xj em X tais que
{ } − ̸
f (x1 ) = f (x2 ). Assim, f (X ) = f (x1 ), f (x2 ),...,f (xn ) teria no m´aximo n 1 elementos e desta forma f (X ) = X,
o que é um absurdo. Logo, f é injetora.

43
Exercı́cio 2.11:
Formule matematicamente e demonstre o seguinte fato(conhecido como princı́pio das gavetas). Se m < n, então
de qualquer modo como se guardem n objetos em m gavetas, haverá sempre uma gaveta, pelo menos, que conter´a
mais de um objeto.

f : In →I m com n > m não é injetiva.

| |
Se f não é sobrejetora, f In também não será. Logo, f In também não será injetiva pelo exercı́cio anterior. E
consequentemente f também não seria injetiva.
Por outro lado, mesmo que f fosse sobrejetiva, se fosse também injetiva, f seria uma bijeção entre um conjunto
finito e um subconjunto pr´oprio dele, que é um absurdo.

44
Exercı́cio 2.12:
Seja X um conjunto com n elementos. Determine o n´umero de funções injetivas f : I p → X.
Princı́pio da contagem. Escolhamos um dos n elementos de X para ser f (1). Daı́ escolhamos 1 dos n 1 −
elementos restantes para ser f (2). E assim sucessivamente temos que o n´umero de fun¸cões injetivas poss´ıveis é

n(n − 1)(n − 2)...(n − p + 1).

45
Exercı́cio 2.13:
Quantos subconjuntos com p elementos possui um subconjunto X, sabendo-se que X tem n elementos?

Se n < p, vem de P1 que n˜ao existe subconjunto de X com p elementos. Caso contrário podemos definir uma
função f : [1, p] N
∩ → X (pelo axioma da escolha). Pelo princı́pio da contagem, temos que f pode ser definida de
n!
modos distintos. Porém, para cada imagem de f, f pode ter sido definida de p! formas. Sendo assim,

p!(n p)!
n!
existem imagens de f .

p!(n p)!

46
Exercı́cio 2.14:
Prove que se A tem n elementos, então P (A) tem 2 n elementos.


Associemos a cada X P (A) uma fun¸cão f X : A →{ } ∈
0, 1 dada por f (x) = 1 se x X e f (x) = 0 se x / X.∈
Temos então que a aplica¸cão X → fX e´ uma bijeção. E como, pelo princ´ıpio da contagem, é poss´ıvel se fazer 2
→{ }
funções f : A 0, 1 diferentes, temos que a ordem de P (A) é exatamente 2.

47
Exercı́cio 2.15:
Defina um fun¸cão sobrejetiva f : N → N tal que, para todo n ∈ N, o conjunto f − (n) seja infinito.
1

n m
Seja f : N → N tal que f (2 3 ) = n e f (x) = 1 para x divis´
ıvel por qualquer primo diferente de 2 e 3. Portanto,
n n 2 n m
f −1 (N) ⊃ {2 3, 2 3 ,..., 2 }
3 ,... .

48
Exercı́cio 2.16:
Prove que se X é infinito enumerável, o conjunto das partes finitas de X também é (infinito) enumerável.

{ }
Seja X = x1 , x2 ,... . Temos que
∞ ∞
P = {A ⊂ {x , x ,...,x }} =
1 2 i Fi .
i=1 i=1

Temos que cardF i = 2i . Como P é uma reunião enumerável de conjuntos enumeráveis, P é enumerável.

49
Exercı́cio 2.17:
Seja f : X →
X uma função. Um subconjunto Y ⊂
X chama-se estável relativamente à f quando f (Y ) Y. Prove ⊂
que um subconjunto X é finito se, e somente existe um função f : X →
X que só admite os subconjuntos est´aveis
∅e X.

⇒ { }
( ) Seja X = x1 , x2 ,...,x n e f : X → ≤
X dado por f (xi ) = x i+1 se 1 i < n e f (xn ) = x 1 . Se f e´ estável em
A e x p A, temos que x q = f q−p(modn) (xp ) A. Logo, A = X.
∈ ∈
( ) Dado x0 X (se X = , X e´ finito) consideremos o conjunto A = x0 , f (x0 ), f 2 (x0 ),...,f n (x0 ),... . Da´ı
⇐ ∈ ̸ ∅ { }
X = A, pois f e´ estável em A e A = .̸∅
Se não existir k N tal que f k (x0 ) = x 0 , A
∈ −{ }
x0 e´ estável por f e logo A −{ }
x0 = X −{ } ∅
x0 = , ou seja,
X = x0 , ou A = X = A x0 , absurdo.
{ } − { ∈} { }
Por outro lado, se existir k N tal que f k (x0 ) = x 0 o conjunto x0 , f (x0 ), f 2 (x0 ),...,f k−1 (x0 ) e´ estável por A
e não vazio, logo é igual a X.

50
Exercı́cio 2.18:
Seja f : X → ̸
X uma função injetiva tal que f (X ) = X. Tomando x ∈ X − f (X ), prove que os elementos
x, f (x), f (f (x)),... são dois a dois distintos.

Provaremos por indu¸cão em n que para todo p ∈ N, temos que f n (x) = f n+p (x) e a proposi¸ c ao estar´a
̸
demonstrada. Com x / f (X ), temos que x = f p (x), para todo p N . Suponhamos que f n (x) = f n+p (x). Então
∈ ̸ ∈ ̸
f n+1 (x) = f n+1+p (x) pois f é injetora. Pelo PIF o resultado segue.
̸

51
Exercı́cio 2.19:
Dado um conjunto finito X, prove que uma função f : X → X é injetora se, e somente se, é sobrejetora.

( ) Temos que g : X → ̸
f (X ) dada por g (x) = f (x) é uma bijeção. Se f (X ) = X ter´
ıamos um absurdo pois
não pode haver bije¸cão entre um conjunto finito e um subconjunto pr´ oprio deste conjunto.
⇐ { } ̸
( ) Seja X = x1 , x2 ,...,x n . Suponha que f não seja injetora, ou seja, existem xi = xj em X tais que
{ } − ̸
f (x1 ) = f (x2 ). Assim, f (X ) = f (x1 ), f (x2 ),...,f (xn ) teria no m´aximo n 1 elementos e desta forma f (X ) = X,
o que é um absurdo. Logo, f é injetora.

52
Exercı́cio 2.20:
(a) Se X e´ finito e Y e´ enumerável, então F (X, Y ) é enumerável.
(b) Para cada fun¸cão f : N N seja Af
→ = {n ∈ N ; f (n)̸= 1}. Prove que o conjunto X das funções f : N → N
tais que A f é finito é um conjunto enumerável.

Item (a)
{
Seja X = x1 ,...,x n }. Definimos
F (X, Y ) → Y n
φ:
f → (f (x ),...,f (x )). 1 n
n
Temos que φ é claramente injetiva. Logo, F (X, Y ) está em bijeção com o conjunto φ(F (X, Y )) ⊂ Y . Como
n n
Y e´ enumerável, Y é enumerável (pois é produto finito de conjuntos enumeráveis). Assim, φ(F (X, Y )) ⊂ Y é
anumerável e, consequentemente, F (X, Y ) é enumerável.
Item (b)
Seja
Fn := Y { ⊂ N;card Y = n}.
Definimos
φ: F → n Yn
{
Y = y1 ,...,y n} → (y1 ,...,y n ).
Claramente, φ e´ injetiva. Como Y n é enumerável, segue que F n e´ enumerável. Portanto,

F := Fn
n=1

é enumerável.
Seja
ψ: X
f → f | ∈. F (Y, N)
∪ Y F
Af

Temos que ψ é injetiva. De fato, se f , g ∈ X são tais que ψ(f ) = ψ(g) temos que
f | = g| Af Ag

implicando que A f = A g ,
f | Af =g | Ag ,

e
f =g
já que
f |\
N Af = 1= g |\
N Af .

Pelo item anterior, Y ∈F (Y, N) é uma união enumerável de conjuntos enumeráveis. Logo,
F ∪ Y ∈F F (Y, N) é
enumerável. Assim, como ψ e´ injetiva, segue que X e´ enumerável.

53
Exercı́cio 2.21:
Obtenha uma decomposição N = ∪∞i=1 Xi tal que os conjuntos X i são infinitos e dois `a dois disjuntos.

Para todo n ∈ N, existe um ´unico k ∈ Z 0 tal que

2k  n < 2 k+1 .

Por isso, fica bem definida a fun¸cão f : N →Z 0 dada por


k
f (n) = n −2 ,
k k+1
onde 2  n< 2 . Desta forma, temos, para
Xi := f −1 (i − 1),
que

N= Xi
i=1

com os conjuntos X i sendo dois `a dois disjuntos. Adiante, como

Xi = 2k + i
{ −1|k ∈Z 0 ,i k
− 1 < 2 },
temos que cada X i e´ infinito.

54
Exercı́cio 2.22:
m
Defina f : N × N → N, pondo f (1, n) = 2n − 1 e f (m + 1, n) = 2 (2n − 1). Prove que f é uma bijeção.
Para cada número natural p, temos, pela unicidade da decomposi¸ cão de n´umeros naturais em n´umeros primos,
que existem únicos m e q Z + tais que p = 2m−1 q e q e´ ı́mpar. Sendo q ı´mpar, existe um único n Z + tal que
∈ ∈
q = 2n 1. Assim, existem ´unicos m e n Z0 tais que p = 2m−1 (2n 1). Portanto, é bem definida a função
− ∈ −
g: Z → + Z+ ×Z +
p = 2m−1 (2n − 1) → (m, n).

Como g é uma inversa para f , temos que f é bijetiva.

55
Exercı́cio 2.23:
Seja X ⊂ N um subconjunto infinito. Prove que existe uma ´ unica bijeção crescente f : N → X .
Definimos, indutivamente f : N → X por
f (1) = min( X )
e
n 1 −
f (n) = min X − {f (i)} ,
i=1
para n > 1. Temos, pelo PIF e pelo fato de X ⊂ N ser bem ordenado, que f está bem definida.
Dados m < n N, temos que
∈ n 1

f (m) < min X − {f (i)} = f (n)
i=1

pois f (m)  x, para todo x ∈ −


X m−1
i=1
∪ X ⊃ − n−1 ∪
i=1 , e f (m) / X
n−1
∈ − ∪
i=1 . Com isso, concl uimos que f é
estritamente crescente e, consequentemente que f e´ injetiva.
Provaremos, agora que f e´ sobrejetiva. Começaremos mostrando, por indução que

n  f (n).

Para n = 1, temos de X ⊂ N, que


1 = min( N)  min(X ) = f (1).
Usando o passo indutivo, temos que
n  f (n) < f (n + 1)
implicando que
n + 1  f (n + 1).
Logo, vale a desigualdade acima. Adiante, dado x X N, provaremos que x
∈ ∈ f (N). Suponhamos por absurdo que
exista x X f (N). Existe, pela arquimedianidade de N , n N tal que
∈ − ∈
x < n  f (n).

Mas, como
n 1−
x ∈ X − {f (i)},
i=1

ter´
ıamos uma contradição com o fato de que
n 1−
x < min X − {f (i)} .
i=1

Portanto, f e´ sobrejetiva.
Provaremos, agora, que se g : N → X é uma bijeção crescente então f = g. Devemos ter que
g(1) = min( X ) = f (1)

pois, caso contrário, existiria n ∈ N, com n > 1, tal que


g(n) = min( X ) < g(1).

Contradizendo o fato de g ser crescente. Usando o passo indutivo,


g(k) = f (k),

para todo k < n + 1, devemos que ter


n n
g(n + 1) = min X − {g(i)} = min X − {f (i)} = f (n + 1)
i=1 i=1

56
pois, caso contrário, existiria p > n + 1 tal que
n
g(p) = min X − {g(i)} < g (n + 1).
i=1

Contradizendo o fato de g ser crescente. E o resultado segue.

57
Exercı́cio 2.24:
Prove que todo conjunto infinito se decomp˜ oe como reuni˜ao de uma infinidade enumer´avel de conjuntos infinitos,
dois a dois disjuntos.

Seja C um conjunto infinito.


Pelo exercı́cio 2.21, existe uma decomposição

N= Xi
i N

na qual os conjuntos X i são infitos e dois `a dois disjuntos.
Se C e´ enumerável, existe uma bije¸cão f : N → C . Logo,
C= f (Xi )

i N

é uma decomposição na qual os conjuntos f (Xi ) são infitos e dois `a dois disjuntos.
Se C e´ não-enumerável, existe uma aplica¸cão injetiva f : N C tal que C f (N) é infinito. Assim,
→ −
C = (C − f (N)) ∪ f (Xi )
i N ∈
é uma decomposição na qual os conjuntos C − f (N) e f (X ) são infitos e dois `a dois disjuntos.
i

58
Exercı́cio 2.25:
Seja A
um conjunto. Dadas funções f , g : N , defina a soma f + g :
A→ N , o produto f g :
A→ · A → N, e dê o
significado da afirmação f  g. Indicando com ξ X a função caracter´
ıstica de um subconjunto X ⊂ A, prove:
a) ξX ∩Y = ξ X ξY ; ·
−ξ
b) ξX ∪Y = ξ X + ξY X Y ∩ . Em particular, ξ X ∪Y = ξX + ξY ⇔ X ∩ Y = ∅;
c) X ⊂ Y ⇔ ξ  ξY ; X

d) ξA− = 1 − ξ .
X X

Definimos
f+g: A → N
a → f (a) + g(a)
e
·
f g: A → N
a → f (a)g(a).
E dizemos que f  g se e somente se
f (a)  g(a)
para todo a ∈A .
a)
Seja a ∈A
.
∈ ∩ ∈ ∈
Se ξX ∩Y (a) = 0, então a / X Y e, consequentemente, a / X ou a / Y . Assim, ξ X ∩Y (a) = 0 implica ξX (a) = 0
ou ξ Y (a) = 0. Logo, ξ X ∩Y (a) = 0 implica ξ X (a)ξY (a) = 0 = ξ X ∩Y (a).
∈ ∩ ∈ ∈
Se ξ X ∩Y (a) = 1, então a X Y e, consequentemente, a X e a Y . Assim, ξ X ∩Y (a) = 1 implica ξ X (a) = 1
e ξ Y (a) = 1. Logo, ξ X ∩Y (a) = 1 implica ξ X (a)ξY (a) = 1 = ξ X ∩Y (a).
Como ξX ∩Y (a) = 0 ou 1, temos que ξX (a)ξY (a) = ξX ∩Y (a) em todos os cas os. E, como a ∈A
e´ arbitrário,
·
temos que ξ X ξY = ξ X ∩Y .
b)
Seja a .
∈A ∈ ∪ ∈ −∈
Se ξ X ∪Y (a) = 0, ent˜ao a / X Y e, consequentemente, a / X e a / Y . Assim, ξ X ∪Y (a) = 0 implica ξ X (a) = 0,
ξY (a) = 0 e ξ X ∩Y (a) = 0. Logo, ξ X ∪Y (a) = 0 implica ξ X (a) + ξY (a) ξX ∩Y (a) = 0 = ξ X ∪Y (a).
∈ ∪ − ∩ − ∪ −
Se ξX ∪Y (a) = 1 e ξX ∩Y (a) = 0, ent˜ao a X Y X Y = (X Y ) (Y X ) e, consequentemente, a X Y ou ∈ −
∈ − ∈ −
a Y X . Se a X Y então ξ X (a) = 1, ξ Y (a) = 0 e, consequentemente, ξ X (a) + ξY (a) = 1. Se a Y X então ∈ −
ξX (a) = 0, ξY (a) = 1 e, consequentemente, ξX (a) + ξY (a) = 1. Logo, ξX ∪Y (a) = 1 e ξX ∩Y (a) = 0 implicam que

ξX (a) + ξY (a) = 1. Portanto, ξ X ∪Y (a) = 1 e ξ X ∩Y (a) = 0 implicam que ξ X (a) + ξY (a) ξX ∩Y (a) = 1 = ξX ∪Y (a).
∈ ∩ ∈ ∈
Se ξX ∪Y (a) = 1 e ξ X ∩Y (a) = 1, ent˜ao a X Y e, consequentemente, a X e a Y . Assim, ξ X ∪Y (a) = 1 e
ξX ∩Y (a) = 1 implicam que ξX (a) = ξ Y (a) = 1. Logo, ξ X ∪Y (a) = 1 e ξ X ∩Y (a) = 1 implicam que ξX (a) +ξY (a) = 2.

Portanto, ξ X ∪Y (a) = 1 e ξ X ∩Y (a) = 1 implicam que ξ X (a) + ξY (a) ξX ∩Y (a) = 1 = ξ X ∪Y (a).

Como ξX ∪Y (a) = 0 ou 1 e ξX ∩Y (a) = 0 ou 1, temos que ξX (a) + ξ Y (a) ξX ∩Y (a) = ξX ∪Y (a) em todos os
casos. E, como a ∈A −
e´ arbitrário, temos que ξ X + ξY ξX ∩Y = ξX ∪Y .
∩ ∅
Em particular, temos que X Y = é equivalente à ξX ∩Y = 0 (i.e. ξX ∩Y é a função nula) que é equivalente
(pelo que foi demonstrado acima) `a ξ X + ξY = ξX ∪Y .
c)
Suponhamos que X Y . Dado a ⊂ ∈A
, temos que ξ X (a) = 0 ou 1. No primeiro caso, temos imediatamente que
∈ ⊂
ξX (a)  ξ Y (a) = 0 ou 1. No segundo, temos que a X Y e, consequentemente, ξX (a) = 1 = ξ Y (a). Em todo
caso, ξ X (a)  ξY (a). Como a ∈A
e´ arbitrário, concluimos que ξ X  ξY .

Suponhamos que ξX  ξY . Dado x X , temos que ξX (x)  ξY (x) e, consequentemente, ξY (x) = 1. Logo,
x X implica que x Y . E temos que X Y .

d) ∈ ⊂
Seja a ∈A
.
Se ξ A−X (a) = 0 temos que a / ∈ A− ∈
X e, consequentemente, a X . Logo, ξ A−X (a) = 0 implica que ξ X (a) = 0.

Assim, ξ A−X (a) = 0 implica que 1( a) ξX (a) = 0 = ξA−X (a).
Se ξ A−X (a) = 1 temos que a ∈A− ∈
X e, consequentemente, a / X . Logo, ξ A−X (a) = 1 implica que ξ X (a) = 0.

Assim, ξ A−X (a) = 1 implica que 1( a) ξX (a) = 1 = ξA−X (a).

Em todos casos, temos que 1( a) ξX (a) = ξ A−X (a). E, como a ∈A −
e´ arbitrário, temos que 1 ξX = ξ A−X .

59
Exercı́cio 2.26:
Prove que o conjunta das sequências crescentes (n1 < n2 < n3 < ...) de n´umeros naturais não é enumerável.

Usaremos o argumento da diagonal de Cantor.


Suponhamos, por absurdo, que exista um enumeração a1 , a2 , a3 ,... das sequências crescentes de números naturais
ai , i N, dadas por

ai1 < ai2 < ai3 < ...
Temos que a sequência b, definida indutivamente por

b1 = a 1 + 1

e
bn+1 = max(bn , an+1 ) + 1
não pertence `a enumeração acima. De fato, temos, pela defini¸cão de b, que

aii < bi

e,consequentemente,
b = ai
̸
para todo i ∈ N.

60
Exercı́cio 2.27:
Sejan ( N, s) e (N′ , s′ ) dois pares formados, cada um, por um conjunto e uma fun¸ cão. Suponhamos que ambos
cumpram os axiomas de Peano. Prove que existe uma única bijeção f : N N′ tal que f (1) = 1 ′ , f (s(n)) = s ′ (f (n)).

Conclua que:
a) m < n ⇔ f (m) < f (n);
b) f (m + n) = f (m) + f (n);
· ·
c) f (m n) = f (m) f (n).

por
Como ( N, s) e (N′ , s′ ) satisfazem os axiomas de Peano, devemos ter que a fun¸ cão f : N → N fica bem definida
f (1) = 1 ′
e
f (s(n)) = s ′ (f (n))
para todo n N. De fato, f está definida em todo N = 1
∈ s(N) pois est´a definida em
{ }∪ {1} e s(N). Como s
é injetiva, segue que f está bem definida. Em particular, segue que f e´ a única função N → N′ satisfazendo as
condições do enunciado. Temos, também, que f é injetiva. De fato, seja

X= n { ∈ N; f (n) ∈/ f (N − {n})}.
Temos que 1 ∈ X pois, pela defini¸cão de f ,
f (N − {1}) = f (s(N)) ⊂ s ′ (N′ ) = N′ − {1′ } = N′ − {f (1)}.

E, se n ∈ X , temos que s(n) ∈ X . De fato, temos que f (s(n)) = ̸ f (1) = 1 ′ pois


f (s(n)) = s ′ (f (n)) ∈ s ′ (N′ ) = N′ − {1′ }

e, se m ̸= n e f (m) ̸= f (s(m)), então temos que

̸ f (m),
f (n) =

pois n ∈ X , e, consequentemente,

f (s(n)) = s ′ (f (n)) ̸= s ′ (f (m)) = f (s(m)).

Implicando, pelo PIF, que f (s(n)) ∈ / f (N − {s(n)} ou, equivalentemente, s(n) ∈ X . Portanto, pelo PIF, temo s
que X = N e, da defini¸cão de X , conclu´ ımos que f e´ injetiva. Temos, também, que f é sobrejetiva. De fato,
1′ = f (1) ∈ f (N) e, se n = f (k) ∈ f (N) temos que

s′ (n) = s ′ (f (k)) = f (s(k)) ∈ f (N).

Assim, pelo PIF, temos que f (N) = N′ . Portanto, f é uma bijeção.


a)
Provaremos primeiro que m < n implica f (m) < f (n). Seja

X= p { ∈ N; f (n + p) > f (n), ∀n ∈ N}.


Temos que 1 ∈ N pois, dado n ∈ N,
f (n + 1) = f (s(n)) = s ′ (f (n)) > f (n).

Supondo que p ∈ X , temos, para qualquer n ∈ N, que


f (n + s(p)) = f (s(n) + p) > f (s(n)) = s ′ (f (n)) > f (n).

61

Logo, p X implica que s(p) ∈ X . Assim, concluı́mos, pelo PIF, X = N. Portanto, se m < n, temos, para p ∈ N
tal que n = m + p, que
f (m) < f (m + p) = f (n).
Agora, provaremos que f (m) < f (n) implica que m < n. Seja

X= p { ∈ N′ ; f (n) = f (m) + p implica m < n }.


Temos que 1′ ∈ X . De fato,
f (n) = f (m) + 1 ′
implica que
f (n) = s ′ (f (m)) = f (s(m))

e, consequentemente, n = s(m) > m.


Supondo que p ∈ X , temos que
f (n) = f (m) + s′ (p)
implica que
f (n) = s ′ (f (m)) + p = f (s(m)) + p
e, consequentemente,
n > s(m) > m.
Logo, p X implica que s ′ (p)
∈ ∈ X . Assim, conclu´ımos que X = N′ . Portanto, se f (m) < f (n), temos, para p ∈ N′
tal que f (n) = f (m) + p, que
m < n.
b)
Seja
X= n { ∈ N; f (n + m) = f (n) + f (m), ∀m ∈ N}.
Provaremos que X = N e teremos, assim, o resultado.
Começamos mostrando que 1 X . Temos que

f (1 + 1) = f (s(1)) = s′ (f (1)) = f (1) + 1′ = f (1) + f (1).

E, se
f (1 + m) = f (1) + f (m),
temos que
f (1 + s(m)) = f (s(s(m))) = s ′ (f (s(m))) = 1 ′ + f (s(m)) = f (1) + f (s(m)).
Logo, pelo PIF, temos que f (1 + m) = f (1) + f (m) para todo m N. Ou seja, 1
∈ ∈ X.

Suponhamos que n X . Provaremos que s(n) X . Temos que∈
f (s(n) + 1) = f (s(s(n))) = s ′ (f (s(n))) = f (s(n)) + 1′ = f (s(n)) + f (1).

Adiante, se
f (s(n) + m) = f (s(n)) + f (m),
temos que
f (s(n) + s(m)) = f (s(s(n) + m))
= s′ (f (s(n) + m))
= f (s(n) + m) + 1 ′
= f (s(n)) + f (m) + 1 ′
= f (s(n)) + f (1) + f (m)
= f (s(n)) + f (1 + m)
= f (s(n)) + f (s(m)).
Assim, concluı́mos, pelo PIF, que f (s(n) + m) = f (s(n)) + f (m) para todo m ∈ N. Logo, n ∈ X implica que
s(n) X .∈
Portanto, conclu´ımos, pelo PIF, que X = N.

62
c)
Seja
X= n { ∈ N; f (n · m) = f (n) · f (m), ∀m ∈ N}.
Provaremos que X = N e teremos, assim, o resultado.

Começamos mostrando que 1 X . De fato, dado m ∈ N, temos que

f (1 · m) = f (m) = 1 · f (m) = f (1) · f (m).

Suponhamos que n ∈ X . Provaremos que s(n) ∈ X . Temos que


f (s(n) · 1) = f (s(n)) = f (s(n)) · 1′ = f (s(n)) · f (1)

Adiante, se · ·
f (s(n) m) = f (s(n)) f (m),
temos que
·
f (s(n) s(m)) = f (s(n)· m + s(n) · 1)
= f (s(n)· m) + f (s(n) · 1) ′
= f (s(n)· f (m) + f (s(n)) ·1
= f (s(n))· (f′ (m) + 1′)
= f (s(n))· s (f (m))
= f (s(n))· f (s(m)).
ımos, pelo PIF, que f (s(n) · m) = f (s(n)) · f (m) para todo m ∈ N. Logo, n ∈ X implica que s(n) ∈ X .
Assim, conclu´
ımos, pelo PIF, que X = N.
Portanto, conclu´

63
Exercı́cio 2.28:
Dada uma sequência de conjuntos A1 , A2 , ...,An , ..., considere os conjuntos lim supAn =
∞ ∞
∩∞ (∪∞
n=1 i=n Ai ) e liminf An =
∪ ∩
n=1 ( i=n Ai ).

a) Prove que lim supAn é o conjunto dos elementos que pertencem à A n para uma infinidade de valores de n e que
liminf An é o conjunto dos elementos que que pertencem a todo An salvo para um n´umero finito de valores
de n.
b) Conclua que lim inf An ⊂ limsup A ;
n

c) Mostre que se A n ⊂A n+1 para todo n então lim infAn = lim supAn = ∪∞ n=1 An ;

d) Por outro lado, se A n An+1 para todo n então liminf An = lim supAn = ∞
n=1 An ;

e) Dê exemplo de uma sequência (A ) tal que li m supA
n = lim infA ; ∩ n n

f) Dê exemplo de uma sequência para a qual os dois limites coincidem mas A ̸⊂ A m n ̸
para todos m = n.

a)
Sejam
X := a { ∈ ∪∞n=1 An |a ∈A n para uma infinidade de valores de n }
e
Y := a { ∈ ∪∞ n=1 An |a ∈A n para todos exceto uma quantidade finita de valores de }
n .
Dado a ∈ X , temos que
{i ∈ N | a ∈ A } i

é ilimitado. Sendo assim, dado n ∈ N, existe i ∈ N tal que n < i e a ∈ A . Isso implica que
i

a ∈ ∪∞ A i=n i

para todo n N. Portanto,


∈ a n=1 ( ∞
∞ ∈∩ ∪
i=n Ai ) = lim supAn .

E, como a é um elemento arbitrário de X , temos que X limsup An . ⊂



Seja a limsup An . Temos que
∞ Ai
a i=n ∈∪
para todo n ∈ N. Segue daı́ que
I := i{ ∈N|a ∈A } i

é infinito, pois, caso contrário, existiria n N tal que n > i para todo i I e, consequentemente, terı́amos que
∈ ∈
a ∞ Ai . Portanto, a X . E, como a é um elemento arbitrário de lim supAn , temos que X limsup An .
̸∈ ∪ ∈ ⊃
i=n

Dado a Y , temos que
I := i N a Ai = N J,
{∈ | ∈ } −
para alum J ⊂
N finito. E, como J é finito e, consequentemente, limitado, existe k ∈ N tal que i  k implica
̸∈ ∈
i J . Logo, para todo i  k, temos que i I e, consequentemente,

a ∈ ∩∞ i=k Ai .

Assim, como ∩∞ ∞ ∞
i=k Ai ⊂ ∪n=1 (∩i=n Ai ), temos que

a ∞ ( ∞ Ai ) = liminf An .
∈∪ n=1 ∩ i=n
Como a é um elemento arbitrário de Y , conclu´ımos da´
ı que Y ⊂ lim infA . n
Seja a liminf An . Temos que existe k N tal que
∈ ∈
a ∈ ∩∞ i=k Ai .

Isso implica que o conjunto


I := i{ ∈N|a ∈A } i

64
é tal que
N−I ⊂ {1,...,k − 1}.

Logo, a Y . Como a é um elemento arbitrário de lim inf An , temos que Y ⊃ lim infA .
n
b)
Pelo ı́tem anterior, temos que

lim supAn := a { ∈ ∪∞ n=1 An |a ∈A n para uma infinidade de valores de n }


e
liminf An := a { ∈ ∪∞n=1 An |a ∈A n para todos exceto uma quantidade finita de valores de }
n .
Assim, segue que
liminf An lim supAn .
c)

Como
∪∞ i=n Ai ⊂ ∪∞ i=1 Ai

para todo n ∈ N, temos, imediatamente que


∩∞ (∪∞ A ) ⊂ ∪∞ A .
limsup An = n=1 i=n i n=1 n

Por outro lado, dado a ∈ ∪∞ A , temos que a ∈ A para algum k ∈ N. Por indu¸cão em p ∈ N, prova-se que
n=1 n k

A ⊂A k k+p

para todo p ∈ N. Portanto,


a ∈ A ⊂ ∩∞
k p=0 Ak+p = ∩∞
⊂ ∪∞ (∩∞ A ) = lim inf A .
i=k Ai n=1 i=n i n


Assim, a lim inf An e, como a é um elemento arbitrário de ∪∞ A , concluimos que ∪∞
n=1 n n=1 An ⊂ lim infA . n
Conclu´ımos, então, do ı́tem b) e do que foi discutido acima, que
∞ An liminf An lim supAn ∞ An .
∪ n=1 ⊂ ⊂ ⊂∪ n=1
Logo, temos que
liminf An = lim supAn = ∪∞n=1 An .

d)
Temos imediatamente que

∩∞ n=1 An = ∩∞ i=1 Ai ⊂ ∪∞ (∩∞


n=1 i=n Ai ) = lim inf An .

Seja a ∈ limsup A . Por indu¸cão em p, temos que


n

Ak ⊂A k+p ,

para todo k e p ∈ N. Assim,


Ak ⊃ ∪∞ p=0 Ak+p = ∪∞ i=k Ai .

E, temos que a ∈ A , para todo k ∈ N, pois


k

a ∈ lim supA n = ∩∞ (∪∞


n=1 i=n Ai ) ⊂ ∪∞ i=k Ai ⊂A . k

Ou Conclu´
seja, a ımos,∞∈∩
n=1 An . Como a é um elemento arbitrário de lim supAn , temos que limsup An
então, do ı́tem b) e do que foi discutido acima, que ⊂ ∩∞ n=1 An .

∩∞ n=1 An ⊂ liminf A ⊂ lim supA ⊂ ∩∞


n n n=1 An .

Logo, temos que


liminf An = lim supAn = ∩∞n=1 An .

e)

65
Definindo
A2k−1 := 1 {}
e
A2k := , ∅
para todo k ∈ N, temos que
lim supAn = ∩∞ (∪∞
n=1 i=n Ai ) = ∩∞ {1} = {1}
n=1

e
liminf An = ∪∞ (∩∞
n=1 i=n Ai ) = ∪∞ ∅ = ∅.
n=1

f)
Definindo
An := n , {}
temos que
An ̸⊂ A m,

̸
para todos n = m,
limsup An = ∩∞ (∪∞
n=1 i=n Ai ) = ∩∞ {k ∈ N | k  n} = ∅
n=1

e
liminf An = ∪∞ (∩∞
n=1 i=n Ai ) = ∪∞ ∅ = ∅.
n=1

66
Exercı́cio 2.29:
(Teorema de Bernstein-Schroeder) Dados conjuntos A e B, suponha que existam fun¸cões injetivas f : A →Be
g:B →
A. Prove que existe uma bije¸cão h : A B. →
Solução 1:


Seja C := g(B) A. Mostraremos que existe uma bijecção H : A C . Assim, como g : B →C é uma bijeção, →
temos que h := g −1 H : A
◦ →
B é uma bijeção.
Como f : A →
Beg:B →
C são funções injetivas, temos que F := g f : A ◦ →
C e´ uma função injetiva. Logo,
− −
F (X Y ) = F (X ) F (Y ), para todos X e Y A. ⊂
Definimos A 1 := A e C 1 := C . E, pelo princı́pio da definiç˜ao recursiva, podemos definir

An := F (An−1 )

e
Cn := F (Cn−1 )
para todo n > 1.
∈ ∈ −
Desta forma, dado x A, temos que x A n Cn , para algum n ∈Z +, ou x ∈ C (pois x ∈/ A − C 1 1 =A −C

implica que x C ). Então, a função H : A C dada por →
H (x) =
∈ −
F (x) se x A n Cn para algum n ∈Z +;
x caso contr´ario,


para todo x A, é bem definida.

Sejam x e y A tais que H (x) = H (y). Se x A n Cn , para algum n ∈ − ∈Z +, então y ∈ A − C , para algum
k k
k Z+ . De fato, se y / Ak Bk , para todo k Z+ , então
∈ ∈ − ∈
y = H (y) = H (x) = F (x) ∈ F (A − B n n) = F (An ) − F (B n) = A n+1 −B n+1

(pois F é injetivo). Uma contradição. Assim, se x An Cn , para algum n Z+ , então


∈ −
F (x) = H (x) = H (y) = F (y)


e, consequentemente, x = y. Além disso, se x / A n Cn , para todo n − ∈Z +, então, como no caso anterior, temos
que y / A n Cn , para todo n Z+ , e, consequentemente,
∈ − ∈
x = H (x) = H (y) = y.

Logo, em todo caso, x = y. Portanto, podemos concluir que H é injetivo.


∈ ∈ − −
Seja x C . Se x A n Cn = F (An−1 ) F (Cn−1 ) = F (An−1 Cn−1 ), para algum n > 1, temos que −
x = F (y) = H (y)

para algum y ∈ A − − C − . Por outro lado, se x ∈/ A − C , para todo n ∈ Z


n 1 n 1 n n +, Temos que

x = h(x).

Logo, podemos concluir que H e´ sobrejetiva.


Portanto, temos que H : A →
C e´ uma bijeção. E o resultado segue.

Solução 2:

A grosso modo analizaremos as quantidades de elmentos das sequências das formas

x, g−1 (x), f −1 g −1 (x), g −1 f −1 g −1 (x),...,


◦ ◦ ◦
x ∈ A, e
y, f −1 (y), g −1 f −1 (y), f −1 g−1 f −1 (y),...,
◦ ◦ ◦
67
y ∈B. Daı́ particionaremos A e B de modo a obter uma fun¸ cão entre A e B cuja bijetividade é herdada da
injetividade de f e g.
Sejam
AI = x A; existe k Z0 tal que ( g f )−k (x) = e g −1 (g f )−k (x) = ,
{ ∈ ∈ ◦ ̸∅ ◦ ◦ ∅}
k
{ ∈ A; existe k ∈ Z tal que g − ◦ (g ◦ f )− (x) ̸= ∅ e (g ◦ f )− (x) = ∅},
AP = x 0
1 (k+1)

k k
A∞ = {x ∈ A; para todo k ∈ Z vale (g ◦ f )− (x)̸= ∅ e g − ◦ (g ◦ f )− (x)̸= ∅},
0
1

B = {y ∈ B; existe k ∈ Z tal que ( f ◦ g)− (y)̸= ∅ e f − ◦ (f ◦ g)− (y) = ∅},


I 0
k 1 k

B = {y ∈ A; existe k ∈ Z tal que f − ◦ (f ◦ g)− (y)̸= ∅ e (f ◦ g)−


P 0
1
(x) = ∅} k (k+1)

e
B = y A; para todo k Z0 vale (f g)−k (y) = e f −1 (f g)−k (y) = .
Segue que
∞ { ∈ ∈ ◦ ̸∅ ◦ ◦ ̸ ∅}
˙ ˙
A = A ∪A ∪A∞
I P

e
B = BI ˙ ˙ ∞.
∪B ∪BP

Agora, provaremos três detalhes técnicos sobre f e g:

(I) f (AI ) = B P

y ∈B P ⇐⇒ ∃k− ∈ Z tal que f − ◦ (f ◦ g)− (y)̸= ∅ e (f ◦ g)− (x) = ∅


1
0
1 k (k+1)

⇐⇒ f = {x} onde k k k
∃k ∈ Z tal que ( g ◦ f )− (x) = f − ◦ (f ◦ g)− (y)̸= ∅ e g − ◦ (g ◦ f )− (x) = (f ◦ g)−
0
1 1 (k+1)
= ∅
⇐⇒ ∃x ∈ A tal que f (x) = y
I
⇐⇒ y ∈ f (A ). I

(II) g(BI ) = AP

Prova-se de modo análogo à prova de (I).

(III) f (A∞ ) = B ∞

k k
y∈ B∞ ⇐⇒ ∀k− ∈ Z vale f − ◦ (f ◦ g)− (y)̸= ∅ e (f ◦ g) (x) ̸= ∅
1
0
1

⇐⇒ f = {x} onde − k k k k
∀k ∈ Z vale (g ◦ f ) (x) = f − ◦ (f ◦ g)− (y)̸= ∅ e g − ◦ (g ◦ f )− (x) = (f ◦ g)− ̸= ∅
0
1 1

⇐⇒ ∃x ∈ A∞ tal que f (x) = y


⇐⇒ y ∈ f (A∞ ).
Agora, segue de A = A ∪˙ A ∪˙ A∞ e de (II), que a fun¸cão
I P

H: A → B
x ∈ A → f (x) ∈ B I P
x ∈A → y ∈ B , tal que g(y) = x
P I
x A f (x) B .
∈ ∞ → ∈ ∞
é bem definida e, da injetividade de f e g e de (I)-(III), que esta é uma bijeção.

68
Capı́tulo 3

Números Reais

69
Exercı́cio 3.01:
Dados a, b, c, d num corpo K , sendo b e d diferentes de zero, prove:
a c ad + bc
(1) + = ;
b d bd

(2)
a c
·
=
a c
.
·
b d b d ·
(1)

Temos que
a = b −1 a = b −1 d−1 da = (bd)−1 (ad) = ad
b bd
e, analogamente,
c bc
= .
d bd
Assim,
a c ad bc
+ = +
b d bd bd
= (bd)−1 (ad) + (bd)−1 (bc)
= (bd)−1 (ad + bc)
ad + bc
= .
bd
(2)

Temos que
a c
· = (b−1 a) (d−1 c) = acb−1 d−1 = (bd)−1 (ac) =
· a c·.
b d b d·

70
Exercı́cio 3.02:

Questão 2-1◦
Propriedade 1. Para todo m inteiro vale
am .a = a m+1.
Demonstraç˜ ao. Para m natural vale pela defini¸cão de potência, agora para m = −n,n > 0 ∈ N um inteiro vamos
provar a −n .a = a −n+1 . Para n = 1 temos
1 1+1 0
a− a = a − = a = 1.
Vamos provar agora para n > 1, n −1>0
a−n = (an )−1 = (an−1 a)−1 = a −n+1 a−1

multiplicando por a de ambos lados a −n .a = a −n+1 como quer´


ıamos demonstrar.
Propriedade 2.
am .an = a m+n .
Demonstraç˜ao. Primeiro seja m um inteiro qualquer e n natural, vamos provar a identidade por indução sobre n,
para n = 0 vale
am .a0 = a m = a m+0
para n = 1 vale
am a1 = a m a = a m+1 .
Supondo válido para n
am .an = a m+n
vamos provar para n + 1
am .an+1 = a m+n+1
temos
am .an+1 = a m an a = a m+n .a = a m+n+1 .
Agora para −n com n natural , se m é natural temos que a propriedade já foi demonstrada
am a−n = a m−n

se m é inteiro negativo temos


am a−n = a m−n
pois o inverso de a a−n é a−m an = a−m+n propriedade que já est´a provada por − m e n serem naturais e
m

am−n an−m = 1 por unicidade do inverso de = a −m an = a−m+n é am a−n logo fica provado para n e m inteiros.
Para potência negativa −n podemos fazer como se segue
am a−n = (a−m )−1 (an )−1 = (a−m an )−1 = (a−m+n )−1 = a m−n .

71
Questão 2-2◦
Propriedade 3.
(am )n = a mn
para m e n inteiros.
ao. Primeiro por indu¸cão para m inteiro e n natural
Demonstraç˜

(am )0 = 1 = a m.0

(am )1 = a m = a m.1 .
Supondo válido para n
m n mn
(a ) = a
vamos provar para n + 1
(am )n+1 = a m(n+1)
temos pela defini¸cão de potência e pela hipótese da indução que

(am )n+1 = (am )n am = a mn am = a mn+m = a m(n+1)

onde usamos a propriedade do produto de potência de mesma base. Para n inteiro negativo

(am )−n = ((am )n )−1 = (amn )(−1) = a −mn .

72
Exercı́cio 3.03:

n

xk xs
Exemplo 1. Se yk = ys para todos k, s ∈I n, num corpo K , prove que dados, a k ∈ K, k ∈ I n tais que
k=1
̸
ak yk = 0
tem-se
∑n

k=1
a k xk
x1
n
∑ = .
y1
a k yk
k=1
x1 xk
Chamando y1 = p temos yk = p logo x k = pyk e a soma
n n
a k xk = p a k yk
k=1 k=1

logo

k=1
n
∑ a k xk
x1
n ∑ =p= .
y1
a k yk
k=1

73
Exercı́cio 3.04:

Definição 1 (Homomorfismo de corpos). Sejam A, B corpos. Uma funç˜ao f : A → B chama-se um homomorfismo


quando se tem
f (x + y) = f (x) + f (y)
f (x.y) = f (x).f (y)
f (1A ) = 1B
para quaisquer x, y ∈ A. Denotaremos nesse caso as unidades 1 A e 1 B pelos mesmos sı́mbolos e escrevemos f (1) = 1.

Propriedade 4. Se f e´ homomorfismo ent˜ao f (0) = 0.


ao. Temos
Demonstraç˜
f (0 + 0) = f (0) + f (0) = f (0)
somando −f (0) a ambos lados segue
f (0) = 0.

Propriedade 5. Vale f ( a) =− −f (a).


ao. Pois
Demonstraç˜
f (a − a) = f (0) = 0 = f (a) + f (−a)

da´ı f ( a) = −f (a).
Corolário 1.
f (a − b) = f (a) + f (−b) = f (a) − f (b).
1 1
Propriedade 6. Se a e´ invert´ ao f (a) é invert´
ıvel ent˜ ıvel e vale f (a ) = f (a) .
− −
Demonstraç˜
ao.
f (a.a−1 ) = f (1) = 1 = f (a).f (a−1 )
então pela unicidade de inverso em corpos segue que f (a)−1 = f (a−1 ).

Propriedade 7. f é injetora.
Demonstraç˜ −
ao. Sejam x, y tais que f (x) = f (y), logo f (x) f (y) = 0, f (x − y) = 0, se x̸= y então x − y seria
invert´ −
ıvel logo f (x y) não seria nulo, então segue que x = y.

Propriedade 8. Se f : A → B com f (x + y) = f (x) + f (y) e f (x.y) = f (x)f (y) para x, y arbitr´arios, ent˜ao

f (x) = 0 x ou f (1) = 1.
Demonstraç˜ ao. f (1) = f (1.1) = f (1)f (1), logo f (1) = f (1)2 por isso f (1) = 1 ou f (1) = 0 . Se f (1) = 0 ent˜ao

f (x.1) = f (x)f (1) = 0, f (x) = 0 x.

74
Exercı́cio 3.05:

Propriedade 9. Se f : Q → Q é um homomorfismo ent˜ao f (x) = x ∀x ∈ Q.


ao. Vale que f (x + y) = f (x) + f (y), tomando x = kh e y = h fixo, tem-se
Demonstraç˜

f ((k + 1)h) − f (kh) = f (h)


aplicamos a soma
∑ n 1
− de ambos lados, a soma é telescópica e resulta em
k=0

f (nh) = nf (h)
p
tomando h = 1 segue que f (n) = n, tomando h = n segue
p p
f (n ) = f (p) = p = nf ( )
n n
⇒ f ( np ) = np .

75
Exercı́cio 3.08:

Propriedade 10. Seja K um conjunto onde valem todos os axiomas de corpo, exceto a existência de inverso
̸
multiplicativo. Seja a = 0. f : K →
K com f (x) = ax e´ bijeç˜
ao a −1 K. ⇔∃ ∈
Demonstraç˜ ⇒
ao. ). A fun¸cão é sobrejetora logo existe x tal que f (x) = 1 = ax portanto a e´ invert´ ıvel com
a−1 = x K. ∈
⇐ ). Dado qualquer y K tomamos x = ya −1 da´
∈ ı f (x) = aa−1 y = y e a fun¸cão é sobrejetiva. f também é
injetiva, pois se f (x1 ) = f (x2 ), ax 1 = ax 2 implica por lei do corte que x 1 = x2 .. Em geral f e´ injetiva ⇔ vale a lei
do corte por essa observa¸cão.
Propriedade 11. Seja K finito. Vale a lei do corte em A ⇔ existe inverso para cada elemento n˜ao nulo de K ,
Demonstraç˜ ⇒
ao. ). Se vale a lei do corte, pela propriedade anterior tem-se que para qualquer a = ̸ 0 em K ,
f :K →K com f (x) = ax é injetiva, como f é injetiva de K em K que é um conjunto finito, então f e´ bijetiva, o
que implica a ser invert´ıvel.
⇐ ). A volta é trivial pois existência de inverso implica lei do corte.

76
Exercı́cio 3.09:

Exemplo 2. O conjunto dos polinˆomios de coeficiente racionais Q[t] n˜ao é um corpo, pois por exemplo o elemento

x n˜ao possui inverso multiplicativo, se houvesse haveria n k
∑∑
k=0 ak x tal que x
n k
k=0 ak x = 1 =
∑ n
k=0 ak x
que n˜ao é possı́vel pois o coeficiente do termo independente x0 é zero em nk=0 ak xk+1 e deveria ser 1.
k+1
o

O conjunto dos inteiros Z n˜ ao é um corpo, pois n˜


ao possui inverso multiplicativo para todo elementos, por
exemplo n˜ao temos o inverso de 2.

77
Exercı́cio 3.10:

2 2
Propriedade 12. Dados x, y ∈ R, x + y = 0 ⇔ x = y = 0.
ao. ⇒ ).Suponha que x ̸
Demonstraç˜ 2 2
= 0, então x > 0 e y ≥ 0 de onde segue que x 2 + y 2 > 0 , absurdo ent˜ao deve
2 2
valer x = 0 ⇒ x = 0 logo temos também y = 0 ⇒ y = 0, portanto x = y = 0.
⇐). Basta substituir x = y = 0 resultando em 0.

78
Exercı́cio 3.11:

Exemplo 3. A fun¸c˜
∏ ∏ ao f : K + → K + com f (x) = x n , n N e´ crescente. Sejam x > y > 0 ent˜
∈ ao xn > yn pois
xn = nk=1 x > nk=1 y = y n , por propriedade de multiplicaç˜ ao de positivos. Se f : Q +→ Q+ , Q + o conjunto dos
ao f n˜ao é sobrejetiva para n = 2, pois n˜ao existe x Q tal que x2 = 2 Q + .
racionais positivos, ent˜ ∈ ∈
f (K + ) n˜
ao é um conjunto limitado superiormente de K , isto é, dado qualquer x K existe y K + tal que
∈ ∈
yn > x. O limitante superior do conjunto, se existisse, n˜ ao poderia ser um n´umero negativou ou zero, pois para
todo y positivo tem-se y n positivo, que é maior que 0 ou qualquer n´umero negativo. Suponha que x positivo seja,
tomando y = x + 1 temos yn = (x + 1)n 1 + nx > x , logo f (K + ) n˜
≥ ao é limitado superiormente.

79
Exercı́cio 3.12:

Propriedade 13. Sejam X um conjunto qualquer e K um corpo, ent˜ ao o conjunto F (X, K ) munido de adi¸c˜ ao
e multiplicaç˜
ao de fun¸c˜
oes é um anel comutativo com unidade, n˜ ao existindo inverso par a todo elemento. Lem-
brando que em um anel comutativo com unidade temos as propriedades, associativa, comutativa, elemento neutro e
existência de inverso aditivo, para adiç˜
ao. valendo também a comutatividade, associatividade, existência de unidade
1 para o produto e distributividade que relaciona as duas operaç˜oes.
Demonstraç˜
ao. • Vale a associatividade da adi¸cão
((f + g) + h)(x) = (f (x) + g(x)) + h(x) = f (x) + (g(x) + h(x)) = ( f + (g + h))(x)
• Existe elemento neutro da adição 0 ∈ K e a fun¸cão constante 0(x) = 0 ∀ x ∈ K , da´ı
(g + 0)(x) = g(x) + 0(x) = g(x).

• Comutatividade da adição
(f + g)(x) = f (x) + g(x) = g(x) + f (x) = (g + f )(x)

• Existe a fun¸cão simétrica, dado g(x), temos f com f (x) = −g(x) e da´ı
(g + f )(x) = g(x) − g(x) = 0.

• Vale a associatividade da multiplicação


(f (x).g(x)).h(x) = f (x).(g(x).h(x))

• Existe elemento neutro da multiplicação 1 ∈ K e a fun¸cão constante I (x) = 1 ∀ x ∈ K , da´ı


(g.I )(x) = g(x).1 = g(x).

• Comutatividade da multiplicação
(f.g)(x) = f (x)g(x) = g(x)f (x) = (g.f )(x)

Por último vale a distributividade (f (g + h))(x) = f (x)(g(x) + h(x)) = f (x)g(x) + f (x)h(x) = (f.g + f.h)(x).
Não temos inverso multiplicativo para toda função, pois dada uma fun¸cão, tal que f (1) = 0 e f (x) = 1 para
̸
todo x = 1 em K , não existe fun¸cão g tal que g(1)f (1) = 1, pois f (1) = 0, assim o produto de f por nenhuma
outra função gera a identidade.

80
Exercı́cio 3.13:

Propriedade 14. Sejam x,y > 0 . x < y ⇔ x− 1


> y −1 .
Demonstraç˜ ao. ⇒ ). Como y > x e x −1 e y −1 são positivos, multiplicamos a desigualdade por x −1 y −1 em ambos
lados x −1 y −1 y > x −1 y −1 x implicando x −1 > y −1 , então se y > x temos x1 > y1 .
⇐). Se x−1 > y −1 . x, y são positivos, multiplicamos a desigualdade por xy em ambos lados, de onde segue que
y > x.

81
Exercı́cio 3.14:

n
Propriedade 15. Sejam a > 0 em K e f : Z → K com f (n) = a oes f é crescente se a > 1,
. Nessas condiç˜
decrescente se a < 1 e constante se a = 1.
Demonstraç˜ao. Para qualquer n Z vale f (n + 1) f (n) = a n+1 an = a n (a 1), an é sempre positivo, então
∈ − − −
− ∀ ∈
o sinal da diferen¸ca depende do sinal de a 1. Se a = 1 vale f (n + 1) = f (n) n Z logo f e´ constante, se
− − −
a 1 < 0, a < 1 então f (n + 1) f (n) < 0, f (n + 1) < f (n), f é decrescente e finalmente se a 1 > 0, a > 1 então
f (n + 1) > f (n) e a fun¸cão é crescente.

Perceba que as propriedades citadas valem para todo n Z , por exemplo no caso de a > 1 temos

· · · < f (−4) < f (−3) < f (−2) < f (−1) < f (0) < f (1) < f (2) < f (3) < · · · < f (n) < f (n + 1) < · · ·
analogamente para os outros casos.

82
Exercı́cio 3.15:

Exemplo 4. Para todo x = 0 real, prove que (1 + x)2n > 1 + 2nx.


̸
− −
Se x > 1 tomamos a desigualdade de bernoulli com 2n no expoente. Se x < 1 vale 1 + x < 0 por´
em elevando
a uma potência par resulta num n´
2n
− −
umero positivo, por outro lado 2nx < 2n logo 1 + 2nx < 1 2n < 0 ent˜
2n
ao
(1 + x) é positivo e 1 + 2nx é negativo, logo nesse caso vale (1 + x) > 1 + 2nx .

83
Exercı́cio 3.16:


Exemplo 5. Se n N e x < 1 ent˜ ao (1 x)n 1
− ≥ − nx, pois de x < 1 segue que −x > −1 e da´ı aplicamos a
desigualdade de Bernoulli (1 + y)n 1 + ny com y =
≥ −x.

84
Exercı́cio 3.17:

Corolário 2. Se a e a + x s˜ao positivos, ent˜


ao vale

(a + x)n ≥a n
+ nan−1 x.
a+x x x
Pois a = (1+ a) ao podemos aplicar a desigualdade de Bernoulli (1 + y)n
> 0 ent˜ ≥ 1 + ny com y = a, resultando
em

(a + x)n ≥a n
+ nan−1 x.

̸
Se a = 0, arbitr´ario em R, podendo agora ser negativo, substituı́mos y = x
a em (1 + x)2n > 1 + 2nx. chegando
na desigualdade
(a + x)2n > a2n + a2n−1 2nx.
x n x
Se vale a < 1 ent˜
ao da desigualdade (1 − y) ≥ 1 − ny, novamente tomamos y= a de onde segue

(a − x) ≥ a − a − nx.
n n n 1

85
Exercı́cio 3.18:

a1
Propriedade 16. Sejam sequências (ak ) , (bk ) em um corpo ordenado K onde cada bk e´ positivo, sendo b1 o
m´ aximo dos termos da sequência de termo abkk ent˜
ınimo e abnn o m´ ao vale

a1
∑ n
ak

b1
≤∑ k=1
n ≤ ab n
n
.
bk
k=1

ao. Para todo k vale


Demonstraç˜
ambos lados, de onde segue
a
b11 ≤ a
bk
k
≤ a
bn
n
⇒b ka
b11 ≤a ≤b k ka
bn
n
pois bk > 0, aplicamos a soma ∑ n
k=1 em
n n n
a1 an
bk
b1
≤ ak ≤ bk
bn
k=1 k=1 k=1

dividindo por
∑ n
k=1 bk que é positivo, temos finalmente

a1
∑ n
ak

b1
≤∑ k=1
n ≤ ab n
n
.
bk
k=1

86
Exercı́cio 3.19:

Propriedade 17 (Multiplicatividade).
|a||b| = |a.b|
para a e b reais quaisquer.
Demonstraç˜ ao. Vale que x.y 2 = (x.y)2 = x2 y 2 e ( x y )2 = x 2 y
| | | || | | | | | 2
= x2 .y 2 os quadrados desses n´umeros são
iguais e eles s˜ao não negativos, então segue que x.y = x y .| | | || |
2. a.b = (a.b)2 =
√a .b = √a .√b
2 2 2 2 = a b.
| |
Propriedade√ 18. Se x ̸ ao | | = | || . || |
= 0 ent˜ 1
x
1
x

1 x 1 1
ao. Vale |x|| | = | | = 1 da´ı | | e´ inverso de |x|, sendo | | .
Demonstraç˜ x x x x

Corolário 3 (Preserva divisão).


| xy | = ||xy|| .

87
Exercı́cio 3.20:

Propriedade 19.
n n
|a | = |
k ak |
k=1 k=1

ao. Por indução, para n = 1 vale, supondo para n números


Demonstraç˜
n n
|a | = |
k ak |
k=1 k=1

vamos provar para n + 1


n+1 n+1
|a | = |
k ak |
k=1 k=1

temos
n+1 n n n n+1
|a | =
k |a |.|a | = |
k n+1 ||
ak an+1 = | | | |
ak an+1 = ak | .
k=1 k=1 k=1 k=1 k=1

Propriedade 20 (Desigualdade triangular generalizada). Sejam g(k) definida para k inteiro ,a, b ∈ Z , ent˜ao vale
b b
| g(k) |≤ |g(k)|.
k=a k=a

ao. Para cada k vale


Demonstraç˜
−|g(k)| ≤ g(k) ≤ |g(k)|
aplicando o somatório em ambos lados segue
b b b
− |g(k)| ≤ g(k) ≤ |g(k)|
k=a k=a k=a

que implica
b b b
| g(k) | ≤| |g(k)|| = |g(k)|
k=a k=a k=a

| |
pois os termos g(k) somados são não negativos ,logo a soma desses termos é não-negativa e o módulo da soma é
igual a soma.

Propriedade 21. A identidade que provamos acima vale para n´umeros reais, vamos provar agora por induç˜
ao que
|
se vale z + w | ≤| | | |
z + w para quaisquer z , w ent˜
ao vale
n n
| zk |≤ |z |
k
k=1 k=1

de maneira que possa ser usada para n´umeros complexos , normas e outras estruturas que satisfazem a desigualdade
triangular.
2. Por indução sobre n, para n = 1 tem-se
1 1
| | | |≤
zk = z1 |z | = |z |
k 1
k=1 k=1

88
logo vale. Supondo a validade para n
n n
| zk |≤ |z |
k
k=1 k=1

vamos provar para n + 1


n+1 n+1
| zk |≤ |z |.
k
k=1 k=1

| |
Da hipótese da indução somamos zn+1 em ambos lados, logo
n+1 n n n+1
| | |
zk = zn+1 + zk | ≤ |z | + |
n+1 zk |≤ |z |
k
k=1 k=1 k=1 k=1

Vejamos outras1 demonstrações da desigualdade triangular

1
Essas demonstrações aprendi com Pedro Kenzo, obrigado por compartilhar as solu¸ cões.

89
Exercı́cio 3.22:

Vamos resolver um caso mais geral do problema.


Definição 2 (Mediana). Dada uma sequência finita (yk )n1 seus termos podem ser rearranjados para forma uma

ao-decrescente (xk )n1 . A mediana X é definida da seguinte maneira
sequência n˜


• Se n e´ ı́mpar X = x n+1
2
.
x n +1 +x n
• Se n e´ par X = 2
2
2
.
n
Exemplo 6. Seja (xk )n1 uma sequência crescente f : R
 → R com f (x) = ∑ k=1 |x − x |. Se x < x
k 1 ent˜
ao
n
f (x) = −nx + xk
k=1

logo f e´ decrescente para x < x1 . Tomando x > xn


n
f (x) = nx − xk
k=1

logo f e´ crescente para x > x n .



Seja agora x [xt , xt+1 ), t variando de 1 até n −1
t n t n
f (x) = (x −x )−
k (x − x ) = (2 t − n)x +
k xk − xk
k=1 k=t+1 k=1 k=t+1

portanto a fun¸c˜ao é decrescente se t < n2 e crescente se t > n2 , de t = 1 até t = n


⌊ ⌋ em cada intervalo [x , x ) a
2 t t+1
ao é decrescente, sendo n2 segmentos decrescentes, de t = n2 + 1 até n
funç˜ ⌊ ⌋ ⌊⌋ n
− 1, temos n − 1 − ⌊ ⌋ segmentos
2
crescentes.

• Se n e´ ı́mpar f e´ decrescente em [x⌊ n


2 ⌋ , x⌊ 2 ⌋+1 ) e crescente em [x⌊ 2 ⌋+1 , x⌊ 2 ⌋+2 ) logo o ponto x ⌊ 2 ⌋+1 = x
n n n n n+1
2
é o unico
´ ponto de m´
ınimo.
• Se n e´ par a funç˜ao é constante em [x n
2
, x n2 +1 ), todos os pontos desse intervalo s˜ao pontos de mı́nimo. Em
x n +x n +1
especial o ponto 2
2
2
é ponto de m´
ınimo.

Conclu´
ımos que um ponto de mı́nimo acontece sempre na mediana da sequência.
Exemplo 7. Achar o mı́nimo da funç˜
n
ao f (x) = k=1 x k para n ı´mpar e para n par.
∑ | − |
Trocandon por 2n temos que o mı́nimo acontece no ponto x 22n = x n = n, substitu´
ımos ent˜
ao tal valor na funç˜
ao

2n n 2n n 2n
|n − k| = |n − k| + |n − k| = (n − k) + −
( n + k) =
k=1 k=1 k=n+1 k=1 k=n+1

n n n
= (n − k) + (k) = n = n.n = n2 .
k=1 k=1 k=1
2n
portanto o m´
ınimo de ∑ |−|
• min{|x − 1| + x 2 = 1
| − |}
k=1 x k é n 2 .

• min{|x − 1| + |x − 2| + |x − 3| + |x − 4|} = 4
• min{|x − 1| + |x − 2| + |x − 3| + |x − 4| + |x − 5| + |x − 6|} = 9
• min{|x − 1| + |x − 2| + |x − 3| + |x − 4| + |x − 5| + |x − 6| + |x − 7| + |x − 8|} = 16.
90
Agora para n ı´mpar, trocamos n por 2n + 1 o mı́nimo acontece no ponto x (2n+1)+1 = x n+1 = n + 1, aplicando na
2
funç˜
ao temos
2n+1 n+1 2n+1 n+1 2n+1
|n + 1 − k | = |n + 1 − k | + |n + 1 − k | = (n + 1 − k) + −(n + 1) + k =
k=1 k=1 k=n+2 k=1 k=n+2

n n n
= (n + 1 − k) + k= (n + 1) = n(n + 1).
k=1 k=1 k=1

• min{|x − 1| + |x − 2| + |x − 3|} = 2
min x 1 + x 2 + x 3 + x 4 + x 5 =6
•• min{|{|x −− 1|| + ||x −− 2|| + ||x −− 3|| + ||x −− 4|| + ||x −− 5|}| + |x − 6| + |x − 7|} = 12
• min{|x − 1| + |x − 2| + |x − 3| + |x − 4| + |x − 5| + |x − 6| + |x − 7| + |x − 8| + |x − 9|} = 20.

91
Exercı́cio 3.23:

Propriedade 22. a | − b| < ε ⇒ |a| < |b| + ε.


ao. Partindo da desigualdade a
Demonstraç˜ | − b| < ε, somamos |b| a ambos lados
|a − b| + |b| < ε + |b|
e usamos agora a desigualdade triangular

a a b + b < ε+ b

da´
ı segue
| | ≤| − | | | ||
|a| ≤ ε + |b|.
Da mesma forma vale se |a − b| < ε então |b| ≤ ε + |a| ⇒ |b| − ε ≤ |a| e com |a| ≤ ε + |b|. temos

|b| − ε ≤ |a| ≤ ε + |b|.


Vimos que |a − b| < ε implica |a| < |b| + ε, mas como a ≤ |a| segue a < |b| + ε.

92
Exercı́cio 3.24:

Propriedade 23. Dado um corpo ordenado K , s˜


ao equivalentes
1. K e´ arquimediano.
2. Z e´ ilimitado superiormente e inferiormente.
3. Q e´ ilimitado superiormente e inferiormente.
Demonstraç˜
ao.• 1 ⇒ 2. N ⊂ Z então Z é ilimitado superiormente. Suponha por absurdo que Z seja limitado
inferiormente,
n> a− ⇒− n então
 ∈
< a oexiste a K tal
que contraria ∀ ∈ Z , logo −a > −x, porém existe n natural tal que
que a < x x
a hip´otese.
∈Z
• 2 ⇒ 3 . Z ⊂ Q portanto Q e´ ilimitado superiormente e inferiormente.
a a
• 3 ⇒ 1 . Para todo y ∈ K existe ∈ Q com a, b > 0 naturais tal que > y, da´ıa > yb , podemos tomar
x
b b
y = , logo a > x, a ∈ N , portanto N é ilimitado superiormente e o corpo é arquimediano.
b

93
Exercı́cio 3.25:

1
Propriedade 24. Seja K um corpo ordenado. K é arquimediado ⇔ ∀ε > 0 em K existe n ∈ N tal que < ε.
2n
1 1
ao. ⇒ ). Como K é arquimediano, então ∀ε > 0 existe n ∈ N tal que n > ⇒ n + 1 > n > por
Demonstraç˜ ε ε
n 1 1
desigualdade de Bernoulli temos 2 > n + 1 > ⇒
ε 2n
< ε.
1 1 n
n
⇐). Se ∀ε > 0 em K existe n ∈ N tal que < ε, tomamos ε = , x > 0 arbitrário então x < 2 , com
2n x
2 = m ∈ N então K é arquimediano, N não é limitado superiormente.

94
Exercı́cio 3.26:

n
Propriedade 25. Seja a > 1, K corpo arquimediano, f : Z → K com f (n) = a , ent˜ao

• f (Z ) n˜ao é limitado superiormente.


• inf( F (Z )) = 0 .
n
Demonstraç˜ao. • Vale que a > 1 então a = p + 1 onde p > 0, por desigualdade de Bernoulli temos ( p + 1) ≥
x n
1 + pn. ∀ x > 0 ∈ K existe n tal que n > ⇒ pn > x ⇒ ( p + 1) ≥ 1 + pn > x , logo f (Z ) não é limitado
p
superiormente.
• 0 é cota inferior de f (Z ) pois vale 0 < an n Z . Suponha que ex ista x tal que 0 < x < a m
∀ ∈ ∀ m ∈ Z,
sabemos que existe n ∈ N tal que a n
> x1 d
a´ı x > a1n = a −n , absurdo, então 0 deve ser o ı́nfimo.

95
Exercı́cio 3.27:

̸
Propriedade 26. Se s e´ irracional e u = 0 é racional ent˜
ao u.s é irracional.
p
Demonstraç˜ ao. Suponha que s e´ irracional e u.s seja racional, ent˜ao u.s = q ̸ ̸
com p = 0 e q = 0 inteiros e como
u = 0 é racional ele é da forma u = vj , j = 0 e v = 0, inteiros, logo
̸ ̸ ̸
j p
s=
v q
v
multiplicando por ambos lados segue
j p.v
s=
j.q
que é um número racional, logo chegamos a um absurdo.

Propriedade 27. Se s e´ irracional e t racional, então s + t é irracional.


Demonstraç˜ao. Suponha s + t racional, então s + t = pq da´ı s = p
q
− t que seria racional por ser diferen¸ca de dois
racionais, um absurdo então segue que s + t e´ irracional.

Exemplo 8. Existem irracionais a e b tais que a + b e a.b sejam racionais. Exemplos a = 1 +


√5 , b = 1 − √ 5

ı a + b = 2 e a.b = 1 5 = 4.
da´ −

96
Exercı́cio 3.28:

Propriedade 28. Sejam a,b,c,d racionais ent˜


ao
√ √ ⇔ a = c e b = d.
a+b 2= c+d 2
√ √
ao. ⇐ ). Se a = c e b = d a temos a + b 2 = c + d 2.
Demonstraç˜
⇒). Suponha a + b√2 = c + d√2 então a − c = √2(d − b), se d = b então a = c e terminamos, se n˜ao vale que
a−c √
= 2
d b
√ −
o que é absurdo pois 2 é irracional.

97
Exercı́cio 3.29:

Exemplo 9. O conjunto da forma {x + y√p} onde x e y s˜ao racionais é subcorpo dos n´umeros reais.
• O elemento neutro da adi¸c˜ao 0 pertence ao conjunto. Pois 0 = 0 + 0 √p
• O elemento neutro da multiplicaç˜ao 1 pertence ao conjunto. Pois 1 = 1 + 0 √p
• A adiç˜ao é fechada. Pois x + y√p + z + w√p = x + z + (y + w)√p.
• O produto é fechado. Pois (x + y√p)(z + w√p) = xz + xw√p + yz √p + y.wp.
• Dado x ∈ A implica −x ∈ A. Pois dado x + y√p temos o simétrico −x − y√p.
• Dado x ̸= 0 ∈ A tem-se x− ∈ A. Pois dado x + y√p temos inverso
1

x−y p

x −y p
2 2

como inverso multiplicativo.

Exemplo 10. O conjunto dos elementos da forma a + bα onde α = 3 2 n˜



ao é um corpo pois o produto n˜
ao é
fechado, vamos mostrar que α 2 n˜ao pertence ao conjunto.
2 3 2
Suponha que α = a + bα ent˜ ao α = aα + bα = 2 substituindo a primeira na segunda temos que

aα + b(a + bα) = aα + ab + b2 α = α(b2 + a) + ab = 2 ⇒ α(b2


+ a) = 2 − ab
2 ab
seb2 ao α = −
+ a ̸= 0 ent˜ o que é absurdo pois α e´ irracional, ent˜ao devemos ter
b2 +a
2
a = −b , multiplicamos a
ao aα + bα2 = 2 por α, de onde segue aα 2 + 2b = 2α, substituindo α2 = a + bα nessa ´ultima temos
express˜

a(a + bα) + 2b = a 2 + abα + 2b = 2α ⇒ α(2 − ab) = 2b + a2

2
se 2 = ab chegamos num absurdo de α = 2b+a
̸ 2−ab
2 de onde segue 2 =
, temos que ter ent˜ao 2 = ab e a = −b b3 ,−
porém n˜ ao podemos escrever α2 da forma a + bα com a e b
ao existe racional que satisfaz essa identidade, daı́ n˜
racionais, portanto o produto de elementos n˜ao é fechado e assim não temos um corpo.

98
Exercı́cio 3.30:

Propriedade 29. Sejam a, b ∈ Q . √a + √b e´ racional ⇔ √a e √b s˜ao racionais.


+

ao. ⇒ ).√
Demonstraç˜ √ √
Se a = b então 2 a ∈ Q o que implica a = b ∈ Q. Agora o caso de a ̸ = b.√ √
√ √
Suponha que a + b é racional então seu inverso também racional , que é −a b √ √
√ √ √ √ √ √ − , da´ı a − b ∈ Q , a soma
a b
(√a + √b) + (√ a − √ b) = 2 a ∈ √
Q logo√ a ∈ Q, a diferen¸ca de n´umeros racionais também é um número racional
( a + b) − a = b, portanto a e b são racionais.
). A volta vale pois a soma de racionais é um racional.

99
Exercı́cio 3.31:

Propriedade 30. Sejam A ⊂ R n˜ao vazio limitado e c ∈ R, ent˜ao



1. c sup(A) ⇔ ∀ ε > 0 x ∈ A tal que c − ε < x.

2. c ≥ inf (A) ⇔ ∀ ε > 0 ∃ x ∈ A tal que c + ε > x.

Demonstraç˜ao. 1. ⇒ ). Para todo ε > 0 vale que c − ε < sup(A). Dado ε > 0 fixo, se n˜ao existisse x ∈ A tal que
c − ε < x então c − ε seria cota superior menor que o supremo, o que é absurdo, contraria o fato do supremo
ser a menor das cotas superiores.
⇐). Suponha por absurdo que fosse c > sup (A), poderı́amos tomar c − sup(A) = ε daı´ c − c + sup(A) =
sup(A) < x o que é absurdo.
2. ⇒ ). Para todo ε > 0 vale que c + ε < inf (A). ∈
Dado ε > 0 fixo, se n˜ao existisse x A tal que c + ε > x
então c + ε seria cota superior menor que o ı́nfimo, o que é absurdo, contraria o fato do ı́nfimo ser a menor
das cotas inferiores.
⇐). Suponha por absurdo que fosse c < inf (A), poder´
ıamos tomar inf (A) − c = ε da´ı x < c + inf (A) − c =
inf (A) o que é absurdo.

100
Exercı́cio 3.32:

Exemplo 11. Seja A = n1 n N . Mostre que inf A = 0. Sabemos que 0 e´ uma cota inferior, agora vamos
{ | ∈ }
ao pode ser cota inferior, pois existe n natural tal que n1 < x, logo
mostrar que 0 é a menor delas. Dado 0 < x, x n˜
0 e´ o ı́nfimo.

101
Exercı́cio 3.33:

Propriedade 31. Se A é limitado inferiormente e B ⊂ A ent˜ao inf (A) ≤ inf (B).


Demonstraç˜ ao. infA é cota inferior de A, logo também é cota inferior de B, sendo cota inferior de B vale infA ≤
infB , pois inf B é a maior cota inferior de B .

Propriedade 32. Se A é limitado superiormente e B ⊂ A ent˜ao sup(A) ≥ sup(B).


ao. Toda cota superior de A e´ cota superior de B , logo o sup(A) é cota superior de B, como sup(B) é
Demonstraç˜
a menor das cotas superiores de B segue que sup(A) ≥ sup(B).
Corolário 4. Se A e B s˜ ⊂
ao conjuntos limitados com B A ent˜
ao vale sup(A) sup(B) ≥ ≥ inf (B) ≥ inf (A) pois
≥ ≤
temos sup(A) sup(B) e inf (A) inf (B), tendo ainda que sup(B) inf (B). ≥

102
Exercı́cio 3.31:

Propriedade 33. Sejam A ⊂ R n˜ao vazio limitado e c ∈ R, ent˜ao



1. c sup(A) ⇔ ∀ ε > 0 x ∈ A tal que c − ε < x.

2. c ≥ inf (A) ⇔ ∀ ε > 0 ∃ x ∈ A tal que c + ε > x.

Demonstraç˜ao. 1. ⇒ ). Para todo ε > 0 vale que c − ε < sup(A). Dado ε > 0 fixo, se n˜ao existisse x ∈ A tal que
c − ε < x então c − ε seria cota superior menor que o supremo, o que é absurdo, contraria o fato do supremo
ser a menor das cotas superiores.
⇐). Suponha por absurdo que fosse c > sup (A), poderı́amos tomar c − sup(A) = ε daı´ c − c + sup(A) =
sup(A) < x o que é absurdo.
2. ⇒ ). Para todo ε > 0 vale que c + ε < inf (A). ∈
Dado ε > 0 fixo, se n˜ao existisse x A tal que c + ε > x
então c + ε seria cota superior menor que o ı́nfimo, o que é absurdo, contraria o fato do ı́nfimo ser a menor
das cotas inferiores.
⇐). Suponha por absurdo que fosse c < inf (A), poder´
ıamos tomar inf (A) − c = ε da´ı x < c + inf (A) − c =
inf (A) o que é absurdo.

103
Exercı́cio 3.32:

Exemplo 12. Seja A = n1 n N . Mostre que inf A = 0. Sabemos que 0 e´ uma cota inferior, agora vamos
{ | ∈ }
ao pode ser cota inferior, pois existe n natural tal que n1 < x, logo
mostrar que 0 é a menor delas. Dado 0 < x, x n˜
0 e´ o ı́nfimo.

104
Exercı́cio 3.33:

Propriedade 34. Se A é limitado inferiormente e B ⊂ A ent˜ao inf (A) ≤ inf (B).


Demonstraç˜ ao. infA é cota inferior de A, logo também é cota inferior de B, sendo cota inferior de B vale infA ≤
infB , pois inf B é a maior cota inferior de B .

Propriedade 35. Se A é limitado superiormente e B ⊂ A ent˜ao sup(A) ≥ sup(B).


ao. Toda cota superior de A e´ cota superior de B , logo o sup(A) é cota superior de B, como sup(B) é
Demonstraç˜
a menor das cotas superiores de B segue que sup(A) ≥ sup(B).
Corolário 5. Se A e B s˜ ⊂
ao conjuntos limitados com B A ent˜
ao vale sup(A) sup(B) ≥ ≥ inf (B) ≥ inf (A) pois
≥ ≤
temos sup(A) sup(B) e inf (A) inf (B), tendo ainda que sup(B) inf (B). ≥

105
Exercı́cio 3.34:

Propriedade 36. Sejam A, B ⊂ R tais que para todo x ∈ A e todo y ∈ B se tenha x ≤ y. Ent˜ao sup A ≤ inf B.
Demonstraç˜ ao. Todo y ∈ B é cota superior de A, logo sup A ≤ y para cada y pois sup A e´ a menor das cotas

superiores, essa relação implica que sup A é cota inferior de B logo sup A inf B, pois inf B é a maior cota inferior.

Propriedade 37. sup A = inf B ⇔ para todo ε > 0 dado , existam x ∈ A e y ∈ B com y − x < ε.
Demonstraç˜
ao. , usamos a contrapositiva. Não podemos ter inf B < sup A pela propriedade anterior, ent˜ao

∈ ≥⇐ ≥
e y B pois y inf B e sup A x de onde segue x − ≥− − − ≥
temos forçosamente que inf B > sup A, tomamos então ε = inf B sup A > 0 e temos y x ε para todo x A
sup A, somando esta desigualdade com a de y tem-se

− ≥ −
y x inf B sup A = ε.
⇒ , Se sup A = inf B. Então sendo para qualquer ε > 0, sup A 2ε não é cota superior de A, pois é menor que

o sup A (que é a menor cota superior), da mesma maneira inf A + 2ε não é cota inferior de B , então existem x A ∈

e y B tais que
ε ε
sup A −
2
≤ ≤
< x sup A = inf B y < inf B +
2
ε ε
inf B
2
− ≤
< x y < inf B +
2
de onde segue inf B ε2 < x, x < 2ε inf B e y < inf B + 2ε somando ambas tem-se y x < ε.
− − − −

106
Exercı́cio 3.35:

Propriedade 38. Se c > 0 ent˜


ao sup(c.A) = c. sup A.
Demonstraç˜ ∈ ≤ ≤
ao. Seja a = sup A. Para todo x A tem-se x a, de onde segue cx ca, assim ca e´ cota superior
de cA. Seja d tal que d < ca então dc < a logo dc não é cota superior de A, implicando a existência de pelo menos
um x tal que dc < x, d < cx de onde segue que d não é cota superior de cA, assim ca é a menor cota superior de
cA logo o supremo.

Propriedade 39. Se c > 0, inf cA = c inf A.


Demonstraç˜ ≤ ≤
ao. Seja a = inf A, então vale a x para todo x, multiplicando por c segue ca cx de onde concluı́mos
que ca e´ cota inferior de cA. Seja d tal que ca < d , então a < dc , implicando que dc não é cota inferior de A assim
existe x A tal que x < dc
∈ ⇒ cx < d , logo d não é cota inferior de cA, implicando que c.a é a maior cota inferior,
logo o ı́nfimo do conjunto.

Propriedade 40. Se c < 0 ent˜


ao inf( cA) = c sup A.
Demonstraç˜ao. Seja a = sup A . Tem-se x ≤ ∈
a para todo x A, multiplicando por c segue cx ≥
ca para todo
x A. Então ca e´ uma cota inferior de cA. Se d > ca tem-se dc < a como a é supremo, isso significa que existe

d

x A tal que c < x logo d > cx , assim esse d não é cota inferior, implicando que ca é a menor cota inferior, então
ı́nfimo do conjunto.
A questão 35 segue da pr´oxima propriedade com c = −1.
Propriedade 41. Se c < 0 ent˜
ao sup(cA) = c inf A.
Demonstraç˜ ≤ ∈ ≥
ao. Seja b = inf A então vale b x para todo x A, multiplicando por c segue cb cx assim cb e´ cota
superior de cA. Agora tome d tal que cb > d segue b < dc , como b é ı́nfimo existe x A tal que x < dc , cx > d

assim esse d não pode ser cota superior de cA, então cb é a menor cota superior, logo o ı́nfimo.

107
Exercı́cio 3.37:

Item I
Sejam A, B ⊂ R, conjuntos limitados .
Propriedade 42. O conjunto A + B = x + y x{ | ∈ A, y ∈ B } também é limitado.
ao. Se A é limitado , existe t tal que |x| < t para todo x ∈ A e se B é limitado existe u tal que |y | < u
Demonstraç˜
∀y ∈ B. Somando as desigualdades e usando desigualdade triangular segue |x| + |y| < u+t e |x+y| ≤ |x| + |y| < u+t
logo o conjunto A + B é limitado.

Item II
Propriedade 43 (Propriedade aditiva). Vale sup(A + B) = sup( A) + sup(B).
Demonstraç˜ao. Como A, B são limitidados superiomente, temos sup A := a e sup B := b, como vale a x e b y≥ ≥
∈ ≥
para todos x, y A, B respectivamente segue que a + b x + y logo o conjunto A + B é limitado superiormente.
Para todo e qualquer ε > 0 existem x, y tais que
ε ε
a < x + , b<y +
2 2
somando ambas desigualdades-segue-se que
a+b< x+y+ε
que mostra que a + b é a menor cota superior, logo o supremo, fica valendo então

sup(A + B) = sup( A) + sup(B).

Item III
Propriedade 44. inf( A + B) = inf A + inf B.
Demonstraç˜ ∀ ∈
ao. Sejam a = infA e b = infB então x, y A, B tem-se a ≤
x, b ≤
y de onde segue por adi¸ cão
a + b x + y, assim a + b e´ cota inferior de A + B. x, y A, B tal que ε > 0 vale x < a + 2ε e y < b + 2ε pois a
≤ ∃ ∈ ∀
e b são as maiores cotas inferiores, somando os termos das desigualdades segue x + y < a + b + ε, que implica que
a + b é a maior cota inferior logo o ı́nfimo.

108
Exercı́cio 3.38:

Definição 3 (Fun¸ cão limitada). Seja A R, f : A ⊂ →


R é dita limitada quando o conjunto f (A) = f (x) x { | ∈
}
A , se f (A) e´ limitado superiormente ent˜ao dizemos que f e´ limitada superiormente e caso f (A) seja limitado
inferiormente dizemos que A é limitado inferiormente.
Seja uma fun¸cão limitada f : V → R.
Definição 4.
{
sup f := sup f (V ) = sup f (x) x | ∈V}
Definição 5. {
inf f := inf f (V ) = inf f (x) x| ∈V }
Propriedade 45. A funç˜ao soma de duas fun¸c˜
oes limitadas é limitada.
ao. Vale f (x)
Demonstraç˜ | | ≤ M e |g(x)| ≤ M ∀x ∈ A então
1 2

|f (x) + g(x)| ≤ |f (x)| + |g(x)| ≤ M 1 + M2 = M

portando a fun¸cão soma f + g de duas fun¸cões limitadas é também uma fun¸


cão limitada.

Sejam f , g : V → R funções limitadas e c ∈ R.


Propriedade 46.
sup(f + g) ≤ sup f + sup g.
ao. Sejam
Demonstraç˜

{
A = f (x) x | ∈ V }, {
B = g(y) y | ∈ V }, {
C = g(x) + f (x) x | ∈V }
temos que C A + B, pois basta tomar x = y nos conjuntos, logo

≥ sup(f + g)
sup(A + B)

sup(A) + sup(B) = sup f + sup g ≥ sup(f + g)

Propriedade 47.
inf(f + g) ≥ inf( f ) + inf( g).
ao. De C
Demonstraç˜ ⊂ A + B segue tomando o ı́nfimo
inf(A + B) = inf( A) + inf( B) = inf( f ) + inf( g) ≤ inf( C ) = inf( f + g).

Exemplo 13. Sejam f , g : [0, 1] → R dadas por f (x) = x e g (x) = −x


• Vale sup f = 1, sup g = 0, f + g = 0 logo sup(f + g) = 0 vale ent˜ao
sup f + sup g = 1 > sup(f + g) = 0.

• Temos ainda inf f = 0, inf g = −1, f + g = 0, inf (f + g) = 0 logo


inf f + inf g = −1 < inf( f + g) = 0.

As desigualdades estritas também valem se consideramos as fun¸ −


oes definidas em [ 1, 1], nesse caso sup f +sup g =


2 e inf f + inf g = 2 e sup(f + g) = 0 = inf( f + g).

109
Exercı́cio 3.39:

Definição 6. Sejam A e B conjuntos não vazios, definimos A.B = x.y x{ | ∈ A, y ∈ B }.


Propriedade 48. Sejam A e B conjuntos limitados de n´ ao vale sup(A.B) = sup( A). sup(B).
umeros positivos, ent˜
Demonstraç˜ ≤ ≤ ∀ ∈
ao. Sejam a = sup(A) e b = sup(B) então valem x a e y b, x A, y B d ∈ a´ı x.y≤ a.b, logo
a.b é cota superior de A.B. Tomando t < a.b segue que at < b logo existe y B tal que at < y da´ı yt < a logo

existe x A tal que yt < x logo t < x.y então t não pode ser uma cota superior, implicando que a.b e´ o supremo

do conjunto.

Propriedade 49. Sejam A e B conjuntos limitados de n´umeros positivos, então vale inf( A.B) = inf( A). inf(B).
Demonstraç˜ ≥ ≥ ∀ ∈ ∈ ≥
ao. Sejam a = inf( A) e b = inf( B) então valem x a e y b, x A, y B da´ı x.y a.b, logo a.b é
cota inferior de A.B. Tomando t > a.b segue que at > b logo existe y B tal que at > y da´ı yt > a logo existe x A
∈ ∈
t
tal que y > x logo t < x.y então t não pode ser uma cota inferior, implicando que a.b e´ o ı́nfimo do conjunto.

110
Exercı́cio 3.40:

Propriedade 50. Sejam f, g : A → R funç˜oes limitadas ent˜ao f .g : A → R é limitada.


Demonstraç˜ | | | | | |
ao. Vale que f (x) < M1 e g(x) < M2 então f (x)g(x) < M1 M2 = M ∀ x ∈ A , portanto f.g : A → R
é limitada.

→R +
Propriedade 51. Sejam f, g : A limitadas superiormente, ent˜
ao

sup(f.g) sup(f )sup( g).



ao. Sejam C = {g(x).f (x) | x ∈ A } , B = {g(y). | y ∈ A } e A = {f (x) | x ∈ A } . Vale que C ⊂ A.B
Demonstraç˜
para ver isso basta tomar x = y nas defini¸cões acima, da´
ı

sup(A.B) ≥ sup(C )
sup(A) sup(B) ≥ sup(C )
sup(f ) sup(g) ≥ sup(f.g).

+
Propriedade 52. Sejam f, g : A →R limitadas inferiormente, ent˜
ao

≥ inf( f )inf( g).


inf(f.g)

ao. Sejam C = {g(x).f (x) | x ∈ A } , B = {g(y). | y ∈ A } e A = {f (x) | x ∈ A } . Vale que C ⊂ A.B,


Demonstraç˜
da´ı
inf(A.B) ≤ inf( C )
inf(A)inf( B) inf( C )
inf(f )inf( g) ≤≤inf( f.g).
1
Exemplo 14. Sejam f , g : [1, 2] → R dadas por f (x) = x e g(x) = x, vale sup f = 2, sup g = 1 sup f. sup g = 2 e
sup(f.g) = 1, pois f.g = 1 logo
sup f sup g > sup(f.g).
1 1
Da mesma maneira inf f = 1, inf g = 2
vale inf f. inf g = 2
e inf( f.g) = 1 portanto

inf f. inf g < inf( f.g).

Propriedade 53. Seja f : A → R+ ao sup(f 2 ) = (sup f )2 .


limitada superiormente ent˜
Demonstraç˜ao. Seja a = sup f tem-se f (x) a x daı´ f (x)2 a 2 então a2 é cota superior de f 2 , e é a menor
√ ≤ ∀ ≤ √
cota superior pois se 0 < c < a 2 então c < a logo existe x tal que c < f (x) < a e da´ı c < f (x)2 < a2 logo a 2 é
a menor cota superior sup( f 2 ) = sup( f )2 .

+ 2 2
Propriedade 54. Seja f : A → R ent˜ao inf( f ) = (inf f ) .
Demonstraç˜
2
√ f (x) ≥ a ∀x da´ı f (x) ≥ a então√a é cota inferior de f , e é a maior cota
ao. Seja a = inf f tem-se 2 2 2
2
inferior pois se a < c então a < c logo existe x tal que a < f (x) < c e da´ı a < f (x) < c logo a é a maior 2
2
2

cota inferior inf( f 2 ) = inf( f )2 .

111
Exercı́cio 3.42:

Teorema 1 (Teorema das raı́zes racionais). Se o polinˆomio


n
f (x) = a k xk
k=0
r
de coeficientes inteiros, tem uma raiz racional x = s tal que mdc(r, s) = 1 ent˜
ao s an e r a0 . | |
n
r k
ao. Se x =
Demonstraç˜ s e´ raiz de f (x) = ak x , então temos
k=0

∑f
r
s
=
n
ak
r
s
k
=0
k=0

multiplicando por s n em ambos os lados temos


n
ak rk .sn−k = 0
k=0

|
como s 0 então s |
n

k=0
ak rk .sn−k , na soma s não aparece como fator apenas quando n − k = 0, n = k, logo
abrindo o limite superior do somat´orio temos
n 1
− n 1

ak rk .sn−k + an rn .sn−n = ak rk .sn−k + an rn = 0
k=0 k=0

da´ı s deve dividir an r n , como s e´ primo com r implica que também é primo com rn , portanto s deve dividir an .
|
Pelo mesmo argumento, temos que r 0 logo r deve dividir
k=0
∑n
ak r k .sn−k , como o ´unico fator onde r não aparece é
quando k = 0, abrimos o limite inferior do somat´orio
n n
a0 r0 .sn−0 + ak r k .sn−k = a 0 .sn + ak rk .sn−k = 0
k=1 k=1

logo r deve dividir a 0 .sn , mas como r e´ primo com s n , ele deve dividir a 0 .

Corolário 6. Se o polinˆomio de coeficientes inteiros


∑n

k=0
ak xk possui raı́zes racionais ent˜
ao elas devem pertencer
ao conjunto
A= { pq | p|a q |a }.
0 n

Corolário 7. Se an = 1 em um polinˆomio de coeficientes inteiros P (x) =


∑n

k=0
ak xk ent˜
ao suas raı́zes racionais
devem ser inteiras, pois

A= { pq | p|a q |1}0

ent˜ −
ao q = 1 ou q = 1, e de qualquer forma implica que as solu¸ c˜ oes são da forma x = p para algum p ∈ Z . Ent˜ao
, nessas condi¸c˜ omio P (x) s˜
oes, as raı́zes do polinˆ ao inteiras ou irracionais.
Propriedade 55. Seja P (x) = x n a, a > 0 Z , se a n˜
√− ∈
ao é n-ésima potência de um n´
umero natural ent˜
ao a
unica
´ raiz positiva de P , que é n a , e´ irracional.
Demonstraç˜ ao. Como P possui coeficiente an = 1 então ele possui raiz irracional ou inteira, se a raiz positiva m
fosse inteira (logo natural) ter´ıamos m n a = 0 e da´
− ı a = m n é potência de um número natural, o que contraria a

hipótese de a não ser n-ésima potência de um número natural, logo n a é irracional.

112
Exercı́cio 3.43:

m
Propriedade 56. Sejam I um intervalo n˜ao degenerado e k > 1 natural. O conjunto A = { ∈ I | m, n ∈ Z } é
kn
denso em I .
Demonstraç˜ao. Dado ε > 0 existe n N tal que k n > 1ε , da´ı os intervalos [kmn , m+1
∈ kn ] tem comprimento kn
m+1
− m
kn =
1
k n < ε.
Existe um menor inteiro m +1 tal que x + ε m+1≤ kn a d´ı kmn (x ε, x + ε) pois se fosse x + ε < kmn iria contrariar
∈ −
a minimalidade de m + 1 e se fosse kmn < x ε então [ kmn , m+1
− kn ] teria comprimento maior do que de ( x − ε, x + ε),
que é ε, uma contradição com a suposi¸cão feita anteriormente.

113
Exercı́cio 3.44:

Propriedade 57. O conjunto dos polinˆ


omios com coeficientes racionais é enumer´
avel.
n+1
ao. Seja P n o conjunto dos polinômios com coeficientes racionais de grau
Demonstraç˜ ≤ n a função f : P → Q
n
tal que
n
P( ak xk ) = (ak )n1
k=0

é uma bijeção. Como Q n+1 é enumerável por ser produto cartesiano finito de conjuntos enumeráveis, segue que P n

é enumerável.
Sendo A o conjunto dos polinˆomios de coeficientes racionais, vale que

A= Pk
k=1

portanto A e´ união enumerável de conjuntos enumeráveis , sendo assim A é enumerável.

Definição 7 (Número algébrico). Um n´umero real (complexo) x é dito algébrico quando é raiz de um polinˆ
omio
com coeficientes inteiros.
Propriedade 58. O conjunto dos n´umeros algébricos é enumer´
avel.
{
1. Enumeramos A = P1 , P2 , ··· ···}
, Pn , , o conjunto dos polinˆomios com coeficientes inteiros, definimos B k como
conjunto das raı́zes reais de f k , então vale que

B= Bk
k=1

como cada B k e´ finito B fica sendo união enumerável de conjuntos finitos, então B é enumerável.

2. Seja B o conjunto dos algébricos e A o conjunto dos polinˆomios com coeficientes inteiros. Para cada algébrico
x escolhemos um polinômio P x tal que P x (x) = 0.
Definimos a fun¸cão f : B → A tal que F (x) = P x . Dado P x F (B), temos que o conjunto g −1 (Px ) dos valores


x B tal que f (x) = P x é finito pois Px possui um número finito de ra´
 ızes e da´ı tem-se
=y

B= g −1 (y)
y f (B)

logo B e´ união enumerável de conjuntos enumeráveis ( no caso finitos), ent˜ao B é enumerável.

Corolário 8. Existem n´umeros reais que n˜ao s˜ao algébricos, pois se todos fossem algébricos R seria enumer´avel.
Definição 8 (Números transcendentes). Os n´umeros reais que n˜ao s˜ao algébricos s˜
ao ditos transcendentais
Propriedade 59. O conjunto dos n´umeros algébricos é denso em R, pois todo racional é algébrico, o racional ab
é raiz do polinˆ
omio com coeficientes inteiros
ax − b = P (x)
b
ax − b = 0 ⇔ ax = b ⇔ x = a. E Q é denso em R.

114
Exercı́cio 3.45:

Propriedade 60. Seja A enumer´ \ ∩


avel e B = R A, ent˜ao para cada intervalo (a, b), (a, b) B e´ n˜ao enumer´
avel,
em especial B é denso em R.
Com esse resultado garantimos que o complementar de um conjunto enumer´ avel é denso em R.
ao. Sabemos que ( a, b) é n˜
Demonstraç˜ ao enumerável, escrevemos

(a, b) = [( a, b) ∩ A] ∪ [(a, b) ∩ (R \ A)] = [( a, b) ∩ A] ∪ [(a, b) ∩ B],


sabemos que ( a, b) A é enumerável se ( a, b) B tamb´em o fosse, chegarı́amos no absurdo de (a, b) ser enumerável,
∩ ∩ ∩
por ser uni˜ao finita de conjuntos enumeráveis , portanto ( a, b) B e´ não enumerável e B é denso em R.

ao ser denso em R, como (a, b).


Exemplo 15. Um conjunto pode n˜ao ser enumer´avel e também n˜

115
Exercı́cio 3.46:

Corolário 9. O conjunto T dos n´umeros transcedentais é n˜


ao enumer´avel e denso em R. Pois A o conjunto dos
n´ \
avel, T = R A, como complementar dos n´umeros algébricos T e´ n˜ao enumer´avel e
umeros algébricos é enumer´
denso em R.

116
Exercı́cio 3.47:

|
Propriedade 61. Seja L K uma extens˜ao de corpo. Se α, β ∈ L s˜ao algébricos sobre K , ent˜ao α ± β,α.β e α
β
̸
com β = 0 s˜
ao algébricos sobre K , Desse modo

{α ∈ L|α é algébrico sobre K }


é um subcorpo de L que contém K .
Demonstraç˜ ao. Seja δ α β, α.β α
∈{ ± ̸ }
β β = 0 então δ ∈
K (α, β ) e K ⊂ ⊂
K (δ ) K (α, β )). Vamos mostrar que
[K (α, β ) : K ] < .
∈∞
Sejam f , g K [x] os polinˆomios mı́nimos de α e β sobre K , com graus m e n respectivamente temos que

[K (α) : K ] = m, [K (β ) : K ] = n.

∈ ⊂
f (x) k(x) K (β )[x] é tal que f (α) = 0, logo α e´ algébrico sobre K (β ), sendo P o polinômio mı́nimo de α
≤ ≤
sobre K (β ) de grau s, ele divide f (x) em K (β )[x] logo s m, portanto [ K (β )(α) : K (β )] = s m o grau é finito
e a extensão total [ K (α, β ) : K ] = sn é finita por multiplicatividade dos graus. Como a extensão [K (α, β ) : K ] é
finita ela é algébrica.
|
Definição 9 (Fecho algébrico de Q). Consideremos a extens˜ao de corpos C Q. Chamamos de fecho algébrico de
Q ao subcorpo Q de C definido por

Q= α { ∈ C, α e´ algébrico sobre Q}
Q é realmente corpo pela propriedade anterior. O conjunto dos n´umeros algébricos é um corpo.

117
Exercı́cio 3.48:

Exemplo 16. Sendo A k = [k, ∞) temos uma sequência de intervalos que s˜ao conjuntos fechados porém a interseç˜ao

Ak = A
k=1


é vazia, pois suponha que exista t A, da´ ∈ ∞
ı existe k > t e t / [k, ) = A k logo n˜
ao pode pertencer a interseç˜
ao te
todos esses conjuntos.
Da mesma maneira existe uma sequência decrescente de intervalos abertos limitados com interse¸

ao vazia, sendo
Bk = (0, k1 )

Bk = B
k=1
1
B é vazio, pois se houvesse um elemento nele x > 0, conseguimos k tal que k < x da´
ı x n˜
ao pertence ao intervalo
(0, k1 ) = B k portanto n˜
ao pode pertencer a interseç˜
ao.

118
Exercı́cio 3.49:

Propriedade 62. Sejam B ⊂ A n˜ao vazios, A limitado superiormente, se x∀ ∈ A existe y ∈ B tal que y ≥ x
ao sup(B) = sup(A).
ent˜
Demonstraç˜ ≥
ao. B é limitado superiormente pois está contido em um conjunto limitado e vale que sup(A) sup(B),
⊂ −
pois B A, suponha que fosse c = sup(A) > sup (B), então tomando ε = sup(A) sup(B) > 0, existe x A tal ∈
− − ≥
que x > c ε = sup(A) sup(A) + sup(B) = sup(B), por hip´otese existe y x > sup (B) com y B , o que é ∈
absurdo, pois n˜ao pode existir um elemento maior que o supremo.

Propriedade 63. Sejam B ⊂ A n˜ao vazios, A limitado inferiormente, se ∀x ∈ A existe y ∈ B tal que y ≤ x ent˜ao
inf (B) = inf (A).
Demonstraç˜ ≤
ao. B é limitado inferiormente pois está contido em um conjunto limitado e vale que inf (A) inf (B),
⊂ −
pois B A, suponha que fosse c = inf (A) < inf (B), então tomando ε = inf (B) inf (A) > 0, existe x A tal ∈
− ≤ ∈
que x < c + ε = inf (A) sup(A) + inf (B) = inf (B), por hip´otese existe y x < inf (B) com y B , o que é
absurdo, pois n˜ao pode existir um elemento menor que o ı́nfimo.

119
Exercı́cio 3.50:

Definição 10 (Corte de Dedekind). Um corte de Dedekind é um par ordenado (A, B) onde A, B ∈ Q n˜ao vazios,
tais que A n˜ ∪ ∀ ∈ ∈
ao possui m´aximo, A B = Q e x A, y B vale x < y.
Seja C o conjunto dos cortes de Dedekind.
Propriedade 64. Em (A, B) vale sup(A) = inf (B).
Demonstraç˜ao. Já sabemos que vale sup(A) ≤ inf (B), pois ∀ ∈
x A, y ∈ B vale x < y implica sup(A) < y
e sup(A) ser cota inferior implica sup(A) ≤ inf (B), suponha por absurdo que fosse sup(A) < inf (B), então o
intervalo (sup(A),inf (B)) não possui valores x A, pois se n˜ao x > sup (A), nem y B pois da´ı y < inf (B), mas
∈ ∪ ∈
como existem racionais em tal intervalo, pois Q e´ denso e A B = Q, chegamos em um absurdo.

ao entre R e C o conjunto dos cortes.


Propriedade 65. Existe bije¸c˜
ao. Definimos f : C
Demonstraç˜ → R como f (A, B) = sup(A) = inf (B).
• f e´ injetora, suponha f (A, B) = f (A′, B′ ) então sup(A) = inf (B) = sup(A′) = inf (B′ ).
Dado x ∈ A vamos mostrar que x ∈ A′ .

x < sup(A′ ) = inf (B ′ ) ≤ y ′ , ∀ y ′ ∈ B ′ , da´ı x ∈ A ′

a inclusão A ′ ⊂ A e´ análoga. Então vale A = A′ .


• Dado y ∈ B, vamos mostrar que y ∈ B ′.
x′ < sup(A) < inf (B ′ ) ≤ y

com isso y ∈ B ′ . De maneira similar, B ′ ⊂ B portanto B = B ′ . Como vale B = B ′ e A = A ′ então a função


é injetiva.
• A função é sobrejetiva. Para qualquer y ∈ R, tomamos os conjuntos ( −∞, y) ∩ Q = A e B = [y, ∞) ∩ Q, A
não possui m´aximo, para todo x ∈ A e y ∈ B tem-se y > x e Q = [(−∞, y) ∩ Q] ∪ [ [y, ∞) ∩ Q], além disso
vale sup(A) = y = inf (B), portanto f (A, B) = y e a fun¸cão é sobrejetora, logo sendo também injetora f é
bijeção.

120
Exercı́cio 3.51:
Sejam X , Y conjuntos não-vazios e f : X Y R uma função limitada. Para cada x0 X e cada y0
× → ∈ ∈ Y,
{
definimos s1 (x0 ) = sup f (x0 , y); y ∈ }
Y e s2 (y0 ) = sup f (x, y0 ); x {
X . Isto define funções s1 : X ∈ } →Re
s2 : Y →R. Prove que se tem sup x∈X s1 (x) = sup y∈Y s2 (y). Em, outras palavras,
supx [supy f (x, y)] = sup y [supx f (x, y)].

Primeiramente, verificaremos que supx∈X s1 (x) R . De fato, se s1 (x); x X fosse ilimitado superiormente,
∈ { ∈ }
dever´
ımos ter que f é ilimitada superiormente. Pois, neste caso, dado A > 0 existiria x 0 X tal que ∈
A + 1 < s1 (x0 ),
{
também existiria, pois s 1 (x0 ) = sup f (x0 , y); y ∈ Y }, y ∈ Y tal que
0

s (x ) − 1 < f (x , y )
1 0 0 0

e, consequentemente, terı́amos que


A < f (x0 , y0 ).
Assim, concluirı́amos que f e´ ilimitada superiormente. Um absurdo.
De forma an´aloga, mostra-se que sup y∈Y s2 (y) R. ∈
Provaremos que
supx∈X s1 (x) = sup (x,y)∈X ×Y f (x, y).
Seja ε > 0 arbitrário. Temos que existe x 0 ∈ X tal que
ε
sup ∈ s (x) − < s (x ).
x X 1 1 0
2
Além disso, como s (x ) = sup {f (x , y); y ∈ Y }, temos que existe y ∈ Y tal que
1 0 0 0
ε
s (x ) − < f (x , y ).
1 0 0 0
2
Então, segue das inequa¸cões acima, que
supx∈X s1 (x) − ε < f (x , y )  sup
0 0 ∈ × f (x, y)
(x,y) X Y

E, como ε > 0 é arbitrário, conclui-se que


supx∈X s1 (x)  sup(x,y)∈X ×Y f (x, y).
Novamente, tomamos ε > 0 arbitrário. Existe ( x0 , y0 ) ∈ X × Y tal que
sup(x,y)∈X ×Y f (x, y) − ε < f (x , y ). 0 0

{
Como s 1 (x0 ) = sup f (x0 , y); y ∈ Y }, devemos ter que
sup(x,y)∈ × f (x, y) − ε < f (x , y )  s (x )  sup ∈
X Y 0 0 1 0 x X s1 (x).

E, pela arbitrariedade de ε > 0, conclu´ımos que


sup(x,y)∈X ×Y f (x, y)  supx∈X s1 (x).
Portanto, temos que

supx∈X s1 (x) = sup (x,y)∈X ×Y f (x, y).


De forma an´aloga, demonstra-se que
supy∈Y s2 (y) = sup (x,y)∈X ×Y f (x, y).
De onde conclu´
ımos que
supx∈X s1 (x) = sup y∈Y s2 (y).

121
Exercı́cio 3.52:
Enuncie e demonstre um resultado an´alogo ao anterior com inf ao invés de sup. Considere, em seguida, o caso
“misto” e prove que
supy [infx f (x, y)] = inf x [supy f (x, y)].

Definimos funções i 1 : X →Rei 2 :Y → R por i (x ) = inf {f (x , y); y ∈ Y } e i (y ) = inf {f (x, y ); x ∈ X }.


1 0 0 2 0 0
Provaremos que
infx∈X i1 (x) = inf y∈Y i2 (y),
ou, equivalentemente,
infx∈X [infy∈Y f (x, y)] = inf y∈Y [infx∈X f (x, y)].
Primeiramente, verificaremos que infx∈X i1 (x) R. De fato, se i1 (x); x X fosse ilimitado inferiormente,
∈ { ∈ }
dever´
ımos ter que f é ilimitada inferiormente. Pois, neste caso, dado A > 0 existiria x 0 X tal que ∈
i1 (x0 ) < −A − 1,
{
também existiria, pois i 1 (x0 ) = inf f (x0 , y); y ∈ Y }, y ∈ Y tal que
0

f (x0 , y0 ) < i1 (x0 ) + 1

e, consequentemente, terı́amos que


f (x0 , y0 ) < −A.
Assim, concluirı́amos que f e´ ilimitada inferiormente. Um absurdo.
De forma an´aloga, mostra-se que inf y∈Y i2 (y) R. ∈
Provaremos que
infx∈X i1 (x) = inf (x,y)∈X ×Y f (x, y).
Seja ε > 0 arbitrário. Temos que existe x 0 ∈ X tal que
i1 (x0 ) < inf x∈X i1 (x) + ε .
2
{
Além disso, como i 1 (x0 ) = inf f (x0 , y); y ∈ Y }, temos que existe y ∈ Y tal que
0

ε
f (x0 , y0 ) < i1 (x0 ) + .
2
Então, segue das inequa¸cões acima, que

inf(x,y)∈X ×Y f (x, y)  f (x0 , y0 ) < inf x∈X i1 (x) + ε

E, como ε > 0 é arbitrário, conclui-se que

inf(x,y)∈X ×Y f (x, y)  infx∈X i1 (x).

Novamente, tomamos ε > 0 arbitrário. Existe ( x0 , y0 ) ∈ X × Y tal que


f (x0 , y0 ) < inf (x,y)∈X ×Y f (x, y) + ε.

{
Como i 1 (x0 ) = inf f (x0 , y); y ∈ Y }, devemos ter que
infx∈X i1 (x)  i1 (x0 )  f (x0 , y0 ) < inf (x,y)∈X ×Y f (x, y) + ε.

E, pela arbitrariedade de ε > 0, conclu´ımos que

infx∈X i1 (x)  inf(x,y)∈X ×Y f (x, y).

Portanto, temos que


infx∈X s1 (x) = inf (x,y)∈X ×Y f (x, y).

122
De forma an´aloga, demonstra-se que

infy∈Y i2 (y) = inf (x,y)∈X ×Y f (x, y).

De onde conclu´
ımos que
infx∈X i1 (x) = sup y∈Y i2 (y).
Agora, provaremos a desigualdade

supy [infx f (x, y)]  infx [supy f (x, y)].

Sejam
{
I := infx∈X f (x, y) y | ∈Y}
e
{
S := supy∈Y f (x, y) x | ∈ X }.
Dados elementos arbitrários inf x∈X f (x, y0 ) em I e sup y∈Y f (x0 , y) de S , temos que

infx∈X f (x, y0 )  f (x0 , y0 )  supy∈Y f (x0 , y).

Assim, pelo resultado do exercı́cio 3.34, temos que

supy [infx f (x, y)] = sup( I )  inf(S ) = inf x [supy f (x, y)].

{ }
Por fim, um exemplo onde ocorre a desigualdade estrita. Sejam X = x1 , x2 , Y = y1 , y2 e f : X { } ×Y → R
definido por f (x1 , y1 ) = f (x2 , y2 ) = 2 e f (x1 , y2 ) = f (x2 , y1 ) = 1. Assim, temos que

supy∈Y f (x1 , y) = sup y∈Y f (x2 , y) = 2

e
infx∈X f (x, y1 ) = inf x∈X f (x, y2 ) = 1.
Portanto,
sup [inf f (x, y)] = 1 < 2 = inf [inf f (x, y)].
y x x y

123
Exercı́cio 3.53:
Sejam x e y números reais positivos. Prove que se tem
√xy  x + y .
2

Temos que √ − √y) 2


( x  0.
Logo,
x − 2√xy + y  0
e, portanto,
x+y √
 xy.
2

124
Exercı́cio 3.54:
A desigualdade entre a média aritmética e a média geométrica, vista no exerc´ ıcio anterior, vale para n números

reais positivos x1 , x2 , ..., xn . Sejam G = n x1 x2 ...x n e A = x1 +x2 + ···+xn. Para provar a desigualdade no caso
n
geral, considere a opera¸cão que consiste em substituir o menor dos n´ umeros dados, digamos xi e o maior deles,
x x
digamos x j , respectivamente por x ′i = iG j e x ′j = G. Isto não altera a média geométrica e, quanto à aritmética,
ela não aumenta, pois, como é fácil ver, x′i + x′j  xi + xj . Prove que, repetida a opera¸cão no m´aximo n vezes,
obtemos n números todos iguais `a G e, portanto, sua média aritmética é G. Como em cada operação a média

aritmética não aumentou, conclua que G  A, ou seja, n x1 x2 ...x n  x1 +x2 + ···+xn .
n


Provaremos, por indução em k = 0, 1,...,n , que se exatamente n k termos da sequência x 1 ,...,xn são iguais
à G, então G  A.
Para k = 0, temos que
A=
x1 + x2 + + xn
=
nG
= G,
···
n n
como quer´ ıamos.

Suponhamos que exatamente n k termos da sequência x1 ,...,xn são iguais `a G, para k > 0. Sejam xi e xj ,
respectivamente, elementos de maior e menor valor da sequência em questão. Desta forma temos que

xi < G < x j .

De fato, temos que


xni  x1 x2 ...x n  xnj ,
e, consequentemente,
xi  G  xj .
Assim, como k > 0, devemos ter que x i < G  xj ou x i  G < xj . Se tivéssemos x i < G = x j , então

Gn = x 1 x2 ...x n  (xj )n−1 xi < Gn .

Absurdo. Da mesma forma, n˜ao podemos ter x i = G < xj . Portanto, devemos ter, de fato x i < G < x j . Adiante,
consideremos a sequência dada por 
xp
xi xj
, p = i, j ̸
x′p = G ,p=i
G , p = j.

Temos que
xi xj xi G
x′i + x′j = +G< + xj = x i + xj .
G G
Assim,
x′1 + x′2 + · · · + x′ n x1 + x2 + ··· + x n
< .
n n
Tamb´
em temos que √
G′ :=

x′1 x′2 ...x ′n = x1 x2 ...x n = G.
n n

ı que a sequência x1 , x2 , ... , xn possui no m´aximo k − 1 elementos diferentes de G′ = G. Pela


Conclu´ımos da´ ′ ′ ′
hipótese de indu¸cão, temos que
′ ′ ′
G = G′ = n
x′1 x′2 ...x ′n  x1 + x2 + · · · + xn < x1 + x2 + · · · + xn = A.
n n
E o resultado segue por indu¸cão. √

125
Exercı́cio 3.55:
Seja K um corpo ordenado e completo. Indique com 0 ′ e 1′ o zero e a unidade de K . Para cada n N, sejam ∈

n′ = 1′ + ... + 1′ (n vezes) e ( n)′ = n′ . Definamos uma função f : R


− − K pondo f ( pq ) = pq para todo pq Q e
→ ′


p p
para x irracional, seja f (x) = sup { ∈ K ;
q′
q }
< x . Prove que f é um homomorfismo sobrejetivo e conclua que f é
uma bijeção, ou seja, um isomorfismo de R sobre K .

A solução será dada em 11 passos:

(I) f (m + n) = f (m) + f (n) para todos n, m ∈ Z:


Provaremos a igualdade por indução em n N que f (m + n) = f (m) + f (n) e f (m n) = f (m) + f ( n), para
todo m Z.∈ ∈ − −
Temos que f e´ definido em Z indutivamente por f (m + 1) = f (m) + f (1) e f ( m − − 1) = −f (m) − f (1), para
m N. Daı́ segue o resultado para n = 1.

Adiante, temos que

f (m + (n + 1)) = f ((m + 1) + n) = f (m + 1) + f (n) = f (m) + f (n) + f (1) = f (m) + f (n + 1).

E de forma an´aloga mostra-se que f (m − (n + 1)) = f (m) + f (−(n + 1)). E o resultado segue pelo PIF.
(II) f (mn) = f (m)f (n) para todos n, m ∈ Z:

Demonstração análoga à anterior.

(III) f (m) < f (n) ⇐⇒ m < n, para todos n, m ∈ Z:


Observemos que pela defini¸cão indutiva de f em N (i.e. f (n + 1) := f (n) + 1) e pelo fato de f (1) := 1 ′ > 0 ′ ,
temos que n Z com n > 0 implica f (n) > 0′ . E por f ( n) = f (n) temos que se n Z com n < 0 então
∈ − − ∈
f (n) < 0 ′ . Ou seja, n > 0 em Z se e somente se f (n) > 0 ′ .
Assim, temos que
f (m) < f (n) f (m) f (n) < 0 ′
⇐⇒
f (m n) < 0 ′ −−
m n < 0′
⇐⇒ −
⇐⇒
m < n.

(IV) f (s + t) = f (s) + f (t) para todos s, t ∈ Q:


p m
Sejam s = q et= n com p, m ∈ Z e q, n ∈ N . Ent˜ao, temos que

f (s + t) = f ( np+qm
qn )
f (np+qm)
= f (qn)
f (n)f (p)+f (q)f (m)
= f (q)f (n)
f (p) f (m)
= f (q) + f (n)
= f (s) + f (t).

(V) f (st) = f (s)f (t) para todos s, t ∈ Q:


Análoga à anterior.

(VI) f (s) < f (t) ⇐⇒ s < t, para todos s, t ∈ Q:


Sejam s = pq e t = m
n com p, m Z e q, n
∈ ∈ N . Ent˜ao, temos que
f (s) < f (t) ⇐⇒ f (n)f (p) < f (q )f (m)
⇐⇒ f (np) < f (qm)
⇐⇒ np < qm
⇐⇒ s < t.
126
{
(VII) f (x) = sup f (r); r  x, r ∈ Q} para todo x ∈ R:
Pela defini¸cão de f nos irracionais, basta provar a igualdade para x ∈ Q. De fa to, por (IV ) tem os qu e
f (x)  sup f (r); r  x, r Q . E por outro lado, f (x) sup f (r); r  x, r
{ ∈ } ∈ { ∈ Q}. E saı́ segue o resultado.
(VIII) f é um homomorfismo:

De fato, temos, para quaisquer x, y ∈ R, que


f (x) + f (y) = sup f (r); r  x, r Q + sup f (r); r  y, r Q
{ ∈ } { ∈ }
= sup f (r1 ) + f (r2 ); r1  x, r2  y e r 1 , r2 Q
{ ∈ } (pelo exercı́cio 3.37)
= sup f (r); r  x + y, r Q
{ ∈ } (por (IV))
= f (x + y).

Analogamente, prova-se que f (xy) = f (x)f (y) utilizando-se do exercı́cio 3.39 e do item (V).

(IX) f (x) < f (y) ⇐⇒ x < y, para todos x, y ∈ R:


Temos, por (VII), que
f (x) < f (y) ⇐⇒ ∃r ∈ Q, f (x) < f (r)  f (y)
⇐⇒ ∃r ∈ Q, x < r  y
⇐⇒ x < y.
{ ∈ Q; f (r)  y}):
(X) y = f (sup r

Seja x = sup {r ∈ Q; f (r)  y}. Observemos que x ∈ R pelo fato de K ser completo e ordenado (logo,
arquimediano). Então, pela escolha de x,

{
f (x) = sup f (r); r  x, r ∈ Q}  y.
Por outro lado, se f (x) < y, haveria, novamente pela arquimedianeidade de K , r ∈ Q, tal que f (x) < f (r) < f (y).
Contradizendo a escolha de x.

(XI) f e´ bijetiva:
O item (IX) implica que f é injetiva e o item (X) implica que f e´ sobrejetiva.

127
Exercı́cio 3.56:
Seja f : R R um isomorfismo de R em si mesmo. Prove que f = identidade. Conclua que se K e L são corpos

ordenados completos, existe um único isomorfismo de K sobre L.

Seja f : R → R um isomorfismo de corpos. Provaremos que f é igual a identidade I R : R → R.


Pelo exerc´ıcio 3.4, temos que f (0) = 0 e f (1) = 1.
Por indução em n N, segue que

f (n) = n = f ( n). − −
E, assim, para q ∈Z + temos que
q 1 1
1= f = f (q )f = qf .
q q q
1
De onde conclu´
ımos que f q = 1q . Portanto, para p Z, temos que

p 1 p
f = f (p)f = .
q q q
Ou seja, f (r) = r para todo r Q. ∈
Agora, observemos que se x > 0, então f (x) > 0. De fato, temos que
√√
f (x) = f ( x x) = f ( x)2 > 0.

Seja x ∈ R. Provaremos que f (x) = x mostrando que para quaisquer r, s ∈ Q tais que
s<x<r
tem-se que
s < f (x) < r.
De fato, temos que
0<x −s
e

implicam que
−x 0< r
0 < f (x − s) = f (x) − f (s) = f (x) − s
e
0 < f (r − x) = f (r) − f (x) = r − f (x)
pela observação do parágrafo anterior. Portanto, temos que
s < f (x) < r
e conclu´ımos que
f (x) = x
para qualquer x R. ∈
Provaremos agora que se K e L são corpos ordenados completos, existe um ´unico isomorfismo entre K e L.
Pelo exercı́cio 3.55, temos que existem isomorfismos fK : R K e fL : R →
L. Desta forma, existe um→
isomorfismo ϕ = fL fK −1 : K L.
◦ →
Suponhamos que ψ : K →
L seja um isomorfismo de corpos. Provaremos que ψ = ϕ. De fato, como
fL−1 ψ −1 ϕ fK : R
◦ ◦ ◦ →R
é um isomorfismo, pelo que foi provado acima, temos que
−1 ψ −1 ϕ f K = I R .
fK ◦ ◦ ◦
Consequentemente,
ψ −1 ϕ = I K

e
ψ = ϕ.

128
Exercı́cio 3.57:
x
Verifique que f : R → (−1, 1), definida por f (x) = √ 1+x2
, é uma bijeção de R no intervalo ( 1, 1).


Seja g : ( 1, 1) → R definida por
g(x) = √1 x− x 2
.

Provaremos que g = f −1 .
∈−
Seja x ( 1, 1). Então,
f ◦ g(x) = f √1x−x2
x

2
√1 x
= 2
1
2
√1 x
1+
−x2

√1x
− x2
= 1
x2 2
1+ 1−x2

√ x

= √1 1 2 +2 2

x
x

x
√1 2 − x

√1 2 x
− x
=
√11 2− x

= x.
Seja x ∈ R. Então,
x

g f (x) = g √1+x 2

√1+ x
x2
= 2
1
2
x
1− √1+ x2
x

√1+ x2
= 1
1+x2 −x2 2
1+x2

√1+ x
x2
=
√1+1 x2

= x.
Portanto, g = f −1 .

129
Exercı́cio 3.58:
Um conjunto G de n´umeros reais chama-se grupo aditivo quando 0 G e x, y G x y∈ ∈ ⇒ − ∈
G. Então,
∈ ⇒− ∈
x G x G e x, y G ∈ ⇒x + y G. Seja então G R um grupo aditivo de n´ umeros reais. Indiquemos
∈ ⊂
com G+ o conjunto dos n´umeros reais pertencentes `a G. Excetuando o caso trivial G = 0 , G+ é não-vazio.
{}
̸ {}
Suponhamos pois G = 0 . Prove que:
(i) Se inf G+ = 0, então G é denso em R ;
(ii) Se inf G+ = a > 0, então a ∈G +
{ ± ±
e G = 0, a, 2a,... ; }
(iii) Conclua que, se α R é irracional, os números reais da forma m + nα com m, n
∈ ∈ Z constituem um
subconjunto denso em R .

(i)
Provaremos que dado um intervalo arbitrário ( a, b) em R, existe g G e m Z tais que mg (a.b). Como
∈ ∈ ∈
mg G (prova-se por indução em m), conclu´ımos da´ı que G intercepta todo intervalo aberto em R. Logo, G é

denso em R .
Como inf G+ = 0, temos que existe g G + tal que

0<g<b − a.
Assim, tomando-se m = min n { ∈ Z; a < ng }, teremos que
a < mg = (m − 1)g + g  a + g < a + (b − a) = b.

Logo, mg (a, b).
(ii)
Primeiramente, provaremos que a ∈ G. De fato, se a = inf G+ / G ter´ıamos que existiriam h e g
∈ ∈G +
tais que
a < h < g < 2a.
Da´
ı
0< g −h< a
− h ∈ G, temos que
e, como g
a = inf G+  g − h < a.
Uma contradição. Logo, a ∈ G.
Agora, provaremos que todo g ∈ G é da forma na para algum n ∈ Z. Seja n = max {n ∈ Z; na  g} e r = g − na.
Pela escolha de n, temos que
0  r < a.
Assim, como r = g − na ∈ G devemos ter que r = 0 pois, caso contr´ario, ter´ıamos que
a = inf G+  r < a.
Portanto, g = na.
(iii)
Seja
{
G := m + nα; m, n ∈ Z} .
Temos que G e´ um grupo aditivo. Como G = 0 , nos basta provar que a = inf G+ = 0.
̸ {}
Suponhamos o contrário. Então a > 0 e, pelo item (ii), temos que
{ ± ±
G = 0, a, 2a,... . }
Assim, como α ∈ G, temos que a = kα ∈ R\Q.
Da´
ı, temos que
{ ± ±
G = 0, a, 2a,... } ⊂ R\Q.
Mas, por outro lado,
{
G := m + nα; m, n ∈ Z} ⊃ Z.
Uma contradição.

130
Exercı́cio 3.59:
Sejam f , g : R 2
→ R e φ, ψ : R 3 R as funções definidas por f (x, y) = 3x y, g (x, y) = (x 1)2 + (y + 1) 2 9,
→ − − −
ϕ(x,y,z ) = 3z e ψ(x,y,z ) = x2 + y 2 z. Interpretando ( x, y) como as coordenadas cartesianas de um ponto

no plano R2 e (x,y,z ) como as coordenadas de um ponto no espa¸ co R3 , descreva geometricamente os conjuntos
f −1 (0), g −1 (0), ϕ −1 (0) e ψ −1 (0).

• f − (0) = {(x, y) ∈ R ; y = 3x}: uma reta;


1 2

• g− (0) = {(x, y) ∈ R ; (x − 1) + (y + 1) = 3 }: uma circunferência de raio 3 e centrada no ponto (1, −1);


1 2 2 2 2

1 3
• ϕ−− (0) = {(x,y,z ) ∈ R ; z = 0}: um plano;
1 3 2 2
• ψ (0) = {(x,y,z ) ∈ R ; z = x + y }: dois cones.

131
Exercı́cio 3.60:
Seja a um número real positivo. Dado um n´umero racional p/q (onde p
√ ∈ Z e q ∈ N), defina a potência de base a
e expoente racional p/q como a p/q = q ap . Prove:
r s
(1) Para quaisquer r, s ∈ Q tem-se a .a = a r+s e (ar )s = a rs ;
+ r
(2) Para todo r ∈ Q , a fun¸cão f : (0, +∞) → (0, +∞), dada por f (x) = x , é uma bijeção crescente;
r
(3) A fun¸cão g : Q → R definida por g(r) = a (onde a e´ um número real positivo fixado) é crescente se a > 1, e
decrescente se 0 < a < 1.

(1)
Sejam r = p/q e s = p ′ /q ′ , onde p e p ′ ∈ Z e q e q ′ ∈ N. Temos que
p p′
ar as = aq aq ′


pq p′ q
= a aq q
qq ′
√ ′


′ ′
qq qq
= apq
√ ap q ′ ′


qq
= a ap q
pq

′ ′


qq
= apq +p q ′ ′

pq′ +p′ q
= a qq ′

p p′
= aq+q ′

= ar+s
e
p p′
(ar )s = (a q ) q ′

= q′
√ q p p ′

=
qq ′
√(a a ) pp ′

pp′

qq
= ars
= a .
(2)
1
A função f tem inversa g : (0, + ∞) → (0, +∞) dada por g (x) = x r . De fato, dado x ∈ (0, +∞), temos que
1
f ◦ g(x) = f (x ) r
1
= (x r )r
= x1
= x.
e

g f (x) = g(xr )
1
= (xr ) r
= x1
= x.
Portanto, f é uma bijeção.

Dados x < y em (0, + ), temos que
xp < y p
e, consequentemente,
f (x) = xr

= q
√ xp
< q
yp
= yr
= f (y).
Portanto, f é crescente.

132
(3)
′ ′
pq pq
Dados r = p/q e s = p′ /q ′ , onde p e p′ ∈ Z e q e q′ ∈ N, com r < s. Então, r = qq ′
< qq ′
= s e,
consequentemente,
pq ′ < p′ q.
Caso a > 1, temos que
′ ′

pq ′ < p′ q pq
⇒ a√ < a √ pq
qq ′ pq qq ′ pq
⇒ a < a ′


pq pq
⇒ g( ) < g( )
qq ′
qq ′

⇒ g(r) < g(s).


Conclu´ımos da´
ı que, g é crescente caso a > 1.
Caso 0 < a < 1, temos que
′ ′

pq ′ < p′ q ⇒ a >apq pq

⇒ √a > √a
qq ′


pq ′ qq ′


pq

pq pq
⇒ g( ) > g( )
qq ′
qq ′

⇒ g(r) > g(s).


Conclu´ımos da´
ı que, g e´ decrescente caso 0 < a < 1.

133
Capı́tulo 4

Sequências e Séries de Números Reais

134
Exercı́cio 4.1:
| | ||
Se lim xn = a, então lim xn = a . Dê um contra-exemplo mostrando que a recı́proca é falsa, salvo quando a = 0.

Como lim xn = a, dado ε > 0 existe n 0 ∈ N tal que


||x | − |a|| ≤ |x − a| < ε,
n n

para todo n ≥ n . Logo, temos que


0
| | ||
lim xn = a .
| | ||
Temos que lim xn = a não implica que lim xn = a. Basta tomar x n = 1, para todo n ∈ N e a = −1.

135
Exercı́cio 4.2:
{| | | | | |}
Seja lim xn = 0. Para cada n, ponha y n = min x1 , x2 ,..., xn . Prove que y n −→ 0.
Dado ε > 0, existe n 0 N tal que se n
∈ ≥ n , então |x | < ε. Como |y | = y
0 n n n {| | | |} ≤ |x |, temos
= min x1 ,..., xn n
≥ | | ≤| |
que para n n 0 , yn xn < ε. Logo,
lim yn = 0.

136
Exercı́cio 4.3:
Se lim x2n = a e lim x2n−1 = a, prove que lim xn = a.

Dado ε > 0, existem n p , ni ∈ N tais que


n ≥ n ⇒ |x − a| < ε
p 2n

e
n ≥ n ⇒ |x − − a| < ε.
i 2n 1

{
Tomemos n 0 = max 2np , 2ni − 1}. Assim, para n ≥ n , temos 0

• Se n = 2k, então
2k = n ≥ n ≥ 2n .
0 p

Logo, k ≥n p e consequentemente
|x − a| = |x − a| < ε.
n 2k

• Se n = 2k − 1, então
2k − 1 = n ≥ n ≥ 2n − 1. 0 i

Logo, k ≥n i e consequentemente
|x − a| = |x − − a| < ε.
n 2k 1

Em ambos os casos, se n ≥ n , temos |x − a| < ε. Logo, lim x = a.


0 n n

137
Exercı́cio 4.4:
Se N = N1 ∪ N ∪ ... ∪ N
2 k e lim xn = lim xn = ... = lim xn = a, então lim xn = a.

n N1 ∈
n N2 ∈
n Nk ∈
n N

Dado ε > 0, existem n 1 , n2 ,...,n k N1 , N2 ,..., Nk tais que se n Ni com n n i , então xn a < ε. Tomando
∈ ∈ ≥ | − |
{ }
n0 = max n1 , n2 ,...,n k , temos que se n ≥ ≥
n0 , então n | − |
ni , para todo i = 1,..,k. Desta forma, xn a < ε.
Portanto,
lim xn = a.

138
Exercı́cio 4.5:
Dê exemplo de uma sequência (xn ) e uma decomposi¸cão N = N1 N 2 ... N k ... de N como reuni˜ao de
∪ ∪ ∪ ∪
uma infinidade de conjuntos infinitos tais que, para todo k, a sequência (xn )n∈Nk tenha limite a, mas não se tem
lim xn = a.

Seja k ∈ N. Definimos
Nk = {2k−1 .n ∈ N; n ∈ N, }
n e´ ı́mpar .


Temos dessa definição que N = Nk . Assim, podemos definir
k=1

x: N → 1, se n = 2k−1 para algum k .


n → x(n) =
1/n, se n = 2k−1 m para algum m > 1.

Provemos que lim xn = 0. Dado ε > 0, existe p 0 ∈ N tal que


1
< ε2k−1 .
3p0

Assim, se n = 2k−1 m ∈ N , com n ≥ n


k 0 = 2k−1 3p0 , então
1
xn =
2k−1 m
≤ 2 −1 3
k 1 p0
< ε.

Logo, lim = 0. Por outro lado, lim x2k − 1 = 1. Portanto, lim xn não existe.

n Nk ∈
n N

139
Exercı́cio 4.6:
Se lim xn = a e lim(xn −y n) = 0, então lim yn e´ igual a a.

Dado ε > 0, existem n 1 , n2 ∈ N tais que


n ≥ n ⇒ |x − a| < ε/2
1 n

e
≥ n ⇒ |x − y | < ε/2.
n 2 n n

{ }
Tomando n 0 = max n1 , n2 , teremos que se n ≥ n , então
0

|y − a| ≤ |x − y | + |x − a| < ε.
n n n n

Logo, lim yn = a.

140
Exercı́cio 4.7:
yn
̸
Seja a = 0. Se lim
a
= 1, então lim yn e´ igual a a.

̸ ||
Sejam ε > 0 e a = 0. Neste caso ε/ a > 0. Assim, existe n 0 ∈ N tal que
 − 
yn
a
1 <
ε
|a| ,
ou seja,
|y − a| < ε.
n

Logo, lim yn = a.

141
Exercı́cio 4.8:
xn a
̸
Seja b = 0. Se lim xn = a e lim
yn
= b, então lim yn = .
b

Pelo ı́tem 3 do Teorema 6, temos que


yn 1
lim = .
xn b
Pelo ı́tem 2 do mesmo teorema,
yn
lim yn = lim xn .
xn
yn
= lim x . lim
n xn
1 a
= a. = .
b b

142
Exercı́cio 4.9:
xn a
̸
Seja b = 0. Se lim xn = a e lim
yn
= b, então lim yn = .
b

Pelo ı́tem 3 do Teorema 6, temos que


yn 1
lim = .
xn b
Pelo ı́tem 2 do mesmo teorema,
yn
lim yn = lim xn .
xn
yn
= lim x . lim
n xn
1 a
= a. = .
b b

143
Exercı́cio 4.10:
Sejam k ∈ N e a > 0. Se a ≤ x ≤ n
n
k
para todo n, então lim
√x
n
n = 1.

Temos que √a = lim √n = 1.


n n
lim
Assim,

n (√  k
lim nk = lim n
n
=

lim n
n
k
= 1k = 1.

Logo, lim xn = 1. ( 

144
Exercı́cio 4.10:
Sejam k ∈ N e a > 0. Se a ≤ x ≤ n
n
k
para todo n, então lim
√x
n
n = 1.

Temos que √a = lim √n = 1.


n n
lim
Assim,

n (√  k
lim nk = lim n
n
=

lim n
n
k
= 1k = 1.

Logo, lim xn = 1. ( 

145
Exercı́cio 4.11:
Use a desigualdade entre as média aritmética e geométrica dosn + 1 números 1 1/n, 1 1/n, ..., 1 − − − 1/n, 1 e prove
que a sequência (1 1/n)n é crescente. Conclua que (1 1/n)n 1/4 para todo n > 1.
− − ≥
Pela desigualdade envolvendo a média aritmética e a média geométrica, temos que

1
n n 1 +1
n+1 1 n
1 − n
.1 ≤ n+1

n
n+1
1 − n1 ≤ n +n 1 = 1 − n +1 1
n n+1
1 − n1 ≤ 1 − n +1 1 .

Logo, a sequência (xn )n∈N dada por


n
xn = 1 − n1
é crescente. Da´ı, para n ≥ 2, temos 2 n
1
4
= 1 − 12 ≤ 1 − n1 .

146
Exercı́cio 4.11a:
1 n+1
Sejam x n = (1 + 1/n)n e y n = (1 − n+1 ) . Mostre que lim xn yn = 1 e deduza daı́ que lim(1 n
− 1/n) = e −1 .

Segue, pelas definições de x n e y n , que


n n+1
1
xn yn = 1+
n
1 − n +1 1
n n
n n
=
n+1 n+1
1 − n +1 1
1
= 1 .
− n+1
Logo,
lim xn yn = li m 1 − n +1 1 = 1.
Como lim xn = e, temos que
n
lim 1 − n1 = lim yn1

= lim yn = e −1 .

147
Exercı́cio 4.12:
k 1
∑− k 1
∑−
Fazendo yn = x 1/k e b = a 1/k na identidade y k bk = (y b)
− − yi bk−i−1 , obtenha x a = (x1/k a1/k )
− − xi/k a1−(i+1)/k
e use isto para provar que se lim xn

= a > 0, então lim x n
i=0
n =
√a. Conclua daı́, que lim(x
n
n)
r
i=0
r
= a para todo
racional r.

Fazendo y = x 1/k e b = a 1/k na identidade


k 1

yk −b k
= (y − b) y i bk−i−1 ,
i=0

obtemos

k 1 i+1
x− a = (x − a ) x a − k .
1/k 1/k i/k
1

i=0

como lim xn = a > 0, existe c > 0 em e um n ∈ N tal que para todo n ≥ n , teremos
c c

0 < c < x n.

Logo, para n ≥n c temos que

k 1
− i+1
1 −
xn 1/k
− a = (x − a
n
1/k
) xi/k
n a k
i=0

> (x1/k
n −a 1/k
).S,

onde
k 1 − i+1
1−
S= ci/k a k > 0.
i=0

dado ε > 0, existe n 1 ∈ N tal que para todo n ≥ n , temos 1

εS > |x − a|. n

Da´
ı, fazendo n 0 = max {n , n }, temos que para todo n ≥ n
1 2 0

1/k 1/k
εS > |x − a| > |x n− a |.S. n

Logo,
ε > x1/k
n | −a | 1/k

para n ≥ n . Concluimos da´ı que


0 √ √ n
lim x = a.
n
n
p
Sendo r = , temos que
q

lim xp/q = lim( q p)



n
= (lim q xn )p

q p p/q r
= ( a) = a√ =a .

148
Exercı́cio 4.14:

Propriedade 66. Seja a, b ≥ 0 e ent˜ao lim √a + b = max{a, b}.


n n n

n n n n
ao. Seja c = max {a, b} então vale Vale a ≤ c , b ≤ c
Demonstraç˜ e da´ı an + b n ≤ 2c n
da mesma maneira
n n n
c ≤ a + b , pois c é a ou b, logo
n n n n
c ≤ a + b ≤ 2c

c≤ a +b ≤ 2c
√ n n n n

tomando limites, temos pelo teorema do sanduı́che

lim √a
n n + bn = c.

m
≥ 0) {
Propriedade 67. Sejam (ak 1 e c = max ak , k
∈ } I m ent˜

m
ao

lim n
ank = c.
n →∞ k=1

ao. Vale a nk
Demonstraç˜ ≤c n
, tomando a soma, tem-se
∑ m n
k=1 ak ≤ m.c n
, tem-se também c n ≤ ∑ m
k=1 ank então vale
m
cn ≤ ank ≤ m.c n

k=1

tomando a raiz  m
c ≤ n
ank ≤ √m.c n

k=1

e novamente por teorema do sandu´


ıche tem-se 
lim
 n
m
ank = c.
k=1

149
Exercı́cio 4.15:

Definição 11 (Termo destacado). Dizemos que x n é um termo destacado quando x n ≥x p para todo p > n. Isto é
quando xn é maior ou igual a todos seus sucessores.
Propriedade 68. Toda sequência possui subsequência mon´
otona .
ao. Seja A
Demonstraç˜ ⊂ N o conjunto dos ı́ndices s da sequência (x n ), tais que x s é destacado, existem dois casos
a serem analisados
Se A é infinito, então podemos tomar uma subsequência (xn1 , xn2 , ) de termos destacados formada pelos
• elementos com ı́ndices em A que é não-crescente com n < n < n ·<· · · · · e com x ≥ x ≥ ·· · .
1 2 3 n1 n2

• Se A e´ finito, tomamos um n maior que todos elementos de A da´ı x não é destacado, existindo x ≥ x
1 n1 n2 n1
com n > n , por sua vez x não é destacado logo existe n > n tal que x ≥ x , assim constru´
2 1 n2 3 2 n3 n2 ımos uma
subsequência não-decrescente .

150
Exercı́cio 4.18:

Generalizamos o exercı́cio em dois resultados.


Propriedade 69. Sejam (an ) e (bn ) sequˆ encias limitada tais que an + bn = 1 n ∀ ∈ N , (z ) e (t
n n) com o mesmo
limite a, ent˜ao lim an .zn + bn .tn = a.
ao. Escrevemos
Demonstraç˜

an .zn + bn .tn = a n .zn − a.a n + a. an +bn .tn = a n (zn − a) + a(1 − b n ) + bn .tn =


=1 bn

= a n (zn − a) + a − a.b n 
+ bn .tn = a n (zn − a) + a + b n (tn − a)
da´ı
lim an (zn − a) + a + b (t − a) = a = lim a
n n n .zn + bn .tn
pois a n e b n são limitadas e z n − a, t − a tendem a zero.
n

Propriedade 70. Se limn→∞ zk (n) = a


∑ ∀ k e cada (xk (n)) e´ limitada com
∑ p
k=1 xk (n) = vn → b ent˜
ao
limn→∞ pk=1 xk (n)zk (n) = a.b.
ao. Vale x 1 (n) = v n
Demonstraç˜ − ∑ p
k=2 xk (n).
p p
xk (n)zk (n) = x 1 (n)z1 (n) + xk (n)zk (n) =
k=1 k=2
p p
= z 1 (n)vn − xk (n)z1 (n) + xk (n)zk (n) =
k=2 k=2
p

= z1 (n)vn +
  
→a.b k=2
 −  →
xk (n) (zk (n)
→0
z1 (n)) a.b.

151
Exercı́cio 4.19:

Definição 12 (Sequência de variação limitada) . Uma sequência (xn ) tem variaç˜ao limitada quando a sequência
(vn ) com
n
vn = |∆x | é limitada.
k
k=1

Propriedade 71. Se (xn ) tem variaç˜


ao limitada ent˜ao (vn ) converge.
ao. (vn ) é limitada e não-decrescente, pois ∆vn = ∆xn+1
Demonstraç˜ | | ≥ 0, logo é convergente.
Propriedade 72. Se (xn ) tem variaç˜
ao limitada ent˜ao existe lim xn .
ao. A série
Demonstraç˜
∑ ∞ |∆x | converge portanto
k=1 k
∑ ∞ ∆x converge absolutamente e vale
k=1 k

n 1
− n 1

xn −x 1 = ∆xk ⇒x n = ∆xk + x1
k=1 k=1

logo x n é convergente.

Exemplo 17. Se ∆xn+1 | c ∆xn |≤ |n N com 0 |∀ ∈ c < 1 ent˜ ≤


ao (xn ) possui variaç˜
ao limitada. Definimos
n−1 ∏
| |
g(k) = ∆xk logo a desigualdade pode ser escrita como g(k + 1) cg (k), Qg(k) c aplicamos k=1 ≤
de ambos ≤
lados, da´
ı
n 1
g(n) = ∆xn | | ≤ c − g(1)
somando em ambos lados temos n n
∆xk ck−1 g(1)
k=1 | |≤ k=1
como o segundo termo converge por ser série geométrica segue que (xn ) e´ de variaç˜
ao limitada, logo converge.
Propriedade 73. (xn ) tem variaç˜
ao limitada ⇔x n = yn −z n onde (yn ) e (zn ) s˜
ao sequências n˜
ao-decrescentes
limitadas.
ao. ).
Demonstraç˜ ⇐

Seja x n = yn zn onde (yn ) e (zn ) são sequências não-decrescentes limitadas, então x n tem variação limitada.
n n n n n n
vn = |∆x | =
k |∆y − ∆z | ≤ |∆y | + |∆z | ≤ |
k k k k ∆yk +| | ∆zk |
k=1 k=1 k=1 k=1 k=1 k=1

= |y − y | + |z − z | < M
n+1 1 n+1 1

pois ( yn ) e (zn ) são limitadas, logo ( vn ) é limitada, isto é, (xn ) tem variação limitada.
⇒ ). Dada ( xn ) com varia¸cão limitada. (xn ) tem varia¸cão limitada ⇔
(xn + c) tem varia¸cão limitada, pois ∆
aplicado as duas sequências tem o mesmo valor. Escrevemos
n 1 −
xn −x 1 = ∆xk
k=1

Para cada n definimos P n o conjunto dos k da soma nk=1 k



−1 ∆x tais que ∆ x 0 e N o conjunto dos k da mesma
k n ≥
soma tais que ∆ xk < 0, com isso temos uma parti¸cão do conjunto dos ı́ndices e vale
n 1

−x
xn 1 =
k=1
∆xk =
   −  − 
k Pn

∆xk

yn
k Nn

( ∆xk )

zn

152
(yn ) é n˜
ao decrescente, pois yn+1 = y n caso não seja adicionado ı́ndice a P n+1 em relação a P n e y n+1 ≥y n caso

seja adicionado um ı́ndice a P n+1 , pois adicionamos um termo da forma ∆ xk 0 o mesmo para ( zn ).
(yn ) é limitada pois
n 1

∆xk ≤ |∆x | =
k |∆x | +
k |∆x | =
k ∆xk + −
( ∆xk ) < M
k Pn
∈ k=1 k Pn
∈ k Nn
∈ k Pn
∈ k Nn

da mesma maneira ( zn ) é limitada.
ao limitada, como por exemplo xn =
Exemplo 18. Existem sequências convergentes que n˜ao possuem varia¸c˜
n−1 (−1)k (−1)n n−1 1
k=1 k , que é convergente porém ∆xn = n ∆xn = n1 e k=1
⇒| k n˜|
ao é limitada.

∑ ∑

153
Exercı́cio 4.20:

1
Exemplo 19. Seja (xn ) definida como x1 = 1, xn+1 = 1 + xn , ent˜ao vale que

1
|∆x | ≤ |
n+1
2
∆xn . |
• Primeiro vale que x ≥ 1 para todo n pois vale para n = 1, supondo validade para n, ent˜ao vale para n + 1,
1
n
pois x n+1 = 1 + xn .

1
Vale que xn+1 xn 2 para todo n, pois, substituindo xn+1 = 1 + x isso implica que x n+1 xn xn + 1 2.
•• De |x x| | ≥ 2 segue
| ≥ que | 1 1
n

| ≤ multiplicando por |x − x | em ambos lados segue que ≥
n+1 n xn+1 xn 2, n+1 n

| x − x | ≤ |x − x |
n n+1
xn+1 xn
n+1
2
n

| x 1 − x1 | = | (1 + x 1 ) − (1 + x1 ) | ≤ |x 2− x |
n+1 n n+1 n
n+1 n

     
xn+2 xn+1

1
|
portanto ∆xn+1 | ≤ |∆x | portanto a sequência é convergente. Calculamos seu limite lim x
2 n n =a

1 2
a= 1+
a
⇔ a −a−1= 0
cujas raı́zes s˜
ao 1±√5, ficamos com a raiz positiva pois a sequência é de termos positivos, logo
2

1+ 5

lim xn = .
2

154
Exercı́cio 4.21:

Exemplo 20. Estudar a convergˆ encia da sequência xn+1 = 1 + xn com x1 = 1.



A sequência é crescente , poisx 2 = 2 > x1 , supondo x n+1 > xn temos
√x n+1 >
√x ⇒ 1 + √x
n n+1 > 1+
√x ⇒ x
n n+2 > xn+1 .

A sequência é limitada superiormente, por 3, por exemplo, pois x1 < 3, supondo x n < 3 < 4 tem-se
√x n <2 ⇒ 1 + √x n <3 ⇒x n+1 < 3.

Agora calculamos o limite da sequência

a= 1+
√a ⇒ (a − 1) 2
=a 2
⇒ a − 3a + 1 = 0
cujas ra´
ızes são ±√5 , n˜ao podendo ser 3−√5 que é menor que 1 logo o limite é
3

3+ 5
2 2 2 .

155
Exercı́cio 4.22:

Propriedade 74. (xn ) n˜


ao possui subsequência convergente ⇔ lim |x | = ∞.
n

Demonstraç˜ ⇒
ao. ).
| | ∞
Se (xn ) não possui subsequência convergente então lim xn = .
| | ∞ ∀ | |
Se não fosse lim xn = , existiria A > 0 tal que n0 , existe n 1 > n0 tal que xn1 < A, aplicando o resultado
| |
com n1 no lugar de n0 , existe n2 > n1 tal que xn2 < A e assim constru´ ···
ımos uma subsequência (xn1 , xn2 , )
limitada , que possui uma subsequência convergente , o que é absurdo.
⇐ ).
Suponha
definição por absurdo
de limite infinito,que | | ∞
sabemos xn existe
lim que = n ) possui subsequência convergente, convergindo para a. Por
en(xtal
0 | | ||
que n > n0 implica xn > a +10, por ( xn ) ter subsequência
| − | ⇒| | | |
que converge para a, existe n 1 tal que n > n 1 e n ı́ndice da subsequência, implica xn a < 10 xn < a + 10,
| | || | | ||
podemos tomar ı́ndice da subsequência tal que n > n 1 e n > n2 , logo valeria xn < a + 10 e xn > a + 10 o q ue
é absurdo, portanto (xn ) não pode possuir subsequência convergente.

156
Exercı́cio 4.25:

xn+1
Propriedade 75 (Teste da razão para sequências.). Se xn > 0 ∀n ∈ N e xn ≤ c < 1 para n suficientemente
ao lim xn = 0.
grande ent˜
Demonstraç˜ao. Existe n 0 tal que para k > n0 vale 0 < xxk+1
k
≤ c < 1, aplicamos o produt´orio ∏ n
k=n0 +1 em ambos
, de onde segue
n n
xk+1
0<
xk
≤ c
k=n0 +1 k=n0 +1
n n0
0 < xn+1 < x(n0 +1) c −
como lim cn = 0, tem-se pelo teorema do sanduı́che que lim xn = 0.
| |
Corolário 10. Dada uma sequência de termos n˜ao nulos (xn ), então ( xn ) é uma sequência de termos positivos,
| |
se ela satisfaz a propriedade anterior ent˜ao lim xn = 0 o que implica lim xn = 0.
xn+1
Propriedade 76. Seja (xn ) sequˆ
encia de termos positivos, se xn ≥ c > 1 para n suficientemente grande ent˜ao

lim xn = .
xk+1
ao. Existe n0
Demonstraç˜ ∈ N tal que k > n 0 implica xk ≥ c, onde c > 1. Aplicando o produtório na desigualdade
tem-se n
xk+1
> cn−n0
xk
k=n0 +1
xn0 +1 n
xn+1 > c
cn0
como lim cn = ∞ segue que lim x n = ∞.
| |
Corolário 11. Na propriedade anterior podemos trocar xn por xn onde xn n˜ao se anula, pois ( xn ) e´ uma | |
sequência de positivos.
Corolário 12. Se lim xxnn+1 = a < 1 ent˜
ao para n suficientemente grande vale xn+1
xn ≤ c < 1, logo também vale
lim xn = 0.
Corolário 13. Se lim xxnn+1 = c > 1 a propriedade também se verifica pois existe n0 ∈ N tal que n > n 0 implica
xn+1
xn > a > 1 para algum a.

Propriedade 77.
n!
lim = 0.
nn
n!
ao. Definimos x n =
Demonstraç˜ nn e vale x n > 0, aplicamos a regra da raz˜ao
n
xn+1 (n + 1)! nn n 1
= = =
xn (n + 1)n+1 n! n+1 (1 + n1 )n

o limite é lim xxnn+1 = 1e < 1.


nn cresce mais r´apido que n!

an
Propriedade 78. Para todo a > 0 real temos lim n! = 0.
an xn+1 an+1 n! a
Demonstraç˜ ao. Pelo teste da raz˜ao, definimos xn = n! temos xn > 0 segue xn = (n+1).n!an
= n+1 e temos
lim xxnn+1 = 0, logo lim xn = 0.

A propriedade nos diz que n! cresce mais r´apido que a n .

157
n
Corolário 14. lim an!n = , pois lim an! = 0, isso significa que A > 0
∞ ∀ ∃ n ∈ N tal que n > n ⇒
0 0
n!
an > A, em
especial para A = 1, tem-se n! > an para n suficientemente grande.
Propriedade 79. Se a > 1 e p natural fixo vale
np
lim = 0.
an
np
ao. Definimos x n =
Demonstraç˜ an , vale x n > 0 daı́ podemos aplicar o teste da razão
p
xn+1 (n + 1)p an n+1 1 1
xn
=
an+1 np
=
n a
⇒ lim x n+1 xn =
a
<1
0<

da´
ı o limite é zero.
an np

Corolário 15. Se a > 1, p ∈ N ent˜ao lim np = ∞ pois lim an = 0.
Tal propriedade mostra que a exponencial a cresce muito mais r´apido que n p para n grande.
n

158
Exercı́cio 4.31:

Exemplo 21. Mostrar que


∑n

k=1
kp
1
lim = .
np+1 p+1
Iremos calcular o limite das diferenças do inverso da sequência
p 1
− p 1

p+1 p+1
[ nk ] + ( p + 1)np nk
(n + 1)p+1 np+1 k=0
k
k=0
k
( p + 1)np
lim −
(n + 1)p = lim ∑(  (n + 1)p = lim
∑ (     
(n + 1)p +lim (n + 1)p = p + 1
→0 →p+1
da´
ı n

k=1
kp
1
lim = .
np+1 p+1

159
Exercı́cio 4.33:

Questão digitada errada

Propriedade 80. Se lim xn = ∞ , com x n > 0 ent˜


ao lim(
∏ n
1

k=1 xk )
n
= ∞
n 1
1 1 n
ao. Se lim xn =
Demonstraç˜ ∞ então lim xn = 0 daı́ lim (
xk
) = 0 que implica
k=1
   =yn

n
1
lim 1 =
yn
∞ = lim( xkn ).
k=1

n (2n)! (2n)! xn+1 (2n+2)(2n+1)(2n)! n!


Exemplo 22. Provar que lim n! = ∞. Tomamos xn = n! da´
ı temos xn = (n+1)n! (2n)! =
(2n+2)(2n+1) n (2n)!
(n+1) = 2(2 n + 1) → ∞ logo lim n! = ∞.
n (2n)! 4
Exemplo 23. Mostrar que lim n!nn = e .
Tomamos xn = (2n)!
n!nn , ı xxnn+1 =
da´ 2(2n+1) 1
1 n
n+1 (1+ n )
→ 4
e.

160
Exercı́cio 4.35:


∑ ∑ ∞ ∞

Propriedade 81. Sejam
n=u
an e
n=s
bn s´
eries de termos positivos. Se
n=s
bn = ∞ e existe n0 ∈ N tal que
an+1 bn+1 ∑ ∞
an
≥ bn para todo n > n0 ent˜
ao
n=u
an = ∞.
ao. anan+1
Demonstraç˜ bn+1

bn , Qak ≥ Qb k tomando o produt´orio com k variando de k = n0 + 1 até n − 1 na
desigualdade em ambos lados segue

n 1 −
n 1

Qak = a an
n0 +1
≥ Qbk = b bn , an
n0 +1
≥ ab n0 +1 bn
n0 +1
k=n0 +1 k=n0 +1

pois temos termos positivos, tomando a série temos


∞ ∞
an ≥ ab n0
n0 n=n +1
bn = ∞
n=n0 +1 0

logo a série tende ao infinito por comparação.

161
Exercı́cio 4.36:

Propriedade 82. 1. Sejam duas séries


∑ ∑
ak e bk de termos positivos, se existe lim abkk = a = 0 ent˜
ao ̸ ∑ ak
converge ⇔∑ bk converge .
2. Se lim abkk = 0 ent˜
ao a convergˆ
encia de
∑ bk implica convergência de
∑ ak .
Demonstraç˜
ao. 1. Existe n 0 ∈ N tal que para k > n tem-se 0

a k
0< t < a−ε< 1< a+ε <t 2
b k

como b k > 0 tem-se


t1 bk < ak < t2 bk
aplicamos a soma
∑ n
k=n0 +1 , da´ı
n n n
t1 bk < ak < t 2 bk
k=n0 +1 k=n0 +1 k=n0 +1

usando essa desigualdade temos por comparação que se


∑ ∑ bk converge ent˜
ao
∑ ak converge e se
∑ ak converge
então bk converge.
2. De maneira similar ao item anter ior.
Existe n 0 ∈ N tal que para k > n 0 tem-se

0 ≤ ab k
k
<ε<t2

como b k > 0 tem-se


0 a k < t2 bk
aplicamos a soma ∑ n
k=n0 +1 , da´ı ≤
n n
0 ≤ ak < t 2 bk
k=n0 +1 k=n0 +1

usando essa desigualda de temos por comparação que se


∑ bk converge então
∑ ak converge.

Exemplo 24. Pode valer que


∑ ∑ ak converge, valendo lim
∑ ∑
ak
bk =0e bk n˜ao converge, tome por exemplo a k = k12 ,
bk = k1 , bk n˜ao converge, lim abkk = lim kk2 = lim k1 = 0 e ak converge, logo a rec´
ıproca do item 2 da propriedade
anterior n˜
ao vale.

162
Exercı́cio 4.40:
Prove que para todo a ∈ R, a série
a2 a2
a2 + + + ...
1 + a2 (1 + a2 )2
é convergente e calcule a soma.

Se a = 0, a série tende à 0 trivialmente.


̸
Suponhamos que a = 0. Ent˜ao, segue que
1
< 1.
1 + a2
Assim, pelo exemplo 7 deste capı́tulo, temos que
∞ 1
∞ 1
i
1 1 + a2
= = 1 = .
i=0
(1 + a2 )i i=0
1 + a2 1 − 1+a2
a2

Logo,
∞ a2
∞ 1
= a2 = 1 + a2 .
i=0
(1 + a2 )i i=0
(1 + a2 )i

163
Exercı́cio 4.41:
Para todo p ∈ N fixado, a série
1
n
n(n + 1)...(n + p)
converge.

Para todos n e p ∈Z +, temos que


1 1
< p+1 .
n(n + 1)...(n + p) n
Como p + 1 > 1, temos, pelo exemplo 29, que
1
n
np+1
é convergente. Logo, conclu´ımos de
i i
1 1
0< <
n=1
n(n + 1)...(n + p) n=1 np+1

que a série
1
n
n(n + 1)...(n + p)

é limitada (e monotona). Portanto,


∑ 1
n n(n+1)...(n+p) converge.

164
Exercı́cio 4.42:
Se
∑ an converge e a n > 0 então
∑ ∑(an )2 e 1
1+an convergem.

Como
∑ an e´ convergente, temos que lim n→∞ an = 0. Assim, como também temos que an > 0 para todo
n ∈Z + , existe n 0 Z+ tal que

0 < an < 1,
para todo n  n0 . Logo, para n  n0 , temos que

0 < a2n < an .

Assim, i i
a2n  an ,
n=n0 n=n0

para todo i ∈Z +, e, como existe


∑ ∞
n=n0 an , temos que existe
∑ ∞
n=n0 a2n . Logo, existe

∞ n0 1
− ∞
a2n = a2n + a2n .
n=1 n=1 n=n0

Como a n > 0, para todo n ∈Z +, temos que


an
< a n,
1 + an
para todo n ∈Z +. Assim,
i i
an
< an ,
n=1
1 + an n=1

para todo i ∈Z ∞ a existe, temos que existe ∞ an


+, e, como n=1 n n=1 1+an .

∑ ∑

165
Exercı́cio 4.43:
Se Σ( an )2 converge então Σ ann converge.

Sejam
Z<
+ := {n ∈ Z ; |a | < 1/n}
+ n

e
Z
+ := {n ∈ Z ; |a |  1/n}.
+ n

Então, Z + = Z<
+ ∪Z + . Além disso, para cada n ∈Z +, temos que

an 1 2
|n| < n 2 + an .
<
De fato, se n ∈Z +, temos que
|a | < n1
n

implicando que
|a | <
n 1

1
+ a2n .
n n2 n2

Se n ∈Z + então temos que
1
n
 |a | n

e, consequentemente,
|a |  a
n 2
<
1
+ a2n .
n
n n2
Assim, temos que
i i i
|a | <
n 1
+ a2 ,
n=1
n n=1
n2 n=1 n

para todo i Z+ . Pelo exe mplo 29 e pela hipt ´otese, temos que

desigualdadade acima, devemos ter que ∞

| an |
∑ ∞ 1
n=1 n2 e ∑ ∞ a2n existem. Assim, pela
n=1

n=1 n existe.
Como ∞ a
n
=
∞ a
n
  | |
n=1
n n=1
n

existe, temos que


∑ ∞ an
n=1 n também deve existir.

166
Exercı́cio 4.44:
Se (an ) é decrescente e Σan converge então lim nan = 0.

Primeiramente observemos que como ( an ) é decrescente e existe


∑ ak , então an  0 para todo n ∈Z +. De
fato, se a n0 < 0 para algum n 0 Z+ , ter´
∈ ıamos que
n0 +i n0 +i
ak  an0 = (i + 1)an0 < 0
k=n0 k=n0

e, consequentemente,
n0 +i
  ak  | |
(i + 1) an0 ,
k=n0

para todo i Z+ . Contradizendo o fato de


∈ ∑
ak existir.
Assim, temos que (Σ nk=1 ak )n∈Z+ é uma sequência crescente de Cauchy. Em especial,

n+p
ak −
n
ak =
 n+p
ak ,
 k=1 k=1
 k=n+1

para todos n e p Z+ .

Seja ε > 0 arbitrário. Como (Σ nk=1 ak )n∈Z+ é de Cauchy, existe n 0 ∈Z + tal que para todo n  n0 vale

ε
>
 n
ak −
n0
ak =
 n
ak (n − n )a
 0 n.
2 
k=1 k=1
 k=n0 +1

Por outro lado, como



an é convergente, devemos ter que lim n→∞ an = 0 e, por conta disso, existe n 1 ∈Z + tal
que para todo n  n1 temos que
ε
> an
2n0
e, consequentemente,
ε
> n 0 an .
2
{ }
Assim, para n > n 2 = max n0 , n1 , temos que
ε ε
ε= + > (n
2 2
− n )a 0 n + n0 an = na n = nan . | |
Portanto, concluimos que limn→∞ nan = 0.

167
Exercı́cio 4.45:
Se (an ) é decrescente e Σan = + , então,

a1 + a3 + · · · + a2n−1
lim
n →∞ a2 + a4 + · · · + a2n = 1.

Como (an ) é decrescente, temos que


a2n−1  a2n ,
para todo n ∈Z +. Assim, para todo n ∈Z +, temos que

a1 + a3 + ···+ a 2n 1 
− a2 + a4 + ··· + a 2n

e, consequentemente,
1
a1 + a3 + ··· + a − 2n 1
=: q n .
a2 + a4 + ··· + a 2n
Provaremos agora que para todo c > 1 existe n0 tal que qn < c para todo n  n0 . Daı́ conclui-se que
limn→∞ qn = 1. Como quer´ıamos demonstrar.
Suponhamos, por contradição, que exista c > 1 e uma subsequência (qnk ) de ( qn ) tais que

qnk  c

para todo k Z+ .

Então temos, para todo k ∈Z +, que

a1 + a3 + ··· + a −
2nk 1  c(a2 + a4 + ··· + a 2nk )
 c(a3 + a5 + ··· + a 2nk +1 )

e, consequentemente,
a1  a1 ca2nk +1
− 1)(a + a + · · · + a − ).
 (c 3 5 2nk 1

E chegamos a um absurdo. De fato, como Σ a = ∞, existe k ∈ Z tal que


n +

2a 1
+ a < a +a +a +·+a 1 1 2 3 2nk +1
c−1
e, consequentemente,
2a1
− + a1 < a1 + (a2 + a3 ) + (a4 + a5 ) + · · · + a2n + a2n
c 1 k k +1

 a1 + 2a3 + 2a5 + · · · + 2a 2nk +1


 a1 + 2(a3 + a5 + ··· + a 2nk +1 ).

De onde temos que existe k ∈Z + tal que

a1 < (c − 1)(a 3 + a5 + ··· + a 2nk +1 ).

168
Exercı́cio 4.46:
Seja (an ) uma sequência não crescente, com lim an = 0. A série
∑ an converge se e somente se
∑ 2n a2n converge.

Como a sequência (an ) é n˜


ao cresente, devemos ter que a n  0, para todo n ∈Z +, caso
∑ ∑
an ou 2n an sejam
convergentes. De fato, se a k0 < 0, temos que
k k0 1
− k k0 1
− k

k
lim
→∞ n=1
an = an + lim
k →∞ n=k
an  an + lim
n→∞ n=k
ak0 = −∞
n=1 0 n=1 0

e, analogamente,
k
lim 2n a2n = −∞.
k →∞ n=1
Em especial, temos, do fato de ( an ) ser uma sequência de termos não negativos, que ∑an e 2n an são
∑ ∑
sequências crescentes. Mostraremos, adiante, que se
∑ an é uma sequência limitada (i.e. convergente) então
2n an também é limitada e vice-versa.

Suponhamos que an seja convergente. Como (an ) é n˜
ao crescente, temos que
2n 1− 2n 1−
2n a2n = 2 a2n  2 ak ,
k=2n −1 k=2n − 1

para todo n ∈Z +. Assim, para todo p ∈Z +, temos que


p p 2n 1− 2p 1
− 2p 1

2n a2n  2 ak  2 ak  lim 2 ak < ∞.
p →∞
n=1 n=1 k=2n − 1 k=1 k=1

Assim, temos que



2n a2n é uma sequência crescente e limitada. Portanto, 2n a2n é convergente.

Suponhamos que 2n a2n seja convergente. Como (an ) é n˜ao decrescente, temos, para todo k Z+ , que ∈
∑ 2k+1
−1
an 
2 k+1
−1
a2k = 2 k a2k .
n=2k n=2k

Assim, dado p ∈Z +, temos, para q ∈Z + tal que p  2q , que

p 2q q 2k+1 1− q q
an a1 + an = a 1 + an  a1 + 2k ak a1 + lim k

n=1

n=1 k=1 n=2k k=1

q →∞ k=1 2 ak < .

Logo,
∑ an é uma sequência crescente e limitada. Portanto,
∑ an é convergente.

169
Exercı́cio 4.47:
Prove que o conjunto de valores de aderência da sequência x n = cos(n) é o intervalo fechado [0, 1].

Seja x ∈ [0, 1]. Como cos : R [0, 1] é sobrejetivo1 , existe θ


→ ∈ R tal que cos θ = x. Encontraremos uma
sequência crescente (nk )k∈Z+ em Z + tal que

x = cos θ = lim cos nk = lim xnk .


k Z+∈ ∈
k Z+

Assim, como x e´ arbitrário, poderemos concluir que o conjunto de valores de aderência da sequência (xn ) é o
intervalo [0, 1].
Seja
G := n + m2π; n e m Z .
{ ∈ }
ıcio 3.58, temos que G é denso em R .
Pelo Exerc´
Para todo k Z+ , temos que existe θ k G tal que
∈ ∈
θ − 1/k < θ k < θ.

Desta forma, temos que


lim θk = θ.
k →∞
Como 2 π é irracional, a sequência (θk )k∈Z+ define unicamente sequências (ñk )k∈Z+ e (mk )k∈Z+ em Z pela
igualdade
θk = ñk + mk 2π,
para todo k Z+ . Por fim, definimos a sequência (nk )k∈Z+ por

| |
nk = ñk ,

para todo k Z+ . Assim, como lim k→∞ θk = θ, temos que

limk→∞ cos(nk ) = lim k→∞ cos(nk ) = lim k→∞ cos(ñk )



= lim k→∞ cos(θk mk 2π) = lim k→∞ cos(θk )
= cos( θ).

{
O conjunto nk ; k ∈ Z } e´ infinito.
+ Caso contrário, o conjunto {cos(n );k k ∈ Z } é finito e, assim, existiria
+
k Z+ tal que

cos θk = cos θ.
Consequentemente, θ k = θ + m2π, para alguns k e m Z. Porém, como θ 1/k < θk < θ, temos uma contradição.
∈ −
Logo, (nk )k∈Z+ é uma sequência ilimitada em Z+ . Assim, ( nk )k∈Z+ admite uma subsequência crescente. Em outros
termos, uma subsequência que também é subsequência de (n)n∈Z+ . Denotaremos tal subsequência também por
(nk )k∈Z+ .
Portanto, temos que a subsequência (xnk ) é tal que

lim xnk = lim cos(nk ) = cos( θ) = x.


k →∞ k →∞

1
Isso pode ser verficado utilizando a continuidade da fun¸ cão cos e o teorema do valor intemedi´ario. Mas, como o Elon assume que
podemos usar os nossos conhecimentos prévios sobre as funções trigonométricas... Vamos tomar a afirma¸ cão como fato.

170
Exercı́cio 4.48:
Sejam a e b números reais positivos. Defina indutivamente as sequˆ
encias (xn ) e (yn ) pondo x1 = ab, y1 = (a+b)/2,


xn+1 = xn yn e yn+1 = (xn + yn )/2. Prove que xn e yn convergem para o mesmo limite, chamado de a média
aritmético-geométrica entre a e b.

ıpio da Definição Indutiva, podemos as sequências (xn )n∈Z0 e (yn )n∈Z0 em R + por
Pelo Princ´

x0 := a, y0 := b, xn+1 :=
√x n yn e yn+1 :=
xn + yn
.
2
Se a = b temos que x n = yn = a, para todo n ∈Z 0 . De fato, se tomamos como hip´otese de indu¸cão que

xn = y n = a
então √
xn+1 =
√x n yn = a2 = a
e
xn + yn 2a
yn+1 = = = a.
2 2
Assim, como x0 = a = b = y0 , temos, pelo Princı́pio da Indução Finita, que xn = yn = a, para todo n ∈Z 0 .
Desta forma,
lim xn = a = lim yn .
n →+∞ n →+∞
Portanto, nos resta mostrar o resultado para o caso em que a < b. Suponhamos, até o fim desta demonstração,
que
b a > 0. −
Primeiramente, provaremos as desigualdades
xn < xn+1 < yn+1 < yn , (4.1)

para todo n ∈Z 0 , por indu¸


cão em n. Suponhamos que, para algum n fixo, D := yn −x n > 0. Então,

x2n < xn yn = x 2n+1 ,

x2n+1 = xn yn
= xn (xn + D)
D2
< x2n + xn D + 4
(( 
= xn + D 2
xn +yn 2
2

= 2
2
= yn+1
e
xn + yn D
yn+1 =
2
= yn
2
= yn. −
Logo, se xn < yn , temos ( 4.1). Como x0 = a < b = y 0 , temos, pelo Princı́pio da Indução Finita, que ( 4.1) vale
para todo n Z0 .

Das desigualdades (4.1) segue que as sequências (xn ) e (yn ) são monótonas e limi tadas. Logo, exist em
limn→+∞ xn e lim n→+∞ yn .
Provaremos, agora, que
lim (yn xn ) = 0.
n →+∞

Usando ( 4.1), temos que
2
(yn −x n) = (yn + xn )2 4xn yn−
2
= 4yn+1 4x2n+1

2
= 4(yn+1
( x2n+1 )
− 
= 4 (yn+1 xn+1 )2 + 2yn+1 xn+1
− − 2x 2
n+1
= 4(yn+1 xn+1 )2 + 8xn+1 (yn+1
− −x n+1 )
> 4(yn+1 xn+1 )2 .

171
e, consequentemente,
yn −x n
> yn+1 −x n+1 .
2
Em particular,
b −a = y −x 0 0
> y1 −x . 1
2 2
Supondo que
b −a >y −x n n,
2n
como hipótese de indu¸cão, temos que
b −
a
>
yn xn
> y n+1
− xn+1 .
n+1
2 2 −
Assim, pelo Princ´ıpio da Indução Finita, temos que
b −a >y −x n n | − x |,
= yn n
2n
para todo n ∈Z +. Assim, pelo Teorema do Confronto (Teorema 8 do Capı́tulo 4), temos que

lim |yn − xn | = 0.
n → +∞

Logo, pelo exercı́cio 4.1, temos que

n
lim yn
→ +∞
− n
lim xn = lim (yn − xn ) = 0.
→ +∞ n→+∞

E o resultado segue.

172
Exercı́cio 4.49:
n 1
Sejam a1  a2  ···
 0 e sn = a1 −a 2 + · · · + (−1) − a . n Prove que a sequência ( sn ) é limitada e que

lim sup sn lim inf sn = lim an .

Valem as desigualdades
s2k  s2(k+1)  s2(k+1)+1  s2k+1 , (4.2)
para todos k ∈Z +. De fato, para todo k ∈ Z , temos que
+

s2k = a − a + ··· + a − − a
1 2 2k 1 2k
 a − a + ··· + a − − a
1 2 2k 1 2k + a2k+1 −a 2(k+1)

= s2(k+1) ,
s2(k+1) = a1 a2 + − ··· + a 2k+1 −a 2(k+1)
 a1 a2 + − ··· + a 2k+1 −a 2(k+1) + a2(k+1)+1
= s2(k+1)+1,
e
s2(k+1)+1 = −
a1 a2 + ··· + a 2k+1 −a 2(k+1) + a2(k+1)+1
 −
a1 a2 + ··· + a 2k+1
= s2k+1 .
Segue, das desigualdades ( 4.2), que a sequência (s2k )k∈Z+ e´ n˜ao decrescente e limitada e que a sequência
(s2k+1 )k∈Z+ e´ não crescente e limitada. Em especial, temos que

lim s2k = sup s2k e lim s2k+1 = inf s2k+1 .


k →+∞ ∈
k Z+ k →+∞ k Z+

Tamb´
em valem as desigualdades
s2k  sn  s2k+1 , (4.3)
para todo k Z + e n  2k + 1 em Z + . De fato, se n  2k + 1, temos que n = 2p ou 2p + 1 para algum p
∈  k.
Assim, como ( s2k )k Z+ e´ não decrescente, (s2k+1 )k Z+ e´ não crescente e por ( 4.2), temos que
∈ ∈
s2k  s2p  s2p+1  s2k+1 .

Logo, obtemos ( 4.3). Em especial, concluimos que ( sn )n∈Z+ é limitada.


Como as sequências (s2k )k∈Z+ e (s2k+1 )k∈Z+ são mon´otonas e limitadas, temos que exist em lim k→∞ s2k e
limk→∞ s2k+1 . Provaremos que

lim inf sn = lim s2k e lim sup sn = lim s2k+1 .


k →∞ k →∞
Como a sequência (an )n∈Z+ é monotona e limitada, temos que esta é convergente. Daı́, segue das igualdades acima,
que

lim sup sn lim inf sn = lim k→+∞ s2k+1 limk→+∞ s2k −
= lim k→+∞ (s2k+1 s2k ) −
= lim k→+∞ a2k+1
= lim n→+∞ an .
Suponhamos que A seja um valor de aderência de (sn )n∈Z+ . Então, devemos ter que

s2k  A  s2k+1 ,

para todo k ∈Z +. De fato, se s 2k+1 < A, para algum k ∈Z +, temos que

sn  s2k+1 <A

para todo n  2k + 1 em Z + , por ( 4.3). Isto é,

|A − s |  |A − s
n 2k+1 |,

173
para um k Z+ fixo e todo n  2k + 1 em Z + . E isso contradiz o fato de A ser um valor de aderência de (sn )n∈Z+ .

Analogamente, prova-se que é impossı́vel se ter A < s 2k , para algum k Z+ . Portanto, devemos ter que

lim s2k = sup s2k  A  inf s2k+1 = lim s2k+1 .
k →∞ k Z+
∈ ∈
k Z+ k → +∞
Em particular, os valores de aderência lim inf sn e limsup sn satisfazem

lim s2k  liminf sn  limsup sn  lim s2k+1 .


k →∞ k →+∞
Logo, como lim inf sn e lim sup sn são, respectivamente, o menor e o maior valor de aderência de (sn )n∈Z+ , temos
que
lim s2k = lim inf sn e lim s2k+1 = lim sup sn .
k →∞ k →+∞

174
Capı́tulo 5

Topologia da Reta

175
Exercı́cio 5.01:

Um conjunto A é aberto se, e somente se, cumpre a seguinte condição: “se uma sequência (xn ) converge para
∈ ∈
um ponto a A então x n A para todo n suficientemente grande”.


( ) Seja ( xn )n∈N uma sequência que tende à a ∈ (b, c) ⊂ A. Existe n ∈ N tal que n ≥ n
0 0 implica que, para
{ − − }
ε = min a b, c a ,
|x − a| < ε,
n

ou seja,
xn ∈ (a − ε, a + ε) ⊂ (a, b) ⊂ A.
1 1
⇐ ∈ ∈ ⊂
( ) Se a A e a (b, c) A, para n suficientemente grande a
1 1
∈ (a − n , a + n ) ⊂ (b, c) ⊂ A. Temos então que

a intA se e somente se ( a , a + ) A para algum n N.
− ⊂ ∈
n n
Por hipótese, existem n − e n + N tais que

a − n1 ∈ A, ∀n ≥ n−
e
1
a+
n
∈ A, ∀n ≥ n +.

{ }
Assim, tomando-se n 0 = max n− , n+ , temos que

(a − n1 , a + n1 ) ⊂ A.
0 0

Logo, a ∈ intA.

176
Exercı́cio 5.02:
Tem-se que lim xn = a se, e somente se, para todo aberto A contendo o ponto a, existe n0 ∈ N tal que n > n
0

implica x n A.


( ) Repetir a ideia do exercı́cio anterior.
( ) Dado ε > 0, existe n 0 N tal que, para todo n
⇐ ∈ ≥ n , temos que
0

xn ∈ (a − ε, a + ε),
ou seja,
xn a < ε.
segue daı́ que limxn = a. | − |

177
Exercı́cio 5.03:
Seja B ⊂ ∈
aberto. Então, para todo x , o conjunto x + B = x + y; y{ ∈ B} e´ aberto. ̸
Analogamente, se x = 0,
{ ∈ }
então o conjunto x.B = x.y; y B e´ aberto.

Seja B ⊂ ∈
aberto. Dado y x + B, temos que y = x + b para algum b ∈ B. Como B e´ aberto, existe ε > 0 tal
− ⊂
que (b ε, b + ε) B. Desta forma, temos

(x + b − ε, x + b + ε) = (y − ε, y + ε) ⊂ x + b.
De fato, seja z ∈ (y − ε, y + ε) = (x + b − ε, x + b + ε). Então

x+b −ε<z<x +b+ε


b − ε < z − x < b + ε.
− ∈ ∈
Logo, z x B e consequentemente z x + B, como quer´ ıamos.
̸ ∈
De maneira an´aloga, temos que se x = 0, então x.B e´ aberto. Seja y x.B. Então y = x.b para algum b ∈ B.
− ⊂ ||
Como B é aberto, existe ε > 0 tal que ( b ε, b + ε) B. Desta forma, temos que para δ = ε. x > 0, temos

(y − δ, y + δ) ⊂ x.B.

178
Exercı́cio 5.04:
{
Sejam A e B abertos. Ent˜ao os conjuntos A + B = x + y; x ∈ A, y ∈ B } e A.B = {x.y; x ∈ A, y ∈ B } são abertos.
∪ ∪
Como A + B = a∈A (a + B) e A.B = a∈A (a.B) e cada a + B e a.B são abertos (pelo exercı́cio anterior),
temos que A + B e A.B são uniões de abertos. Logo, também são abertos.

179
Exercı́cio 5.05:
⊂ ∩
Para quaisquer X, Y , tem-se int(X Y ) = int(X ) ∩ int(Y ) e int(X ∪ Y ) ⊃ int(X ) ∪ int(Y ). Dê um exemplo em
que a inclus˜ao não se reduza a uma igualdade.


int(X ∩ Y ) ⊂ int(X ) ∩ int(Y ).
Dado w ∈ int(X ∩ Y ), existem a, b ∈ tais que w ∈ (a, b) ⊂ X ∩ Y . Da´ı, temos que w ∈ (a, b) ⊂ X e

w (a, b) ⊂ Y. Assim, w ∈ int(X ) ∩ int(Y ).
• ∩ Y ) ⊃ int(X ) ∩ int(Y ).
int(X
Dado w ∈ ∩
int(X ) int(Y ), existem ax , ay , bx , by ∈
tais que w ∈ (a , b ) ⊂ X e w ∈ (a , b ) ⊂ Y. Então
x x y y
{ } { } ∈ ⊂ X ∩ Y. Assim, w ∈ int(X ∩ Y ).
tomando a = max ax , ay eb = min bx , by , temos que w (a, b)

int(X ∪ Y ) ⊃ int(X ) ∪ int(Y ).
∈ ∪
Dado w int(X ) int(Y ), existem ax , ay , bx , by ∈ tais que w ∈ (a , b ) ⊂ X e w ∈ (a , b ) ⊂ Y. Então,
x x y y
∈ ⊂ ∪ ∈
w (ax , bx ) X Y. Donde w int(X Y ). ∪
• Sejam X = [−1, 0) e Y = [0, 1]. Então,

int(X ) = ( 1, 0)
int(Y ) = (0, 1)

int(X ) ∪
∫ − \ { 0}
(Y ) = ( 1, 1)

int(X ∪ Y ) = (−1, 1).

180
Exercı́cio 5.06:
Se A ⊂ é aberto e a ∈ A então A\{a} e´ aberto.
Seja x ∈ A \ {a}. Existe ε > 0 tal que x ∈ (x − ε, x + ε) ⊂ A. Tomando ε̃ = min {|x − a |, ε} teremos que
x ∈ (x − ε̃, x + ε̃) ⊂ A \ {a}. Logo, x ∈ int(A \ {a}). Segue da´ı que A \ {a} e´ aberto.

181
Exercı́cio 5.07:
Considere as fun¸cões f,g,h : , dadas por f (x) = ax + b (a = 0), g(x) = x 2 e h(x) = x 3 . Mostre que, para cada
→ ̸
A , f −1 (A), g−1 (A) e h −1 (A) são abertos.

Dado um intervalo (c1 , c2 ) ⊂ . Temos que
 (
c1 − b , c − b ),
2
a> 0
a
f −1 ((c1 , c2 )) =
 (
c2 − b , c a− b ),
1
a< 0
a a

g −1 ((c1 , c2 )) =  −√
−−√√cc ),∪√(c√),c , √c ), 0c ≤<c0 <≤ cc
(
∅,
c2 , ( 1
2
c < c ≤0
2
1 2
1
1
1

2
2
2

1 √ √
h− ((c , c )) = ( c , c ).
1 2
3
1
3
2

Assim, para cada A ⊂ aberto, temos que A = ∪I , I intervalo aberto, e


λ λ

f − (A)] ∪ f − (I ), g − (A)] ∪ g − (I ) e h − (A)] ∪ h− (I ).


1 1
λ
1 1
λ
1 1
λ

E como f −1 (Iλ ), g −1 (Iλ ) e h −1 (Iλ ) são abertos, temos que f −1 (A), g−1 (A) e h −1 (A) também o são.

182
Exercı́cio 5.08:
No exercı́cio anterior, mostre que, para cada A ⊂ aberto, f (A) e h(A) são abertos. Dê exemplo de A aberto tal
que g(A) não seja aberto.

Seja A aberto em e a ∈ A. Então, existem c , c ∈ tais que a ∈ (c , c ) ⊂ A. Da´ı,


1 2 1 2

(ac1 + b,ac 2 + b), a > 0


f ((c1 , c2 )) =
(ac2 + b,ac 1 + b), a < 0

h((c1 , c2 )) = ( c31 , c32 ),

com f (a)
mesmo ∈ f ((c1h.
vale para , c2Logo,
)) e h(a)∈
f (A) eh((c
h(A), csão
1 2 )). Segue daı́ que dado um b = f (a)
abertos. ∈ f (A) qualquer, b ∈ intf (A). E o

Exemplo: g(( 1, 1)) = [0 , 1) não é aberto.

183
Exercı́cio 5.09:
Toda coleção de abertos n˜ao-vazios, dois a dois disjuntos é enumerável.

{ }
Seja Aλ λ∈Λ uma coleção de abertos disjuntos. Para cada λ ∈ Λ escolhemos r ∈ A ∩ Q. Como A ∩ A
λ λ λ µ ∅
= ,
̸
para λ = µ, a função
Λ −→Q
λ →rλ
é injetiva. Segue da´ı que Λ é finito ou enumer´
avel.

184
Exercı́cio 5.10:
O conjunto dos valores de aderência de uma sequência é um conjunto fechado.


Seja A o conjunto dos valores de aderência da sequência (xn )n∈N e a um ponto de aderência de A. Existe uma

sequência (ak ) em A que tende `a a. E, para cada a k (ak ), existe uma subsequência (x(k,n) ) de ( xn ) que tende `a
ak .
Definimos uma sequência (x(0,n) ) de ( xn ). Para cada n N, existe k 0 N tal que
∈ ∈
1
|a − a |
k0 <
2n

e n0 ∈ N tal que |a − x | < 2n1 .


k0 (k0 ,n0 )

Assim, tomando-se x ( 0, n) = x(k0 ,n0 ) teremos que

|a − x | < n1 .
(0,n)

Segue da´
ı que a = lim x(0,n) e, consequêntemente, a ∈ A. Portanto, A e´ fechado.

185
Exercı́cio 5.11:
Se X ⊂ F e F e´ fechado então X̄ ⊂ F.

Para cada x X̄, existe uma sequência em X que tende `a x. Logo, x ∈ F̄ = F já que ( x n) é uma sequência em
F. Segue da´ ⊂
ı que X̄ F.

186
Exercı́cio 5.12:
{
Se lim xn = a e X = x1 , x2 ,...,x n ,... } então X̄ = X ∪ {a}.
Segue diretamente da defini¸cão que X̄ ⊃ ∪{ }
X ∈
a . Seja b X̄ e (yn ) uma sequência em X que tende `a b.
Consideremos o conjunto n N; xn
{ ∈ ∈{ }} ∈
y1 , y2 ,...,y k ,... = I . Se I é finito , então b = x n para algum n I . Caso
I seja infinito, a subsequência y ki = n n tal que n = min n N ; xn
{ ∈ ∈{ }}
yki +1 ,...,y k de (yk ) é uma sequência de
(xn ). E como lim xn = a, temos que b = lim yki = a. Segue da´ ⊂ ∪{ }
ı que X̄ a .

187
Exercı́cio 5.13:
1
O número 4 pertence ao conjunto de Cantor.

O conjunto K de Cantor é dado por


K= ∩∞
n=0 Kn ,
1
com K n+1 sendo obtido de K n retirando-se de cada intervalo de K n um subintervalo de comprimento n+1 de seu
3
centro. p
2
Temos que 2 é o ı́nfimo de um intervalo de K2 . Supondo que
3 n=1 3

2
2n
é o ı́nfimo de um intervalo de K2p ,
p 2 2
temos que n=1 32n + 32(p+1) é o ı́nfimo de um intervalo em K2(p+1) . Segue daı́ que cada somatório reduzido da
∑ ∑
série
2
32n
pertence à K.
E como
∞ 1
n=1
= 2
1 − −1 1
9
9
= 2
8
− 1 = 14 ,
1
temos pela compacidade de K que
4
∈ K.

188
Exercı́cio 5.14:
Sejam F e G conjuntos fechados disjuntos tais que F ∪ G seja um intervalo fechado (limitado ou não). Então F = ∅

ou G = .


Sejam F e G conjuntos fechados tais que F G seja um intervalo fechado.
Suponhamos que existam x ∈F e y ∈G. Suponhamos, sem perda de generalidade, que x < y. Assim,
⊂ ∪
[x, y] F G.
∩ ∩ ∈ ∩ ∈
Seja m = inf G [x, y]. Como G [x, y] é fechado, m G [x, y] e, consequêntemente, m G. Por outro lado,
⊃ ⊃
F [x, m) e, consequêntemente, F = F̄ [x, m) = [x, m]. Segue da´ı que m G F. ∈ ∩
∩ ∅
Portanto, se tivéssemos F G = , dever´ ∅
ıamos ter também que F = ou G = . ∅

189
Exercı́cio 5.15:

Se E enumerável. Consiga uma sequência cujo conjunto dos valores de aderência éĒ. Use este fato para mostrar

que todo conjunto fechado F e´ o conjunto dos valores de aderência de alguma sequência.

Seja E = ak k∈N . Definimos N k = 2k−1 .m N; m e´ ı́mpar e, para cada n N, xn = a k onde n Nk .


{ } { ∈ } ∈ ∈
Assim, ak = lim xn e, consequentemente, o conjunto dos valores de aderência da sequência (xn ), A, contém
n Nk∈

E. Dado a Ē, existe uma sequência (aki )i∈N em E tendendo à a. Por sua vez, a subsequência (x2ki 1 )i∈N e´ igual


a sequência (aki )i∈N . Logo, a A. Concluı́mos que A ⊃Ē. Por outro lado, A ⊂
Ē já que a sequência (xn ) ( e
consequentemente todas as suas subsequências) pertence à E . Portanto, A = Ē.

Dado um conjunto fechado X , pelo Teorema 6, existe um conjunto enumerável E denso em X. Por conseguinte,
existe uma sequência (xn ) cujo conjunto dos valores de aderência é Ē = X (pois X é fechado).

190
Exercı́cio 5.16:
Com a nota¸cão do Exercı́cio 4, se α e´ irracional, os conjuntos F Z e G = αZ são fechados porém F + G não é
fechados. Tamb´ { }
em H = 0, 1, 1/2,..., 1/n, ... e´ fechados mas F.H não é fechado.

Os conjuntos F e G são, respectivamente, complementares dos conjuntos abertos


∪ ∪
(n, n + 1) e (αn,α (n + 1)).
n Z
∈ n Z

Logo, F e G são fechados.
Temos que F + G = Z + αZ é um grupo aditivo em R . Assim,
(i) inf([ G + F ] ∩ (0, ∞)) = a > 0.
Neste caso, pelo exerc´ 1 F + G = {0, ±a, ±2a,... } = a Z. Da´
ıcio 3.58, terı́amos que ı, F + G = a Z ∋ ak = α ∈
Z + αZ = F + G e, consequentemente, a = ∈ Q. Por outro lado, F + G = a Z ∋ ap = α ∈ Z + αZ = F + G,
k
para algum p ∈ Z, e consequentemente, α = ap ∈ Q. Contradição.
Portanto, inf([F + G] ∩ (0, ∞)) = 0.
(ii) inf([ F + G] ∩ (0, ∞)) = 0.
1
Novamente pelo exercı́cio 3.58, temos que F +G é denso em R. Por outro lado, / Z +αZ, pois α é irracional.

2
Logo, F + G  F + G = R. Ou seja, F + G não é fechado.
O conjunto H é fechado, pois é o complementar do conjunto (1, ∞) ∪ ( ∪
n N

( n1 , n+1
1
)). Por outro lado, F .H =
Q  Q = R. Ou seja, F.H não é um fechado.

191
Exercı́cio 5.17:
Seja K o conjunto de Cantor. Mostre que {|x − y|; x ∈ K, y ∈ K } = [0, 1].
Não é difı́cil provar que toda extremidade de subintervalo em cada passo da constru¸
cão do conjunto de Cantor
é do tipo
p
an
3n
[0, 1] ∈
n=0

com a n = 0 ou 2 , 0 ≤ n < p, a p = 0, 1 ou 2 .
Dado p
cn
n=0
3n ∈ [0, 1], (5.1)
com c n = 0, 1 ou 2 , definiremos a, b ∈ R tais que a, b ∈ K e c = b − a. Sejam
p p+1
an bn
a= eb= :
n=0
3n n=0
3n

•c 0 = 0 ou 1
a0 = c 0 e b 0 = 0.

•c 1
c1 s 1 a1
3
+ 2 =
3 3
− b3 − b9
1 2
(5.2)

c1 = 0
a1 = b 1 = b 2 = 0, s1 = 0
c1 = 1
a1 = 2, b1 = 0, b2 , s1 = 1

c1 = 2 a1 = 2, b1 = 0, b2 = 0, s1 = 0

•c n, 1 ≤n <p
cn sn an
+
3n 3n+1
= n
3
− 3bn+1
n+1
+
s n −1
3n
(5.3)

cn = 0
sn−1 = 0
an = 0, bn+1 = 0, sn = 0
sn−1 = 1
an = 0, bn+1 = 2, sn = 1

cn = 1
sn−1 = 0
an = 2, bn+1 = 2, sn = 1
sn−1 = 1
an = 0, bn+1 = 0, sn = 0

cn = 2
sn−1 = 0
an = 2, bn+1 = 0, sn = 0
sn−1 = 1
an = 2, bn+1 = 2, sn = 1

192
•c p
cp ap
3p
= p
3
− 3b p+1
p+1
+
s p−1
3p
(5.4)

cp = 0
sp−1 = 0
ap = 0, bp+1 = 0
sp−1 = 1
ap = 1, bp+1 = 2

cp = 1
sp−1 = 0

ap = 1, bp+1 = 0
sp−1 = 1
ap = 0, bp+1 = 0

cp = 2
sp−1 = 0
ap = 2, bp+1 = 0
sp−1 = 1
ap = 1, bp+1 = 0

Assim, somando-se as equações (5.2), (5.3) e (5.4) obtemos que c = b a e a, b K. − ∈


{| − | ∈ }
O conjunto D = x y ; x, y K contém todos os elementos do tipo (5.1). E como o conjunto de todos os
elementos do tipo ( 5.1) é denso em [0, 1] teremos o resultado assim que provarmos que D e´ fechado.
2n

Seja K n =
i=1

[ai , bi ] o conjunto resultante da n ésima etapa da construção do conjunto de Cantor. O conjunto
Dn = {|x − y|; x, y ∈ K } é dado por
n

Dn = Dnij , onde D nij = {|x − y|; x ∈ [a , b ], y ∈ [a , b ]},


i i j j
n
1 i,j 2
≤ ≤


é fechado, pois cada D nij e´ fechado. Assim, D = Dn também é fechado.
n=1

193
Exercı́cio 5.18:
Dado qualquer n´umero real a > 0, existe x 1 , x2 ,...,x n no conjunto de Cantor tais que x 1 + x2 + ... + xn = a.

Dado a > 0 em R podemos escrever a na forma


∞ a
n
k+
n=1
3n

∞ an
com k ∈Nea n = 0, 1 ou 2 . Se provarmos que
n=1 3
n
pode ser escrito como soma de dois elementos α, β ∈ K,
temos o resultado definindo x 2 = x 2+k = 1, x1 = α e x 2 = β.
É pos´ıvel demonstrar que
K=
cn
{

; cn = 0 ou 2 . }
3n
Assim, definiremos α n e β n tais que
αn βn
α= eβ= .
3n 3n
Definimos
•a 1 tal que
a1 α1 β 1 s 1
= + + (5.5)
3 3 3 3
a1 = 0
α1 = β 1 = 0, s1 = 0
a1 = 1
α1 = β 1 = 0, s1 = 1
a1 = 2
α1 = 2, β1 = 0, s1 = 0

•a n, n > 1, tal que


an sn−1 αn β n s n
+ n −1 = n + n + n (5.6)
3n 3 3 3 3
an = 0 sn−1 = 0 αn = β n = 0, sn = 0 sn−1 = 1 αn = 2, βn = 0, sn = 1
an = 1 sn−1 = 0 αn = β n = 0, sn = 1 sn−1 = 1 αn = 2, βn = 2, sn = 0
an = 2 sn−1 = 0 αn = 2, βn = 0, sn = 0 sn−1 = 1 αn = 2, βn = 2, sn = 1
Somando as equações (5.5) e (5.6), temos para todo k ∈ N que
k k k
an αn βn sk
= + + .
n=1
3n n=1
3n n=1
3n 3k

Assim,
∞ a ∞ α 3n
n n βn
= +
n=1
3n n=1
3n 3n
βn

e, consequentemente, a k = α + β.

194
Exercı́cio 5.19:
Seja K o conjunto de Cantor. Dado ε > 0 arbitrário, existem um intervalos abertos J 1 = (a1 , b1 ),...,J n = (an , bn )
tais que que K ⊂ J ∪ J ∪ ... ∪ J
1 2 n e
n

i=1
(ai − b ) < ε.
i

Seja K p o conjunto resultante do p-ésimo passo da contração do conjunto K de Cantor. Sabemos que
2p
˙ 1
Kp = [ci , ci + ].
i=1 3p
1 2
Assim, tomando-se a i = c i − 3p e bi = 3p temos que

2p
J= (ai , bi ) ⊃K ⊃K
p
i=1

e
2p
p 1 2
(bi −a )= 2
i = 3( )p .
i=1
3p−1 3
Assim, dado ε > 0, basta tomar p ∈ N tal que
2
3( )p < ε
3
e teremos J que satisfaz as condi¸cões do enunciado.

195
Exercı́cio 5.20:
Para X, Y ⊂ R qualquer, tem-se X ∪Y =X ∪Y eX ∩ Y ⊂ X ∩ Y . Dê um exemplo no qual a inclusão não se
reduz a uma igualdade.

• X ∪Y ⊃ X ∪Y .
Seja a ∈ X ∪ Y . Então, existe uma sequência (x ) em X ou (y ) em Y que tende `a a. Em ambos os casos
n n
existe uma sequência em X ∪ Y que tende `a a.
• X ∪Y ⊂ X ∪Y .
Seja ∈ X ∪ Y . Então,
uma asubsequência em X existe uma
(no caso emsequência n
que possui(zinfinitos ∪ Y queemtende
) em Xtermos X ) `ou
a a.uma
E, assim, esta sequência
subsequência em Y (no possui
caso
em que possui infinitos termos em Y ). No primeiro caso a ∈ X e no segundo caso a ∈ Y . Em todo caso,
a ∈ X ∪Y.
• X ∩Y ⊂ X ∩Y .
Seja a ∈ X ∩ Y . Então, existe uma sequência (z ) em X ∩ Y que tende `a a. Assim, ( z ) é uma sequência em
n n
X que tende `a a, e, consequentemente, a ∈ X . Analogamente, a ∈ Y . Portanto, a ∈ X ∩ Y .

[−1, 0) ∩ (0, 1] = ∅ = ∅.
[−1, 0) ∩ (0, 1] = [ −1, 0] ∩ [0, 1] = {0}.

196
Exercı́cio 5.21:
Um conjunto A ⊂ R é aberto se, e somente se, A ∩ X ⊂ A ∩ X para todo X ⊂ R.
( ) Seja A um conjunto aberto. Dado X R e a A X. Então, existe uma sequência (xn ) em X que tende
⇒ ⊂ ∈ ∩
à a. Assim, como A e´ aberto, existe n0 N tal que para todo n n 0 , xn A. Da´
∈ ≥ ∈ ı, existe uma sequência em A
∈ ∈ ∩ ⊂ ∩
que tende à a. Ou seja, a A. Logo, a A X A X.
( ) Seja A R tal que A X A X para todo X R.
⇐ ⊂ ∩ ⊂ ∩ ⊂
Dado a A, suponhamos que para todo n N, existe xn (a n1 , a + n1 ) A. Seja X = xn n∈N . Então,
∈ ∈ ∈ − − { }
{}a = A X A X = = . Contradição. Logo, existe n N tal que ( a n1 , a + n1 ) A. E, assim, a int(A).
∩ ⊂ ∩ ∅ ∅ ∈ − ⊂ ∈

197
Exercı́cio 5.22:
Sejam F 1 ⊃ F ⊃ ... ⊃ F ⊃ ... não vazio se os F
2 n n são apenas fechados ou apenas limitado s.

•F n apenas fechado
Fn = [n, ∞) ⇒ ∅
Fn = .
n N

•F n apenas limitado
1
Fn = 0,
n
⇒ ∅
Fn = .
n N

198
Exercı́cio 5.23:
Um conjunto n˜ao vazio X ⊂ R é um intervalo se, e somente se, satisfaz a condição seguinte: “a, b ∈ X,a < x <
b⇒ ∈
x X ”.


( ) Seja X um intervalo não-vazio com α = inf X e β = sup X.
Dados a e b X, a < b, e x R tal que a < x < b. Assim,
∈ ∈
X ⊃ (α, β) ⊃ (a, b) ∈ X.

( ) Seja X um conjunto que satisfaz a condi¸cão

∈ X, a < x < b ⇒ x ∈ X ”.
“a, b

Sejam α = inf X e β = sup X. Temos então que X (α, β ). De fato, dado x ∈ (α, β), existem a e b ∈ X tais
que

α a<x<b β.≤
Assim, pela propriedade, x ∈ X. Por outro lado, X − {α, β } ⊂ (α, β ) pela defini¸cão de α e β . Portanto,

X = [α, β ] ou X = [α, β ) ou X = (α, β ] ou X = (α, β ).

199
Exercı́cio 5.24:
Mostre que a interse¸cão de uma sequência decrescente I1 ⊃ I ⊃ ... ⊃ I ⊃ ... de intervalos é um intervalo ou o
2 n
conjunto vazio.

Sejam a n = inf In e b n = sup In , então I n ⊃ (a n , bn ) e [an , bn ] ⊃I n. Assim,

[an , bn ] ⊃ [a n+1 , bn+1 ]

e
(an , bn ) ⊃ (a n+1 , bn+1 ).

Da´
ı,
{ } { }
[sup an , inf bn ] = [an , bn ]
⊃ In
⊃ (an , bn )
{ } { }
= (sup an , inf bn ).

Assim, como −∞ ≤ sup{a } ≤ inf {b } ≤ ∞,


n n

{
In = (sup an , inf bn ) ou } { }
{ }
= (sup an , inf bn ] ou { }
{ }
= [sup an , inf bn ) ou { }
{ }
= [sup an , inf bn ] { }
e temos o resultado.

200
Exercı́cio 5.25:
Um conjunto é denso em R se, e somente se, seu complementar tem interior vazio.

Seja D um conjunto denso em R. Dado x R D e ε > 0, temos que (x ε, x+ε) D = . Assim, x / int(R D).
∈ − − ∩ ̸ ∅ ∈ −
E, como int(R D) R D, temos que int(R D) = .
− ⊂ − − ̸∅

201
Exercı́cio 5.26:
Se F é fechado e A e´ aberto então F − A e´ fechado.
Como A e´ aberto, Ac é fechado. Então, F ∩A c
=F − A é uma interseção de dois fechados. Logo, F − A é
fecado.

202
Exercı́cio 5.27:
Dê exemplo de um aberto A tal que A ⊃ Q mas R − A seja não-enumerável.


Seja (xn ) uma enumeração de Q . Definimos A = In , onde
n=1

In = (xn − 21 , x
n n +
1
2n
).

Temos que A e aberto, pois é uma união de abertos e também A Q. ⊃


Provemos que R A = . Consideremos o intervalo compacto [0 , 10] e suponhamos que R
− ̸ ∅ − A = ∅. Então,
k
∈ N tais que
existem n 1 , n2 ,...,n k ∪
i=1
Ini ⊃ [0, 10].
Sgue daı́, então, que
k
10 < (sup Ini − inf I ni )
i=1
k
< (sup In − inf I n)
i=1
∞ 1
= = 2.
i=1
2n−1

Absurdo. Assim, R A = .
− ̸∅
Se R A é finito ou enumeráel, podemos adicionar todos os elementos de R A em uma sequência (yn ) ( de termos
− −
repetidosse necessário ) que enumera os elementos de R A. Da´ −
ı, Ã = A (

n=1
I˜n ), onde I˜n = (yn
∪ 1
2n
∪ − 1
, yn + n )
2
é tal que Ã⊃ A e à ⊃ −R A. Mas, por argumento an´alogo ao acima, podemos mostrar que R à = . Uma − ̸ ∅
contradição. Portanto, R A e´ não enumerável.

203
Exercı́cio 5.28:
Dê exemplo de um conjunto fechado, não-enumerável, formado apenas por n´umeros transcendentes.

Seja (xn ) uma enumeração do conjunto dos n´umeros algébricos (exerc´ıcio 3.44). Seja

A= In ,
i=1

1 1
onde In = (xn − 2n
, xn + n ). Temos que A e´ um aberto que contém todos os números naturais. Assim, pelo
2
mesmo argumento usado no execı́cio anterior, F = R A e´ não enumerável. E F é um fechado que contém somente
números algébricos. −

204
Exercı́cio 5.29:
Defina a distˆancia de um ponto a ∈ R a um conjunto n˜ao-vazio X ⊂ R como d(a, X )inf {|x − a|; x ∈ X }. Prove:
1) d(a, X ) = 0 ⇔ a ∈ X.
2) Se F ⊂ R é fechado, então para todo a ∈ R existe b ∈ F tal que d(a, F ) = |b − a|.

(1) (⇒) Para cada n ∈ N, existe x ∈ N tal que |x − a| < n1 . Assim, ( x ) é uma sequência em X que tende à
n n n

a. Logo, a ∈ X .
(⇐) Seja ( x ) uma sequência em X que tende à a. Então, para todo ε > 0, existe n ∈ N tal que |x − a| < ε.
n 0 n0
Assim,

{| − | ∈ }
0 = inf xn a ; xn (xn )
≥ {| − | ∈ } ≥
inf x a ; x X 0,

ou seja, d(a, X ) = 0.
(2) Consideremos o compacto C = F ∩ B[a; 2d(a, F )]. Temos, pela definição de d(a, F ), que C̸= ∅. Seja d =
d(a, F ) e xn ∈ C tal que |x − a| < d + n1 . Como C é compacto, existe uma subsequência (x ) de ( x )
n nk n
ε ε
tal que x → b ∈ C. Assim, dado ε > 0, existe n + k ∈ N tal que | x − a | < d + e | b − x | > .
nk nk nk
2 2
Consequentemente,
|b − a| ≤ |b − x | + |x − a| < d + ε.
nk nk

Portanto, | b − a | ≤ d. Mas, como b ∈ C ⊂ F e pela defini¸cão de d = d(a, F ), temos que | b − a | ≥ d e,


consequentemente, |b − a| = d.

205
Exercı́cio 5.30:
Se X é limitado superiormente, seu fecho X também o é. Além disso, supX = sup X. Enuncie e prove um resultado
análogo para inf .

Temos que X é limitado somente se X e´ limitado. De fato, se A > 0 em R e´ tal que existe a X com a > A,

| − | ∈ | − |
tomando-se ε = A a , existe x X tal que x a < ε. Consequentemente, x > A.
Como X ⊂ X, temos imediatamente que (pelo exercı́cio 3.33) que sup X ≥ sup X. Assim, para mostrar que
sup X sup X, basta provar que para qualquer c R tal que c < sup X, existe x X tal que c < x. Então, dado
≥ ∈ ∈
∈ ∈ | − | | −|
c tal que c < sup X, existe x X tal que c < x. Assim, tomando algum x X tal que x x < x c , temos que
x > c. Portanto, sup X = sup X.
O resultado an´alogo seria inf A = inf A. E a demonstra¸cão desse resultado é também análoga.

206
Exercı́cio 5.31:
Para todo X ⊂ R limitado superiormente, sup X e´ aderente a X. Resultado análogo para inf .
∈ X tal que sup X − ε < x ≤ sup X.
Pela definição de supremo, para todo ε > 0 existe x
Assim, para n ≥ 1 em N, tomando x ∈ X tal que sup X − n1 > x ≤ sup X. Temos, assim, que para todo
n n
1
n ∈ N, |x − sup X | < . Logo, a sequência (x ) em X tende à sup X e sup X ∈ X .
n n n

207
Exercı́cio 5.32:
Para todo X ⊂ R, X ′ e´ fechado.
Seja a X ′ . Dado ε > 0, existe x X ′ tal que x (a ε, a+ε). E como x X ′ , existem infinitos elementos de X
∈ ∈ ∈ − ∈
− {| − ± |} − ⊃ −
em (x δ, x+δ ), onde δ = min x (a ε) . Assim, infinitos elementos de X pertencem à (a ε, a+ε) (x δ, x+δ ).
Isso implica que a X ′ . Conclu´ımos que X ′ X ′ .
∈ ⊃

208
Exercı́cio 5.33:
Um número a e´ ponto de acumulação de X se, e somente se, é ponto de acumulação de X .


( ) Seja a um ponto de acumula¸cão de X. Então, para todo ε > 0 existem infinitos elementos de X em
− ⊂ −
(a ε, a + ε). E como X X , existem infinitos elementos de X em (a ε, a + ε), ou seja, a é ponto de acumulação
de X .
⇐ ∈ ∈ − −{ }
( ) Seja a um ponto de acumulação de X. Dado ε > 0, existe x X tal que x (a ε, a+ε) a . Tomando-se
{| − | | − ± |} ∈
δ = min x a , x (a ε) , temos que existe x X tal que x x < δ. | − |
∈ − ⊂ −
Assim, x (x δ, x + δ ) (a ε, a + ε) −{ }
a . Segue da´ı que a e´ ponto de acumulação de X.

209
Exercı́cio 5.34:
(X ∪ Y )′ = X ′ ∪ Y ′ .

• (X ∪ Y )′ ⊂ X ′ ∪ Y ′ .
Seja a ∈ (X ∪ Y ) e (a n ) uma sequência em X Y ∪ −{ }
a que tende `a a. Então, existem infinitos termos de
(an ) em Y . Assim, existe uma subsequência de (an ) em X
′ ′
−{ }
a ou em Y −{ }
a . E como esta subsequência
∈ ∪
tende à a, temos que a X Y .
• (X ∪ Y )′ ⊃ X ′ ∪ Y ′ .
Seja ∈ X ′ subsequência
existea uma ∪ Y ′ . Então existe
em X uma
∪ Y −sequência n
{a} que (atende
) em a, −
`a X ou{seja,
a} ouaem Y −∪{Ya)}′ .que tende `a a. Em todo caso,
∈ (X

210
Exercı́cio 5.35:
Todo ponto de um conjunto aberto A e´ ponto de acumulação de A.

∈ −
Seja a A. Então existe δ > 0 tal que ( a δ, a + δ ) ⊂ A.
{ }
Assim, dado ε, para δ 1 = min δ, ε temos que

(a − ε, a + ε) ⊃ (a − δ , a + δ ) ⊂ A.
1 1

Logo, (a − ε, a + ε) contém infinitos elementos de A já que ( a − δ , a + δ ) tem infinitos elementos. Portanto, a é
1 1
ponto de acumulação.

211
Exercı́cio 5.36:
Sejam F fechado e x ∈ F. Então x é um ponto isolado de F se, somente se, F − {x} é ainda fechado.
⇒ {}
( ) Se F = x , temos diretamente que F −{ } ∅
x = é fechado. Suponhamos que F −{ } ̸ ∅
x = . Seja (an ) uma
sequência em F−{ } ∈
x com lim an = a. Como F é fechado e (an ) é uma sequência em F, então a F. Temos que
̸ − ∩ {}
a = x, pois para algum ε > 0, (x ε, x + ε) F = x . Portanto, a F ∈ −{ }x . Conclu´ımo da´ı que F −{ }
x é
fechado.
1
( ) Suponhamos que x não seja um ponto isolado de F. Então, para todo n N , existe xn ((xn
⇐ ∈ ∈ − ,x +
n
1
n
∩ −{ }
) F) x . Assim, a sequência (xn ) em F −{ }
x tende à x. O que implica que F −{ }
a não é fechado.

212
Exercı́cio 5.37:
Seja X R tal que X ′ X = . Mostre que existe, para cada x
⊂ ∩ ∅ ∈ X, um intervalo aberto I x, de centro em x, tal
̸ ⇒ ∩
que x = y I x Iy = . ∅
Para cada x ∈ X definimos
δx = inf x {| − x̃|; x̃ ∈ X − {x}}.
Temos que δ x = 0 somente se x ∈ X ′ . Então, δ x > 0 para cada x ∈ X já que X ′ ∩ X = ∅.
Definimos para cada x X, ∈
Ix = (x − δ2 , x + δ2 ).
x x

̸
Assim, para x = y em X, temos que se z ∈I ∩I
x y

|x − y| ≤ |x − z| + |z − y|
≤ δ2 + δ2 x x

≤ |x −2 y| + |x −2 y| = |x − y|.
Portanto, I x ∩ I = ∅. E o resultado segue.
y

213
Exercı́cio 5.38:
Seja F ⊂ R fechado, infinito enumerável. Mostre que F possui uma infinidade de pontos isolados.
Pelo corolário 1 do Teorema 9, temos que F possui algum pont o isolado. Suponhamos que x1 , x2 ,...,x n são
pontos isolados de F . Por indução no resultado do exercı́cio 3.36, temos que F −{ }
x1 , x2 ,...,x n e´ fechado. Além
disso, F −{ }
x1 , x2 ,...,x n é um conjunto sem pontos isolados e infinito enumerável, contradizendo o Corolário 1 do
Teorema 9.

214
Exercı́cio 5.39:
Mostre que todo n´umero real x e´ limite de uma sequência de números transcendentes dois a dois distintos.

Sejam A o conjunto dos n´umeros algébricos em Q (complementar do conjunto dos trancendentes) e x R. ∈


− − ∪{ }
Dado ε > 0, temos que (x ε, x +ε) (A x ) é infinito. De fato, pelo exerc´ ıcio 3.44, A é enumerável enquanto

(x ε, x + ε) é n˜ − − ∪{ }
ao enumerável. Logo, ( x ε, x + ε) (A x ) é infinito.
Seja ε 1 = 1. Podemos escolher
∈ − − ∪{ }
x1 (x ε1 , x + ε1 ) (A x ).
E, indutivamente, escolher
xn ∈ (x − ε , x + ε ) − (A ∪ {x}),
n n

| − x |. Assim ( x
onde ε n = x n n) é uma sequência de termos trancendentes, dois a dois distintos e que tende à x.

215
Exercı́cio 5.40:
Mostre que se X ⊂ R não é enumerável, então X ∩ X ′̸= ∅.
Se X X ′ = , então todo ponto de X é isolado. Mas, pelo Corolário 2 do Teorema 8, temos que isso implica
∩ ∅
que X e´ enumerável.

216
Exercı́cio 5.41:
Se A e A ∪ {a} são abertos ent˜ao a e´ ponto de acumulação de A à direita e `a esquerda.
Como A ∪ {a} e´ aberto, existe ε > 0 tal que ( a − ε, a + ε) ⊂ A ∪ {a}.
Assim,
[a, a + ε) ⊂ A ∪ {a}
e, consequentemente,
(a, a + ε) ⊂ A ∪ {a}.
Da´
ı segue que A cont´
em infinitos pontos de [a, a + ε). Logo, a e´ ponto de acumulação a direita de A.
Analogamente, mostra-se que a e´ ponto de acumulação à esquerda de A.

217
Exercı́cio 5.42:
Dê explicitamente o significado de cada uma das seguintes afirmações. Em suas explica¸cões, você está proibido de
usar qualquer das palavras grifadas abaixo:
1) a ∈ X não é ponto interior de X ;
2) a ∈ R não é aderente a X ;
3) X ⊂ R não é um conjunto aberto;
4) O conjunto Y ⊂ R não é fechado;
5) a ∈ R não é ponto de acumulação do conjunto X ⊂ R;

⊂∅
6) X ′ = ;
7) X Y mas X não é denso em Y ;
8) int(X ) = ;∅
9) X ∩ X ′ = ∅;
10) X não é compacto.

∈R +
(1) Não existe ε tal que
a ∈ (a − ε, a + ε) ⊂ X.
+
(2) Existe ε ∈R tal que
(a − ε, A + ε) ⊂ R − X.
+
(3) Existe x ∈ X tal que para todo ε ∈ R
(x − ε, x + ε)  X.
+
(4) Existe x ∈ R − X tal que para todo ε ∈ R
− ε, x + ε) ∩ X̸= ∅.
(x
∈R +
(5) Existe ε tal que
(a − ε, a + ε) ∩ X ⊂ {a}.
(6) Para todo a ∈ R, tal que para todo ε ∈ R +

(a − ε, a + ε) ∩ X̸= ∅ então a ∈ X.
+
(7) Existe y ∈ Y e ε ∈ R tais que
(y − ε, y + ε) ∩ X = ∅
mas X ⊂ Y .
+ +
(8) Para todo x ∈ X e ε ∈ R , existe a ∈ R e δ ∈ R tais que
(x − ε, x + ε) ⊃ (a − δ, a + δ ) ⊂ R − X.
+
(9) Para todo x ∈ X, existe ε ∈ R tal que
(x − ε, x + ε) ∩ X = {x}.

(10) Existe uma sequência (xn ) em R e (εn ) em R + tais que


X ⊂ (xn − ε , x + ε ),
n n
n N ∈
mas para nenhum Ñ ⊂ N finito temos que
X ⊂ (xn − ε , x + ε ).
n n
n Ñ∈

218
Exercı́cio 5.43:
Se todo ponto de acumula¸cão de X é unilateral, X e´ enumerável.

Seja D o conjunto dos elementos de X que são pontos de acumula¸cão à direita de X.


Dado n Z seja

{ }
aλ λ∈Λ = [n, n + 1] D. ∩
∈ − ∩ ∅
Para cada λ Λ, existe I λ = (aλ ελ , aλ ), ελ > 0, tal que I λ X = . Observemos que para quaisquer λ 1 e λ 2 λ, ∈
λ1 = λ 2 , teremos que I λ1 Iλ2 = . Se Λ é não enumerável, existe ε > 0 tal que para cada n N existe λ n tal que
̸ ∩ ∅ ∈
ελn > ε. Assim,

n=1 ε
λn = ∞
e como os I λ′ s são disjuntos e est˜ao contidos em um intervalo de comprimento 1, obtemos uma contradição. Logo,
Λ é finito ou enumerável. E como
D= ∩
[n, n + 1] D,
n Z

temos que D é enumerável.
De forma análoga, mostra-se que o conjunto E dos pontos de acumulação à esquerda de X é finito ou enumerável.
E como X X ′ é finito ou enumerável pelo Corol´ario 2 do Teorema 8, temos que

X = (X − X ′) ∪ E ∪ D
é finito ou enumerável.

219
Exercı́cio 5.44:
Seja X R um conjunto arbitrário. Toda cobertura de X por meio de abertos possui uma subcobertura enumerável.

(Teorema de Lindelöf).

Seja {A } ∈
λ λ Λ uma cobertura aberta de X. Tomemos E = xn { } ∈ um subconjuntto denso em
n N X. Para cada
n ∈ N fixemos +
{ ∈R
Rn = ε ; (xn − ε, x n + ε) ⊂ A , λ ∈ Λ }.
λ

̸ ∅
Sabemos que R n = . Logo, existe sup Rn = 2εn .
+

Além disso, existe ε R n tal que ε n < ε. Ou seja, existe ε ∈R tal que

(x − ε, x + ε) ⊂ A λ

para algum λ ∈ Λ. Existe, também n ∈ N tal que


x ∈ (x − ε/4, x + ε/4).
n

Para qualquer
yn ∈ (x − 3ε/4, x
n n + 3ε/4)
temos que
y ∈ (x − ε, x + ε).
Da´
ı, 3ε/4 ∈R n. E, assim,
εn ≥ 3ε8 > 4ε .
Logo,
x ∈ (x − ε , x
n n n + εn ) ⊂A λn .

Conclu´ımos da´
ı que Aλn { } ∈ ⊂ {A } ∈
n N λ λ Λ e´ uma cobertura de X.

220
Exercı́cio 5.45:
Com a nota¸cão do Exerc´
ıcio 4, prove:
a) Se A e´ compacto e B e´ fechado então A + B e´ fechado;
b) se A e B são compactos, então A + B e A.B são compactos;
c) se A é fechado e B é compacto, A.B pode n˜ao ser fechado.

(a) Seja x ∈ A + B e (c n = a n + bn )n∈N uma sequência em A + B tendendo à x. A sequência (an )n∈N em A possui
uma subsequência (ank )k∈N tendendo a algum a ∈ A. Dado ε > 0, existe n ∈ N tal que, para todo n ≥ n ,
0 0

|a + b − x| < ε/2.
n n

E também, existe k 0 ∈ N tal que, para todo k ≥ k , 0

|a − a| < ε/2.
nk

Assim, para
{
k1 = max k0 , min k { ∈ N; n ≥ n }}
k 0

e todo k ≥ k , temos que


1

|b − (x − a)|
nk = |(a + bnk x) (a ank )
nk − − − |
≤ |a + bnk x + a ank
nk − | | − |
< ε/2 + ε/2 = ε.

Logo, lim bnk = x


n→∞
− a. Segue da´ı que x − a ∈ B e, consequentemente, x ∈ A + B. Conclu´ımos da´ı que A + B
é fechado.
(b) A + B é fecado pelo item (a). E, também, temos que

sup(A + B) = sup A + sup B < ∞


e
inf(A + B) = inf A + inf B > −∞.

Seja x A.B e (cn = a n .bn )n∈N uma sequência em A.B tendendo à x. A sequência (an )n∈N em A possui uma

subsequ?ncia (an )n∈N1 tendendo `a algum a A. Por sua vez a sequência (bn )n∈N1 possui uma subsequência

(bn )n∈N2 tendendo a algum b B. Assim,

x = lim an bn
n N2

= lim an lim bn
n N2
∈ n N2

= a.b ∈ A.B
Segue da´ı que A.B é fechado. E, como

sup(A.B) = sup A. sup B <

e

inf(A.B) = inf A. inf B > −∞,
temos que A.B e´ limitado. Portanto, A.B é compacto.
(c) Tome B = 0, 1, 1/2,..., 1/n,... e A = Z. Temos que A.B não é fechado. (Vide exercı́cio 5.16)
{ }

221
Exercı́cio 5.46:
Obtenha coberturas abertas de Q e de [0 , ∞) que n˜ao admitam subcoberturas finitas.
{−
Temos que ( n, n) }∈
n N e´ uma cobertura aberta tanto de Q quanto de [0 , + ∞) que n˜ao admite subcobertura
finita.

222
Exercı́cio 5.47:
Considere as funções f,g,h do Exercı́cio 7. Mostre que para K e L compactos arbitrários, f (K ), g(K ), h(K ), f −1 (L), g −1 (L)
e h −1 (L) são compactos.

Seja (xn ) uma sequência em f −1 (L) tendendo `a x ∈ R. Temos que


lim f (xn ) = lim( axn + b)
= a lim xn + b
= ax + b = f (x)
1
Como
Segue da´ (xnf) −é1uma
ı fque sequência
(L). Segue convergente
da´ı que f −1 (L) énofechado. ∈ ∈
fechado L, temos que f (x) L e, consequentemente, x f − (L).
Seja (yn = f (xn ))n∈N uma sequência em f (K ). Como K é compacto, existe uma subsequência (xnk ) de ( xn )

que converge à algum x K. Assim,

y = lim f (xn ) = lim f (xnk ) = f (x).


Ou seja, y f (K ). Conclu´ımos da´ı que f (K ) é fechado.
Analogamente, prova-se que g −1 (L), g(K ), h−1 e h −1 (K ) são fechados.
Por fim, temos que
f (K ) = aK + b
g(K ) ⊂ K.K
e
K.K.K.
E como os conjuntos aK + b, K2 e K3 são limitados(ver exercı́cio 5.45(b)), temos que f (K ), g(K ) e h(K ) são
compactos. E, também,
f (f −1 (L))
⊂ L,
1
g(g− (L)) ⊂ L
e
h(h− (L)) ⊂ L.
1

Assim, pelas definições de f, g e h temos que se f −1 (L), g −1 ou h −1 (L) fosse limitado, terı́amos que f (f −1 (L)),
g(g−1 (L)) ou h(h−1 (L)) seria limitado contradizendo o fato de L ser limitado.

223
Exercı́cio 5.48:
As seguintes afirmações a respeito de um conjunto X ⊂ R são equivalentes:
(1) X é limitado;
(2) Todo subconjunto infinito de X possui ponto de acumula¸cão (que pode n˜ao pertencer a X );
(3) Toda sequência de pontos de X possui uma subsequência convergente.

(1) ⇒ (2) Como X é limitado, temos que X e´ compacto. Então, todo conjunto infinito em X ⊂ X possui um
ponto de acumulação por ser subconjunto de um compacto.
(2) ⇒∈ { }
(3) Seja ( xn )n∈N uma sequência em X. Se o conjunto P = xn n∈N for finito, ent˜ao para algum p P e ∈
infinitos n N, temos que x n = p. Logo, existe uma subsequência de x n tendendo à p. Se P for infinito, então pela
hipótese, P possui um ponto de acumula ção x. Logo, é possı́vel encontrar uma subsequência de (xn ) tendendo à x.
(3) ⇒ (1) Se X não fosse limitado, seria possı́vel encontrar uma sequência crescente e ilimitada em X. Tal
sequência não teria subsequência convergente. O que contradiz a hipótese.

224
Exercı́cio 5.49:
Seja X R um conjunto compacto cujos pontos, com exceção de a = inf X e b = sup X, são pontos de acumulação

{ }
à direita e `a esquerda. Ent˜ao X = [a, b] ou X = a, b .

∈ ̸ { }
Como X é compacto, temos que a = inf X e b = sup X X. Suponhamos que X = a, b . Então, existe c X ∈
∈ ≤ ∩ ≤ ≤ ̸
tal que a < c < b. Seja x (a, b). Se c x temos que s = sup([ a, x] X ) é tal que a < c s x. Assim, se s = x
∈ −{ }
temos que s X ∩ {}
a, b e [s, x) X = s . Ou seja, s não seria um ponto de acumula¸cão à direita. Absurdo.
∈ ≤ ∈
Logo, c = s X. Do mesmo modo, se x c teremos que x X. Concluı́mos que X = [a, b].

225
Exercı́cio 5.50:

Se (Kλ )λ∈L é uma famı́lia qualquer de compactos, então Kλ e´ compacto. Se K1 ,...,K n são compactos ent˜ao
∪ ∪ ∪
K1 K2 ... Kn e´ compacto. Se K e´ compacto e F é fechado, então K F é compacto. ∩

• K , λ ∈ Λ, compactos ⇒
λ
∩ ∈ Kλ é compacto.
λ Λ
Como cada K λ e´ fechado, temos que ∩K λ é fechado. Além disso, temos que, dado algum λ 0 ∈ Λ qualquer

inf( Aλ ) ≥ inf A λ0 > −∞
e

sup( Aλ ) ≤ inf A λ0 < ∞.
Da´ı, ∩A é limitado. Portanto, ∩A é compacto.
λ λ

i
∪ ∪ n
• K , i = 1, 2,...,n, compacto ⇒ K é compacto.
i=1 i
n
Como cada K i e´ fechado, temos que i=1 Ki e´ fechado. Além disso,
n
n
sup( {
Ki ) = sup sup Ki } i=1 < ∞
i=1

e n
n
inf( {
Ki ) = inf inf Ki } i=1 > −∞.
i=1

Da´ı,
∪ n
i=1 Ki e´ limitado. Portanto,
∪ n
i=1 Ki é compacto.
• K e´ compacto e F é fechado ⇒ K ∩ F é compacto.
Como K e F são fechados, K ∩ F e´ fechado. Além disso,

sup(K ∩ F ) ≤ sup K < ∞

e
inf(K ∩ F ) ≥ inf K > −∞.
Da´ı, K ∩ F é limitado. Portanto, K ∩ F é compacto.

226
Exercı́cio 5.51:
Seja X ⊂ R. Uma função f : X → R diz-se não-decrescente no ponto a ∈ X quando existe δ > 0 tal que
− ≤ ≤
a δ < x a y < a+δ ⇒f (x)≤ f (a) ≤ f (y). (Bem entendido: x, y ∈ X.) Mostre que se f e´ não-decrescente

em [a, b] (isto é, x, y [a, b], x y ≤ ⇒ f (x) ≤ f (y)).
Sejam x, y ∈ [a, b] com x ≤
y. Provaremos que f (x) f (y). Para cada α ≤ [x, y] existe δx R+ tal que
∈ ∈
α −δ α< z ≤ ≤ ≤
α w < α + δα implica f (z) f (α) f (z)(para z , w [a, b]). ≤ ∈
∪ n
Temos então que [ x, y] ⊂∪ α∈[x,y] (α −
δ α , α + δ α ). Asssim, existem α1 , α2 ,...,α n [x, y] tais que [ x, y] ∈ ⊂
i=1 (αi− δα1 , αi + δα1 ). Podemos mostrar, por indu¸cão em n, que podemos decompor [ x, y] como

[x, β1 δβ1 ) (β1 δβ1 , β1 + δβ1 ),


− ⊂ −
[βi−1 −δ βi −1 ) ⊂ (β − δ , β + δ
i βi i βi ), i = 1, 2,...,p − 1,
e
[βp−1 −δ βp− 1, y] ⊂ (β − δ , β + δ )
p βp p βp
p n
para βi { } ⊂ {α }
i=1 i i=1 e β i < βi+1 , i = 1, 2,...,p − 1. Da´ıescolhemos γ ∈ R, i = 1, 2,...,p − 1, tais que
i

βi+1 −δ i+1 < γi < βi + δβi e β i < γi < βi+1 .

Assim,
f (x) ≤ f (γ ) ≤ f (γ ) ≤ ... ≤ f (γ − ) ≤ f (y).
1 2 p 1

227
Exercı́cio 5.52:
Seja [a, b]⊂∪ Aλ onde cada A λ é aberto. Mostre que é poss´ıvel decompor [a, b] em um n´umero finito de intervalos
justapostos de modo que cada um deles esteja contido em algum A λ .


Pelo Teorema 2, cada A λ pode ser decomposto como j ∈N A(λ,j) , Nλ N, onde os A′(λ,j) s são intervalos abertos

disjuntos.
Existe uma subcobertura Bi ni=1
{ } ⊂{ }
A(λ.j) de [a, b]. Provaremos que é possı́vel decompor [a, b] como

[a, x ) ⊂ C ,
1 1 [) ⊂ C , i = 2,...,p,
i e [xp , b) ⊂C p+1 (5.7)

para Ci p+1
i=1 Bi ni=1 .
{ } ⊂{ }
Se n = 1, temos o resultado diretamente. Suponhamos que o resultado seja v´ alido para n

no caso em que n = k + 1. Como a B i , para algum i = 1, 2, ...n, tomemos C 1 = B i e assim
≤ k e que estejamos
n
[sup C1 ] ⊂ ({B } − {B }).
j j=1 i

Pela hipótese indutiva, existem y1 , y2 ,...,y p e C̃1 , C̃2 ,..., C̃p que decomp˜oe [sup C1 , b] como em ( 5.7). Assim,
tomando-se
x1 = sup C1
xi = yi , i = 2,...,p + 1
Ci = C̃i−1 , i = 2,...,p + 2
temos o que quer´ıamos.
Então, como cada intervalo da decomposição de [ a, b] está contido em algum C i e este por sua vez contido em
algum A λ , temos o nosso resultado.

228
Exercı́cio 5.53:
No exercı́cio anterior, mostre que os intervalos nos quais se decompôs [a, b] podem ser tomados com o mesmo
comprimento.

No exerc´
ıcio anterior obtivemos
C1 = [a, x1 )
Ci = [xi−1 , xi ), i = 2,...,n,
e
Cn+1 = [xn , b]
n
{ }
tais que Ci i=1 e´ uma partição de [ a, b] sendo que C i

Temos que x i Aλi +1 . Assim, para cada i = 1,...,n,
⊂ existe
A . λ1
ε i > 0 tal que

(xi − ε ,x + ε ) ⊂A
i i i λi +1 .

+
Tomemos q ∈Z tal que
b − a < min{ε } n
i i=1 .
q
Consideremos, agora, os intervalos

Ii = [a +
i − 1 (b − a), a + i (b − a)), i = 1,...,q.
q q
Assim, temos que, para cada p = 1,...,q,

(i) a ou x i ≤ a + p −q 1 (b − a) < a + pq (b − a) < x i+1 ou b

Ip ⊂C ⊂A i+1 λi +1

(ii) a + p − 1 (b − a) < x i < a + p (b − a)


q q

Ip ⊂ (x − ε , x + ε ) ⊂ A
i i i i λi +1 .

Concluimos daı́ que, para p = 1,...,q,I p ⊂A λ, para algum λ. E temos o nosso resultado.

229
Exercı́cio 5.54:
∪ ∪ ∪ ∪
(Teorema de Baire) Se F 1 , F2 ,...,F n ,... são fechados com interior vazio então S = F 1 F2 ... Fn ... tem interior
vazio. ( É possı́vel mostrar que, dado arbitrariamente um intervalo aberto I , existe algum x I (R S ). Imite a
∈ ∩ −
demonstração do Teorema 6, Capı́tulo III, onde se tem pontos em vez dos fechados F n .)

∈ −
Seja I um intervalo aberto. Como F tem interior vazio, devemos ter x1 I F1 . E como F1 e´ fechado e I
aberto, existe ε1 > 0 tal que ( x1 ε, x1 + ε) I (R F ). Adiante, J1 = [x1 ε/2, x1 + ε/2] I (R F1 ).
− ⊂ ∩ − − ⊂ ∩ −
− −
Definimos I 1 = (x 1 ε/2, x1 + ε/2).
Da mesma forma podemos obter J 2 compacto contido em I 1 (R F2 ) e que desta forma J 2 J 1 .
∩ − ⊂
Por indução, podemos tomar intervalos compactos J n J n−1 , n N [2, ), tais que F n Jn = .
⊂ ∈ ∩ ∞ ∩ ∅
Logo, existe, pelo Teorema 12 do Capı́tulo V, x N Jn . E como J n Fn = , x F n e, consequentemente

x / S. ∈ j∈
∩ ∩ ∅ ∈
Asim, I  S. E como I e´ arbitrário, segue daı́ que S tem interior vazio.

230
Exercı́cio 5.55:
O conjunto R Q dos n´umeros irracionais não pode ser expresso como reuni˜ao enumerável de fechados. Analoga-

mente, Q não é intersecção de uma famı́lia enumerável de abertos.

Suponhamos, por absurdo, que existam conjuntos fechados F n , n Z+ , em R tais que R Q = n∈Z+ Fn .
∈ − ∪
Como cada F n está contido em R Q e R Q possui interior vazio, conclu´
− − ımos que F n tem interior vazio, para
cada n Z+ .

Seja ( qn )n∈Z+ uma enumeração do conjunto Q. Como o conjunto qn , para todo n Z+ , é fechado e tem
{ } ∈
interior vazio, temos que R é uma união enumerável de conjuntos fechados com interior vazio pois

R = (R Q) Q = ( n Z+ Fn ) ( n Z+ qn ).
− ∪ ∪∈ ∪∪∈ { }
Assim, pelo Teorema de Baire, R é um conjunto de interior vazio (em R ). Uma contradi¸cão. Portanto, n˜ao podem
existir fechados F , n ∈ Z , em R tais que R − Q = ∪ ∈ F .
n + n Z+ n
Suponhamos, por absurdo, que existam abertos A , n ∈ Z , em R tais que Q = ∩ ∈ A .
n + n Z+ n
Consideremos os fechados F := R − A , para todo n ∈ Z . Desta forma, terı́amos que
n n +

R − Q = R − ∪ ∈ A = ∪ ∈ (R − A ) = ∪ ∈ F .
n Z+ n n Z+ n n Z+ n

Ou seja, desta forma R − Q seria uma uni˜ao enumerável de conjuntos fechados. Uma contradição. Portanto, não
podem existir abertos A , n ∈ Z , em R tais que Q = ∩ ∈ A .
n + n Z+ n

231
Exercı́cio 5.56:
Se [a, b] n
⊂∪
i=1 [ai , bi ], então b −
a  i=1 (bi ai ). Tamb´
∑ n
em [a, b] − ⊂ ∪∞ n=1 [an , bn ] implica b −a ∑ ∞ (b − a ).
n=1 n n
Finalmente, resultados análogos valem para ( a, b) em vez de [ a, b].

n n
(I) [ a, b] ⊂ [ai , bi ] ⇒b − a  (bi −a ) i
i=1 i=1

Para todo k ∈Z + e i = 1,...,n , temos que

1 1
[ai , bi ] ai , bi + .
⊂ − 2k 2k
Logo,
n n
[a, b] ⊂ [ai , bi ] ⊂ ai − 2k1 , b + 2k1 i .
i=1 i=1

Assim, pela Proposi¸cão 1 deste capı́tulo, segue que


n
1
b −a< bi +
2k
− ai − 2k1
i=1

e, consequentemente,
n n
b −a< bi − a + k1
i = (bi − a ) + nk .
i
i=1 i=1

Portanto,
n n
b −a k
lim
→∞
(bi − a ) + nk
i = (bi − a ).i
i=1 i=1

∞ ∞
(II) [ a, b] ⊂ [ai , bi ] ⇒b − a  (bi −a ) i
i=1 i=1

Para todo k e i ∈Z +, temos que

[ai , bi ] ⊂ ai − 2 1 k,b + 2 1 k
i+1 i i+1
.

Logo,
∞ ∞
[a, b] ⊂ [ai , bi ] ⊂ ai − 2 1 k,b + 2 1 k
i+1 i i+1
.
i=1 i=1

Assim, pela Proposi¸cão 2 deste capı́tulo, segue que


∞ 1
b −a< bi +
2i+1 k
− ai −21k i+1
i=1

e, consequentemente,

b −a< ∞
bi − a + 21k
i i
=

(bi − a ) + k1
i
∞ 1
i
=

2
1
(bi − ai ) + .
k
i=1 i=1 i=1 i=1

Portanto,
∞ ∞
b −a k
lim
→∞
(bi − a ) + k1
i = (bi − a ).i
i=1 i=1

232
n n
(III) ( a, b) ⊂ [ai , bi ] ⇒b−a (bi −a )
i
i=1 i=1

Temos que
n
[a, b] = [a, a] ∪ [b, b] ∪ [ai , bi ].
i=1

Assim, pelo item (I), temos que


n n
b − a  (a − a) + (b − b) + (bi −a )= i (bi − a ).
i
i=1 i=1

∞ n
(IV) ( a, b) ⊂ [ai , bi ] ⇒b−a (bi −a )
i
i=1 i=1

Temos que

[a, b] = [a, a] ∪ [b, b] ∪ [ai , bi ].
i=1

Assim, pelo item (I), temos que


∞ ∞
b − a  (a − a) + (b − b) + (bi −a )= i (bi − a ).
i
i=1 i=1

233
Exercı́cio 5.57:
Seja X ⊂ R . Uma fun¸cão f : X → R chama-se localmente limitada quando para cada x ∈ X existe um intervalo
aberto I x , contendo x, tal que f | ∩ é limitada. Mostre que se X é compacto, toda função f : X → R localmente
Ix X
limitada é limitada.

Sejam X um subconjunto compacto de R, f : X R uma função localmente limitada, para cada x


→ ∈ X, Ix
um intervalo contendo x e A x > 0 tais que
f (y) < Ax , | |
para cada y ∈X ∩I . x
Como x ∈ I , para todo x ∈ X , temos que
x

X= Ix .
x X ∈
Sendo X um conjunto compacto, existem x 1 ,. . . ,xn em X tais que

X = I x1 ∪···∪ Ixn .

Para A ∈ R definido por


{
A := max Ax1 ,...,A xn }
temos, para y ∈I xi , que
|f (y)| < A xi  A.
Ou seja,
|f (y)| < A
n
para todo y ∈X =∪ i=1 Ixi . Portanto, f é limitada.

234
Exercı́cio 5.58:
Dado X ⊂ R não-compacto, defina uma função f : X → R que seja localmente limitada mas n˜ao seja limitada.
Sendo X não-compacto, basta provarmos que existe uma função f : X → R que seja localmente mas n˜ao seja
limitada nos casos:
• X não é limitado;
Seja f : X → R dada por
f (x) = x,
para todo x X . Desta forma, para cada x X , o intervalo
∈ ∈ Ix = (x − 1, x + 1)
é tal que f | X Ix
∩ é limitada pois
x − 1  f (y)  x + 1,
para todo y I x X .∈ ∩
Por outro lado, f ainda não é limitada já que se existe A > 0 tal que
|f (x)| < A,
para todo x ∈ X , ter´ıamos que
|x| < A,
para todo x ∈ X , contradizendo o fato de X ser ilimitado.
• X não é fechado;
Como X não é fechado, existe a ∈ X ′ − X . Definimos f : X → R por
1
f (x) =
a − x,
para todo x X . ∈
Desta forma, para cada x ∈ X , o intervalo
Ix = x − |x −2 a| , x + |x −2 a|
é tal que f | ∩ é limitada. De fato, para y
X Ix ∈ I , temos que
x

|y − a| = |(x − a) + (y − x)|
 |x − a| − |y − x|
| −| x a
 |x − a| − 2
|−|
x a
2

e, consequentemente,
|f (y)| = |y −1 a|  |x −2 a| .
E f não é limitada já que, dado A > 0 existe x ∈ X tal que
|f (x)| > A.
De fato, como a ∈ X ′ − X , existe x ∈ X tal que
|x − a| < A1
e, desta forma,
|f (x)| = |x −1 a| > A.

235
Exercı́cio 5.59:
Sejam C compacto, A aberto e C ⊂ A. Mostre que existe ε > 0 tal que x ∈ C , |y − x| < ε ⇒ y ∈ A.
Como A é aberto e A ⊃ C , temos que, para todo x ∈ C , existe ε > 0 tal que x

(x − 2ε , x + 2ε ) ⊂ A.
x x

A fam´ılia
{(x − ε , x + ε ) : x ∈ C }
x x

é uma cobertura aberta para o conjunto C . Como C e´ compacto, existem x 1 ,. . . ,xn ∈ C tais que
n
C ⊂ (xi −ε x i , xi + εxi )
i=1

Tomemos
ε := min {ε ,...,ε } > 0.
x1 xn

Supondo que x ∈ C e |x − y| < ε temos que y ∈ A. De fato, como x ∈ C , temos que


x ∈ (x − ε , x + ε ),
k xk k xk

para k = 1,. . . ,n − 1 ou n. Assim,

|x − y|  |x − x| + |x − y| < ε + ε  2ε
k k xk xk

e, consequentemente,
y ∈ (x − 2ε
k xk , xk + 2εxk ) ⊂ A.

236
Exercı́cio 5.60:
{
Dada uma sequência (xn ), seja Xn = xn , xn+1 ,... } para todo n ∈ Z +. Mostre que ∩∞
n=1 Xn é o conjunto dos
valores de aderência de (xn ).

Seja A o conjunto dos valores de aderência de (xn ).


Provaremos separadamente que A ∞ ⊂∩
n=1 Xn e que

n=1 Xn ∩
A. ⊂

Seja a A. Então, existe uma subsequência (xnk )k∈Z+ de (xn ) tal que lim k →∞ xn = a. Dado n
k
∈Z +, temos
que existe k 0 Z+ tal que

n  nk0 < n k0 +p ,
para todo p ∈Z +. Assim, ( xnk0 +p )p∈Z+ e´ uma sequência em X n tal que

lim xk0 +p = lim xk = a.


p
→∞ →∞ k

Desta forma, a ∈ X n , para n ∈ Z+ arbitrário. Ou seja, a ∈ ∩∞


n=1 Xn .
Seja a ∈ ∩∞n=1 Xn . Provaremos que existe uma subsequência (xn )k∈Z+ de (xn )n∈Z+ tal que lim k →∞ xn = a.
k k
Ou seja, que a ∈ A. Comecemos observando que para todo m e k ∈ Z+ existe n k ∈ Z+ tal que

nk > m e |x − a| < k1 .
nk (5.8)

∈ {
De fato, como a Xm+1 e Xm+1 = xn : n  m + 1 , temos que existe xnk } Xm+1 tal que xnk a < k1 e,
∈ | − |
desta forma, n k satisfaz (5.8). Assim, pelo princı́pio da definição recursiva, fica bem definida a sequêcia de ı́ndices
(nk )k∈Z+ tal que
n1 := 1
e }
nk := min n ∈Z +: n  | − a| < k1
nk−1 , xn ,

para k > 1 em Z + . Logo, a subsequência (xnk )k∈Z+ e´ tal que

|x − a|  k1 ,
nk

para todo k ∈Z +, e, consequentemente,


lim xnk = a.
k →+∞

237
Exercı́cio 5.61:
Uma famı́lia de conjuntos (Kλ )λ∈L chama-se uma cadeia quando, para quaisquer λ e µ L tem-se Kλ ∈ ⊂K µ ou

Kµ K λ . Prove que se ( Kλ )λ∈L é uma cadeia não vazia de compactos não-vazios então a interseção K = ∩∈λ L Kλ
é não vazia (e compacta).

Primeiramente, mostraremos que, dada um conjunto finito L ′ contido em L, o conjunto



λ L∈ ′

não é vazio. Procederemos por indução no n´umero de elementos de L′ . Para L′ = {µ}, temos a afirma¸cão
trivialmente já que

λ L ∈ ′
Kλ = K µ = . ̸ ∅
Suponhamos, como hipóteses de indu¸cão, que, para cada conjunto finito L ′′ de cardinalidade menor ou igual que
n Z+ , seja verdade que

̸ ∅
Kλ = .

λ L ′′

Sejam L ′ um subconjunto de L com cardinalidade n + 1 e λ ∈ L′ . Definimos s conjuntos 0

L′− := {λ ∈ L ′ \{λ } : K ⊂ K } 0 λ λ0

e
L′+ := λ { ∈ L′ \{λ } : K ⊂ K }. 0 λ0 λ
Temos que
Kλ0 ∩ ∩ λ L+
∈ ′ Kλ =
∩ λ L+ (Kλ
∈ ′ ∩K λ0 ) =
∩ λ L+
∈ ′ K λ0
= Kλ0
e, como L ′− tem cardinalidade menor ou igual que n,

Kλ0 ∩ λ L
∈ ′

Kλ = ∈ (Kλ ∩ Kλ0 )
λ L ′

∩ =
̸
=
∩∅ .
∈ Kλ
λ L ′

Assim,
∩ ∈ Kλ =
λ L ′ Kλ0 ∩ ∩∩ λ L+∈ ′ Kλ ∩ ∩ λ L
∈ ′


= Kλ0 λ∈L Kλ ∩ ′

= . ̸ ∅
Portanto, provamos a afirmação do inı́cio do parágrafo.

Fixemos λ 0 L.
Suponhamos, por absurdo, que

∅= K µ = K λ0 ∩ Kλ .

λ L λ L ∈
Desta forma terı́amos que
K λ0 ⊂ (R Kλ ), \
λ L ∈
ou seja, R Kλ λ∈L seria uma cobertura de K λ0 por conjuntos abertos em R . Como K λ0 é compacto, existiria um
{\ }
subconjunto finito L ′ de L tal que
K λ0 (R Kλ ). ⊂ \
λ L ∈ ′

Porém, isso implicaria que

Kλ = K λ0 ∩ Kλ = . ∅
λ L
∈ ∪{λ0 }

λ L ∈ ′

Contradizendo, já que L ′ ∪ {λ } é finito, o que foi provado no primeiro parágrafo desta demonstração.
0

238
Exercı́cio 5.62:
Se X ⊂ R e´ não-enumerável, então X ′ também o é.
Todos os pontos de X X ′ são isolados. De fato, dado x X X ′ , temos que x não é um ponto de acumulação
\ ∈ \
∩ −
de X e, consequentemente, existe ε > 0 tal que X (x ε, x + ε) = x . {}
Como todos os pontos de X X ′ são isolados, temos, pelo corolário 2 do Teorema 8, que este conjunto é
\
enumerável.
O conjunto X X ′ não é enumerável. De fato, como X = (X X ′ ) (X X ′ ), X não é enumerável e X X ′ é
∩ \ ∪ ∩ \
enumerável, devemos ter que X X ′ e´ não enumerável.

Por fim, como X ′ contém o conjunto não enumerável X X ′ , devemos ter que X ′ e´ não enumerável.

239
Exercı́cio 5.63:
Para todo X ⊂ R, X − X ′ e´ enumerável.
Todos os pontos de X X ′ são isolados. De fato, dado x X X ′ , temos que x não é um ponto de acumula¸
− ∈ − cão
∩ − {}
de X e, consequentemente, existe ε > 0 tal que X (x ε, x + ε) = x .
Como todos os pontos de X X ′ são isolados, temos, pelo corol´ario 2 do Teorema 8, que este conjunto é

enumerável.
Por fim, como X = X X ′ , temos que X X ′ = X X ′ . Logo, X X ′ e´ enumerável.
∪ − − −

240
Exercı́cio 5.64:
ao de um conjunto X R quando todo intervalo aberto de centro
Um número real a chama-se ponto de condensaç˜ ⊂
a contém uma infinidade não-enumerável de pontos de X . Seja F0 o conjunto dos pontos de condensa¸cão de um
conjunto F R. Prove que F0 e´ um conjunto perfeito (isto é, fechado, sem pontos isolados) e que F F 0 é
⊂ −
enumerável. Conclua daı́ o Teorema de Bendixon: todo fechado da reta é reunião de um conjunto perfeito com um
conjunto enumerável.

(I) F0 e´ fechado:

Mostraremos que todo a F0 é um ponto de condensação de F . Isto é, que, para um ε > 0 arbitrário,
(a −Deε,fato,
a + ε) ∩ F não é enumerável.

sendo a um elemento do feixo de F 0 , devemos ter que existe x ∈ (a − ε, a + ε) ∩ F . Assim, para ˜ε > 0
0
tal que
(a − ε, a + ε) ⊃ (x − ε̃, x + ε̃),
temos que
(a − ε, a + ε) ∩ F ⊃ (x − ε̃, x + ε̃) ∩ F.
Como x é um ponto de condensação de F , (x − ε̃, x + ε̃) ∩ F não é enumerável. Logo, ( a − ε, a + ε) ∩ F também
não é enumerável.

(II) Se I é um intervalo finito tal que F0 ∩ I = ∅ então F ∩ I é finito ou infinito enumerável


Suponhamos que F ∩ I infinito e n˜ao é enumerável. Provaremos que F ∩ I̸= ∅. 0
Definiremos uma sequência de intervalos compactos ([an , bn ])n∈Z+ contidos em I tais que, para todo n ∈Z +,

[an , bn ] ∩ F é infinito e não é enumerável, (5.9)

[a , b ] ⊃ [a
n n ,b ]
n+1 n+1 (5.10)
e
bn = b n−a1 .
−a
2
n − (5.11)

Verificaremos que a sequência ([an , bn ])n∈Z+ fica bem definida por

[a1 , b1 ] := I

e  an−1 , bn −1 −a
2
n−1
, se an−1 , bn 1 −2 an 1

F −

[an , bn ] := é infinito e não é enumerável; (5.12)
 bn −1 −a
2
n−1
, bn−1 , caso contr´ario,

para n > 1, e possui as propriedades ( 5.9), (5.10) e (5.11). O intervalo [ a1 , b1 ] = I , satisfaz ( 5.9) e (5.11) pelas
hipóteses sobre I . Suponhamos que os intervalos [ a1 , b1 ],..., [an−2 , bn−2 ] e [an−1 , bn−1 ] estejam bem definidos por

(5.12) e satisfazem as condi¸cões ( 5.9), (5.10) e (5.11). Como [ an−1 , bn−1 ] F é infinito e não é enumerável, temos
− −
que [ an−1 , (bn−1 an−1 )/2] ou [( bn−1 an−1 )/2, bn−1 ] são infinitos e n˜ao são enumeráveis. Desta forma, (5.12)
define [an , bn ] de forma que ( 5.9) e (5.10) são prontamente satisfeitos. Também temos que [an , bn ] satisfaz ( 5.11)
pois
bn an =
bn−1 an−1 b a
= n 1.
− −
− 2 2 −
Assim, temos, pelo Principio da Definição Indutiva, que existe uma sequência de intervalos compactos ([an , bn ])n∈Z+
contidos em I que satisfaz ( 5.9), (5.10) e (5.11).
Pelo Teorema 12 do Capı́tulo 5, a propriedade (5.10) da sequência de compactos ([an , bn ])n∈Z+ implica que
existe x 0 ∈∩ ∞ [an , bn ]. Mostraremos que x 0 F 0 . ∈
n=1
Seja ε > 0. Para algum n Z+ , temos que

b a
ε > n−1 .

2

241
Como x 0 ∈ [a n , bn ], segue que
(x − ε, x + ε) ⊃ [a , b ].
0 0 n n

Logo, (x0 − ε, x 0 + ε) ∩ F é infinito e não enumerável já que


(x − ε, x + ε) ∩ F ⊃ [a , b ] ∩ F
0 0 n n


e [an , bn ] F é infinito e não é enumerável por ( 5.9). Com isso, devemos ter que x 0 ∈F .0
∈ ∩
Portanto, x 0 I F0 = [a1 , b1 ] F0 . ∩
(III) F0 não possui pontos isolados


Suponhamos que exista x0 em F0 e ε > 0 tais que (x0 2ε, x0 +2ε) F0 = x0 . Provaremos que (x0 ε, x0 +ε) F ∩ { } − ∩
é finito
Paraou infinito
todo n Zenumerável.

+ , a inclus˜ao

x0 − ε, x − n +ε 1
0 = x0 − ε, x − n +ε 1 ⊂ (x − 2ε, x )
0 0 0

implica que
x0 − ε, x − n +ε 1 ∩ F
0 0 = . ∅
Assim, pelo item (II), temos que
x0 − ε, x − n +ε 1 ∩ F
0

é finito ou infinito enumerável.


Analogamente,
ε
x0 +
n+1
, x0 + ε ∩F
é finito ou infinito enumerável, para todo n Z+ . ∈
Segue que

(x0 ε, x0 + ε) F = ( ∩  ( 
= (x0 ε, x0 ) F −
( x0 ∩ ∪ { } ∩ F ) ∪ (x , x + ε) ∩ F 0 0
ε
= n∈Z+ x0 ∪
ε, x0 n+1 − − ∩ F ∪ ({x } ∩ F ) 0
ε
∪ ∪ ∈ x + ,x + ε ∩F
n Z+ 0 n+1 0

é finito ou infinito enumerável já que

∪∈
n Z+ x0 − ε, x − n +ε 1 ∩ F
0

e
ε
∪∈
n Z+ x0 +
n+1
, x0 + ε ∩F
são uniões enumeráveis de conjuntos finitos ou infinitos enumeráveis.

(IV) F −F 0 e´ enumerável

Para cada x ∈ F − F , o conjunto


0

{ε ∈ R; 0 < ε < 1 e (x − ε, x + ε) ∩ F é finito ou infinito enumerável}


não é vazio e é limitado. Logo, podemos definir, para cada x ∈ F − F , 0

ε := sup {ε ∈ R; 0 < ε < 1 e (x − ε, x + ε) ∩ F é finito ou infinito enumerável}.


x

Desta forma,
(x − ε ,x + ε ) ∩ F
x x

242
é finito ou infinito enumerável.

Seja E um subconjunto denso e finito ou infinito enumerável de F F0 (cuja existência é garantida pelo Teorema
6 do Capı́tulo 5). Provaremos que
F F0 − ⊂∪
e∈E (e −
εe , e + εe ) F. ∩

E, deste fato, concluiremos que F F0 é finito ou infinito enumerável já que cada ( e εe , e + εe ) − ∩ F , para e ∈ E ,
é finito ou infinito enumerável e E é finito ou infinito enumerável.
∈ − − ∈ −
Seja x F F0 . Como E é denso em F F0 , existe e (x εx /2, x + εx /2). Desta forma

(e − ε /2, e + ε /2) ∩ F ⊂ (x − ε , x + ε ) ∩ F
x x x x

é finito ou infinito enumerável. Logo, pela defini¸cão de ε e , temos que ε e  εx /2. Assim,

x ∈ (e − ε /2, e + ε /2) ⊂ (e − ε , e + ε ).
x x e e

Portanto, concluimos que F − F ⊂ ∪ ∈ (e − ε , e + ε ) ∩ F .


0 e E e e

(V) Teorema da Bendixon: Todo fechado de R é uma união de um conjunto perfeito e um conjunto enumerável.

Seja F um conjunto fechado. Denotemos por F 0 o conjunto dos seus pontos de condensa¸cão.
Todo ponto de condensa¸cão de F e´ um ponto de acumulação de F . De fato, para todo x F0 , temos que ∈
− ∩ ∈
(x ε, x + ε) F é infinito e enumerável, para todo ε > 0. Logo, para todo x F 0 , temos que ( x varepsilon, x + −
ε) (F x ) = , para todo ε > 0. Assim, todo ponto de F 0 é um ponto de acumulação de F . Isto é, F 0 F ′ .
∩ −{ } ̸ ∅ ⊂
Como
F0 F ′ F, ⊂ ⊂
∪ −
Temos que F = F0 (F F0 ). Assim, pelos intens (I), (III) e (IV), temos que F e´ a união do conjunto perfeito F 0
e o conjunto finito ou infinito enumer´avel F F0 . −

243
Capı́tulo 6

Limites de Fun¸cões

244
Exercı́cio 6.01:

x
→a
̸
Na definição do lim f (x), retire a exigência de ser x = a. Mostre que esta nova defini¸cão coincide com a anterior

no caso a / X mas, para a ∈ X, o novo limite existe se, e somente se, o antigo existe e é igual a f (a).
Seja L = lim f (x) pela defini¸cão antiga.
x
→a
• a ∈/ X
Dado ε > 0, existe δ > 0 tal que 0 < x a < δ, x
| − | ∈ | − |
X, implica f (x) L < ε. Então, como a ∈ X, se
x a < δ, x X, então f (x) L < ε. Portanto, ainda temos L = lim f (x).
| − | ∈ | − | x →a
• a ∈ X e f (a) ̸= L.
Tomando ε = |L − f (a)| > 0 temos que para todo δ > 0 existe x ∈ X tal que |x − a| < δ e |f (x) − L| ≥ ε ( a
saber x = a). Portanto, lim f (x) não mais existe.
• a ∈ X e f (a) = L
Temos que dado ε > 0 existe δ > 0 tal que 0 < |x − a| < δ, x ∈ X, implica |f (x) − L| < ε. Mas, além disso,
|f (a) − L| = 0 < ε. Assim, para todo x ∈ X tal que |x − a| < δ temos que |f (x) − L| < ε. Portanto, ainda
temos lim f (x) = L.

Por fim, se lim f (x) = L pela defini¸cão nova, então f e L satisfazem também as condições da defini¸cão antiga.
→a
x
Logo, lim f (x) = L tamb´
em pela definição antiga.
x →a

245
Exercı́cio 6.02:
Considere o seguinte erro tipográfico na defini¸cão de limite:

∀ε > 0∃δ > 0; x ∈ X, |x − a| < ε ⇒ |f (x) − L| < δ.


Mostre que f cumpre esta condi¸cão se, e somente se, é limitada em qualquer intervalo limitado de centro a. No
caso afirmativo, L pode ser qualquer n´umero real.


( ) Seja I um intervalo de comprimento ε e centrado em a. Então,

(a − ε, a + ε) ⊃ I
e, pela hip´otese, existe δ > 0 tal que
|f (x)| − |L| ≤ |f (x) − L| < δ,
para todo x ∈ (a − ε, a + ε). Logo, para todo x ∈ I temos que
|f (x)| < δ + |L|
e, consequentemente, f e´ limitada em I .
( ) Seja ε > 0. Existe A R tal que para todo x
⇐ ∈ ∈ (a − ε, a + ε) temos que |f (x)| < A. Da´ıtemos que para
todo x R tal que x a < ε temos
∈ | − |
|f (x) − L| ≤ |f (x)| + |L| < A + |L|.
||
Assim, tomando δ = A + L teremos a condição que querı́amos. E como ε e´ arbitrário a afirma¸cão está provada.

246
Exercı́cio 6.03:
Seja X = Y ∪ Z, com a ∈ Y ′ ∩ Z ′. Dada f : X → R, tomemos g = f |Y e h = f |Z. Se lim→
x a
g(x) = L e lim h(x) = L
x→a
então lim f (x) = L.
x →a

Seja ε > 0 dado. Ent˜ao existem δ 1 e δ 2 positivos tais que se

x ∈ (a − δ , a + δ ) ∩ Y
1 1

ou
x ∈ (a − δ , a + δ ) ∩ Z
2 2

implicam
|f (x) − L| < ε.
{
Fixemos δ = min δ1 , δ2 . }
Seja
x ∈ (a − δ, a + δ ) ∩ X.
Temos que x ∈Y ou x ∈ Z. No primeiro caso
x ∈ (a − δ, a + δ ) ∩ Y ⊂ (a − δ , a + δ ) ∩ Y.
1 1

Isso implica que


|f (x) − L| < ε.
Analogamente, no segundo caso, também temos que

|f (x) − L| < ε.
Conclu´
ımos, assim, que em todo caso

x a <δ,x X f (x) L < ε.


| − |
Como ε e´ arbitrário, então temos o resultado. ∈ ⇒| − |

247
Exercı́cio 6.04:
Seja f : R \ → R definida por f (x) = 1 +1e 1/x
. Então lim+ f (x) = 0 e lim f (x) = 1.
x →0 x →0

Seja f : R \ {0} → R \ {0} dada por f (x) = x1 . Temos então que


1

lim f (x) = +∞ e lim f (x) = −∞.


1 1
x → 0+ → x 0

y
Seja f 2 : R → R \ {0} dada por f (y) = 1 + e . Então,
2

lim f (y) = +∞ e lim f (y) = 1.


2 2
y + y
→ ∞ →−∞
Por fim, seja f 3 : R \ → R dada por f (z) = z1 . Então
3

z →lim
+∞
f3 (z) = 0 e lim f3 (z) = 1.
z →1

Pelo Teorema 9, temos que


x
lim+ f (x) = lim+ (f3 f2 f1 )(x) = 0
→0 x →0
◦ ◦
e
x
lim f (x) = lim (f3 f2 f1 )(x) = 1.
→0 −
→0
x −
◦ ◦

248
Exercı́cio 6.05:
Seja f (x) = x + 10 sin x para todo x ∈ R . Então x
lim f (x) = +∞ e lim f (x) = −∞. Prove o mesmo para a
→ +∞ x→−∞
x
função g(x) = x + sin x.
2

Da relação
−1 ≤ sin x ≤ 1,
para todo x ∈ R, segue a rela¸cão
x
2
=x − x2 ≤ g(x) ≤ x + x2 = 3x2 ,

∈∈
para todo x R. 2
Seja A R arbitrário. Tomemos B = 2A e C = A. Se x
3
∈ R e x > B = 2A, temos que
g(x) ≥ x2 > A.
E como B só depende de A, segue que
x
lim g(x) = +
→ +∞
∞.
Se x ∈ R e x < C = 23 A, temos que
g(x) ≤ 32 x < A.
E como C só depende de A, segue que
x
lim g(x) =
→−∞
−∞.

249
Exercı́cio 6.06:
Seja f : X R monótona, com f (X ) [a, b]. Se f (X ) é denso no intervalo [a, b] então, para cada c
→ ⊂ ∈ X ′ ∩ X−′ ,
+

x −
x→c

tem-se lim f (x) = lim+ f (x). Se c X então este limite é igual a f (c).
→c

Como c ∈ X ′ ∩ X−′ e f : X → R é monótona, pelo Teorema 12, existem os limites


+

L = lim + f (x) e M = lim f (x).


x→c x →c −

Além disso, como f (X ) ⊂ [a, b] e [a, b] é compacto temos que L = lim f (xn )( para alguma sequência em (c, b]) e
n →∞
M = lim f (yn )( para alguma sequência em [a, c)) estão em [ a, b].
n

Suponhamos
→∞ que f é não-decrescente. Os casos em que f é crescente, decrescente ou não-crescente são demons-
trados de forma an´aloga.

Seja x X. Se x < c então
≤ { ∈
f (x) M = sup f (y); y X e y < c . }
Também, f (x) ≤ f (y) para todo y > c e, consequentemente,
f (x) ≤ L = inf {f (y); y ∈ X e y > c}.

Assim, f (x) ∈
/ (min{L, M }, max{L, M }). Se x > c, obtemos de forma análoga que f (x) ∈ / (min{L, M }, max{L, M }).
Se x = c, então
M ≤ f (x) ≤ L
pois f (y) ≤ f (x) para todo y < c = x e f (x) = f (y) para todo y > c = x já que f e´ não-decrescente.
Se c ∈/ X, então
∅ = (min {L, M }, max{L, M }) ∩ f (X ),
o que implica que L = M pois caso contr´ario haveria um aberto em [ a, b] sem nenhum ponto de f (X ).

Se c X, devemos ter
M = f (c) = L

pois caso contrário haveria um aberto


(M, f (c)) ou ( f (c), L)
não vazio em [ a, b] sem nenhum elemento de f (X ) pelo que foi dito acima. Neste caso, pelo Teorema 11, temos
que lim f (x) = f (c).
x →c

250
Exercı́cio 6.07:
Demonstre o Teorema 2.

Suponhamos que lim f (x) = L. Dado ε > 0, existe δ > 0 tal que se x
x→ a
∈ X e 0 < |x − a| < δ, então |f (x) − L| < ε.
Da´ı segue que se y ∈ Y ⊂ X e 0 < |y − a| < δ, então |g(y) − L| = |f (x) − L| < ε. Como ε e´ arbitrário, temos que
lim g(x) = L.
x→a
∩ −
Suponhamos que Y = I X, com I aberto contendo a. Então, existe δ0 > 0 tal que ( a δ0 , a + δ0 ) I . Seja ⊂
∈ | − | | − |
ε > 0 arbitrário. Existe δ1 > 0 tal que y Y e 0 < y a < δ1 temos que g(y) L < ε. Então, tomando-se
{ } ∈ | − |
δ = min δ0 , δ1 , temos que se x X e 0 < x a < δ então

x ∈ (X \ {a}) ∩ (a − δ, a + δ ) ⊂ Y .
|
Assim, f (x) − L| = |g(x) − L| < ε. Conclu´ımos que lim
→x
f (x) = L.
a

251
Exercı́cio 6.08:
Sejam f : X R monótona e a X+′ . Se existir uma sequência de pontos xn
→ ∈ ∈ X com x n > a, lim xn = a e
lim f (xn ) = L, então lim+ f (x) = L.
x →a

Consideremos o caso em que f e´ não-decrescente e ( xn )n∈N e´ tal que xn > a para todo n ∈ N , lim x n =ae
lim f (xn ) = L.

Dado x X tal que x > a, devemos ter
L < f (x).
∈ ∩ ∞
De fato, se existisse x X (a, + ) tal que f (x) < L existiria uma subsequência de (f (xn )) que n˜ao tende a L.
Por exemplo, seja ( xnk ) dada por
{ xn0 ∈ (a, x) ∩ {x } ∈
n n N xnk ∈ (a, x nk 1 )
− ∩ {x }
n n>nk −1 .

Então,
f (xnk ) ≤ ... ≤ f (x ) ≤ f (x) < L.
n0

Logo, L não é limite da sequência (f (xnk )). Assim, dado ε > 0, tomemos n0 N tal que f (xn0 ) L < ε e
∈ | − |

δ = x xn0 a. Da´ −
ı, se 0 < x a < δ teremos que a < x < x n0 e, pelo que foi dito acima, L < f (x) f (xn0 ). Logo, ≤
|f (x) − L| ≤ |f (x ) − L| < ε.
n0

Portanto, lim+ f (x) = L.


x →a
Os casos em que f é crescente, não-crescente e decrescente são análogos.

252
Exercı́cio 6.09:
Se f : X → R e´ monótona então o conjunto dos pontos a ∈ X ′ para os quais n˜ao se tem lim f (x) = lim+ f (x) é
x →a −
→a
x
enumerável.

Seja D o conjunto dos a ∈ X−′ ∩ X ′


+ tais que

x→a
̸
lim+ f (x) = lim f (x).
x →a −

Tais limites existem pelo Teorema 12. Suponhamos que f seja não-crescente. Pela demonstração do Teorema 12,

para cada a D, temos que
xlim
→ a −
{
= sup f (x); x ∈ X,x < a } (6.1)
e
lim+ = inf {f (x); x ∈ X,x > a }. (6.2)
x→a
Para cada a ∈ D, seja
Ia = ( lim f (x), lim+ f (x)).
x →a −
x →a
Dados a, b ∈ D tais que a < b temos que existe x ∈ (a, b) ∩ X. Assim, por ( 6.1) e (6.2) temos que
lim f (x) ≤ f (x) ≤ lim f (x).
→ x a+ → x b −

Da´ı, segue que I ∩ I = ∅.


a b
Do parágrafo anterior conclu´ ̸ b então I ∩ I = ∅. Assim, qualquer função h : D → Q
ımos que se a e b ∈ D e a = a b
tal que h(a) ∈ I ∩ Q (tais fun¸cões existem gra¸cas ao axioma da escolha) é injetiva. Conclu´ımos daı́ que D é
a
enumerável.
Os casos em que f é crescente, não-crescente e decrescente são análogos.

253
Exercı́cio 6.10:
Enuncie e demonstre para funções o análogo do Teorema 14 do Capı́tulo IV.

(1) Sejam f , g : X → R e a ∈ X ′ . Se g e´ limitada inferiormente e lim ∞


→ f (x) = + , então lim
x a

→ f (x) + g(x) = + .
x a
De fato, seja c ∈ R um limitante inferior de g. Seja dado A ∈ R arbitrário. Existe δ > 0 tal que se x ∈
(a − δ, a + δ ) ∩ (X \ {a}), então f (x) > A − c.
Assim, se x ∈ (a − δ, a + δ ) ∩ (X \ {a})

f (x) + g(x) > (A c) = (c) = A.

Segue daı́ que lim f (x) + g(x) = + ∞. −


→a x

(2) Sejam f, g : X → R e a ∈ X ′ . Se existe um limitante inferior c > 0 de g e lim f (x) = +∞, então lim f (x).g(x) =
x→a x→a
+ ∞.
A
De fato, seja A ∈ R arbitrário. existe δ > 0 tal que se x ∈ (a − δ, a + δ) ∩ (X \ {a}), então f (x) > c.
Assim, se x ∈ (a − δ, a + δ) ∩ (X \ {a}), então
A
f (x).g(x) > ( ).(c) = A.
c
Segue daı́ que lim f (x).g(x) = + .
→a x

(3) Sejam f : X → R tal que f (x) > 0 para todo X e a ∈ X ′ . Temos neste caso que
1
lim f (x) = 0
x →a
⇔ limx a

→ f (x) = + .

De fato,
1

( ) Seja A ∈ R ∩ (0, +∞) arbitrário. Existe δ > 0 tal que se x ∈ (a − δ, a + δ) ∩ (X \ {a}), então 0 < f (x) < A.
Assim, dado x ∈ (a − δ, a + δ ) ∩ (X \ {a}), então
1
A< .
f (x)

Segue daı́ que lim f (x) = + .


→a
x

1 1

( ) Seja ε > 0 arbitrário. Existe δ > 0 tal que x ∈ (a − δ, a + δ) ∩ (X \ {a}) implica
f (x)
> . Assim, para
ε
todo x ∈ (a − δ, a + δ) ∩ (X \ { } a ), temos que 0 < f (x) < ε. Segue daı́ que lim f (x) = 0.
→ x a

(4) Sejam f , g : X → R, tais que f e g s ao fun¸cões positivas e a ∈ X ′ .


f (x)
(a) Se existe c > 0 tal que f (x) > c para todo x ∈ X e lim g(x) = 0, então lim =+ ∞.
→ x a→ x a g(x)
f (x)
(b) Se f é limitada e lim g(x) = +
x→a
∞, então lim→ x a g(x)
= 0.

De fato,
1
(a) Temos que lim
g(x)
=+ ∞ pelo item (3). Daı́ o resultado segue aplicando-se o item (2) `as funções f e g1 .

254
1 1
(b) Seja k > 0 tal que 0 < f (x) < k. Então,
f (x)
> para todo x
k
∈ X.
1
Como lim = +
x→a
∞ , temos que lim
x→a g(x)
= 0. Assim,

1
g(x) f (x)
x
lim
→a f (x)
= lim
x →a 1
=+ , ∞
g(x)

pelo item (a).


Portanto, temos pelo item (3) que
f (x) 1
lim = lim g(x)
= 0.
→a g(x)
x x →a
f (x)

255
Exercı́cio 6.11:
Dado a > 1, defina f : Q → R pondo, para cada pq ∈ Q, f p
q
= a p/q . Prove que lim f (x) = 1. conclua que para
x →0
′ ′
b b b
cada b ∈ R existe lim f (x), sendo este limite igual a f (b) se b ∈ Q. Chame este limite de a . Prove que a .a = a b+b
→ x b

′ b b
e que b < b ⇒ a < a .

Como a > 1, temos pelas propriedades básicas da multiplicação que f é crescente.


Seja n 0 N tal que se n 0 > a. Então, para todo n > n0

√n > √a > 1.
n n

Logo, como lim n → +∞ √n = 1, temos que n


n
lim
→ +∞
f (1/n) = 1. Segue do exercı́cio 8 que lim+ f (x) = 1. Também
x →0
temos que
1 1 1
n→+∞

lim f ( 1/n) = lim
n→+∞ f (1/n)
=
lim f (1/n)
= = 1.
1
n →+∞
E, novamente pelo resultado do exercı́cio 8, temos que lim f (x) = 1. Concluı́mos, então, que lim f (x) = 1.
x→0 − x →0
Seja b ∈ R.
1. b ∈ Q.
lim f (b + 1/n) = lim f (b)f (1/n) = f (b)
n→ +∞ n→+∞

n
lim f (b
→ +∞
− 1/n) = lim −
→+∞ f (b)f ( 1/n) = f (b).
n

E do exercı́cio 6.8 temos que lim f (x) = f (b).


x →b
2. b ∈ R \ Q.
m u m u
Sejam xn = tal que m = max u Z ; < b e yn = tal que m = min u Z ; > b . Segue que se
{ ∈ } { ∈ }
n n n n
m m+1

xn = n então y n = n (pois b / Q). As sequências (xn ) e (yn ) s ao, respectivamente, n ao-decrescente e
n ao-crescente. Isso implica que as sequências (f (xn )) e ( f (yn )) também o são. Daı́ segue que lim f (xn ) e
n →∞
lim f (yn ) existem pois f (xn ) < f (yn ) para todo n N, isto é, tais sequências são limitadas. Por fim, temos

n →∞
que
1 f (yn ) lim f (yn )
1 = lim f = lim = .
n f (xn ) lim f (xn )

Da´ı, pelo exercı́cio 6.8, temos que

lim f (x) = lim f (xn ) = lim f (yn ) = lim+ f (x).


x →b − n→∞ n →∞ x →b
b
Portanto, o limite lim f (x) existe e denotamos tal limite por a .
x
→b
Passaremos agora a verificação das duas propriedades.
′ ′
b b
• a .a = a b+b
Sejam (xn ) e (yn ) sequências que tendem à b e b ′ , respectivamente. Então (xn + yn ) tende `a b + b′ e

ab .ab = lim f (x). lim f (x) ′

x→b x→b
= lim f (xn ). lim f (yn )
n→∞ n→∞
= lim f (xn ).f (yn )
n →∞
= lim f (xn + yn )
n →∞

= lim f (x) = a b+b .


x →b+b ′

256

• b < b′ ⇒ a b
< ab .
Sejam r 1 , r2 ∈ Q ∩ (b, b′ ) tais que r 1 < r2 . Então, como f é crescente
b
{
a = inf f (r); r ∈ Q \ {0} tal que r < b }
e ′

ab = sup f (r); r
{ ∈ Q \ {0} tal que r < b }.
Assim, ′

ab ≤ f (r ) < f (r ) ≤ a
1 2
b

e o resultado segue.

257
Exercı́cio 6.12:
Dado a > 1, defina g : R → R, pondo g(x) = ax (veja o exercı́cio anterior). Prove que lim g(x) = +∞ e
x→+∞
lim g(x) = 0.
x
→−∞

Seja a = 1 + ε. Dado A > 0 arbitrário, tomemos n0 N tal que n0 ε > A. Assim, dado x > n0 temos, pela

desigualdade de Bernoulli, que
A < 1 + n0 ε (1 + ε)n0 = g(n0 ) < g(x).

Segue daı́ que lim g(x) = + .
x →+∞

1
Dado, novamente, A > 0, tomemos n 0 N tal que n 0 ε > . Então, dado x < n0 temos que

1
∈ A −
A>
n0 ε + 1
≥ (1 +1ε) n0

= g( n0 ) > g(x) > 0.

Daı́ segue que lim g(x) = 0.


x →−∞

258
Exercı́cio 6.13:
Seja p : R →R, um polinˆomio real. Se o coeficiente do termo de grau mais elevado de p é positivo então
x→+∞

lim p(x) = + e lim p(x) é igual a + ou
x →−∞
∞ −∞
, conforme o grau de p seja par ou ı́mpar.

Seja p(x) = an xn + a n−1 xn−1 + ... + a 1 x + a 0 . Provaremos por indução em n que lim p(x) = + . O caso ∞
n→∞
n = 1 é trivial. Suponhamos que o resultado seja verdadeiro para o polinômio de grau n − 1. Seja p(x) o polinômio
tomado inicialmente. Temos que
p(x) = x(an xn−1 + ... + a1 ) + a0 .
Pela hipótese de indução, temos que lim an xn−1 + ... + a1 = + , o que nos garante que lim
∞ ∞ pelos
n→ +∞ →+∞ p(x) = +
n
resultados do exerc´
ıcio 6.10.
Suponhamos agora n = 2k − 1. Provaremos por indução sobre k que x
lim
→−∞ p(x) =
−∞. Para k = 1, o resultado
é trivial. Temos que
a1
p(x) = x 2 [(a2k−1 x2(k−1)−1 + ... + a2 ) + ] + a0 .
x
a1
Assim, sabendo que lim x = 0 e supondo que lim (a2k−1 x2(k−1)−1 + ... + a2 ) = −∞, temos novamente pelos
x →−∞ x→−∞
resultados do exercı́cio 6.10 que lim p(x) =
x→−∞
−∞.
Suponhamos agora que n = 2k. Então

p(x) = x(a2k x2k−1 + ... + a1 ) + a0 .

E, pelo resultado do parágrafo anterior, temos que

lim (a2k x − + ... + a1 ) = −∞.


2k 1
x→−∞
Assim, pelos resultados do exercı́cio 6.10 temos que lim p(x) = +
x→−∞
∞.

259
Exercı́cio 6.14:
Determine o conjunto dos valores de aderência da função f : R \ {0} → R, f (x) = sin(1/x)
1+e 1/x
, no ponto x = 0.

Temos que
1/x
0 ≤e e − 1 ≤ sin(1/x) ≤ 1
para todo x ∈ R \ {0}. Então,
−1 ≤ sin(1/x)
1+e
≤ 1.
1/x

Logo, o conjunto A dos pontos de aderência de f no ponto 0 é tal que A [ 1, 1]. ⊂−


1 1/n
∈−
Seja λ [ 1, 1]. Tomemos θ = arcsin λ. Então, definindo x n = θ 2πn temos que nlim
− →+∞ xn = 0 e lim e = 0,
1
pois lim
n→+∞ xn
= −∞
. Assim,

f (xn ) =
sin(θ 2πn) −
=
λ
λ →
1 + eθ−2πn 1 + eθ+2πn
quando n → +∞. Logo, λ ∈ Λ. E segue que A ⊃ [−1, 1]. Conclu´ımos que A = [−1, 1].

260
Exercı́cio 6.15:
| | || | | ||
Se lim x→a f (x) = L então lim x→a f (x) = L . Se lim x→a f (x) = L então o conjunto dos valores de aderência
de f no ponto a é L ,{ } {− } {− }
L e L, L .

Consideremos a função f : X R e a X.
→ ∈
Suponhamos que lim x→a f (x) = L. Dado ε > 0, existe δ > 0 tal que

|f (x) − L| < ε,
para todo x ∈ (X \{a}) ∩ (a − δ, a + δ). Desta forma, dado ε > 0, existe δ > 0 tal que

||f (x)| − |L||  |f (x) − L| < ε,


∈ \{ } ∩ − | | ||
para todo x (X a ) (a δ, a + δ ). Logo, lim x→a f (x) = L .
| | ||
Suponhamos que lim x→a f (x) = L . Provaremos que o conjunto A
dos pontos de aderência de f é {−L, L},
{ } {− }
L ou L . O faremos mostrando que L, L{ − } ⊃ A̸ ∅ = .
Seja L ′ ∈A . Então, existe uma sequência (xn )n∈Z+ em X tal que lim n→∞ f (xn ) = L′ . Segue que

|L′| = | lim n
| →∞ |f (x )| = |L|
→∞ f (x ) = lim
n
n
n

já que lim → |f (x)| = |L|. Logo, L ′ ∈ {L, −L}. Concluimos daı́ que {L, −L} ⊃ A.
x a

Pelo Teorema 13 do Capı́tulo VI, temos que L é um ponto de aderência da função g : X → R no ponto a se e
somente se para todos ε e δ > 0 vale

(L′
( 
− ε, L′ + ε) ∩ g (X \{a}) ∩ (a − δ, a + δ) ̸= ∅.
Suponhamos, por absurdo, que L e −L não são valores de aderência de f no ponto a. Segue que existem ε +,
ε− , δ e δ − > 0 tais que
+ ( 
(L − ε , L + ε ) ∩ f (X \{a}) ∩ (a − δ , a + δ ) = ∅
+ + + +

e
(L − ε− , L + ε− ) ∩ ( \{ } ∩ −
f (X a ) ∅ (a δ− , a + δ− ) = .
| | ||
Como lim x→a f (x) = L , existe

f (x) ∈ (L − ε, L + ε) ∩ | |( \{ } ∩ −
f (X
a ) (a δ, a + δ ) ,

com x ∈ (X \{a}) ∩ (a − δ, a + δ). Desta forma, terı́amos que


|f (x) − L| = ||f (x)| − |L|| < ε  ε , +

caso |f (x)| = ±f (x) e |L| = ±L, e

|f (x) − (−L)| = ||f (x)| − |L|| < ε  ε−,


caso |f (x)| = ±f (x) e |L| = ∓L. Consequentemente, seguiria que

+ +
(
f (x) ∈ (L − ε , L + ε ) ∩ f (X \{a}) ∩ (a − δ , a + δ + +)

,

pois x ∈ (X \{a}) ∩ (a − δ, a + δ ) ⊂ (X \{a}) ∩ (a − δ , a + δ ), ou


+ +

∈− − − ∩ (
f (x) ( L ε− , L + ε− ) f (X a ) (a δ− , a + δ− ) , \{ } ∩ − 
∈ \{ } ∩ − ⊂ \{ } ∩ −
pois x (X a ) (a δ, a + δ ) (X a ) (a δ− , a + δ− ). Em todos os casos, chegam os a uma contradição

com as hipóteses de que L e L não são valores de aderência de f no ponto a. Portanto, L ou L são valores de −
aderência de f .

261
Exercı́cio 6.16:
Dados um número real a e um conjunto compacto não-vazio K , obtenha uma fun¸cão f : R → R tal que o conjunto
dos valores de aderência de f no ponto a seja K .

Consideremos o ponto a R. Provaremos que para todo conjunto F fechado em R existe uma função f : R
∈ →R
tal que o conjunto dos pontos de aderência de f no ponto a e´ igual a F .
{ }
Seja E = en n∈Z+ um subconjunto enumerável de F tal que E = F (veja o Teorema 6 do Cap´ıtulo V).
Para cada número inteiro positivo n, definimos o conjunto
n
Cn := x ∈ R : |x − a| = 32 k
, para algum k ∈Z + .

Se n e m são inteiros distintos devemos ter que C n Cm = . De fato, se x C n Cm , existiriam k e k ′


∩ ∅ ∈ }∩ ∈Z + tais
que
3n 3m
2k
= x a = k
2
|− | ′

Como 2 e 3 s˜ao números inteiros coprimos, temos uma contradição. Logo, C n Cm = . ∩ ̸ ∅


Assim, podemos definir uma fun¸cão f : R R por →
f (x) =
en , se x ∈C , n
e1 , se x ∈ R\ ∪ ∈ n Z+ Cn.

Para todo δ > 0 e n ∈Z + existe k ∈Z + tal que 3 n /2k < δ e, consequentemente,

3n ( 
en = f a+
2k
∈ f (a − δ, a + δ)\{a} .
Desta forma, temos que f (a − δ, a + δ) = E . Logo, para todo δ > 0,
f (a δ, a + δ ) a = E = F.
− VI, o conjunto
Pelo Colorário 3 do Teorema 13 do Capı́tulo
( \{ } dos valores de aderência de f no ponto a é

(
f (a − δ, a + δ)\{a}  = E = E = F.
δ>0 δ>0

262
Exercı́cio 6.17:
Seja f : R→ R definida por f (x) = x se x e´ irracional, f ( pq ) = q se p
q e´ uma fração irredut´
ıvel com p > 0, f (0) = 0.
Mostre que f é ilimitada em qualquer intervalo não-degenerado.

Suponhamos que I seja um intervalo não degenerado com a = inf( I ) e b = sup( I ).


∈ ⊂
Mostraremos que, dado A > 0, arbitr´ario existe x (a, b) I tal que f (x) > A.
Sejam n Z+ , tal que

2
2n > A e ,
b a −
e
m
p := inf m Z; a < n
.

Desta forma temos que


∈ 2
p −1 a < p
2n 2n
e
p+1 p 1 2 −
= n + n < a + (b − a) = b.
2n 2 2
Logo,
p 2p + 1 p+1
a< < n+1 < n < b.
2n 2 2
2p+1
Assim, x = 2n+1
pertence a I e
f (x) = 2n+1 > 2 n > A.

263
Exercı́cio 6.18:
Sejam X , Y ⊂ R, f : X → R, g : Y → R com f (X ) ⊂ Y . Se, para a ∈ X ′ e b ∈ Y ′ tem-se lim
→ f (x) = b, lim
x a→
g(y) = c
y b
̸ ∈ X \{a}, então lim→ g(f (x)) = c. Mostre que a condi¸cão b ∈ Y ′ decorre de ser
e, além disso, f (x) = b para todo x
x a
f (x)̸= b para x =
̸ a.
Primeiramente, mostraremos que se f (x) = b para x = a então b Y ′ . De fato, se ε > 0, temos que existe,
̸ ̸ ∈
pois limx→a f (x) = b, δ > 0 tal que f (x) b < ε para todo x (a δ, a + δ ) (X a ). Como a X ′ , segue que
| −| ∈ − ∩ \{ } ∈
− ∩ \{ } ̸ ∅ ∈ −
(a δ, a + δ ) (X a ) = . Logo, existe x (a δ, a + δ ) (X a ), tal que ∩ \{ }
f (x) (b ε, b + ε) (Y b ).

Portanto, devemos ter que b ∈ Y ′ . ∈ − ∩ \{ }


→a

Provaremos, agora, que lim g f (x) = c. Seja ε > 0. Como lim g(y) = c, existe ν > 0 tal que
x y →b
|g(y) − c| < ε,
para todo y ∈ Y \{b} que satifaz a desigualdade
|y − b| < ν.
Tamb´
em existe δ > 0, pois lim f (x) = b, tal que
→a
x

|f (x) − b| < ν,
para todo x ∈ X \{a} que satisfaz a desigualdade
|x − a| < δ.
Assim, para todo x ∈ X \{a} tal que
temos que
|x − a| < δ,
|g ◦ f (x) − c| < ε.
x→a

Portanto, segue que lim g f (x) = c.

264
Exercı́cio 6.19:
Para todo número real x indiquemos, com [ x] o maior inteiro  x. Mostre que se a e b são positivos então
x b b b x
lim+ = e lim = 0.
x →0 a x a →0+ x a
x

Prove também que, no primeiro caso o limite à esquerda seria o mesmo mas no segundo caso o limite é + ∞ quando
x →
0 por valores negativos.

x b x b b
• lim+
x 0 a x
= lim
x 0 a x −
=
a
→ →
·
Pela definição de [ ], temos, para todo x ∈ R, que
b b b
x
−1< x

x
.

Assim, segue que


b
a
− xa = xa b
x
−1 <
x b
a x

xb
ax
b
= ,
a
para x > 0, e
b xb x b x b b
a
=
ax

a x
<
a x
−1 =
a
− xa ,
para x < 0. Logo,    
x b
a x
− ab <
x
a
,

para todo x R 0 .∈ \{ }
Dado ε > 0 e tomando-se δ = εa, para todo x ∈ R\{0} tal que
|x| < δ,
temos que    
x b
a x
− ab <
x
a
<
εa
a
= ε.

Portanto,
x b b
lim =
x→0 a x a
e, consequentemente,
x b x b b
lim+ = lim = .
x →0 a x →0 a x
x −
a
b x b x
• lim+
→0 x a
x
= 0 e lim
→0 x a
x −
=+ ∞
Consideremos a função f : R \{0} → R dada, no ponto x ∈ R\{0}, por
b x
f (x) = .
x a

Se x ∈ (0, a), temos que 0 < x/a < 1 e, consequentemente, [x/a] = 0. Desta forma, para todo x ∈ (0, a),
b x b
f (x) = = 0 = 0.
x a x
Por outro lado, se x ∈ ( −a, 0), temos que −1 < x/a < 0 e, consequentemente, [ x/a] = −1. Assim, para todo
x ∈ (−a, 0),
b x
f (x) =
x a
= −xb .
265
Portanto, temos que
b x
x
lim+
→0 x a
= lim+ f
x→0
| (0,a) (x) = lim+ 0 = 0.
x
→0
e
b x
lim
x→0 x a

= lim f
x→0 −
|−( a,0) (x) =
x
lim+
→0
− xb = + ∞.

266
Exercı́cio 6.20:
Dadas funções f e g : X → R, defina h = max {f, g} : X → R pondo
f (x), se f (x)  g(x),
h(x) =
g(x), se g(x)  f (x).

Seja a X ′ . Prove que se lim f (x) = L e lim g(x) = M então lim h(x) = N , onde N é o maior dos dois números
∈ x→a x→a x→a
L e M.

Suponhamos que L > M . Como lim f (x) = L e lim g(x) = M , tomando-se ε := (L


x→a x→a
− M )/2 existe δ > 0 tal
que para todo
∈ −
x (a δ, a + δ ) (X a ) ∩ \{ }
vale
f (x) ∈ (L − ε, L + ε) = L + M 3L M
,

2 2
e
∈ (M − ε, M + ε) =
g(x)
3M − L, L + M .
2 2
Assim, para todo x ∈ (a − δ, a + δ ) ∩ (X \{a}),

g(x) < f (x)

e, consequentemente,
h(x) = f (x).
Desta forma, temos que h | ∩ \{a}) = f |(a−δ,a+δ)∩(X \{a}) . Portanto, quando L > M ,
(a δ,a+δ) (X

x →a →a
x
|
lim h(x) = lim h (a−δ,a+δ)∩(X \{a}) (x) = lim f
→a
x
| − ∩ \{a}) (x) = L = N.
(a δ,a+δ) (X

De modo análogo, mostra-se que se M > L então lim


x→a h(x) = N .
Por fim, consideremos o caso em que L = M . Seja ε > 0. Como lim f (x) = L = N e lim g(x) = M = N , existe
x→a x→a
δ > 0 tal que para todo
∈ −
x (a δ, a + δ ) (X a ) ∩ \{ }
vale
f (x) ∈ (N − ε, N + ε)
e
g(x) ∈ (N − ε, N + ε).
Assim, para todo x ∈ (a − δ, a + δ) ∩ (X \{a}), tem-se que
h(x) ∈ {f (x), g(x)} ⊂ (L − ε, L + ε).

Desta forma, concluimos que


lim h(x) = N.
x →a

267
Exercı́cio 6.21:
Sejam f e g : X → R funções limitadas numa vizinhança do ponto a ∈ X ′ . Mostre que
lim sup(f + g)(x)  lim sup f (x) + lim sup g(x)
x →a x →a x →a
e que (− 
lim sup
x
→a f (x) = − lim
→ inf f (x).
x a

Enuncie e prove resultados análogos para lim inf( f + g) e para o produto de duas fun¸cões.

Pelos exercı́cios 3.38 e 3.40, temos que a soma e o produto de funções limitadas s˜ao também funções limitadas.
· →
Para funções limitadas R
Isto é, f + g e f g : X são funções limitadas.′
f eg:X R e a X , provaremos que
→ ∈
lim sup(f + g)(x)  lim sup f (x) + lim sup g(x),
x →a x →a x →a
lim inf(f + g)(x)  lim inf f (x) + lim inf g(x)
x→a x →a x →a
e (− 
lim sup
x
→a f (x) = − lim
→ inf f (x).
x a

Além disso, mostraremos que se f e g forem funções não negativas (i.e. f (X ) e g(X ) ⊂R +) então

lim sup(f g)(x) 


x →a
· ( lim sup f (x)
x →a
( lim sup g(x)
x →a

e ( ( 
x
lim inf(f g)(x) 
→a
· lim inf f (x)
x →a lim inf g(x) .
x
→a
→ R não forem não negativas, podemos ter que
Se f ou g : X

lim sup(f · g)(x) > lim sup f (x) lim sup g(x) .
x a x a x a

De fato, definindo f e g : R → R por
→ ( ( → 
0, se x Q,∈
g(x) = f (x) =
−1, se x / Q,

temos que ( ( 
x→0
·
lim sup(f g)(x) = 1 > 0 = 0 0 = · lim sup f (x)
x
→0
lim sup g(x) .
x→0
Se (xn )n∈Z+ e´ uma sequência em X então (f (xn ))n∈Z+ e (g(xn ))n∈Z+ são sequências em f (X ) e g (X ), respec-
tivamente. Como f (X ) e g(X ) são compactos (pois são fechos de conjuntos limitados, veja exercicio 5.30), existem
subsequências de (f (xn ))n∈Z+ e (g(xn ))n∈Z+ que são convergentes. Em particular, existe 1 uma subsequência
(xnk )n∈Z+ de (xn )n∈Z+ tal que ( f (xnk ))k∈Z+ e (g(xnk ))k∈Z+ são convergentes.

• lim sup(f + g)(x)  lim sup f (x) + lim sup g(x) e lim inf(f + g)(x)  lim inf f (x) + lim inf g(x)
x
→a x →a x →a x →a x
→a x →a

Sejam S ∈ R um valor de aderência da função f + g no ponto a e (x n )n Z+


∈ uma sequência em X tal que

lim xn = a e lim
n→ +∞ n →+∞(f + g)(xn ) = S.
Mostraremos que
lim inf f (x) + lim inf g(x)  S  lim sup f (x) + lim sup g(x).
→a
x x →a x →a x →a
1
Tome uma subsequência convergente (f (xmp ))p∈Z+ de ( f (xm ))m∈Z+ e extraia dela uma subsequência convergente (g(xmpk ))k∈Z+
de (g (xmp ))p∈Z+ . Assim, ( xnk )k∈Z+ , para n k := m pk , é como desejado.

268
E, como S ∈ R um valor de aderência arbitrário da fun¸cão f + g no ponto a, concluiremos que
lim sup(f + g)(x)  lim sup f (x) + lim sup g(x)
→a
x x →a x →a
e
lim inf(f + g)(x)  lim inf f (x) + lim inf g(x).
x →a x →a x →a
Como ( xn )n∈Z+ e´ uma sequência em X , segue que existe uma subsequência (xnk )k∈Z+ de (xn )n∈Z+ tal que
(f (xnk ))k∈Z+ e (g(xnk ))k∈Z+ são convergentes. Segue que lim k→+∞ f (xnk ) e lim k→+∞ g(xnk ) são ponto s de
aderência de f e g, respectivamente, no ponto a = lim k→+∞ xnk . Logo,
S = lim (f + g)(x k )
n
k → +∞
= lim f (xnk ) + lim g(xnk )
k → +∞ k → +∞

 xlim
→a sup f (x) + lim
x→a sup g(x)

e, analogamente,
S  lim inf f (x) + lim inf g(x).
x→a x →a

• lim sup
x
→a
(−  f (x) = − lim
→ inf f (x)
x a

Sejam VA (f ; a) e VA (−f ; a) os conjuntos dos valores de aderência no ponto a das funções f e −f .


Dado L ∈ V A(f ; a), existe uma sequência (x ) ∈ tal que a = lim → ∞ x e L = lim → ∞ f (x ). Assim,
n n Z+ n + n n + n

−L = − →lim∞ f (x ) = lim
→ ∞
(−f )(x ) ∈ V A(−f ; a).
n +
n
n +
n

Portanto, como L ∈ VA(f ; a) é arbitrário, concluimos que −VA (f ; a) ⊂ VA(−f ; a).


Analogamente, prova-se que −VA (f ; a) ⊃ V A(−f ; a). E, com isso, concluimos que −VA (f ; a) = V A(−f ; a).
Portanto, ( 
lim sup − f (x) = sup V A(−f ; a)
→ x a
= sup − V A(f ; a)
= inf (f ; a)
= − limVinf A f (x). ( 
→ x a

• x→0
·
lim sup(f g)(x) 
( lim sup f (x)
x →0
( x →0

lim sup g(x) e lim inf(f g)(x) 
x →a · ( x →a
(
lim inf f (x)

lim inf g(x) se f e
→a
x
g 0

Sejam P ∈ R um valor de aderência da função f · g no ponto a e (x ) ∈ uma sequência em X tal que n n Z+

→lim x = a e
∞ n
lim
+
·
→ ∞(f g)(x ) = S.
n
n +
n

Mostraremos que ( (  ( ( 
lim inf f (x) lim inf g(x)  P  lim sup f (x) lim sup g(x) .
x→a x →a x →a →a
x
∈ R um valor de aderência arbitrário da fun¸cão f g no ponto a, concluiremos que
E, como P
lim sup(f · g)(x)  lim sup f (x) lim sup g(x)
→ x a → →
( · (
x
a x a
e
lim inf(f g)(x) 
x →a
· lim inf f (x)
→a
x
lim inf g(x) .
x →a
Como ( xn ) n Z +
∈ (
e´ uma sequência em X , segue que existe uma subsequência (xnk )k∈Z+ de (xn )n∈Z+ tal que
(f (xnk ))k∈Z+ e (g(xnk ))k∈Z+ são convergentes. Segue que lim k→+∞ f (xnk ) e lim k→+∞ g(xnk ) são ponto s de
( 
aderência de f e g, respectivamente, no ponto a = lim k→+∞ xnk . Logo,
P = lim
k
(( →+∞ n ·
(f g)(x k )
=

lim f (xn )
k → +∞
lim g(xn )
k → +∞
k

lim sup f (x) lim sup g(x)


x→a x→a
(( k

269
e, analogamente, ( ( 
P  lim inf f (x) lim inf g(x) .
x
→a x
→a

270
Exercı́cio 6.22:
Seja f : [0, + ∞) → R uma função limitada em cada intervalo limitado. Se
lim [f (x + 1) − f (x)] = L
→ ∞ x +

então
f (x)
lim = L.
x→ +∞ x

Como f é limitada em todos os intervalos limitados de [0 , + ), para todo C ∞ ∈Z + existe MC > 0 tal que
f (λ) < M , para todo λ [C 1, C ).
| | ∈ −∈ − ∈
Para C C Z+ , λ [C 1, C ) e k Z+ , temos que


−L  f (λ+k)
λ+k
  f (λ+k)
λ+k
f (λ+k)
k − + f (λ)k

+ f (λ+k)

    k

 − f (λ)
k −

 L

 f (λ+k)
λ+k
f (λ+k)
k − + k + f (λ)

+ f (λ+k) f (λ)


λ
λ+k

  
f (λ+k)
k
 +
k
f (λ)
k
  −  − − − 
+
k

k
L
f (λ+k) f (λ)
k L
λ f (λ+k) f (λ) f (λ)
 λ+k


k

+ f (λ)k
k

+
 −  −  −  −| |
L +
f (λ+k)
k
k
f (λ)
k
+ L
L
+

 C
k
f (λ+k)
k
f (λ)
k
−  −    | | 
L + f (λ) k + L +
+ f (λ)k + f (λ+k)k
f (λ)
   − − − − 
k L
C C f (λ) | C f (λ+k)
| f (λ)

< 1
k C
(| || |
k L + (1 + k ) k + (1 + k )

L + (1 + C
k )M C + (1
  − −
+ C
k )
k
f (λ+k)
k
k
f (λ)
k
L
L

Seja ε > 0. Provaremos que existem C e n0 Z tal que para todo k  n0 e λ [C 1, C ) valem as desigualdades
1 C
∈ε   − ∈ −−  
C f (λ + k) f (λ) ε
k
||
C L + 1+
k
M <
2
e 1+
k k k
L <
2
.

Assim, dado x  (C − 1) + n , existem λ ∈ [C − 1, C ) e k ∈ Z tais que x = λ + k, k  n e, consequentemente,


0 + 0
  
f (x)
−L =
x
−L f (λ+k)
λ+k

(
< 1
C |L| + (1 + )M + 1 +
k − −L C
k C
 (   C
k
f (λ+k)
k
f (λ)
k

ε ε
< 2 + 2
= ε.

Com isso, conclui-se que lim x→+∞ f (x)


x = L.( 
Por hipótese, lim x→+∞ f (x + 1) f (x) = L. Fixemos C Z+ tal que para todo numero real x
− ∈  C − 1 vale
a desigualdade
ε
f (x + 1) f (x) L < .
3
| − − |
Assim, para todo λ [C 1, C ) e k Z+ , temos que
∈ − ∈
f (λ + k) f (λ)

k 1
f (λ + i + 1) f (λ + i) L
 k − k −L   =
i=0
k − −kk 
k 1−

|f (λ + i + 1) − f (λ + i) − L|
i=0
k
k 1− ε/3
<
i=0
k
ε
= 3.

271
Como
1
lim =0
k → +∞ k
e
C
k
lim
→ +∞
||
C L + 1+
k
MC = C L + M,||
segue que
1 C
k
lim
→ +∞ k
||
C L + 1+
k
MC = 0.

Além disso,
C
lim 1+ = 1.
k → +∞ k
Fixemos n 0 ∈Z + tal que para todo k  n0 valem as desigualdades
 1
||
C L + 1+
C
MC
 <
ε
k k 2
e
C 3
1+ < .
k 2
Por fim, para λ ∈ [C − 1, C ) e k  n , temos que
0
 1
||
C L + 1+
C
MC
 <
ε
k k 2
e  
C f (λ + k)
1+
k k
− f (λ)
k
−L <

23
ε
= .
2
Como quer´
ıamos demonstrar.

272
Exercı́cio 6.23:
Seja f : R → R definida por
f (x) = x + ax sen(x).·
||
Mostre que se a < 1 então
x
lim f (x) = +
→ +∞
∞ e lim
x →−∞ f (x) =
−∞.

Suponhamos que 1 −| |
a > 0.
Seja A > 0. Provaremos que
A
x> 1 − |a| > 0 ⇒ f (x) > A
e
x< − 1 −A|a| < 0 ⇒ f (x) < −A.
Assim, como A > 0 é arbitrário, podemos concluir que

x
lim f (x) = +
→ +∞
∞ e lim
x →−∞ f (x) =
−∞.
Para todo x ∈ R, temos as desigualdades
−|a| = −|a| · 1
 −|a|| sen(x)|
= −|a · sen(x)|
 a · sen(x).
A
Assim, para x > 1 a −| | > 0,
f (x) = x + xa sen(x) ·
 x xa
= x(1
> A
− −||a||)
A
e, para x < − −| | < 0,
1 a
f (x) = x + xa sen(x)·
 x xa − ||
= x(1 a ) −| |
< A. −

273
Exercı́cio 6.24:
Seja p : R → R um polinômio não constante. Dado b ∈ R, suponha que exista uma sequência (x n ), tal que

lim p(x ) = b ∈ R. n
→ ∞ n +

Prove que ( xn ) é limitada é o conjunto dos seus pontos de aderência n˜ ao é vazio e está contido em p−1 (b). Em
particular, se existe uma sequência (xn ), tal que lim p(xn ) = 0, ent˜ao p tem alguma raiz real.

Suponhamos que lim p(xn ) = b ∈ R.


Pelo exerc´
ıcio 6.13, temos que
lim p(x) = + ou
e
x →+∞ ∞ −∞
lim p(x) = +∞ ou − ∞.
→−∞
x

Assim, se ( xn )n∈Z+ possuisse uma subsequência (xnk )k∈Z+ tal que

k
lim xnk = +
→ +∞
∞ ou − ∞
ter´
ıamos que
k
lim p(xnk ) = +
→ +∞
∞ ou − ∞.
Contradizendo o fato de que toda subsequência de (xn )n∈Z+ converge para b R. Portanto, a sequência (xn ) é

limitada.
Como a sequência (xn ) é limitada, segue que o seu conjunto de pontos de aderência não é vazio.
Sejam (xnk )k∈Z+ uma subsequência convergente de (xn )n∈Z+ e a = lim k→+∞ xnk . Suponhamos que c 0 , c 1 , . . . ,
cm R são tais que

p(x) = c m xm + + c1 x + c0 ,···
para todo x ∈ R. Assim,
p(a) = cm am + ··· + c a + c 1 0
m
= cm + ··· + c
lim xnk lim x 1 nk + c0
→ ∞k (+ → ∞ k +

m
= lim c x + · · · + c x + c
m nk 1 nk 0
k → ∞
+
= lim p(xnk )
k →+∞
= b.

Logo, a p −1 (b). Com isso, concluimos que o conjunto dos pontos de aderência de (xn ) está contido em p −1 (b).

Em especial, se b = 0, o conjunto dos pontos de aderência de ( xn ) está contido no conjunto de raı́zes de p.
Logo, neste caso, conclui-se que p possui raizes reais.

274
Capı́tulo 7

Funções Contı́nuas

275
Exercı́cio 7.38:
A função R f : [0, + )∞ → ∞ √
[0, + ), definida por f (x) = n x, n > 1, não é Lipschitziana num intervalo da forma
[0, a], a > 0, embora seja uniformemente continua nestes intervalos. Por outro lado, f e´ Lipschitziana, com
constante
1
c := n
n an−1
, √

no intervalo [a, + ). Concluir que f é uniformemente continua em [0, + ). ∞
Mostraremos, primeiramente, que f : [0, a] → R não é Lipschitziana, para todo a > 0. Faremos isso verificando
que o conjunto
f (x) f (y)
L := | | − | |
x y ∈
: x e y [0, a] }
é ilimitado superiormente e, consequentemente, não existe c R tal que

|f (x) − f (y)|  c|x − y|,

para todo x e y [0, a].
Seja A > 0. Para todo z ∈ R tal que }
max n−1
√1 A , √1a n
<z

temos que
1
z n−1 > A e ∈ [0, a].
zn
Assim, }
S :=
1
: z > max 1/
n
{ −
√A, 1/ √a} ⊂ [0, a]
1 n

zn
1
e, para todo x = zn ∈ S, n
|f (x) − f (0)| = |f (1/z ) − f (0)| = z n 1
− > A.
| x − 0| |1/z − 0| n

Desta forma, temos que L é ilimitado superiormente.


Verificaremos, a seguir, que f : [a, +∞) → R e´ Lipschitziana com
1
c := √ n
n a − n 1

como constante de Lipschitz.



Sejam x e y = x + λ [a, 0], λ > 0. Pela Desigualdade de Bernulli
n
1 1
1+n  1+ ,
nx nx
temos que
n λ λ
1+  1+ .
x nx
Assim,
λ
n
1+
x
− 1  nxλ
e, consequentemente,
n n
λ
√x + λ − √ x  n √x − . n n 1

Logo,
|f (x) − f (y)| = |√f (x + λ) −√f (x)|
n
= x+λ− x n

 √λ
n n−1
n x
 √λ
n 1
n an −

 cx | − y|.
276
∞→
Com isso, concluimos que f : [a, + ) R e´ Lipschitziana com c como constante de Lipschitz.
Por fim, provaremos que f : [0, ) ∞→R é uniformemente contı́nua.
Seja ε > 0. Para todos x e y [0, + ), com y = x + λ e λ [0, εn ), temos que
∈ ∞ ∈
|f (x) − f (y)| = |√f (x + λ) −√f (x)|
n
=

√x + λ√− x √
(√ x + λ) − x
n n
n

n
= √λε
< n n

= ε.
n
Logo, se x e y ∈ [0, +∞) e |x − y| < ε |
temos que f (x) − f (y)| < ε. Portanto, f e´ uniformemente continua.

277
Exercı́cio 7.39:
Sejam }
1
Z∗+ := n + : n ∈ Z+
n
e F := Z+ ∪ Z∗ . Defina a fun¸cão f : F → R por
+

f (x) =
2, se x ∈ Z∗ ;
+
x, se x ∈Z .+

Mostre que os conjuntos Z+ e Z∗+ são fechados, que f |Z+ e f |Z+ são funções uniformemente cont´
∗ ınuas, mas f : F →R
não é uniformemente cont´
ınua.

Primeiramente, verifiquemos que  + é fechado. Pela construção de R , temos que

(n − 1, n + 1) ∩ Z + {}
= n ,

para todo n Z+ . Desta forma, se ( xn )n∈Z+ é uma sequência convergente (de Cauchy) em Z + existe n 0
∈ ∈Z + tal
que para todos p  n0 vale a desigualdade
xn0 xp < 1. | − |
Logo, como
(xn0 − 1, x n0 + 1) ∩Z + { }
= xn 0 ,
devemos ter que xn0 = x p , para todo p  n 0 . Assim, lim xn = x n0 Z + . Portanto, concluimos que o limite de

n→∞
toda sequência convergente de elmentos de Z + concverge para um elemento de Z + . Desta forma, Z + e´ fechado.
Pela equivalência
1 1
n <m n+ <m + ,
n
⇐⇒
m
para todos n e m Z+ , obtemos que os intervalos

Cn :=
 3, 5 ,
2 2
1 1 1 1
se n = 1;

 0, 2 + 1 ,
2
1 1
se n = 1;

 n+
n 2

,n+ +
n 2
, se n = 1. ̸  n −1+ n− 1
,n+1+
n+1
̸
, se n = 1.

são tais que }


1
Cn ∩ Z∗ =
+ n+
n
.

Assim, de modo an´alogo ao caso de Z+ , verifica-se que toda sequência de elementos de Z∗+ converge para um
elemento de Z ∗+ e, por isso, Z ∗+ e´ fechado.
| |
Como f Z+ e´ constante e f Z+ é a função identidade, temos que estas funções são uniformemente cont´
∗ ınuas.
Provaremos, agora, que f não é uniformemente contı́nua verificando que para todo δ > 0 existem x e y F ∈
| − | |
tais que x y < δ e f (x) f (y) < 1. − |
Seja δ > 0. Podemos escolher n Z+ tal que ∈
1
< δ e n > 3.
n
Assim, para x = n + 1/n e y = n, temos que

|x − y| = n1 e |f (x) − f (y)| = n − 2 + n1 > 1.


Portanto, f não é uniformemente cont´ınua.

278
Exercı́cio 7.40:
Dê um exemplo de dois abertos A e B e uma função cont´
ınua f : A B∪ → R tal que f | A ef | B sejam uniformemente
contı́nuas, mas f não seja.

Sejam A e B os subconjuntos abertos ( −∞, 0) e (0 , +∞), respectivamente, em R. Podemos definir uma fun¸cão
f:A B∪ → R por
f (x) =
− 1, se x ∈ (−∞, 0) = A;
1, se x ∈ (0, +∞) = B.
| |
As restrições f A e f B são constantes e, por isso, s˜ao uniformemente cont´
ınuas.
Mostraremos, agora, que f não é uniformemente contı́nua verificando que para todo δ > 0 existem x e y ∈ A ∪B
tais que
|x − y| < δ e |f (x) − f (y)| > 1.
Seja δ > 0. Para x = δ/4 e y = −δ/4, temos que
|x − y| = δ2 < δ e |f (x) − f (y)| = 2 > 1.
Portanto, concluimos que f não é uniformemente cont´
ınua.

279
Exercı́cio 7.41:
Toda função contı́nua monótona limitada f : I → R, definida num intervalo I , é uniformemente cont´ınua.
Seja I um intervalo em R .
Suponhamos que f : I R seja crescente, contı́nua e limitada.


Seja ε > 0. Provaremos que existe δ > 0 tal que se x e y I satisfazem x | − y| < δ então
|f (x) − f (y)| < ϵ. (7.1)

Daı́, obtemos diretamente que f é uniformemente contı́nua.


Sejam
A := inf f (I ) e B := sup f (I ).
Se B − A < ε a desigualdade ( 7.1) se verifica automaticamente pois, neste caso
|f (x) − f (y)|  B − A < ε,
para todos x e y ∈ I . Desta forma, basta varificarmos o caso em que B − A  ε.
Como
ε ε
A = inf f (I ) < A +  B − < sup f (I ) = B
3 3
e I é um intervalo, temos, pelo Teorema do Valor Intermediário, que f −1 (A + 3ε ) e f −1 (B − ε
3) são conjuntos não
vazios.
Tomemos
ε ε
c inf f −1 A +
∈ e d sup f −1 B
∈ . −
3 3
Sejam
a := inf I e b := sup I.
Como f −1 (A + 3ε ) e f −1 (B ε3 ) I , devemos ter que a =  c e d  b. Por fim, como f é crescente, todo elemento
− ⊂
de f −1 A + ε3 é menor que qualquer elemento de f −1 B 3ε . Logo, −
(  ( 
a  c < d  b.

∈ ∩
Seja x [a, c) I . Como A = inf f (I ), devemos ter que f (x)  A. Por outro lado, como f e´ crescente e x < c,
devemos ter que f (x)  f (c) = A + 3ε . Portanto, f (x) [A, A + ε3 ].

De modo análogo, prova-se que se x (d, b] então f (x) [B 3ε , B].
∈ ∈ −

Como c e d I , temos que [ c, d] é um intervalo compacto contido em I . Sendo f contı́nua, devemos ter que
|
f [c,d] também é cont´
ınua. Assim, como toda função cont´
ınua com dom´ınio compacto é uniformemente cont´ ınua,
devemos ter que f[c,d] é uniformemente contı́nua. Logo, existe δ0 > 0 tal que para quaisquer x e y [c, d], com ∈
| − |
x y < δ0 , a desigualdade
ϵ
| −
f (x) f (y) <
2
|
é v´
alida.
Por fim, tomemos
{ −}
δ := min δ0 , d c .
Se x e y ∈ I , com x < y , são tais que |x − y| < δ, pela escolha de δ  d − c, devemos ter um dos casos:
• x e y ∈ [a, c) ∩ I ;
• x ∈ [a, c) ∩ I e y ∈ [c, d];
• x e y ∈ [c, d];
• x ∈ [c, d] e y ∈ (d, b] ∩ I ;
• x e y ∈ (d, b] ∩ I .
Verificaremos, em cada um destes casos, que a desigualdade ( 7.1) é v´
alida:

280
ε
• Se x e y ∈ [a, c) ∩ I temos que f (x) e f (y) ∈ [A, A + ]. Logo, a desigualdade ( 7.1) é v´alida;
3
ε
• Se x ∈ [a, c) ∩ I e y ∈ [c, d] temos que f (x) e f (c) ∈ [A, A + ] e |c − y| < δ  δ . Logo,
3 0

|f (x) − f (y)|  |f (x) − f (c)| + |f (c) − f (y)| < 3ε + 2ε < ε;


ε
• Se x e y ∈ [c, d] temo imediatamente que |f (x) − f (y)| < < ε;2
ε
• Se x ∈ [c, d] e y ∈ (d, b] ∩ I temos que |d − x| < δ  δ e f (y) e f (d) ∈ [B − , B]. Logo,
0 3

|f (x) − f (y)|  |f (x) − f (d)| + |f (d) − f (y)| < 2ε + 3ε < ε;


• Se x e y ∈ (d, b] ∩ I temos que f (x) e f (y) ∈ [B − , B]. Logo, a desigualdade ( 7.1) é v´alida.
ε
3

Suponhamos agora que f : I → R seja decrescente, contı́nua e limitada. Provaremos que f é uniformemente
cont´ınua.
Como f : I → R e´ decrescente, contı́nua e limitada, temos que − f : I → R e´ crescente, cont´ ınua e limitada.
Pelo que foi demonstrado acima, f é uniformemente cont´ ınua.
Seja ε > 0. Como −f é uniformemente cont´ ınua, existe δ > 0 tal que, para todos x e y ∈ I com |x − y | < δ , a
desigualdade
|(−f )(x) − (−f )(y)| < ε
alida. Logo, para todos x e y ∈ I com |x − y| < δ , a desigualdade
é v´

|f (x) − f (y)| = |(−f )(x) − (−f )(y)| < ε


é v´
alida. Desta forma, concluimos que f é uniformemente cont´ınua.

Portanto, toda função f : I R monótona, cont´ınua e limitada é uniformemente cont´ınua.

281
Exercı́cio 7.42:
Seja f : X →R uma função contı́nua. Para que f se estenda continuamente a uma função ϕ : X → R e´ necessário
e suficiente que exista lim f (x) para todo a X ′ . ∈
x →a

Suponhamos que haja uma fun¸cão cont´ınua ϕ : X → R tal que ϕ| X = f . Então, para todo a ∈ X ′, existe
lim f (x) pois, pelo Teorema 2 do Capı́tulo VI,
x →a

x→a x
→a
|
lim f (x) = lim ϕ X (x) = lim ϕ(x).
x
→a
Suponhamos que, para todo a ∈ X ′ , exista lim f (x).
x→a

Seja ϕ : X → R definida pela igualdade


ϕ(a) := lim f (x),
x →a
para todo a ∈ X ′ . Assim, pelo fat o de f ser cont´ınua,
ϕ(a) = lim f (x) = f (a),
→a x

para todo a ∈ X . Mostraremos que, para todo a ∈ X ′ ,

lim ϕ(y) = ϕ(a),


y →a
e concluiremos daı́ que ϕ é uma extensão contı́nua de f .
Fixemos a X ′ e ε > 0. Como ϕ(a) := lim f (x), existe δ > 0 tal que
∈ x→a
|ϕ(a) − f (x)| < 2ε ,
para todo x ∈
X tal que a x < δ . | − | Seja y ∈ X ′ tal que | a − y | < δ . Novamente pela defin ição de ϕ,
ϕ(y) := lim f (x), existe δ 0 > 0 tal que
x y
→ δ0 < δ − |a − y|
e
|ϕ(y) − f (x)| < 2ε ,
para todo x ∈ X tal que |y − x| < δ . Assim, como y ∈ X ′ , existe
0

x ∈ (y − δ , y + δ ) ∩ X ⊂ (a − δ, a + δ ) ∩ X.
0 0 0

Logo,
|ϕ(a) − ϕ(y)|  |ϕ(a) − f (x )| + |f (x ) − ϕ(y)| < 2ε + ε2 = ε.
0 0

Desta forma, concluimos que


|ϕ(a) − ϕ(y)| < ε,
para todo y X ′ tal que a y < δ .
∈ | − |
Portanto, para todo a X ′ , ∈
lim ϕ(y) = ϕ(a).
y →a

282
Exercı́cio 7.43:
Seja f : [a, b] → R cont´ınua. Dado ε > 0, existem
a = a 0 < a1 < ·· · < a −
n 1 < an = b

tais que, para cada i = 1,2, . . . ,n,


x, y ∈ [a − , a ] ⇒ |f (x) − f (y)| < ε.
i 1 i

Seja ε > 0. Mostraremos que existem

a = a 0 < a1 < ·· · < a −


n 1 < an = b

tais que, para cada i = 1,2, . . . ,n,


x, y ∈ [a − , a ] ⇒ |f (x) − f (y)| < ε.
i 1 i

Como o intervalo [a, b] é compacto e f : [a, b] → R é cont´ınua, temos que f : [a, b] → R é uniformemente contı́nua.
Assim, podemos escolher um δ > 0 tal que
|f (x) − f (y)| < ε,

para todos x e y [a, b] tais que x | − y| < δ.
A famı́lia de intervalos abertos }
p − δ2 , p + δ2
p [a,b]

é uma cobertura aberta para o intervalo compacto [a, b]. Pelo exercı́cio 5.52, existe uma partição

a = a 0 < a1 < ·· · < a −


n 1 < an = b

do intervalo [a, b] tal que para cada subintervalo [ ak , ak+1 ] existe p k ∈ [a, b] tal que
[ak , ak+1 ] ⊂ pk − 2δ , p k + δ2 .

Por fim, utilizando-se uma parti¸cão de [ a, b] como acima, dados x e y ∈ [a , b ] ⊂


k k
(
pk − δ
2 , pk + δ
2

, é v´
alida a
desigualdade
x y <δ | − |
e, por isso e pela escolha de δ , temos que
|f (x) − f (y)| < ε.
Logo, temos uma partição de [ a, b] com a propriedade desejada.

283
Exercı́cio 7.44:
Uma função cont´
ınua ϕ : [a, b] → R chama-se poligonal quando existe uma partição
a = a < a < ·· · < a = b
0 1 n

|
tais que ϕ [ai 1 ,ai ] é um polinômio de grau  1, para cada i = 1,...,n . Prove que, se f : [a, b]

R e´ cont´ınua, →
então, dado ε > 0, existe uma fun¸cão poligonal ϕ : [a, b] R, tal que f (x) ϕ(x) < ε para todo x [a, b].
→ | − | ∈
Seja ε > 0.
Pelo exercı́cio 7.43, existe uma partição

a = a 0 < a1 < ·· · < a n =b


tal que
|f (x) − f (y)| < 2ε ,

para x e y [ak , ak+1 ].
Podemos definir uma função poligonal ϕ : [a, b] → R pela equa¸cão
(
ϕ(x) := f (ak+1 ) − f (a )
a
x−a
k

−a k+1
k
k
+ f (ak ),

para cada x ∈ [a , a
k k+1 ]. Por esta defini¸cão e pela escolha da parti¸cão de [ a, b], temos que

|ϕ(x) − f (a )| = |f (a
k k+1 ) − f (a )|
k
 −
x ak
  |f (a k+1 ) − f (a )| < 2ε ,
k

ak+1 ak


para cada x [ak , ak+1 ].
Por fim, dado x [a, b], ∈
ε ε
f (x) ϕ(x)  f (x) f (a ) + ϕ(x)
+ = ε, f (a ) <
| − | | − k
2 2 | | − k |
onde x ∈ [ak , ak+1 ]. Portanto, ϕ é uma função poligonal que satisfaz a condi¸cão do enunciado.

284
Exercı́cio 7.45:
Dado ξ : [a, b] → R, se existem a = a 0 < a1 < ··· |
< an = b tais que ξ (ai 1 ,ai ) é constante para cada i = 1, 2, . . . ,n,

ξ é chamada de função escada. Mostre que se f : [a, b] R e´ cont´ınua, então, para cada ε > 0, existe uma fun¸cão


escada ξ : [a, b] R, tal que f (x) ξ (x) < ε qualquer que seja x [a, b].
| − | ∈
Seja ε > 0.
Como f é cont´
ınua, existe, pelo exercı́cio 7.43, uma partição

a = a 0 < a1 < ·· · < a n =b

de [a, b] tal que a desigualdade


| −
f (x) f (y) < ε, |
é v´ ∈
alida sempre que existe k = 1, . . . , n, tal que x e y [ak , ak+1 ].
Podemos definir uma função escada ξ : [a, b] R pela igualdade

ξ (x) =

f (ak ), x [ak , ak+1),
f (an ), x = a n .

Logo, se x ∈ [a , a
k k+1 ) então
|f (x) − ξ(x)| = |f (x) − f (a )| < εk

| − ξ(a )| = 0, segue que a desigualdade


pela escolha da parti¸cão de [ a, b]. Portanto, como f (an ) n

|f (x) − ξ(x)| < ε


é v´
alida para todo x ∈ [a, b].

285
Exercı́cio 7.46:
Dada uma fun¸cão f : X R, suponha que para cada ε > 0 se possa obter uma fun¸cão cont´ınua g : X
→ → R, tal que
| − | ∈
f (x) g(x) < ε qualquer que seja x X . Então f e´ cont´
ınua.

Concluiremos que f é cont´ ınua mostrando que f é cont´ ınua em um ponto arbitrário a ∈ X.
Seja ε > 0.
Pela hipótese sobre f , existe uma fun¸cão cont´
ınua g : X R tal que

|f (x) − g(x)| < 3ε ,
para todo x X .

Como g e´ cont´
ınua, existe δ > 0 tal que
|g(x) − g(a)| < 3ε ,
para todo x ∈ X tal que |x − a| < δ.
Assim,
|f (x) − f (a)|  |f (x) − g(x)| + |g(x) − g(a)| + |g(a) − f (a)|
ε ε ε
< 3
+ 3
+ 3
= ε.
para todo x ∈ X tal que |x − a| < δ. Desta forma, concluimos que f e´ cont´
ınua em a.

286
Exercı́cio 7.47:
Seja X R . Uma fun¸cão f : X
⊂ → R diz-se semicont´
ınua superiormente no ponto a ∈ X quando, para cada ε > 0
dado, pode-se obter δ > 0, tal que se

x ∈ X e |x − a| < δ ⇒ f (x) < f (a) + ε.


Diz-se que f é semicintı́nua superiormente quando ela o é em todos os pontos de X .
(a) Defina fun¸cão semicontı́nua inferiormente e mostre que f é contı́nua num ponto se, e somente se, é semi-
contı́nua superior e inferiormente naquele ponto.
(b) Prove que um subconjunto A ⊂ R é aberto se, e somente se, sua função caracter´ıstica ξ A: R → R (definida

por ξ A (x) = 1 se x A e ξ A (x) = 0 se x ∈
/ A) é semicont´
ınua inferiormente.
(c) Enuncie e prove um resultado análogo ao anterior para conjuntos fechados.
(d) Mostre, mais geralmente, que para todo subconjunto X R, sua fun¸cão caracter´ıstica ξX : R
⊂ R e´ des- →

contı́nua precisamente nos pontos da fronteira de X . Dado a fr X , mostre que ξ X é semicont´ ınua superior-
mente no ponto a se a X e inferiormente se a / X . Conclua que a fun¸cão f : R
∈ ∈ R, definida por f (x) = 1

para x Q e f (x) = 0 para x irracional, é semicont´
∈ ınua superiormente nos números racionais e inferiormente
nos números irracionais.
(e) Seja f : R → R definida por f (x) = sen(1 /x) se x = 0 e f (0) = c. Mostre que f e´ semicont´ınua superiormente
̸

no ponto 0 se, e somente se, c  1. (E inferiormente se, e somente se, c  1.) Tomando 1 < c < 1, mostre−
que f não é semicont´
ınua inferiormente ou superiormente no ponto 0.
(f) As fun¸cões f e g : R →R, onde f (0) = g(0) = 0 e, para x = 0, f (x) = x sen(1/x), g(x) = 1/ x , são
̸ ||
semicont´ ·
ınuas inferiormente, mas seu produto f g não é uma função semicont´
ınua no ponto 0.
(g) Para que f : X → R seja semicontı́nua superiormente no ponto a X X ′ é necessário e suficiente que
∈ ∩
limx→a sup f (x)  f (a) (Errata: é necessário mas não suficinte se f for ilimitada. Contra exemplo: a = 0
e f: R → R definida por f (x) = 1/x, em x = 0, e f (0) = 0). Equivalentemente: para toda sequência de
̸
pontos xn X com lim n xn = a, que seja lim n sup f (xn )  f (a). Vale o resul tado an´alogo para

semicontinuidade inferior. →∞ →∞
(h) A soma de duas fun¸ cões semicontı́nuas superiormente num ponto ainda goza da mesma propriedade. Use

o item (e) com c = 1 e c = 1 para dar exemplo de duas fun¸ cões semicontı́nuas (uma superiormente e
outra inferiormente) cuja soma n˜ao é semicont´ınua. Mostre que se f é semicont´
ınua superiormente, f é −
inferiormente.
(i) Sejam f e g : X → R semicontı́nuas superiormente num ponto. Se f (x)  0 e g(x)  0 para todo x ∈ X,
·
então o produto f g e´ uma função semicontı́nua superiormente no mesmo ponto.
(j) Quando X R é compacto, toda função semicont´
⊂ ınua superiormente f : X R é limitada superiormente e

atinge seu valor máximo num ponto de X . Enuncie e prove um fato an´alogo para semicontinuidade inferior.

(a)

Uma função f : X → R diz-se semicont́ınua inferiormente no ponto a X quando, para cada ε > 0 dado,

pode-se obter δ > 0, tal que se
x X e x a <δ f (a) ε < f (x).
Diz-se que f é semicint´ ∈ | − | ⇒ −
ınua inferiormente quando ela o é em todos os pontos de X .
Suponhamos que f : X R seja uma fun¸cão semicont´ınua inferiormente e superiormente. Provaremos que f é


contı́nua em um ponto arbitrário a X e concluiremos daı́ que f e´ cont´ ınua.
Seja ε > 0. Como f é semicont´ ınua inferiormente e superiormente em a, existem δ − e δ + > 0 tais que

x ∈ X e |x − a| < δ− ⇒ f (a) − ε < f (x)

287
e
x ∈ X e |x − a| < δ ⇒ f (x) < f (a) + ε.
+

{
Assim, para δ := min δ− , δ+ , }
x ∈ X e |x − a| < δ ⇒ f (a) − ε < f (x) < f (a) + ε.
Logo, podemos concluir que f e´ cont´
ınua em a X . ∈
Suponhamos, por outro lado, que f é cont´ınua. Provaremos que f é semicontı́nua inferiormente e superiormente

em um ponto arbitr´ario a X e concluiremos daı́ que f e´ semicontı́nua inferiormente e superiormente.
Seja ε > 0. Como f e´ cont´ınua em a, existe δ > 0 tal que

x ∈ X e |x − a| < δ ⇒ f (a) − ε < f (x) < f (a) + ε.


Assim,
x ∈ X e |x − a| < δ ⇒ f (a) − ε < f (x)
e
x ∈ X e |x − a| < δ ⇒ f (x) < f (a) + ε.
Logo, podemos concluir que f é semicont´
ınua inferiormente e superiormente em a ∈ X.
(b)

Suponhamos que A ⊂ R é aberto. Provaremos que ξ : R → R é semicont´ ınua inferiormente em um ponto


A
p∈ R arbitrário. E conluiremos da´ı que ξ e´ semicontı́nua inferiormente.
A
Seja ε > 0.
Se p ∈ A então existe δ > 0 tal que
(p − δ, p + δ ) ⊂ A.
Assim, neste caso, se x ∈ R e |x − p| < δ então

ξ (p) − ε = 1 − ε < 1 = ξ (x).


A A

Logo, se p ∈ A então ξ é semicont´


A ınua inferiormente em p.
Se p ∈ R\A então, para todo x ∈ R, temos que

ξA (p) −ε= 0−ε<0 ξ A (x).

Logo, se p R A então ξ A é semicont´


∈ \ ınua inferiormente em p.
ınua inferiormente em p R.
Portanto, ξ A é semicont´ ∈
Suponhamos, por outro lado, que ξA seja semicont´ ınua inferiormente. Provaremos que, para um ponto arbitrário
∈ − ⊂
a A, existe δ > 0 tal que ( a δ, a + δ ) A. Desta forma concluiremos que A e´ aberto.
Como ξ A e´ semicont´ ınua inferiormente, tomando-se ε = 1/2, existe δ > 0 tal que, para todo x (a δ, a + δ ), ∈ −
vale
1
2
= ξ A (a) ε < ξA (x). −
̸ ∈ −
Desta forma, ξ A (x) = 0, para todo x (a δ, a + δ ). Logo, ( a δ, a + δ ) A. − ⊂
(c)

Provaremos que F ⊂ R é fechado se, ıstica ξF : R


e somente se, sua fun¸cão caracter´ → R e´ semicont´ınua
superiormente.
que F ⊂ R seja fechado. Provaremos que ξ é semicontı́nua superiormente em um ponto arbitrário
∈ Suponhamos
F
p R e concluiremos daı́ que ξ e´ semicontı́nua superiormente.
F
Seja ε > 0.
Se p ∈ F , para todo x ∈ R, temos que

ξF (x)  1 < ξF (p) + ε.

Logo, se p ∈ F então ξ F é semicont´


ınua superiormente em p.

288
Se p ∈ R\F então existe δ > 0 tal que
(p − δ, p + δ) ⊂ R\F.
Assim, neste caso, se x ∈ R e |x − p| < δ então
ξF (x) = 0 < 0 + ε = ξ A (p) + ε.

Logo, se p R F então ξ F é semicont´


∈ \ ınua inferiormente em p.
ınua superiormente em p R.
Portanto, ξ F é semicont´ ∈
Suponhamos, por outro lado, que ξF seja semicontı́nua superiormente. Provaremos que, para um ponto ar-
bitrário p R F , existe δ > 0 tal que ( p δ, p + δ ) R F . Desta forma, concluiremos que F e´ fechado.
∈ \ − ⊂ \
Como ξ F é semicont´ ∈ −
ınua inferiormente, tomando-se ε = 1/2, existe δ > 0 tal que, para todo x ( p δ, p + δ ),
vale
1 1
ξF (x) < ξF (p) + ε = 0 + 2 = 2 .
Desta forma, ξ F (x) = 1, para todo x ( p δ, p + δ ). Logo, ( p δ, p + δ ) R F .
̸ ∈ − − ⊂ \
(d)

Seja X R.

Para concluirmos que ξ X : R → R é descont´ınuo exatamente nos pontos de fronteira de X , verificaremos que ξ X
é cont´ınua em int(X ) e int(R X ), semicont´
\ ∩
ınua superiormente, mas não inferiormente, em fr(X ) X e semicont´ınua
inferiormente, mas não superiormente, em fr(X ) (R X ). ∩ \
Seja a int(X ). Como int( X ) é aberto, existe δ > 0 tal que ( a δ, a + δ ) int(X ). Assim, para tod o x R
∈ − ⊂ ∈
| − | ∈
tal que x a < δ , temos que x int(X ) e, consequentemente,

|ξ (x) − ξ (a)| = |1 − 1| = 0.
X X

Logo, ξ X e´ cont´ınua em a ∈ int(X ) pois, para todo ε > 0,


|ξ (x) − ξ (a)| = 0 < ε
X X

sempre que x a < δ .


| − |
De modo análogo, verifica-se que ξ X : R →
R e´ cont´ınua em int(R X ). \
∈ ∩
Seja a (fr X ) X . Para quaisquer ε e δ positivos, vale, para todo x (a ∈ − δ, a + δ), a desigualdade
ξX (x)  1 < 1 + ε = ξX (a) + ε.

Logo, ξ X e´ semicontı́nua superiormente em a. Por outro lado, para qualque r δ > 0, existe x ∈ (a − δ, a +δ) ∩ (R\X ).
Assim, para ε = 1/2 e qualquer δ > 0, temos que

ξX (a) − ε = 1/2 > 0 = ξ X (x),

para todo x (a δ, a + δ ) (R X ). Logo, ξ X não é semicont´


∈ − ∩ \ ınua inferiormente.
Seja a (fr X ) (R X ). Para quaisquer ε e δ positivos, vale, para todo x (a
∈ ∩ \ ∈ − δ, a + δ), a desigualdade
ξX (a) −ε= 0−ε<0 ξ X (x).

Logo, ξX é semicont´
ınua inferiormente em a. Por outro lado, para qualq uer δ > 0, existe x ∈ (a − δ, a + δ ) ∩ X .
Assim, para ε = 1/2 e qualquer δ > 0, temos que

ξX (x) = 1 > 1/2 = 0 + 1 /2 = ξX (a) + ε.

paraPor
todo xf =
fim, ∈ −
(aξ δ, a + δ) X
Q é semicont´

. Logo,
ınua ξ X não é semicont´
superiormente em ınua superiormente.

Q = R ∩ Q = (fr Q) ∩ Q
e semicont´
ınua inferiormente em
R\Q = R ∩ (R\Q) = (fr Q) ∩ (R\Q)
pelo que foi provado anteriormente.

289
(e)

Seja f : R → R dada por


sen x1 , se x = 0
̸
f (x) =
c, se x = 0.
Como f é uma composição de funções contı́nuas no aberto R 0 , temos que f e´ cont´
\{ } ınua em R 0 . \{ }
Desta forma, podemos concluir que f e´ semicont´ ınua superiormente (inferiormente) se, e somente se, c  1

(c  1) mostrando que f é f é semicont´ ınua superiormente (inferiormente) em 0 se, e somente se, c  1 (c  1). −
Assim, podemos também concluir que f não é semicont´ ınua superiormente ou inferiormente se 1 < c < 1. −
Suponhamos que f seja semicontı́nua superiormente em 0. Mostraremos, para ε > 0 arbitrário, que 1 ε < c e −
concluiremos daı́ que 1  c. Existe δ > 0 tal que

f (x) < f (0) + ε = c + ε,

para todo x ∈ (−δ, δ). Assim, para n ∈ Z + suficientemente grande, temos que
1
π
+ 2nπ
∈ (−δ, δ)
2

e, consequentemente,
π 1
1 = sen + 2nπ = f π < c + ε.
2 2 + 2nπ
Consideremos, por outro lado, o caso em que 1  c. Seja ε > 0. Para todo x ∈ R\{0}, temos que
1
f (x) = sen  1 < 1 + ε = c + ε = f (0) + ε.
x
Logo, para todo x ∈ R, temos que
f (x) < f (0) + ε.
Portanto, f é semicont´
ınua superiormente em 0.

Suponhamosdaı́
e concluiremos queque
f seja
c  semicontı́nua
− inferiormente
1. Existe δ > 0 tal que em 0. Mostraremos, para ε > 0 arbitrário, que c < −1 + ε
c − ε = f (0) − ε < f (x),
para todo x ∈ (−δ, δ). Assim, para n ∈ Z + suficientemente grande, temos que
1

+ 2nπ
∈ (−δ, δ)
2

e, consequentemente,
1 3π
c −ε <f 3π
+ 2nπ
= sen
2
+ 2nπ = −1.
2
Consideremos, por outro lado, o caso em que c −1. Seja ε > 0. Para todo x ∈ R\{0}, temos que

1
f (0) − ε = c − ε  −1 − ε < −1  sen = f (x).
x
Logo, para todo x ∈ R, temos que
f (0) − ε < f (x).
Portanto, f é semicont´
ınua inferiormente em 0.

(f)

Sejam f e g : R → R definidas por


0, ( se x = 0,
f (x) = 1
x sen x
̸
, se x = 0,

290
e
0, se x = 0,
g(x) = 1
|x| , se x ̸= 0,
ınuas em R 0 , temos que f e g são
Como f e g são produtos, composições e quocientes de fun¸ cões cont´ \{ }
ınuas em R 0 . Logo, f e g são semicont´
cont´ \{ } ınuas inferiormente em R 0 . Desta forma, provando que f e g são
\{ }
semicontı́nuas inferiormente em 0 podemos, pelo item (a), concluir que f e g são semicont´
ınuas inferiormente.
Seja ε > 0. Tomando δ := ε, temos, para todo x ( δ, δ ) 0 , que ∈ − \{ }

x sen
1
 ||
= x sen
 1
  |x| < δ = ε
x x
e
1
−ε < 0 < |x| .
Assim,
1
f (0) − ε = −ε < x sen x
= f (x)

e
g(0) − ε = −ε < |x1| = g(x),
para todo x R tal que 0 < x < δ . Logo, podemos concluir que f e g são semicont´
∈ || ınuas inferiormente em 0.
· ınua em 0. Seja δ > 0. Existe n Z+ tal que
Provaremos, agora, que f g não é semicont´ ∈
1
x := < δ.
(2n + 1)π

Assim, x ∈ (−δ, δ)\{0} e


·
f g(x) =
x
|x| sen
1
x
(
= sen (2n + 1)π =
 −1.
Logo, podemos concluir que para todo δ > 0 existe x ∈ R\{0} tal que |x| < δ e
·
f g(x) = −1 = 0 − 1 = f · g(0) − 1.
·
Portanto, f g não é semicont´
ınua em 0.

(g)

Seja f : X R uma função e a X X ′ .


→ ∈ ∩
Suponhamos que f seja semicont´ ınua superiormente em a. Provaremos que lim x→a sup f (x)  f (a).
Sejam c R um valor de aderência de f em a e (xn )n∈Z+ uma sequência em X a tal que lim n→∞ xn = a e
∈ \{ }
limn→∞ f (xn ) = c. Provaremos, para ε > 0 arbitrário, que

c < f (a) + ε.

Com isso, poderemos concluir que c  f (a). Existe δ > 0 tal que
ε
f (x) < f (a) +
2
sempre que x ∈ X e´ tal que |x − a| < δ. Tamb´em existe n ∈ Z tal que 0 +

|x − a| < δ e |f (x ) − c| < 2ε .
n0 n0

Assim,
c − ε2 < f (x n0 ) < f (a) +
ε
2
e, consequentemente,
c < f (a) + ε.

291
Concluimos que f (a) é maior ou igual que qualquer valor de aderência de f em a. Portanto, lim x→a sup f (x) 
f (a).
Consideremos, agora, que lim x→a sup f (x)  f (a). Para provarmos que provaremos que f e´ semicont´ ınua
superiormente em a precisamos de mais hip´oteses. Por exemplo, se f : R R e´ dada por →
0, se x = 0,
f (x) = 1
x , se x = 0.̸
então
lim sup f (x) =
x
→0
−∞ < f (0).
Desta forma, iremos assumir também que f é limitada em uma vizinhança de a.
Seja ε > 0. Suponhamos, por absurdo, que n˜ao exista δ > 0 tal que

f (x) < f (a) + ε

sempre que x ∈ X e´ tal que |x − a| < δ. Então, para cada n ∈ Z +, exite x n ∈ X \{a} tal que
|x − a| < n1
n

e
f (a) + ε  f (xn ).
\{ }
Assim, temos uma sequência (xn )n∈Z+ em X a tal que lim n→∞ xn = a. Como f é limitada em uma vizinhança
de a, existe uma subsequência (f (xnk ))k∈Z+ de (f (xn ))n∈Z+ que converge para um ponto c R. Logo, ∈
f (a) + ε  lim f (xnk ) = c
k →∞
já que
f (a) + ε  f (xnk ),
para todo k ∈Z +. Por outro lado, como c é um ponto de acumulação de f em a,

c  xlim
→a sup f (x)  f (a).
Uma contradição.
Portanto, f é semicont´
ınua superiormente no ponto a.
De modo análogo, admitindo que f é limitada em uma vizinhança de a, prova-se que f é semicont´
ınua inferi-
ormente em a se, e somente se,
f (a)  lim inf f (x).
→a
x

(h)

Suponhamos que f e g : X →R sejam duas fun¸cões semicontı́nuas superiormente em um ponto a ∈ X . Mos-


traremos que f + g é semicont´ ınua superiormente em a.
Seja ε > 0. Existem δ f e δ g > 0 tais que
ε
f (x) < f (a) + ,
2
∈ | − |
sempre que x X e´ tal que x a < δf , e
ε
g(x) < g(a) + ,
2
sempre que x
temos que
∈ X e´ tal que |x − a| < δ . Denotemos min{δ , δ } por δ. Assim, para x ∈ X tal que |x − a| < δ,
g f g

ε ε
(f + g)(x) = f (x) + g(x) < f (a) + + g(a) + = (f + g)(a) + ε.
2 2
Logo, podemos concluir que f + g é semicont´ınua superiormente em a.
Sejam f e g : R R dadas por

sen x1 , se x = 0, ̸
f (x) =
1, −
se x = 0,

292
e
sen x1 , se x = 0, ̸
g(x) =
1, se x = 0.
Pelo item (e), f é semicont´
ınua inferiormente e g e´ semicont´
ınua superiormente. Provaremos que f + g não é
semicont´ınua superiormente nem inferiormente em 0.
Seja δ > 0. Existe n Z+ tal que
∈  
1
< δ.
2πn + π2
Assim, para
1
x := π
2πn + 2

temos que
| x| < δ
e
(f + g)(x) = f (x) + g(x)
= sen x1 + sen x1
= 2
> 0+1
= f (0) + g(0) + 1.
Desta forma, como δ > 0 é arbitrário, concluı́mos que f + g não é semicont´ınua superiormente em 0.
De forma an´aloga, podemos mostrar que f + g não é semicont´ ınua inferiormente em 0.
Suponhamos que f : X R sejam semicont´
→ ınua superiormente. Provaremos que f e´ semicont´ −
ınua inferior-
∈ −
mente em um ponto arbitrário a X . E concluiremos daı́ que f e´ semicontı́nua inferiormente.
Seja ε > 0. Existe δ > 0 tal que
f (x) < f (a) + ε,
para todo x ∈ X tal que |x − a| < δ. Assim,
−f (a) − ε < −f (x),
para todo x ∈ X tal que |x − a| < δ. Portanto, como ε > 0 é arbitrário, −f é semicont´ınua inferiormente em a.
(i)

Suponhamos que f e g : X R sejam duas fun¸cões semicontı́nuas superiormente em um ponto a


→ ∈ X e que

f (x) e g (x)  0, para todo x X . Mostraremos que f g é semicont´ ·
ınua superiormente em a.
Seja ε > 0. Como f (a) + g(a) > 0, podemos escolher λ R tal que ∈
0 < λ < min 1,
1
} .
f (a) + g(a) + 1

Desta forma, ((
(f (a) + g(a))λ + λ2 = (f (a) + g(a)) + λ λ
< (f (a) + g(a)) + 1 λ

< ε.
Seja δ > 0 tal que
f (x) < f (a) + λ
e
g(x) < g(a) + λ,
para todo x ∈ X tal que |x| < δ. Logo,
(f · g)(x) = f (x) g(x)
( · ( 
= f (a) + λ g(a) + λ
= (f g)(a) + (f (a) + g(a))λ + λ2
·
= ·
(f g)(a) + ε,

293
para todo x ∈ X tal que |x| < δ. Portanto, f · g é semicont´ınua superiormente em a.
(j)

Suponhamos que f : X →R seja uma fun¸cão semicontı́nua superiormente em um conjunto compacto X ⊂ R.



Para cada a X , existe δ (a) > 0 tal que
f (x) < f (a) + 1,
para todo x ∈ X tal que |x| < δ(a).
Como (− 
X ⊂∪∈ a X a δ (a), a + δ (a)
e X e´ compacto, existem a 1 , . . . , a n ∈ X tais que
X⊂∪ n
k=1
( ak − δ(a ), ak k + δ (ak ) .
Seja
{
M := max f (ak ) + 1: k = 1,...,n }.
Provaremos que M é uma cota superior para f (X ) e concluiremos daı́ que f é limitada superiormente.

Seja x X . Para algum k ∈{
1,...,n , x } ∈( −
ak δ (ak ), ak + δ (ak ) já que X n
k=1 ak
 ⊂∪ ( −
δ (ak ), ak + δ (ak ) .

Logo, pela escolha de δ (ak ), temos que
f (x) < f (ak ) + ε  M.
Portanto, M é uma cota superior para f (X ).
Seja (xn )n∈Z+ uma sequência em X tal que

lim f (xn ) = sup f (X ).


n → +∞

Como X é compacto, existe uma subsequência convergente (xnk )k∈Z+ de (xn )n∈Z+ tal que a = lim k→+∞ xnk ∈ X.
Assim, pelo item (g),
sup f (X ) = lim n→+∞ f (xn )
= lim n→+∞ f (xnk )
 limx→a sup f (x)
 f (a).
Logo, f (a) = sup f (X ) e, portanto, f assume seu valor máximo em um ponto de X .
De modo análogo, podemos provar que se f : X R for uma função semicont´ınua inferiormente em um conjunto

compacto X R então f é limitada inferiormente e assume seu valor m´
⊂ ınimo em um ponto de X .

294
Capı́tulo 8

Derivadas

295
Exercı́cio 8.46:
Dadas f e g anal´ ⊂
ıticas no intervalo aberto I , seja X I um conjunto que possui um ponto de acumula¸ cão a I . ∈
∈ ∈
Se f (x) = g(x), para todo x I . Em particular, se f (x) = 0, para todo x X , então f (x) = 0, para todo x I . ∈
Sejam f e g : I R funções analı́ticas definidas no intervalo aberto I e X I . Suponhamos que X é tal que
→ ⊂
f = g X e exista a X ′ I .
| X | ∈ ∩
Pelo exercı́cio 8.45, basta provarmos que f (n) (a) = g (n) (a), n Z+ , para concluirmos que f = g. Assim,

mostrando, por indução em n Z+ , que a fun¸cão ϕ := f g e´ tal que
∈ −
0 = ϕ(n) (a) = f (n) (a) −g (n)
(a),


para todo n Z+ , teremos o resultado do enunciado.
Seja ( xk )k∈Z+ uma sequência em X tal que lim k→+∞ xk = a e xk = a, para todo k ̸ ∈Z +. Denotemos por

(hk )k∈Z+ definida por h k = xk a. Segue que

lim hk = 0.
k →+∞
Para n = 0, temos que
ϕ(0) (a) = −
f (a) g(a)
=
=
=
( − −− 
lim x→a f (x) limx→a g(x)
lim k→+∞ f (xk ) limk→+∞ g(xk )
lim k→+∞ f (xk ) g(xk )
= lim k→+∞ 0
= 0.
Suponhamos que ϕ (m) (a) = 0 para todo 0  m < n. Pela F´ormula da Taylor com Resto de Lagrange (Teorema

8.10), temos que existe θ k (0, 1) tal que
n 1 − ϕ(i) (a) ϕ (n) (a + θk hk ) n ϕ(n) (a + θk hk ) n
0 = ϕ(xk ) = ϕ(a + hk ) = hik + hk = hk .
i! n! n!
i=0
Logo,
lim θk hk = 0
k →+∞
e, como h k = a − x ̸= 0,
k
ϕ(n) (a + θk hk ) = 0.
Desta forma,
ϕ(n) (a) = lim ϕ(n) (x) = lim ϕ(n) (a + θk hk ) = 0.
x →a k →+∞
Portanto, pelo princ´ıpio da indução finita, ϕ (n) (a) = 0, para todo n Z+ . ∈
Em particular, provamos que se f : I R é uma função analı́tica definida no intervalo aberto I , X
→ ⊂ I, f| X =0
∈ ∩
e a X I então f = 0 pois 0 é anal´ıtica em I .

296
Exercı́cio 8.47:

Seja I = (a δ, a + δ ). Dada f : I → R, de classe C ∞, suponha que existam constantes a 0 , a1 , . . . , an , . . . tais

que, para todo x I , se tenha

n
f (x) = an (x − a) .
n=0

Prove que
∑ an (x − a) n
é a série de Taylor de f em torno de a, isto é, que

f (n) (a)
an = ,
n!
para todo n = 0, 1, 2, . . . .

Seja
′′ (n)
f (x) = f (a) + f ′ (a) (x · − a) + f 2!(a) · (x − a) + · · · + f 2 (a)
n!
· − a)
(x n
+ r(x − a),

para x I , a n-ésima expansão de Taylor de f em torno de a.
Provaremos, por indução em n Z0 , que ∈
f (n) (a)
an = .
n!
Para n = 0, temos da igualdade

n
f (x) = an (x − a)
n=0
que
∞ ∞
k
f (a) = ak (a − a) = ak (0)k = a 0 .
k=0 k=0

Suponhamos, como hipótese de indução, que

f (k) (a)
ak = ,
k!
para todo k ∈ Z ∩ [0, n). Assim,
0

f (n 1) (a)
− −
n 1 f (n) (a) n
f (a) + · · · + − (x − a) +
(n 1)!
(x − a) + r(x − a) n!
= f (x) ∑∑
= a + · · · + a − (x − a) − + a (x − a) + ∞
0 n 1
n 1
a (x − a)n
n
k=n+1 k
k

−f (n 1) (a)

∞ n 1 n k
= f (a) + · · · + − (x − a) + a (x − a) +
(n 1)!
a (x − a) n k=n+1 k

e, consequentemente,
f (n) (a)

n n k
n!
(x − a) + r(x − a) = a (x − a)
n + ak (x − a) .
k=n+1

Logo, para x ∈ I \{a}, ∞


f (n) (a) n r(x − a)
k
n!
−a n = (x − a)− ak (x − a) − (x − a) . n
k=n+1

Por fim, ∑∑
f (n) (a) ∞ r(x−a)
n! −a n = lim →a (x − a)−n
x
k
k=n+1 ak (x − a) − (x−a) n

= lim x→a (x − a)−n


∞ k r(x a)

k=n+1 ak (x − a) − limx→a (x a)n

= 0+0
= 0.
Portanto, o resultado segue pelo PIF.

297
Exercı́cio 8.48:
x5
Seja f (x) = 1+x6 . Calcule as derivadas de ordem 2001 e 2003 da fun¸ cão f : R → R no ponto 0.
Para y ∈ (−1, 1) temos, pelo Exemplo 24 do Capı́tulo IV, temos que
1

= yk .
1 −y k=0

Assim, para todo x ∈ (−1, 1), temos que


5
x
f (x) = 6

= x1+x
5 1
1−(−x6 )
= x 5
k=0
∑∑ −
∞ ( x6 )k
= x5 ∞ ( 1)k x6k
∑∑ − −
= ∞ k=0
( 1)k 6k+5
x
k=0
= ∞ n
n=0 an x ,

onde
( 1)k , se n = 6k + 5 para algum k
− ∈Z 0 ,
an :=
0, caso contr´ ario.
Pelo exerc´
ıcio 8.47, temos que

( 1)k n!, se n = 6k + 5 para algum k


− ∈Z 0 ,
f (n) (0) = a n n! :=
0, caso contr´ ario.

Assim,
f (2001) (0) = f (333·6+3) (0) = 0
e
f (2003) (0) = f (333·6+5) (0) = ( 1)333 2003! = 2003!.
− −

298
Exercı́cio 8.49:
Seja f : I →R definida num intervalo. Prove que f é convexa se, e somente se, para quaisquer a e b em I e
0  t  1, vale
− −
f ((1 t)a + tb)  (1 t)f (a) + tf (b).

Suponhamos que f : I → R seja convexa. Mostraremos que


f ((1 − t)a + tb)  (1 − t)f (a) + tf (b), (8.1)
para todos a e b ∈ I e t ∈ [0, 1]. Para a = b, temos que
f ((1 − t)a + tb) = f ((1 − t)a + ta) = f (a) = (1 − t)f (a) + tf (a) = (1 − t)f (a) + tf (b).
Adiante, mostraremos para o caso em que a < b. Deste caso segue o caso em que b < a. De fato, para a′ = b,
′ ′
b = a e t = 1 − t,
f ((1 − t)a + tb) = f ((1 − t′ )a′ + t′ b′ )  (1 − t′ )f (a′ ) + t′ f (b′ ) = (1 − t)f (a) + tf (b).
Portanto, se f e´ convexa, então a desigualdade ( 8.1) é v´ alida para todos a e b ∈ I e t ∈ [0, 1].
Sejam a e b ∈ I , com a < b, e t ∈ [0, 1]. Desta forma x := (1 − t)a + tb e´ tal que
x − a = t(b − a)  0
e
b − x = (1 − t)(b − a)  0.
Ou seja, x ∈ [a, b] ⊂ I . Além disso, segue da definição de x que
x−a
t=
b−a
e
b−x
1−t= .
b−a
Como f é convexa, temos que
f (x) f (a) f (b) f (a)
x−a  b −a
e, consequentemente,
b−x x−a
f (x)  f (a) + f (b).
b−a b−a
Logo,
f ((1 − t)a + tb) = f (x)
b −x x−a
b−a f (a) + b−a f (b)


= (1 t)f (a) + tf (b).
Consideremos, agora, que f satisfaz a desigualdade ( 8.1), para todos a e b I e t ∈ ∈ [0, 1]. Mostraremos que f
é convexa.
∈ ∈
Sejam a e b I , com a < b, e x [a, b]. Definindo

t :=
x a −
b a −
temos que
x = (1 t)a + tb −

e, da desigualdade a  x  b, que t [0, 1]. Da´ı,

f (x) =
 (1 −−
f ((1 t)ft)a
(a)++tb)
tf (b)
= −
b x
f (a) + −
x a
f (b)
b a
− b a

e, consequentemente,
f (x) − f (a)  f (b) − f (a) .
x −a b−a
Com isso, concluimos que f e´ convexa.

299
Exercı́cio 8.50:
x
Verifique que f : R → R, dada por f (x) = e , é convexa e conclua que, para 0  t  1 e x e y ∈ R quaisquer vale
(1 t)x+ty x y
e −  (1 − t)e + te .

Deduza daı́ a desigualdade


aα bβ  αa + βb,
para α, β , a, b não-negativos, com α + β = 1.

Seja exp: R → R a aplicação exponencial. Como exp ′′ = exp  0, temos, pelo Teorema 11 do Capı́tulo 8, que
exp é uma funç˜
ao convexa. Assim, pelo Excercı́cio 8.50,
( 
e(1−t)x+ty = exp (1 t)x + ty  (1 t) exp(x) + t exp(y) = (1
− − − t)ex
+ tey ,

para todos x e y R e t [0, 1].


∈ ∈
Dados a, b, α e β R+ , com α + β = 1, provaremos que vale a desigualdade

aα bβ  αa + βb.

Sejam x := ln(a) e y := ln(b) ∈ R e t = β ∈ [0, 1]. Temos, pela desigualdade provada acima, que
aα b β = eα ln(a) eβ ln(b)
= e(1−t)x ety
= e(1−t)x+ty
 (1 t)ex + tey

= αeln(a) + βe ln(b)
= αa + βb.

300
Exercı́cio 8.51:
Seja F : I
∑ → R convexa no intervalo I . Se a1 , . . . , an pertencem ao intervalo I , t1 , . . . , tn pentencem a [0 , 1] e
n
i=1 ti = 1, prove que
n n
f ti ai  ti f (ai ).
i=1 i=1

Primeiramente, verifiquemos que se a1 , . . . , ak pertencem ao intervalo I , t1 , . . . , tk pentencentem a [0 , 1] e


k
= 1 então ni=1 ti ai I . Sejam
i=1 ti ∈
a := min a1 ,...,a n I { }∈

e ∑ b := max a1 ,...,a { n } ∈ I.
Segue que
n n n
a= ti a= ti a  ti ai
i=1 i=1 i=1
e
n n n
ti ai  ti b = ti b = b.
i=1 i=1 i=1

Logo,
n
ti ai ∈ [a, b] ⊂ I .
i=1

Provaremos o resultado por indução em n.


No caso n = 2, temos que
t1 = 1 −t 2

e, pelo Exercı́cio 8.49, temos que

f (t1 a1 + t2 a2 ) = f ((1 t2 )a1 + t2 a2 ) −


= (1 t2 )f (a1 ) + t2 f (a2 ) −
= t1 f (a1 ) + t2 f (a2 ).

Suponhamos, como hipótese de indução, que o caso n = k 1 seja verdadeiro. − ∑ k


Sejam a 1 , . . . , a k pertencentes ao intervalo I , t 1 , . . . , t k pentencentes a [0, 1] e i=1 ti = 1.
Se t k = 1 segue que t 1 = ·· ·
= t k−1 = 0 e, consequentemente,
k k
f ti ai = f (ak ) = ti f (ai ).
i=1 i=1

̸
Consideremos agora o caso em que t k = 1. Desta forma,
ti
t′i :=
1 −t k

são tais que


k 1− k 1
− −
k 1
ti i=1 ti
′i
i=1 t = i=1 − = 1−t
1 tk ∑ k =1
e
t′ ∈ [0, 1].
i

Seja
k 1

a := t′i ai ∈ I.
i=1

301
Assim, pela hip´otese de indu¸cão,
k k 1

f ti ai = f − t ) 1 −t t a + t a
(1 k
i
k
i k k
i=1
=
(
f (1 − t )a + t ak
i=1
k k

 (1 − t )f (a) + t f (a )
k k k
=

(1 − t )f
(1 − t )
∑ k 1

k
k
∑∑
− t′ a + t f (a )
k 1
k 1
i=1 i i
− t′ f (a ) + t f (a )
i=1 i k i
k

k
k

= i=1 ti f (ai ) + tk f (ak )


k
= i=1 ti f (ai ).

Portanto, o resultado segue pelo PIF. ∑

302
Exercı́cio 8.52:
Sejam x 1 , x 2 , . . . , x n e t 1 , . . . , t n números não-negativos, com t 1 + · · · + t = 1. Prove que
n

xt11 xt22
· ····· xtnn  t1 x1 + t 2 x2 + ··· + t x . n n

Conclua, em particular a desigualdade entre as médias aritmética e geométrica.

Denotemos por exp : R → R a aplicação exponecial. Como exp ′′ = exp  0, temos, pelo Teorema 11 do Cap´ıtulo
8, que exp é uma função convexa.
Sejam x 1 , . . . , x n , t 1 , . . . , t n R0 tais que t 1 +
∈ ···
+ tn = 1. Se algum x i é nulo, temos imediatamente que
t1 tn
x1 ··· xn =0  t1 x1 + ...t n xn .

Se x 1 , . . . , x n ∈ (0, +∞) então


t1 t2
x · x ·····
1 2 xtnn = et1 ln x1 et2 ln x2 · etn ln xn
·····
= et1 ln x1 +t2 ln x2 +···+tn ln xn
= exp( t1 ln x1 + t2 ln x2 + + tn ln xn ). ···
Assim, como exp é convexa, temos, pelo Exerc´
ıcio 8.51, que

xt11 xt22
· ····· xtnn = exp( t1 ln x1 + t2 ln x2 + + tn ln xn ) ···
= t1 exp(ln x1 ) + t2 exp(ln x2 ) + + tn exp(ln xn ) ···
= t1 x1 + t2 x2 + + t n xn . ···
Sejam x 1 , ... x n números reais não negativos,

G :=
√x · x ·····
n
1 2 xn

sua média geométrica e


A :=
x1 + ··· + x n

n
sua média aritmática. Pelo que foi demonstrado acima, temos que
1 1 1
G = x1n x2n
x1 x2
· ····· x n
n
xn
 n + n + ···+ n
= A.

303
Exercı́cio 8.53:

Seja ϕ : [a, b] R duas vezes derivável, com ϕ(a) = ϕ(b) = 0 e ϕ ′′ (x) < 0 para todo x [a, b]. Prove que ϕ(x) > 0,


para todo x (a, b). Conclua que, se f : I R é duas vezes derivável e f ′′ (x) > 0, para todo x I , então f é
→ ∈
estritamente convexa no intervalo I .


Seja ϕ : [a, b] R, a < b, duas vezes deriv´avel, com ϕ(a) = ϕ(b) = 0 e ϕ ′′ (x) < 0 para todo x [a, b].

Como ϕ′′ (x) < 0 para todo x [a, b], temos que ϕ′ é estritamente decrescente (Corolário 6 do Teorema 7 do

Capı́tulo VIII). Além disso, como ϕ(a) = 0 = ϕ(b), pelo Toerema de Rolle, existe c (a, b) tal que ϕ′ (c) = 0.

Desta forma, como ϕ ′ é estritamente decrescente em [a, b] e ϕ ′ (c) = 0, ϕ ′ > 0 em [ a, c) e ϕ ′ < 0 em ( c, b]. Com isso,
concluimos que ϕ é estritamente crescente em [a, c] e estritamente decrescente em [c, b] (novamente pelo Corolário
6 do Teorema 7 do Capı́tulo VIII).
Sendo ϕ(a) = 0 e ϕ estritamente crescente em [ a, c] segue que ϕ > 0 em ( a, c]. Em particular, temos que
ϕ(c) > ϕ(a) = 0. Por outro lado, como ϕ(c) > 0 = ϕ(b) e ϕ é estritamente decrescente em [c, b), ϕ > 0 em [ c, b].
Portanto, ϕ > 0 em ( a, b).

Seja f : I R é duas vezes derivável e f ′′ (x) > 0, para todo x I , então f é estritamente convexa no intervalo

I . Provaremos, para a e b em I , com a < b, arbitrários, que

f (x) − f (a) < f (b) − f (a) ,


x −a b−a

para todo x (a, b). Com isso, concluiremos que f e´ estritamente convexa.
Definimos ϕ : [a, b] R por

ϕ(x) =
f (b) f (a) − −
(x a) + f (a) f (x), −
b a −
para todo x ∈ [a, b]. Desta forma,
ϕ(a) = ϕ(b) = 0,

ϕ′ (x) =
− f (a) − f ′ (x)
f (b)
−a
b
e
ϕ′′ (x) = −f ′′ (x) < 0,

para todo x I . Logo, pelo que foi provado acima, ϕ > 0 em ( a, b).

Assim, para todo x (a, b),

0 < ϕ(x) =
f (b) − f (a) (x − a) + f (a) − f (x)
b −a
e, consequentemente,
f (x) − f (a) < f (b) − f (a) .
x −a b−a

304
Exercı́cio 8.54:
Seja f cont´
ınua num ponto. Prove que se f e´ derivável nesse ponto então existe na m´aximo uma reta que coincide
com o gráfico de f uma infinidade de vezes em qualquer vizinhan¸ca do ponto.

Seja f : X→R derivável em a X ′ .



Suponhamos que c e d R são tais que, para qualquer vizinhan¸ca U de a em X ,

f (x) = cx + d,

para infinitos x ∈ U . Em particular, existe uma sequência (x n )n Z+


∈ em X tal que

→∞ xn = a
nlim

e
f (xn ) = cx n + d,
para todo n Z+ .

Como f e´ derivável em a, temos que

c = lim
(cxn + d) − (ca + d)= lim
f (xn ) − f (a)= lim f (x) − f (a) = f ′ (a).
n→∞ xn −a n→∞ xn −a x → a x −a
Além disso, como f e´ cont´ınua em a,

ca + d = lim (cxn + d) = lim f (xn ) = f (a)


n→∞ n →∞
e, consequentemente,
d = f (a) − ca = f (a) − f ′ (a)a.
Portanto, concluimos que se f coincide em infinitos p ontos, em cada vizinhança de a, com uma reta

(x,ca + d) : x R
{ ∈ }
então c e d são determinados unicamente pelos valores de f (a) e f ′ (a). Em particular, existe no m´aximo uma reta
com esta propriedade.

305
Exercı́cio 8.55:
Seja f : [a, + ∞) → R duas vezes derivável. Se x
lim f (x) = f (a) então existe x
→ +∞
∈ (a, +∞) tal que f ′′ (x) = 0.
Usaremos o seguinte resultado: Se g : [c, + ) R é uma função derivável que adimite uma sequência (xn )n∈Z+
∞ →
em [c, + ) tal que lim n→+∞ xn = + e lim n→+∞ g(xn ) = g(c) então existe x (c, + ) tal que g ′ (x) = 0.
∞ ∞ ∈ ∞
Se g(y) = g(c), para todo y (c, + ), qualquer x (c, + ) é tal que g′ (x) = 0. Suponhamos que exista
∈ ∞ ∈ ∞
∈ ∞ ̸
y (c, + ) tal que g(y) = g(c). Provaremos que exsite x como desejado para o caso em que

g(y) > g(c).

O caso em que g(y) < g(c) é análogo. Como lim n→+∞ xn = + e lim n→+∞ g(xn ) = g(c), deve existir algum
xn ∈ (y, +∞) tal que g(xn

) ∈ (g(c) − 1, g(y)).
Se
g(xn ) ∈ (g(c) − 1, g(c)]
temos que
g(xn )  g(c) < g(y)
e, pelo Teorema do Valor Intermediário, existe um z ∈ (y, x n] tal que

g(z) = g(c).

∈ (a, z) tal que g ′ (x) = 0. Se


Logo, pelo Teorema de Rolle, existe x

g(x ) ∈ (g(c), g(y)),


n

temos, pelo Teorema do Valor Intermadiário, que existe z ∈ (c, y) tal que

g(z) = g(xn ).

∞ →∈
Logo, pelo Teorema de Rolle, existe x (z, xn ) tal que g ′ (x) = 0.
Consideremos, agora, f : [a, + ) R como no enunciado.

Como lim x→+∞ f (x) = f (a), deve existir uma sequência (xn )n∈Z+ em [a, + ) tal que lim n→+∞ xn = + e ∞
limn→+∞ f (xn ) = f (a). Logo, existe um ponto b (a, + ) tal que f ′ (b) = 0.
∈ ∞
Provaremos que existe uma sequência (xn )n∈Z+ em [b, + ) tal que lim n→+∞ xn = + e lim n→+∞ f ′ (xn ) =
∞ ∞
0 = f ′ (b). Assim, concluı́mos que a função derivável f ′ : [b, + ) R admite um ponto x (b, + ) tal que
∞ → ∈ ∞
f ′′ (x) = 0.
Pelo Teorema do Valor Médio, podemos escolher, para cada n Z+ , x n (a + n, a + n + 1) tal que
∈ ∈
f ′ (xn ) = f (a + n + 1) − f (a + n).

Segue da´
ı que limn→+∞ xn = + ∞e
lim f ′ (xn ) =
n
→ +∞
lim
(
→+∞ f (a + n + 1) f (a + n)
n
− 
= lim f (a + n + 1) − lim f (a + n)
n→+∞ n→+∞
= f (a) − f (a)
= 0.

306
Capı́tulo 9

Integral de Riemann

307
Capı́tulo 10

Sequências e Séries de Funções

308
Exercı́cio 10.44:
A sequência de funções fn (x) = nx2 possui derivadas equilimitadas no ponto 0 mas n˜ ao é equicont´
ınua neste
ponto.

Como f n′ (0) = 0 para todo n Z+ temos que a sequência (fn ) possui derivadas limitadas em 0.

Por outro lado, para todo δ > 0, existe x ( δ, δ ) e n Z+ tais que
∈− ∈
|f (0) − f (x)|  1.
n n

De fato, dado δ > 0, existe k ∈Z + tal que 1 /k < δ. Assim, para n = k 2 e x = 1/k, temos que

|f n (0) −f | = |0 − 1| = 1.
n (x)

Portanto, a sequência não é equicont´


ınua em 0.

309
Exercı́cio 10.45:
Um conjunto de polinômios de grau  k, uniformemente limitado em um intervalo compacto, é equicont´
ınuo neste
intervalo.

Seja E tal conjunto de polinˆomios definidos no intervalo compacto I .


Podemos considerar, sem perda de generalidade, que I = [0, b] para algum b > 0. De fato, se I = [a, b], podemos
definir
{
Ẽ := p˜ : [0, b a] R p E, p(x)
− → |∃ ∈ ˜ = p(x + a) x [0, b a] ∀ ∈ − }
e teremos, assim, que E é uniformemente limitado se e somente se Ẽ e´ uniformemente limitado e que E e´ equi-
contı́nuo se e somente se Ẽ e´ equicont´
ıno.
∑ k Provaremos,
i
por indução em k, que existe uma constante C > 0 tal que, para todo p ∈ E dado por p(x) =
i=0 ai x , vale
|a | < C , i = 0,...,k.
i (10.1)
Donde conclui-se que o conjunto das derivadas dos polinˆ omios de E e´ equilimitado e, consequentemente, que E é
equicont´
ınuo.
Para k = 0, temos que ( 10.1) segue do fato de E ser equilimitado.

Suponhamos que exista C > 0 satifazendo ( 10.1) para todo k  n 1. E suponhamos que k = n.
Como E é equilimitado, segue que o conjunto

E ′ := p { − p(0) | p ∈ E }
é também equilimitado. Consideremos

E ′′ := p
{ | ∃p˜ ∈ E ′ , p(x)
˜ = p(x)x, ∀x ∈ I }

Adiante, dada uma constante A > 0 tal que

|p(x)| < A, ∀p ∈ E ′, x ∈ I ,
temos que A ′ = A/b e´ tal que
|p(x)| < A′, ∀p ∈ E ′′, x ∈ I .
Ou seja, E ′′ e´ equilimitada. Além disso, todos os polinômios em E ′′ são de grau
∑  n − 1. Pelo passo indutivo,
n
existe C ′ > 0 tal que, para todo p ∈ E ′′ dado por p(x) = ai xi−1 , vale
i=1

|a | < C ′ , i = 1,...,n.
i

E, como E é equilimitado, existe C ′′ > 0 tal que, para todo p ∈ E , vale

|a | = |p(0)| < C ′′ .
0

Assim, para C = max{C ′ , C ′′ } temos ( 10.1). E o resultado segue pelo PIF.

310
Exercı́cio 10.46:
Diz-se que uma sequência de funções fn : X R converge fracamente para uma fun¸cão f : X
→ R quando →
limn→∞ fn (x) = f (x) para cada ponto x ∈
X na qual f e´ cont´
ınua. Seja D R denso. Prove que se uma

sequência de funções monótonas f n : R
→ R converge simplesmente em D para uma fun¸cão f : R R então (fn )

converge fracamente para f em R .

Seja x 0 ∈ R um ponto de continuidade de f . Provaremos que

lim fn (x0 ) = f (x0 ). (10.2)


n →∞
Para tanto, basta mostrar, para (fnk ) e (fnp ) sendo as subsequência não-decrescentes e não-crescentes de (fn ), que

lim fnk (x0 ) = lim fnp (x0 ) = f (x0 )


k →∞ p→∞
Com igual raz˜ao, basta provar a afirmação (10.2) para o caso em que todas as f n ’s são decrescentes e para o caso
em que todas as f n ’s são não-crescentes.
Suponhamos que todas as f n ’s são não-decrescentes.
Começaremos provando que f e´ não-decrescente em D. Suponhamos, por absurdo, que existam

x− < x+

em D tais que
f (x− ) > f (x+ ).
Então, como
lim fn (x− ) = f (x− ),
n→∞
lim fn (x+ ) = f (x+ )
n →∞
e
f (x− ) + f (x+ )
f (x− ) > > f (x+ ),
2
temos, para n suficientemente grande, que
f (x− ) + f (x+ )
fn (x− ) > > f n (x+ ).
2
Contradizendo o fato de f n ser não-decrescente. Portanto, f e´ não-decrescente em D.
O próximo passo desta demonstração é provar que se

x− +
0 < x 0 < x0 ,

onde x − +
0 e x 0 ∈ D, então
f (x−
0)  f (x0 )  f (x+
0 ).

De fato, se
f (x−
0 ) > f (x0 ),

ınua em x 0 e D é denso em R , existe ˜x


então, como f e´ cont´ ∈ D tal que
x−
0 < x̃ < x0

e
f (x−
0 ) > f (x̃).

Contradizendo o fato de f ser não-decrescente em D. Portanto, devemos ter que

f (x−
0)  f (x0 ).

E, de forma an´aloga, mostra-se que devemos ter que

f (x0 )  f (x+
0 ).

311
Por fim, provaremos que, dado ε > 0, existe n 0 ∈ Z tal que para todo natural
+ n > n0 vale

|f (x ) − f (x )| < ε.
0 n 0

Com efeito, existem (pois f e´ cont´ınua em x ) x − e x ∈ D tais que


0 0
+
0

x− +
0 < x 0 < x0

e
|f (x−) − f (x )| < ε/2.
0
+
0

E, como ( fn ) converge simplesmente para f em D, existe n ∈ Z , tal que, para todo 0 + n > n 0 , vale

f (x−) fn (x−) < ε/2


| − 0 0 |
e
+ +
|f (x ) − f
0 n (x0 ) | < ε/2.
Pelo que foi dito nos parágrafos acima, temos que

f (x−
0)  f (x0 )  f (x+
0)

e
fn (x−
0)  fn (x0 )  fn (x0
+
).
Logo,
f (x−
0) −f +
n (x0 )  f (x0 ) −f n (x0 )  f (x+
0) −f −).
n (x0

E, como
|f (x−) − f
0
+
| |f (x−) − f (x )| + |f (x ) − f
n (x0 )  0
+
0
+
0
+
|
n (x0 )  ε/2 + ε/2 = ε
e
+
|f (x ) − f −)|  |f (x+) − f (x−)| + |f (x−) − fn (x−) |  ε/2 + ε/2 = ε,
n (x0
0 0 0 0 0

para todo n > n 0 , segue que


|
f (x0 ) fn (x0 ) < ε, − |
para todo n > n 0 . E temos o resultado.
O caso em que todas as f n ’s são não-crescentes é análogo.

312
Exercı́cio 10.47:
Seja f (x) = x + |xx| se x = 0 e f (0) = 0. Obtenha uma sequência de funções cont´
̸ ınuas crescentes f n : R → R que
convirjam para f em R −{ }0 , mas ( fn (0)) não converge.

Considere
f (x), x < −1/k e 0 < x
fk (x) = ( 
2+ 1
k
x+1/k
1/k
1
− − 1, −1/k  x  0
k

para k par e
f (x), −
x < 1/k e 0 < x
fk (x) = x
2+ 1 1, 0  x  1/k
(  k 1/k −
para k ´ımpar. Temos, então, que ( fk ) é uma sequência de fun¸
cões contı́nuas e crescentes que converge para f em
R 0 . E, como
−{ }
fk (0) = ( 1)k , −
temos que ( fk (0)) não é convergente.

313
Exercı́cio 10.48:
Uma sequência de funções monótonas fn : R R possui uma subsequência que converge fracamente para uma

função monótona f : R R, a qual podemos tomar contı́nua à direita.

Provaremos o resultado para o caso em que (fn ) é uma sequência de funç˜ oes não decrescentes. A demonstração
para o caso em que ( fn ) é uma sequência de fun¸ cões não-crescentes é análoga. No caso geral, ( fn ) possui uma
subsequência de pelo menos um destes dois tipos. E esta, por sua vez, possuirá um subsequência com a propriedade
do enúnciado.
Seja (fn ) uma sequência simplesmente limitada de funções não decrescentes.
Pelo Teorema de Cantor-Tychonov existe uma subsequência (fnk ) de ( fn ) que converge em Q para uma função
f˜ : Q R. Como ( fnk ), devemos ter que f˜e´ não-decrescente.

Como f˜ e´ monótona, existe lim x→x+
0
f˜(x). Assim, podemos definir

f: R → R
x0 → limx→x+ f˜(x).
0

Provaremos que f e´ cont´


ınua à direita e ( fnk ) converge fracamente para f .
Seja x 0 R. Provaremos que

f (x0 ) = lim + f (x).
x →x0
Seja ( xn ) uma sequência em R que converge `a direita para x0 . Pela definição de f , para cada n ∈Z + existe
yn Q (xn , + ) tal que
∈ ∩ ∞
1
xn yn <
n
| − |
e
1
f (xn ) f˜(yn ) < .
| − |
n
Assim,
lim yn = lim xn = x0
n →∞ n→∞
e, consequentemente,
lim f (xn ) = lim f˜(yn ) = lim + f˜(x) = f (x0 ).
n →∞ n →∞ x →x0
Como (xn ) é uma sequência que converge para x 0 à direita arbitrária, segue que

lim+ f (x) = f (x0 ).


x
→x0
Por sua vez, como x 0 e´ arbitrário, conclu´ımos que f e´ cont´
ınua à direita.
Para todos x 0 R, x −∈ 0 e x0
+
Q tais que

x− +
0 < x 0 < x0 ,

temos que
f˜(x−
0)  f (x0))  f˜(x+
0 (10.3)
pois
f (x0 ) = lim x→x+ f˜(x)
0

e f˜e´ não-decrescente.
Seja x 0 um ponto de continuidade de f e ε > 0. Provaremos que existe k0 ∈Z + tal que para todo k > k0 em

Z temos que
+
|f (x ) − f 0 nk (x0 )| < ε.
Como x 0 é ponto de continuidade de f , existem y 0− e y 0+ ∈ R tais que
y0− < x 0 < y0+

e
|f (y ) − f (y−) | < 2ε .
+
0 0

314
Tomando-se x − +
0 e x0 ∈ Q tais que
y0− < x − + +
0 < x0 < x0 < y 0

temos, por ( 10.3), que


f (y0−)  f˜(x−
0)  f (x0 )  f˜(x+
0)  f (y0+).
Assim,
|f˜(x ) − f˜(x−)| < 2ε .
+
0 0

Como (fnk ) converge simplesmente para f˜ em Q , existe k ∈ Z tal que, para todo k > k
0 + 0 vale

|f˜(x−) − f (x−)| < 2ε


0 nk 0

e |f˜(x ) − f
+ +
| < 2ε .
0 nk (x0 )

Do que foi dito acima, e pelo fato de cada f nk ser não-decrescente, temos que

f˜(x−
0)  f (x0 )  f˜(x+
0 ),

fnk (x−
0)  fnk (x0 )  f (x+
0)

e, consequentemente,
f˜(x−
0) −f +
nk (x0 )  f (x0 ) −f nk (x0 )  f˜(x+
0) −f −).
nk (x0

Por fim, como


|f˜(x−) − f
0
+
| |f˜(x−) − f˜(x )| + |f˜(x ) − f
nk (x0 )  0
+
0
+
0
+
nk (x0 ) | < 2ε + 2ε = ε
e
|f˜(x ) − f
+ −)|  |f˜(x+) − f˜(x−)| + |f˜(x−) − fn (x−)| < ε + ε = ε
nk (x0
0 0 0 0 0 k
2 2
para todo k > k0 em Z + , temos que
|f (x ) − f
0 nk (x0 ) |<ε
para todo k > k0 em Z + .

315
Exercı́cio 10.49:
Seja ( fn ) uma sequência equicontı́nua e simplesmente limitada num compacto X ⊂ R. Se toda subsequência
uniformemente convergente em X tem o mesmo limite f : X →R, então f n f uniformemente.

Suponhamos, por absurdo, que a sequência (fn ) não convirja uniformemente para f . Então, existe uma sub-
sequência (fnk ) de ( fn ) e uma sequência
| − |
f (xnk ) fnk (xnk )  ε (10.4)
para algum ε > 0 fixo. Porém, como X é compacto, (fnk ) é equicont´ ınua e simplesmente limitada, temos, pelo
Teorema 10.23, que alguma subsequência de (fnk ) converge uniformemente em X para uma fun¸cão f˜. E, como
toda subsequência uniformemente convergente de (fn ) converge para f , devemos ter que f˜ = f . O que contradiz
(10.4).

316
Exercı́cio 10.50:
Dê exemplo de uma sequência equicont´
ınua de fun¸
cões f n : (0, 1) → (0, 1) que não possua subsequência uniforme-
mente convergente em (0, 1).

Seja (fn ) a sequência dada por

fn :
x ∈ ([  →→ ( 
(0, 1)
1
0, 2πn
1
(0, 1)
1
4 sin
1
1
x + 1
2
x ∈ →
2πn , 1 2.

Esta sequência converge simplesmente para a função

f : (0, 1) → (0, 1)
1
x → 2.

Porém nenhuma subsequência desta sequencia converge uniformemente.

317
Exercı́cio 10.51:
Dada uma sequência de funções duas vezes deriváveis (fn ) definidas no intervalo compacto I , suponha que f n → f
simplesmente em I , que ( fn′ (a)) é limitada para um certo a I e que ( fn′′ ) é uniformemente limitada em I . Prove

que f C 1 .

Temos que a sequência (fn′ ) é equicontı́nua e uniformemente limitada. De fato, como (fn′′ ) é uma sequência
uniformemente limitada, segue que (fn ) é equicont´ ınua. Adiante, como (fn′′ ) é uniformemente limitada e I é
compacto, existe, pelo teorema do valor médio, C ′ > 0 tal que

|f ′ (x) − f ′ (a)| < C ′, ∀n ∈ Z


n n +, x ∈ I.
E, como ( fn′ (a)) é uma sequência limitada, existe C > 0 tal que
|f ′ (a)| < C, ∀n ∈ Z
n +.

Então,
|f ′ (x)|  |f ′ (x) − f ′ (a)| + |f ′ (a)| < C ′ + C, ∀n ∈ Z
n n n n +, x ∈ I.
Ou seja, ( fn′ ) é uniformemente limitada.
Adiante, temos que toda subsequência uniformemente convergente de (fn′ ) converge para f ′ . De fato, dada uma
subsequência uniformemente convergente (fn′ k ) temos, pelo Teorema 10.7, que f n′ k f ′ já que f nk
→ →f.
Como (fn ) é uma sequência equincontı́nua, uniformemente limitada tal que todas as suas subsequências unifor-

memente convergentes convergem para f , temos, pelo exercı́cio 10.49, que f n ′ ′ →
f uniformemente em I .
Portanto, f ′ é cont´
ınua, pois é o limite uniforme de uma sequência de fun¸cões cont´
ınuas.

318
Exercı́cio 10.52:
Dada uma sequência de funções k+1 vezes deriváveis (fn ) definidas no intervalo I , suponha que existam a0 ,...,a k I
(k) (k+1)

e c > 0, tais que fn (a0 )  c, fn′ (a1 )  c,..., fn (ak )  c para todo n Z+ e que a sequência (fn ) seja
| | | | | | ∈
uniformemente limitada em I . Prove que existe uma subsequência (fni ) que converge, juntamente com suas k
primeiras derivadas, uniformemente em cada parte compacta I .

É suficiente provar que existe uma subsequência (fni ) que converge, juntamente com suas k primeiras derivadas,
uniformemente em I para o caso em que I é um intervalo compacto. Então, assumiremos sem perda de generalidade,
que I e´ um intervalo compacto.
(k) (k+1)
Temos que a sequência (fn ) é equicontı́nua e uniformemente limitada. De fato, como (fn ) é uma sequência
(k) (k+1)
uniformemente limitada, segue que (fn ) é equicontı́nua. Adiante, como (fn ) é uniformemente limitada e I é
compacto, existe, pelo teorema do valor médio, C ′ > 0 tal que

|f (k)
n (x) −f (k)
n (ak ) | < C ′ , ∀n ∈ Z +, x ∈ I.
(k)
E, como ( fn (ak )) é uma sequência limitada, existe C > 0 tal que
(k)
|f n (ak ) | < C, ∀n ∈ Z +.

Então,
|f(k)
n (x) | |f (k)
n (x) −f (k)
n (ak ) | + |f (k)
| < C ′ + C, ∀n ∈ Z
n (ak ) +, x ∈ I.
(k)
Ou seja, ( fn )é uniformemente limitada.
Agora, provaremos o resultado por indução em k.
Consideremos o caso k = 0. Pelo que foi dito acima, temos que ( fn ) é equicont´
ınua e uniformemente limitada.
Segue daı́ e do fato de I ser compacto, pelo Teorema 10.23, que ( fn ) possui uma subsequência uniformemente
convergente.
(k)
Faremos agora o passo indutivo. Como ( fn ) é uniformemente limitada, temos que existe uma subsequência
(k)

(fni ) que converge, juntamente com suas k 1 primeiras derivadas, uniformente. Adiante, temos que ( fni ) é
(k) (k)
equicontı́nua e uniformemente limitada. Isto implica que existe uma subsequência (fnj ) de ( fni ) que converge
uniformemente. Portanto, devemos ter que (fnj ) e suas primeiras k derivadas convergem uniformemente.

319
Exercı́cio 10.53:
Demonstre o corolário de Teorema 22 para intervalos arbitrários (abertos ou n˜ao) I ⊂ R.
∅ ∅
Se int(I ) = , os ´unicos compactos em I são e I . Assim, o Teorema de Arzel´a-Ascoli se aplica imediatamente
e o resultado segue. Adiante assumiremos que existe c int(I ).∈

Consideremos o intervalo J = [a, b] I dado por

a=
c, caso inf( I ) / I ∈
inf(I ), caso contr´ario
e
b= c,
sup(I ), caso
casosup( Iario.
contr´)/I ∈
Se J = I , temos que I é compacto, e, pelo Teorema de Arzelá-Ascoli, temos que existe uma subsequência de (fn )
que converge uniformemente em I . E, logo, em cada parte compacta de I . Assumiremos daqui em diante que
̸
J = I.
Então existe uma sequência de compactos K i I , i Z+ tais que
⊂ ∈
K1 = J,

Ki ⊂K i+1

e
int(I ) = ∪ ∈ int(K ).
i Z+ i

Seja K ⊂ I um compacto. Pela escolha de J , temos que K \J ⊂ int(I ) e que K \J é um compacto. Da´ı segue
que
K \J ⊂ ∪ ∈ int(K )
i Z+ i

e, consequentemente,
\ ⊂K
K J i

para algum i ∈Z +. Logo,


K ⊂J ∪K i = Ki
para algum i Z+ .

Agora, como na demonstração dada no texto, existe uma subsequência de (fn ) que converge uniformemente em
cada Ki , i Z+ . Portanto, como cada parte compacta de I está contida em algum Ki , temos que esta subsequência

converge uniformemente em toda parte compacta de K .

320

Você também pode gostar